Sunteți pe pagina 1din 171

Intellectual

Property
Law
Prof. Susan D. Villanueva
2 n d Semester, AY 2011-2012
Katrina Michelle Mancao
Sp eci al t han k s to K a rich i Sa n tos for hel pi n g m e c omp let e m y no tes ,
a nd to M a rie E strel la a n d Di Bo n ill a fo r sh ari ng thei r correc t a n s w ers to o ur hw J
 
Contents  
I.   INTRODUCTION   1  
II.   CONSTITUTION   2  

III.   INTERNATIONAL  CONVENTIONS   3  


IV.   CIVIL  CODE   5  
V.   COPYRIGHT  AND  RELATED  RIGHTS   6  
VI.   LAW  ON  TRADEMARKS,  TRADENAMES  AND  SERVICE  MARKS   63  
VII.   LAW  ON  PATENTS   126  
 
  -­‐ It   is   separate   and   distinct   from   the   thing  
I. Introduction   created.  
  o Thus,   it   can   be   subject   to   a   separate  
contract.  
Justin  Hughes,   The   Philosophy   of   Intellectual   Property,  
-­‐ Rules  for  IP  are  different  from  rules  governing  
77  Geo.  L.J.  287  (1988)   traditional  property.  
  -­‐ What’s  the  point  of  granting  IP?  
Marshall   A.   Leaffer,   Protecting   United   States   o IP   is   an   incentive   system   designed   to  
encourage   individuals   to   create,   to  
Intellectual   Property   Abroad:   Toward   a   New  
invent,  to  innovate.    
Multilateralism,  76  Iowa  L.  Rev.  273  (1991)   o BUT   the   higher   purpose   of   IP   is   to  
  improve   the   lives   of   the   public.   It  
A.   Samuel   Oddi,   The   International   Patent   System   and   operates   on   the   assumption   that  
Third  World  Development,  1987  Duke  L.  J.  831  (1987)   everyone   would   benefit   from   the  
creation   of   more   works.   Thus,   IP   is  
 
granted   under   the   “beneficial   to   the  
Michael   A.   Heller   and   Rebecca   Eisenberg,   Can   Patents   people”  clause  of  the  Constitution.  
Deter   Innovation?   The   Anti-­‐commons   in   Biomedical   -­‐ IP,   however,   can   be   in   conflict   with   public  
Research,  280  Science  698  (1998)   access.   It   grants   a   form   of   monopoly   that  
  restricts   people’s   access   to   something.   Thus,   IP  
Law  must  be  balanced  with  a  strong  
24  November  2011  
competition  law.  
Intellectual  Property:  
 
-­‐ It   is   a   form   of   property.   (Note   that   under   the  
TRIPS   –   it   is   mandatory   to   those   who   would  
Civil   Code,   intellectual   creation   is   one   of   the  
like   to   enter   or   join   the   WTO   to   agree   to   this  
modes  of  acquiring  ownership.)  
Agreement.  
-­‐ It  is  an  intangible  property  right  that  belongs  to  
the  creator.    
Katrina Michelle Mancao
 
II. Constitution   their   intellectual   property   and   creations,   particularly  
when  beneficial  to  the  people,  for  such  period  as  may  be  
ARTICLE  XII,  SECTION  6.  
provided  by  law.  
The   use   of   property   bears   a   social   function,   and   all  
 
economic  agents  shall  contribute  to  the  common  good.  
ARTS  AND  CULTURE  
Individuals   and   private   groups,   including   corporations,  
cooperatives,   and   similar   collective   organizations,   shall   SECTION  14.    
have   the   right   to   own,   establish,   and   operate   economic   The  State  shall  foster  the  preservation,  enrichment,  and  
enterprises,  subject  to  the  duty  of  the  State  to  promote   dynamic   evolution   of   a   Filipino   national   culture   based   on  
distributive   justice   and   to   intervene   when   the   common   the   principle   of   unity   in   diversity   in   a   climate   of   free  
good  so  demands.   artistic  and  intellectual  expression.  
   
ARTICLE  XII,  SECTION  14.   SECTION  15.    
The   sustained   development   of   a   reservoir   of   national   Arts   and   letters   shall   enjoy   the   patronage   of   the   State.  
talents   consisting   of   Filipino   scientists,   entrepreneurs,   The   State   shall   conserve,   promote,   and   popularize   the  
professionals,   managers,   high-­‐level   technical   manpower   nation's  historical  and  cultural  heritage  and  resources,  as  
and   skilled   workers   and   craftsmen   in   all   fields   shall   be   well  as  artistic  creations.  
promoted   by   the   State.   The   State   shall   encourage    
appropriate  technology  and  regulate  its  transfer  for  the   SECTION  16.    
national   benefit.   The   practice   of   all   professions   in   the   All   the   country's   artistic   and   historic   wealth   constitutes  
Philippines   shall   be   limited   to   Filipino   citizens,   save   in   the  cultural  treasure  of  the  nation  and  shall  be  under  the  
cases  prescribed  by  law.   protection   of   the   State   which   may   regulate   its  
  disposition.  
ARTICLE  XIV,  SECTIONS  10  TO  18.    
  SECTION  17.    
SCIENCE  AND  TECHNOLOGY   The  State  shall  recognize,  respect,  and  protect  the  rights  
of   indigenous   cultural   communities   to   preserve   and  
SECTION  10.    
develop  their  cultures,  traditions,  and  institutions.  It  shall  
Science   and   technology   are   essential   for   national   consider  these  rights  in  the  formulation  of  national  plans  
development   and   progress.   The   State   shall   give   priority   and  policies.  
to  research  and  development,  invention,  innovation,  and    
their   utilization;   and   to   science   and   technology  
SECTION  18.  
education,   training,   and   services.   It   shall   support  
indigenous,   appropriate,   and   self-­‐reliant   scientific   and    
technological   capabilities,   and   their   application   to   the   The   State   shall   ensure   equal   access   to   cultural  
country's  productive  systems  and  national  life.   opportunities  through  the  educational  system,  public  or  
  private   cultural   entities,   scholarships,   grants   and   other  
incentives,   and   community   cultural   centers,   and   other  
SECTION  11.    
public  venues.  
The   Congress   may   provide   for   incentives,   including   tax    
deductions,   to   encourage   private   participation   in   The   State   shall   encourage   and   support   researches   and  
programs   of   basic   and   applied   scientific   research.   studies  on  the  arts  and  culture.  
Scholarships,   grants-­‐in-­‐aid,   or   other   forms   of   incentives    
shall   be   provided   to   deserving   science   students,   08  December  2011  
researchers,   scientists,   inventors,   technologists,   and    
specially  gifted  citizens.   Note   that   under   the   Constitution   (Section   11,   Article   XIV),  
  there   are   “other   forms   of   incentives”   granted   to  
SECTION  12.     encourage   innovation   and   creation.   Why   is   it   still  
The   State   shall   regulate   the   transfer   and   promote   the   necessary   to   have   IP   laws   despite   the   presence   of   this  
adaptation   of   technology   from   all   sources   for   the   clause  in  the  Constitution?  
national   benefit.   It   shall   encourage   the   widest   -­‐ We  are  obligated  by  the  TRIPS,  as  a  signatory  to  
participation   of   private   groups,   local   governments,   and   it,  to  pass  laws  that  will  protect  IP.  
community-­‐based   organizations   in   the   generation   and    
utilization  of  science  and  technology.   What   makes   the   IP   system   more   superior   (than   the   mere  
  granting  of  awards)?  What  makes  it  more  sustainable?  
SECTION  13.     -­‐ Term  and  duration  of  the  benefit.  They  may  be  
The   State   shall   protect   and   secure   the   exclusive   rights   of   enjoyed  for  a  longer  period  of  time.  
scientists,   inventors,   artists,   and   other   gifted   citizens   to   -­‐ IP  System  is  more  sustainable  because  the  rules  

2z
Intellectual Property Law

in   placed   and   the   standards   of   protection   are   III. International  Conventions  


objective   and   open.   The   rights   granted   under    
the   IP   system   attached   even   when   the   work   It  is  important  to  know  about  the  conventions  
produced   is   not   good   in   the   subjective   sense   at   because   the   States   are   mandated   to   amend  
the   time   it   was   granted   (note   that   standards   their  laws  to  comply  with  it.    
change).  
   
It   is   possible   for   2   forms   of   IP   to   protect   a   A. Berne   Convention   for   the   Protection  
single  work.   of   Literary   and   Artistic   Works   as  
-­‐ e.g.   Winnie   the   Pooh   and   Friends   –   revised  in  Brussels  (Brussels  Act)  
protected   by   trademark   and  
It  was  concluded  in  1886.  
copyright.    
 
 
B. Paris   Convention   for   the   Protection  
of   Industrial   Property   Rights   (Paris  
Convention)  
The   Paris   Convention   was   concluded   in   1883.   It   covers  
industrial   property   in   its   broadest   sense   including  
patents,   trademarks,   industrial   design,   utility   models  
(sometimes   called   “little   patents”),   tradenames,  
geographical   indications,   and   methods   of   unfair  
competition.  (Catindig)  
 
C. International   Convention   for   the  
Protection   of   Performers,   Producers  
of   Phonograms   and   Broadcasting  
Organizations  (Rome  Convention)  
It  was  concluded  in  1961.  
 
D. Convention   Establishing   the   World  
Intellectual  Property  Organization  
 
E. Budapest   Treaty  on  the  International  
Recognition   of   the   Deposit   of  
Microorganisms   for   the   Purposes   of  
Patent  Procedure  
Concluded   in   1977,   this   Treaty   requires   a   Contracting  
State,   which   allows   or   requires   the   deposit   of  
microorganisms   for   purposes   of   patent   procedure,   to  
recognize,   for   such   purposes,   the   deposit   of   a  
microorganism   with   any   “international   depositary  
authority.”    
 
F. Agreement  on   Trade  Related  Aspects  
of   Intellectual   Property,   Including  
Trade   in   Counterfeit   Goods   of   the  
General   Agreement   in   Tariffs   and  
Trade  
 
TANADA  V.  ANGARA  (1997)  
Relevant  issue:  4th  issue  
Petitioners   aver   that   paragraph   1,   Article   34   of   the  
General  Provisions  and  Basic  Principles  of  the  Agreement  

o3
Katrina Michelle Mancao
 
on  Trade-­‐Related  Aspects  of  Intellectual  Property  Rights   patent   could   not   determine   the   exact   process   used   in  
(TRIPS)  intrudes  on  the  power  of  the  Supreme  Court  to   obtaining  such  identical  product.    Hence,  the  “burden  of  
promulgate   rules   concerning   pleading,   practice   and   proof”   contemplated   by   Article   34   should   actually   be  
procedures.   understood  as  the  duty  of  the  alleged  patent  infringer  to  
  overthrow   such   presumption.       Such   burden,   properly  
To   understand   the   scope   and   meaning   of   Article   34,   understood,  actually  refers  to  the  “burden  of  evidence”    
TRIPS,  it  will  be  fruitful  to  restate  its  full  text  as  follows:   (burden  of  going  forward)  placed  on  the  producer  of  the  
  identical  (or  fake)  product  to  show  that  his  product  was  
“Article  34.  Process  Patents:  Burden  of  Proof   produced  without  the  use  of  the  patented  process.  
   
1.   For   the   purposes   of   civil   proceedings   in   The   foregoing   notwithstanding,   the   patent   owner   still  
respect   of   the   infringement   of   the   rights   of   the   has   the   “burden   of   proof”   since,   regardless   of   the  
owner   referred   to   in   paragraph   1(b)   of   Article   presumption   provided   under   paragraph   1   of   Article   34,  
28,  if  the  subject  matter  of  a  patent  is  a  process   such   owner   still   has   to   introduce   evidence   of   the  
for  obtaining  a  product,  the  judicial  authorities   existence  of  the  alleged  identical  product,  the  fact  that  it  
shall   have   the   authority   to   order   the   defendant   is   “identical”   to   the   genuine   one   produced   by   the  
to   prove   that   the   process   to   obtain   an   identical   patented   process   and   the   fact   of   “newness”   of   the  
product  is  different  from  the  patented  process.     genuine   product   or   the   fact   of   “substantial   likelihood”  
Therefore,   Members   shall   provide,   in   at   least   that   the   identical   product   was   made   by   the   patented  
one   of   the   following   circumstances,   that   any   process.  
identical   product   when   produced   without   the    
consent   of   the   patent   owner   shall,   in   the   The   foregoing   should   really   present   no   problem   in  
absence   of   proof   to   the   contrary,   be   deemed   changing   the   rules   of   evidence   as   the   present   law   on   the  
to   have   been   obtained   by   the   patented   subject,   Republic   Act   No.   165,   as   amended,   otherwise  
process:   known   as   the   Patent   Law,   provides   a   similar  
  presumption  in  cases  of  infringement  of  patented  design  
(a)   if   the   product   obtained   by   the   or  utility  model,  thus:  
patented  process  is  new;    
  “SEC.   60.   Infringement.   -­‐   Infringement   of   a  
(b)   if   there   is   a   substantial   likelihood   design   patent   or   of   a   patent   for   utility   model  
that   the   identical   product   was   made   shall   consist   in   unauthorized   copying   of   the  
by   the   process   and   the   owner   of   the   patented   design   or   utility   model   for   the  
patent   has   been   unable   through   purpose   of   trade   or   industry   in   the   article   or  
reasonable   efforts   to   determine   the   product   and   in   the   making,   using   or   selling   of  
process  actually  used.   the   article   or   product   copying   the   patented  
  design   or   utility   model.     Identity   or   substantial  
2.  Any  Member  shall  be  free  to  provide  that  the   identity   with   the   patented   design   or   utility  
burden   of   proof   indicated   in   paragraph   1   shall   model   shall   constitute   evidence   of   copying.”  
be   on   the   alleged   infringer   only   if   the   condition   (underscoring  supplied)  
referred   to   in   subparagraph   (a)   is   fulfilled   or    
only   if   the   condition   referred   to   in   Moreover,   it   should   be   noted   that   the   requirement   of  
subparagraph  (b)  is  fulfilled.   Article   34   to   provide   a   disputable   presumption   applies  
  only  if  (1)  the  product  obtained  by  the  patented  process  
3.  In  the  adduction  of  proof  to  the  contrary,  the   is   NEW   or   (2)   there   is   a   substantial   likelihood   that   the  
legitimate  interests  of  defendants  in  protecting   identical   product   was   made   by   the   process   and   the  
their   manufacturing   and   business   secrets   shall   process   owner   has   not   been   able   through   reasonable  
be  taken  into  account.”   effort   to   determine   the   process   used.     Where   either   of  
  these   two   provisos   does   not   obtain,   members   shall   be  
From   the   above,   a   WTO   Member   is   required   to   provide   a   free   to   determine   the   appropriate   method   of  
rule   of   disputable   (note   the   words   “in   the   absence   of   implementing   the   provisions   of   TRIPS   within   their   own  
proof   to   the   contrary”)   presumption   that   a   product   internal  systems  and  processes.  
shown  to  be  identical  to  one  produced  with  the  use  of  a    
patented   process   shall   be   deemed   to   have   been   By   and   large,   the   arguments   adduced   in   connection   with  
obtained  by  the  (illegal)  use  of  the  said  patented  process,   our   disposition   of   the   third   issue   -­‐-­‐   derogation   of  
(1)   where   such   product   obtained   by   the   patented   legislative   power   -­‐   will   apply   to   this   fourth   issue   also.    
product   is   new,   or   (2)   where   there   is   “substantial   Suffice   it   to   say   that   the   reciprocity   clause   more   than  
likelihood”   that   the   identical   product   was   made   with   the   justifies   such   intrusion,   if   any   actually   exists.     Besides,  
use   of   the   said   patented   process   but   the   owner   of   the   Article   34   does   not   contain   an   unreasonable   burden,  

4z
Intellectual Property Law

consistent   as   it   is   with   due   process   and   the   concept   of   ARTICLE  723.    


adversarial   dispute   settlement   inherent   in   our   judicial   Letters  and  other  private  communications  in  writing  are  
system.   owned   by   the   person   to   whom   they   are   addressed   and  
  delivered,  but  they  cannot  be  published  or  disseminated  
So   too,   since   the   Philippine   is   a   signatory   to   most   without  the  consent  of  the  writer  or  his  heirs.  However,  
international   conventions   on   patents,   trademarks   and   the   court   may   authorize   their   publication   or  
copyrights,   the   adjustment   in   legislation   and   rules   of   dissemination   if   the   public   good   or   the   interest   of   justice  
procedure  will  not  be  substantial.   so  requires.  
   
G. Patent  Cooperation  Treaty   ARTICLE  724.    
  Special  laws  govern  copyright  and  patent.  
Question:   Was   it   the   right   time   to   enter   it   in    
2001?     ARTICLE  520.    
  A  trade-­‐mark  or  trade-­‐name  duly  registered  in  the  proper  
IV. Civil  Code   government  bureau  or  office  is  owned  by  and  pertains  to  
the   person,   corporation,   or   firm   registering   the   same,  
ARTICLE  712.  ★  
subject  to  the  provisions  of  special  laws.  
Ownership  is  acquired  by  occupation  and  by  intellectual  
 
creation.  
ARTICLE  521.    
 
Ownership   and   other   real   rights   over   property   are   The   goodwill   of   a   business   is   property,   and   may   be  
acquired  and  transmitted  by  law,  by  donation,  by  testate   transferred   together   with   the   right   to   use   the   name  
and  intestate  succession,  and  in  consequence  of  certain   under  which  the  business  is  conducted.  
contracts,  by  tradition.    
  ARTICLE  522.    
They  may  also  be  acquired  by  means  of  prescription.   Trade-­‐marks   and   trade-­‐names   are   governed   by   special  
  laws.  
ARTICLE  721.      
By   intellectual   creation,   the   following   persons   acquire    
ownership:   • Differences   between   copyright,  
  trademarks  and  patent  
(1)   The   author   with   regard   to   his   literary,   dramatic,  
KHO  V.  COURT  OF  APPEALS  (2002)  
historical,  legal,  philosophical,  scientific  or  other  work;  
  Refresher:  
(2)  The  composer;  as  to  his  musical  composition;   Elidad   C.   Kho,   doing   business   under   the   name   of   KEC  
  Cosmetics  Laboratory,  alleged  that  she  is  the  registered  
(3)  The  painter,  sculptor,  or  other  artist,  with  respect  to   owner   of   the   copyrights   Chin   Chun   Su   and   Oval   Facial  
the  product  of  his  art;   Cream  Container/Case,  and  that  she  has  the  patent  rights  
  on   Chin   Chun   Su   &   Device   and   Chin   Chun   Su   for  
(4)  The  scientist  or  technologist  or  any  other  person  with   medicated   cream.   Petitioner   likewise   alleged   that  
regard  to  his  discovery  or  invention.   respondent  Summerville  General  Merchandising  and  Co.  
  advertised   and   sold   petitioner's   cream   and   products  
under   the   brand   name   Chin   Chun   Su   in   similar   containers  
ARTICLE  722.    
that   petitioner   uses,   thereby   misleading   the   public   and  
The   author   and   the   composer,   mentioned   in   Nos.   1   and   2   resulting   in   the   decline   in   the   petitioner's   sales   and  
of   the   preceding   article,   shall   have   the   ownership   of   income.  
their  creations  even  before  the  publication  of  the  same.    
Once   their   works   are   published,   their   rights   are   The   respondents,   on   the   other   hand,   alleged   as   their  
governed  by  the  Copyright  laws.   defense  that  Summerville  is  the  exclusive  and  authorized  
  importer,   re-­‐packer   and   distributor   of   Chin   Chun   Su  
The  painter,  sculptor  or  other  artist  shall  have  dominion   products   manufactured   by   Shun   Yi   Factory   of   Taiwan;  
over  the  product  of  his  art  even  before  it  is  copyrighted.   that   the   said   Taiwanese   manufacturing   company  
  authorized   Summerville   to   register   its   trade   name   Chin  
The   scientist   or   technologist   has   the   ownership   of   his   Chun   Su   Medicated   Cream   with   the   Philippine   Patent  
discovery  or  invention  even  before  it  is  patented.   Office   and   other   appropriate   governmental   agencies;  
  that   KEC   Cosmetics   Laboratory   of   the   petitioner  
  obtained   the   copyrights   through   misrepresentation   and  
  falsification;   and,   that   the   authority   of   Quintin   Cheng,  

o5
Katrina Michelle Mancao
 
assignee   of   the   patent   registration   certificate,   to   V. Copyright  and  Related  Rights  
distribute   and   market   Chin   Chun   Su   products   in   the    
Philippines   had   already   been   terminated   by   the   said  
A. Legislative   history   of   the   law   on  
Taiwanese  Manufacturing  Company.  
  Copyright  and  Related  rights  
Issue:   1. Spanish  Law  on  Intellectual  
Whether   the   copyright   and   patent   over   the   name   and   Property  
container   of   a   beauty   cream   product   would   entitle   the  
registrant   to   the   use   and   ownership   over   the   same   to  
2. US  Copyright  Law  
the  exclusion  of  others.   3. Act  3134  (Copyright  Law)  
  4. Presidential   Decree   No.   49  
Doctrine:  
(took   effect   on   27  
Trademark,   copyright   and   patents   are   different  
intellectual  property  rights  that  cannot  be  interchanged   December  1972)  
with  one  another.     5. Presidential   Decree   No.  
Ø A   trademark   is   any   visible   sign   capable   of   1988  (amended  by  PD  49  b y  
distinguishing   the   goods   (trademark)   or  
services   (service   mark)   of   an   enterprise   and   inserting  Section  56)  
shall  include  a  stamped  or  marked  container  of   6. RA   8293   –   Intellectual  
goods.     Property  Code  (took  effect  
Ø In   relation   thereto,   a   trade   name   means   the  
on  01  January  1998)  
name   or   designation   identifying   or  
distinguishing  an  enterprise.      
Ø Meanwhile,   the   scope   of   a   copyright   is   The  Philippines  its  one  of  the  first  developing  
confined   to   literary   and   artistic   works   which   countries  to  amend  its  laws.  
are  original  intellectual  creations  in  the  literary    
and   artistic   domain   protected   from   the   B. Law  on  Copyright  
moment  of  their  creation.    
Ø Patentable  inventions,  on  the  other  hand,  refer   SECTION  241.  EFFECTIVITY.  –    
to   any   technical   solution   of   a   problem   in   any   This  Act  shall  take  effect  on  1  January  1998.  
field   of   human   activity   which   is   new,   involves    
an  inventive  step  and  is  industrially  applicable.   SECTION  239.  REPEALS.  
  239.3.   The   provisions   of   this   Act   shall   apply   to   works   in  
Petitioner   has   no   right   to   support   her   claim   for   the   which   copyright   protection   obtained   prior   to   the  
exclusive  use  of  the  subject  trade  name  and  its  container.   effectivity   of   this   Act   is   subsisting:   Provided,   That   the  
The   name   and   container   of   a   beauty   cream   product   are   application   of   this   Act   shall   not   result   in   the   diminution  
proper   subjects   of   a   trademark   inasmuch   as   the   same   of  such  protection.    
falls  squarely  within  its  definition.  In  order  to  be  entitled    
to   exclusively   use   the   same   in   the   sale   of   the   beauty   SECTION  236.  PRESERVATION  OF  EXISTING  RIGHTS.  –    
cream  product,  the  user  must  sufficiently  prove  that  she  
Nothing   herein   shall   adversely   affect   the   rights   on   the  
registered   or   used   it   before   anybody   else   did.   The  
enforcement   of   rights   in   patents,   utility   models,  
petitioner’s   copyright   and   patent   registration   of   the  
industrial   designs,   marks   and   works,   acquired   in   good  
name  and  container  would  not  guarantee  her  the  right  to  
faith  prior  to  the  effective  date  of  this  Act.  
the  exclusive  use  of  the  same  for  the  reason  that  they  are  
 
not   appropriate   subjects   of   the   said   intellectual   rights.  
Consequently,   a   preliminary   injunction   order   cannot   be   1.
Definition  of  Copyright  
issued  for  the  reason  that  the  petitioner  has  not  proven   SECTION  177.  COPYRIGHT  OR  ECONOMIC  RIGHTS.  –    
that   she   has   a   clear   right   over   the   said   name   and   Subject   to   the   provisions   of   Chapter   VIII,   copyright   or  
container   to   the   exclusion   of   others,   not   having   proven   economic   rights   shall   consist   of   the   exclusive   right   to  
that  she  has  registered  a  trademark  thereto  or  used  the   carry  out,  authorize  or  prevent  the  following  acts:  
same  before  anyone  did.    
  177.1.  Reproduction  of  the  work  or  substantial  portion  of  
the  work;  
 
177.2.  Dramatization,  translation,  adaptation,  abridgment,  
arrangement  or  other  transformation  of  the  work;  
 

6z
Intellectual Property Law

177.3.  The  first  public  distribution  of  the  original  and  each   RULE  12,  COPYRIGHT  SAFEGUARDS  AND  REGULATIONS  
copy   of   the   work   by   sale   or   other   forms   of   transfer   of   DEFINITION  OF  TERMS  
ownership;  
SECTION  1.  FIRST  PUBLIC  DISTRIBUTION  OF  WORK.  
 
An  exclusive  right  of  first  distribution  of  work  includes  all  
177.4.  Rental  of  the  original  or  a  copy  of  an  audiovisual  or  
acts  involving  distribution,  specifically  including  the  first  
cinematographic   work,   a   work   embodied   in   a   sound  
importation   of   an   original   and   each   copy   of   the   work  
recording,   a   computer   program,   a   compilation   of   data  
into  the  jurisdiction  of  the  Republic  of  the  Philippines.  
and   other   materials   or   a   musical   work   in   graphic   form,  
 
irrespective  of  the  ownership  of  the  original  or  the  copy  
Standard   for   Copyright   2.
which  is  the  subject  of  the  rental;    
  Protection  
177.5.  Public  display  of  the  original  or  a  copy  of  the  work;   SECTION  172.  LITERARY  AND  ARTISTIC  WORKS.  –    
  172.1.   Literary   and   artistic   works,   hereinafter   referred   to  
177.6.  Public  performance  of  the  work;  and   as   "works",   are   original   intellectual   creations   in   the  
  literary  and  artistic  domain  protected  from  the  moment  
177.7.  Other  communication  to  the  public  of  the  work.   of  their  creation  and  shall  include  in  particular:  
   
RULE  2,  COPYRIGHT  SAFEGUARDS  AND  REGULATIONS   (a)   Books,   pamphlets,   articles   and   other  
DEFINITION  OF  TERMS   writings;  
 
For   the   purpose   of   these   Copyright   Safeguards   and  
(b)  Periodicals  and  newspapers;  
Regulations,  the  following  terms  are  herein  defined:  
 
x  x  x  
(c)  Lectures,  sermons,  addresses,  dissertations  
Copyright   is   a   right   granted   by   statute   to   the  
prepared   for   oral   delivery,   whether   or   not  
author   or   originator   of   literary,   scholarly,  
reduced  in  writing  or  other  material  form;  
scientific,   or   artistic   productions,   including  
 
computer  programs.  A  copyright  gives  him  the  
(d)  Letters;  
legal   right   to   determine   how   the   work   is   used  
 
and  to  obtain  economic  benefits  from  the  work.  
(e)   Dramatic   or   dramatico-­‐musical  
For   example,   the   owner   of   a   copyright   for   a  
compositions;   choreographic   works   or  
book   or   a   piece   of   software   has   the   exclusive  
entertainment  in  dumb  shows;  
rights  to  use,  copy,  distribute,  and  sell  copies  of  
 
the   work,   including   later   editions   or   versions   of  
(f)   Musical   compositions,   with   or   without  
the   work.   If   another   person   improperly   uses  
words;  
material   covered   by   a   copyright,   the   copyright  
 
owner  can  obtain  legal  relief  
(g)   Works   of   drawing,   painting,   architecture,  
x  x  x  
sculpture,   engraving,   lithography   or   other  
 
works   of   art;   models   or   designs   for   works   of  
RULE  11,  COPYRIGHT  SAFEGUARDS  AND  REGULATIONS   art1;  
DEFINITION  OF  TERMS    
SECTION   1.   COMMUNICATION   TO   THE   PUBLIC   OF   COPYRIGHTED   (h)   Original   ornamental   designs   or   models   for  
WORKS.   articles   of   manufacture,   whether   or   not  
registrable   as   an   industrial   design,   and   other  
“Communication  to  the  public”  or  “communicate  to  the  
works  of  applied  art;  
public,”   also   includes   point-­‐to-­‐point   transmission   of   a  
 
work,   including   video   on   demand,   and   providing   access  
(i)   Illustrations,   maps,   plans,   sketches,   charts  
to   an   electronic   retrieval   system,   such   as   computer  
and   three-­‐dimensional   works   relative   to  
databases,  servers,  or  similar  electronic  storage  devices.  
geography,   topography,   architecture   or  
Broadcasting,   rebroadcasting,   retransmission   by   cable,  
science;  
and   broadcast   and   retransmission   by   satellite   are   all   acts  
 
of  “communication  to  the  public”  within  the  meaning  of  
(j)   Drawings   or   plastic   works   of   a   scientific   or  
the  IPC.  
technical  character;  
 
 

                                                                                                                                                       
1
 “Works  of  applied  art”  à  artistic  creation  with  utilitarian  value  

o7
Katrina Michelle Mancao
 
(k)   Photographic   works   including   works   Doctrine:  
produced   by   a   process   analogous   to   A   person   to   be   entitled   to   a   copyright   must   be   the  
photography;  lantern  slides;   original   creator   of   the   work.   He   must   have   created   it   by  
  his   own   skill,   labor   and   judgment   without   directly  
(l)   Audiovisual   works   and   cinematographic   copying   or   evasively   imitating   the   work   of   another.   The  
works   and   works   produced   by   a   process   grant   of   preliminary   injunction   in   a   case   rests   on   the  
analogous   to   cinematography   or   any   process   sound   discretion   of   the   court   with   the   caveat   that   it  
for  making  audio-­‐visual  recordings;   should  be  made  with  extreme  caution.  Its  grant  depends  
  chiefly   on   the   extent   of   doubt   on   the   validity   of   the  
(m)  Pictorial  illustrations  and  advertisements;   copyright,   existence   of   infringement,   and   the   damages  
  sustained   by   such   infringement.   In   our   view,   the   copies  
(n)  Computer  programs2;  and   of  the  certificates  of  copyright  registered  in  the  name  of  
  Ceroilfood   Shandong   sufficiently   raise   reasonable   doubt.  
(o)   Other   literary,   scholarly,   scientific   and   With  such  a  doubt  preliminary  injunction  is  unavailing.  
artistic  works.    
  SAMBAR  V.  LEVI  STRAUSS  (2002)  
172.2.   Works   are   protected   by   the   sole   fact   of   their   Refresher:  
creation,   irrespective   of   their   mode   or   form   of   On   28   September   1987,   Levi   Strauss   &   Co.   and   Levi  
expression,   as   well   as   of   their   content,   quality   and   Strauss   (Phil.),   Inc.   demanded   that   CVS   Garment  
purpose.     Enterprises   (CVSGE)   desist   from   using   their   stitched  
  arcuate   design   on   the   Europress   jeans   which   CVSGE  
CHING  KIAN  CHUAN  V.  CA  (2001)   advertised  in  Manila  Bulletin.  
Refresher:    
Petitioner  Wilson  Ong  Ching  Kian  Chuan  (“Ong”)  imports   Atty.   Benjamin   Gruba,   counsel   of   CVSGE,   replied   that   the  
vermicelli   from   China   National   Cereals   Oils   and   arcuate   design   on   the   back   pockets   of   Europress   jeans  
Foodstuffs   Import   and   Export   Corporation,   based   in   was   different   from   the   design   on   the   back   pockets   of  
Beijing,   China,   under   the   firm   name   C.K.C.   Trading.   He   Levi’s   jeans.     He   further   asserted   that   his   client   had   a  
repacks  it  in  cellophane  wrappers  with  a  design  of  two-­‐ copyright  on  the  design  it  was  using.  
dragons   and   the   TOWER   trademark   on   the   uppermost    
portion.   Ong   acquired   a   Certificate   of   Copyright   Thereafter,  private  respondents  filed  a  complaint  against  
Registration  from  the  National  Library  on  June  9,  1993  on   Sambar,   doing   business   under   the   name   and   style   of  
the  said  design.   CVSGE.     Private   respondents   also   impleaded   the   Director  
  of  the  National  Library.  
Ong   discovered   that   private   respondent   Lorenzo   Tan    
repacked   his   vermicelli   he   imports   from   the   same   Levi   Strauss   and   Co.   (LS&Co.),   an   internationally   known  
company   but   based   in   Qingdao,   China   in   a   "nearly"   clothing   manufacturer,   owns   the   arcuate   design  
identical   wrapper.   On   September   16,   1993,   Ong   filed   trademark   which   was   registered   under   U.S.   Trademark  
against   Tan   a   verified   complaint   for   infringement   of   Registration  No.  404,  248  on  November  16,  1943,  and  in  
copyright   with   damages   and   prayer   for   temporary   the   Principal   Register   of   trademarks   with   the   Philippine  
restraining   order   or   writ   of   preliminary   injunction   with   Patent  Office  under  Certificate  of  Registration  No.  20240  
the   Regional   Trial   Court   in   Quezon   City.   Ong   alleged   that   issued   on   October   8,   1973;   that   through   a   Trademark  
he   was   the   holder   of   a   Certificate   of   Copyright   Technical   Data   and   Technical   Assistance   Agreement   with  
Registration  over  the  cellophane  wrapper  with  the  two-­‐ Levi   Strauss   (Phil.)   Inc.   (LSPI)   in   1972,   LS&Co.   granted  
dragon  design,  and  that  Tan  used  an  identical  wrapper  in   LSPI   a   non-­‐exclusive   license   to   use   the   arcuate  
his  business.  In  his  prayer  for  a  preliminary  injunction  in   trademark   in   its   manufacture   and   sale   of   Levi’s   pants,  
addition   to   damages,   he   asked   that   Tan   be   restrained   jackets  and  shirts  in  the  Philippines;  that  in  1983,  LS&Co.  
from  using  the  wrapper.  He  said  he  would  post  a  bond  to   also   appointed   LSPI   as   its   agent   and   attorney-­‐in-­‐fact   to  
guarantee   the   payment   of   damages   resulting   from   the   protect   its   trademark   in   the   Philippines;   and   that  
issuance  of  the  writ  of  preliminary  injunction.   sometime  in  1987,  CVSGIC  and  Venancio  Sambar,  without  
  the  consent  and  authority  of  private  respondents  and  in  
  infringement   and   unfair   competition,   sold   and  
advertised,   and   despite   demands   to   cease   and   desist,  
                                                                                                                                                       
2
 Class   notes:   This   is   the   outcome   of   the   lobbying   in   the   US.   At  
continued   to   manufacture,   sell   and   advertise   denim  
that   time,   technology   was   not   as   advanced.   They   were   pants   under   the   brand   name   “Europress”   with   back  
concerned   more   on   DURATION   (copyright   protection   is   for   50   pockets   bearing   a   design   similar   to   the   arcuate  
years!).   They   found   out   later   that   duration   is   not   important   in   trademark   of   private   respondents,   thereby   causing  
computer   programs   because   they   are   immediately   replaced   or   confusion   on   the   buying   public,   prejudicial   to   private  
upgraded.   respondents’  goodwill  and  property  right.  

8z
Intellectual Property Law

  He  has  a  right  to  determine  whether  it  shall  be  published  


Doctrine:   at   all,   and   if   published,   when,   where,   by   whom,   and   in  
To  be  entitled  to  copyright,  the  thing  being  copyrighted   what   form.   This   exclusive   right   is   confined   to   the   first  
must  be  original,  created  by  the  author  through  his  own   publication.  When  once  published,  it  is  dedicated  to  the  
skill,   labor   and   judgment,   without   directly   copying   or   public,  and  the  author  loses  the  exclusive  right  to  control  
evasively  imitating  the  work  of  another.   subsequent   publication   by   others,   unless   the   work   is  
  placed  under  the  protection  of  the  copyright  law.  
3. When  does  Copyright  vest?    
  Remarks:  Decided  under  the  old  law.  
SANTOS  V.  MCCULLOUGH  PRINTING  CO.  (1964)    
Refresher:   FILIPINO  SOCIETY  OF  COMPOSERS  V.  BENJAMIN  TAN  (1987)  
The   complaint   alleges   that   plaintiff   Mauro   Malang   Refresher:    
Santos  designed  for  former  Ambassador  Felino  Neri,  for   Plaintiff-­‐appellant   is   a   non-­‐profit   association   of   authors,  
his   personal   Christmas   Card   greetings   for   the   year   1959   composers   and   publishers   duly   organized   under   the  
(Rural   Christmas   time   scene),   the   artistic   motif   in   Corporation   Law   of   the   Philippines   and   registered   with  
question.   The   following   year   the   defendant   McCullough   the   Securities   and   Exchange   Commission.   Said  
Printing  Company,  without  the  knowledge  and  authority   association  is  the  owner  of  certain  musical  compositions  
of   plaintiff,   displayed   the   very   design   in   its   album   of   among   which   are   the   songs   entitled:   "Dahil   Sa   Iyo",  
Christmas  cards  and  offered  it  for  sale,  for  a  price.   "Sapagkat   Ikaw   Ay   Akin,"   "Sapagkat   Kami   Ay   Tao  
  Lamang"  and  "The  Nearness  Of  You."  
Doctrine:    
Paragraph  33  of  Patent  Office  Administrative  Order  No.  3   On  the  other  hand,  defendant-­‐appellee  is  the  operator  of  
(as   amended   dated   September   18,   1947)   entitled   "Rules   a   restaurant   known   as   "Alex   Soda   Foundation   and  
of  Practice  in  the  Philippines  Patent  Office  relating  to  the   Restaurant"   where   a   combo   with   professional   singers,  
Registration  of  Copyright  Claims"  promulgated  pursuant   hired  to  play  and  sing  musical  compositions  to  entertain  
to   Republic   Act   165,   provides,   among   others,   that   an   and  amuse  customers  therein,  were  playing  and  singing  
intellectual  creation  should  be  copyrighted  30  days  after   the   above-­‐mentioned   compositions   without   any   license  
its   publication,   if   made   in   Manila,   or   within   60   day's   if   or   permission   from   the   appellant   to   play   or   sing   the  
made   elsewhere,   failure   of   which   renders   such   creation   same.   Accordingly,   appellant   demanded   from   the  
public   property.   In   the   case   at   bar,   even   as   of   this   appellee   payment   of   the   necessary   license   fee   for   the  
moment,  there  is  no  copyright  for  the  design  in  question.   playing   and   singing   of   aforesaid   compositions   but   the  
We   are   not   also   prepared   to   accept   the   contention   of   demand  was  ignored.  
appellant  that  the  publication  of  the  design  was  a  limited    
one,   or   that   there   was   an   understanding   that   only   Hence,  on  November  7,  1967,  appellant  filed  a  complaint  
Ambassador  Neri  should,  have  absolute  right  to  use  the   with   the   lower   court   for   infringement   of   copyright  
same.  In  the  first  place,  if  such  were  the  condition  then   against   defendant-­‐appellee   for   allowing   the   playing   in  
Ambassador  Neri  would  be  the  aggrieved  party,  and  not   defendant-­‐appellee's   restaurant   of   said   songs  
the   appellant.   In   the   second   place,   if   there   was   such   a   copyrighted  in  the  name  of  the  former.  
limited   publication   or   prohibition,   the   same   was   not    
shown  on  the  face  of  the  design.  When  the  purpose  is  a   Issue:  Whether  or  not  the  playing  and  signing  of  musical  
limited  publication,  but  the  effect  is  general  publication,   compositions   which   have   been   copyrighted   under   the  
irrevocable   rights   thereupon   become   vested   in   the   provisions   of   the   Copyright   Law   (Act   3134)   inside   the  
public,   in   consequence   of   which   enforcement   of   the   establishment   of   the   defendant-­‐appellee   constitute   a  
restriction  becomes  impossible.   public   performance   for   profit   within   the   meaning   and  
  contemplation   of   the   Copyright   Law   of   the   Philippines;  
It   has   been   held   that   the   effect   of   offering   for   sale   a   and   assuming   that   there   were   indeed   public  
dress,  for  example  manufactured  in  accordance  with  an   performances  for  profit,  whether  or  not  appellee  can  be  
original   design   which   is   not   protected   by   either   a   held  liable  therefor.    
copyright   or   a   patent,   is   to   divest   the   owner   of   his    
common   law   rights   therein   by   virtue   of   the   publication   Doctrine:  
of   a   'copy'   and   thereafter   anyone   is   free   to   copy   the   There  were  "public  performances  for  profit.  "  
design   or   the   dress.   When   Ambassador   Neri   distributed    
800   copies   of   the   design   in   controversy,   the   plaintiff   lost   The  word  "perform"  as  used  in  the  Act  has  been  applied  
control   of   his   design   and   the   necessary   implication   was   to   "one   who   plays   a   musical   composition   on   a   piano,  
that   there   had   been   a   general   publication,   there   having   thereby   producing   in   the   air   sound   waves   which   are  
been   no   showing   of   a   clear   indication   that   a   limited   heard   as   music   ...   and   if   the   instrument   he   plays   on   is   a  
publication   was   intended.   The   author   of   a   literary   piano   plus   a   broadcasting   apparatus,   so   that   waves   are  
composition   has   a   light   to   the   first   publication   thereof.  

o9
Katrina Michelle Mancao
 
thrown   out,   not   only   upon   the   air,   but   upon   the   other,   be   noted   that   for   the   playing   and   singing   the   musical  
then  also  he  is  performing  the  musical  composition."     compositions   involved,   the   combo   was   paid   as  
  independent  contractors  by  the  appellant.  It  is  therefore  
In  relation  thereto,  it  has  been  held  that  "The  playing  of   obvious  that  the  expenses  entailed  thereby  are  added  to  
music   in   dine   and   dance   establishment   which   was   paid   the   overhead   of   the   restaurant   which   are   either  
for   by   the   public   in   purchases   of   food   and   drink   eventually  charged  in  the  price  of  the  food  and  drinks  or  
constituted   "performance   for   profit"   within   a   Copyright   to  the  overall  total  of  additional  income  produced  by  the  
Law."  Thus,  it  has  been  explained  that  while  it  is  possible   bigger   volume   of   business   which   the   entertainment   was  
in  such  establishments  for  the  patrons  to  purchase  their   programmed   to   attract.   Consequently,   it   is   beyond  
food  and  drinks  and  at  the  same  time  dance  to  the  music   question   that   the   playing   and   singing   of   the   combo   in  
of   the   orchestra,   the   music   is   furnished   and   used   by   the   defendant-­‐appellee's   restaurant   constituted  
orchestra   for   the   purpose   of   inducing   the   public   to   performance   for   profit   contemplated   by   the   Copyright  
patronize   the   establishment   and   pay   for   the   Law.  
entertainment   in   the   purchase   of   food   and   drinks.   The    
defendant  conducts  his  place  of  business  for  profit,  and   Nevertheless,   appellee   cannot   be   said   to   have   infringed  
it   is   public;   and   the   music   is   performed   for   profit.   In   a   upon   the   Copyright   Law.   Appellee's   allegation   that   the  
similar  case,  the  Court  ruled  that  "The  Performance  in  a   composers   of   the   contested   musical   compositions  
restaurant  or  hotel  dining  room,  by  persons  employed  by   waived   their   right   in   favor   of   the   general   public   when  
the  proprietor,  of  a  copyrighted  musical  composition,  for   they   allowed   their   intellectual   creations   to   become  
the   entertainment   of   patrons,   without   charge   for   property   of   the   public   domain   before   applying   for   the  
admission   to   hear   it,   infringes   the   exclusive   right   of   the   corresponding  copyrights  for  the  same  is  correct.  
owner  of  the  copyright."    
  The   Supreme   Court   has   ruled   that   "Paragraph   33   of  
In   delivering   the   opinion   of   the   Court   in   said   two   cases,   Patent   Office   Administrative   Order   No.   3   entitled   'Rules  
Justice  Holmes  elaborated  thus:   of  Practice  in  the  Philippines  Patent  Office  relating  to  the  
  Registration  of  Copyright  Claims'  promulgated  pursuant  
If   the   rights   under   the   copyright   are   infringed   to  Republic  Act  165,  provides  among  other  things  that  an  
only   by   a   performance   where   money   is   taken   at   intellectual  creation  should  be  copyrighted  30  days  after  
the   door,   they   are   very   imperfectly   protected.   its   publication,   if   made   in   Manila,   or   within   the   60   days   if  
Performances   not   different   in   kind   from   those   made   elsewhere,   failure   of   which   renders   such   creation  
of   the   defendants   could   be   given   that   might   public  property."  Indeed,  if  the  general  public  has  made  
compete  with  and  even  destroy  the  success  of   use   of   the   object   sought   to   be   copyrighted   for   30   days  
the   monopoly   that   the   law   intends   the   prior   to   the   copyright   application   the   law   deems   the  
plaintiffs   to   have.   It   is   enough   to   say   that   there   object   to   have   been   donated   to   the   public   domain   and  
is  no  need  to  construe  the  statute  so  narrowly.   the  same  can  no  longer  be  copyrighted.  
The   defendants'   performances   are   not    
eleemosynary.  They  are  part  of  a  total  for  which   SECTION   2,   PD   49   (DECREE   ON   THE   PROTECTION   OF  
the   public   pays,   and   the   fact   that   the   price   of  
INTELLECTUAL  PROPERTY)  
the   whole   is   attributed   to   a   particular   item  
which   those   present   are   expected   to   order   is   The   Rights   granted   by   this   Decree   shall,   from   the  
not   important.   It   is   true   that   the   music   is   not   moment   of   creation,   subsist   with   respect   to   any   of   the  
following  classes  of  works:  
the   sole   object,   but   neither   is   the   food,   which  
probably  could  be  got  cheaper  elsewhere.  The    
object   is   a   repast   in   surroundings   that   to   (A)   Books,   including   composite   and  
people  having  limited  power  of  conversation  or   encyclopedic   works,   manuscripts,   directories,  
and  gazetteers;  
disliking   the   rival   noise,   give   a   luxurious  
 
pleasure   not   to   be   had   from   eating   a   silent  
meal.  If  music  did  not  pay,  it  would  be  given  up.   (B)   Periodicals,   including   pamphlets   and  
If   it   pays,   it   pays   out   of   the   public's   pocket.   newspapers;  
Whether   it   pays   or   not,   the   purpose   of    
(C)  Lectures,  sermons,  addresses,  dissertations  
employing  it  is  profit,  and  that  is  enough.    
  prepared  for  oral  delivery;  
In  the  case  at  bar,  it  is  admitted  that  the  patrons  of  the    
restaurant   in   question   pay   only   for   the   food   and   drinks   (D)  Letters;  
 
and   apparently   not   for   listening   to   the   music.   As   found  
(E)   Dramatic   or   dramatico-­‐musical  
by  the  trial  court,  the  music  provided  is  for  the  purpose  of  
compositions;   choreographic   works   and  
entertaining  and  amusing  the  customers  in  order  to  make  
entertainments   in   dumb   shows,   the   acting  
the   establishment   more   attractive   and   desirable.   It   will  
form  of  which  is  fixed  in  writing  or  otherwise;  

10 z
Intellectual Property Law

  protection   in   the   country   of   origin   of   the   work.  


(F)   Musical   compositions,   with   or   without   Consequently,   apart   from   the   provisions   of   this  
words;   Convention,   the   extent   of   protection,   as   well   as   the  
  means   of   redress   afforded   to   the   author   to   protect   his  
(G)   Works   of   drawing,   painting,   architecture,   rights,   shall   be   governed   exclusively   by   the   laws   of   the  
sculpture,   engraving,   lithography,   and   other   country  where  protection  is  claimed.  
works   of   art;   models   or   designs   for   works   of    
art;   SECTION  172  AND  172.2,  SUPRA  
   
(H)  Reproductions  of  a  work  of  art;  
RULE   7,   SECTIONS   2-­‐4,   COPYRIGHT   SAFEGUARDS   AND  
 
(I)   Original   ornamental   designs   or   models   for   REGULATIONS  
articles   of   manufacture,   whether   or   not   SECTION   2.   EFFECTS   OF   REGISTRATION   AND   DEPOSIT   OF   WORK.  
patentable,  and  other  works  of  applied  art;   —    
  The   registration   and   deposit   of   the   work   is   purely   for  
(J)  Maps,  plans,  sketches,  and  charts;   recording   the   date   of   registration   and   deposit   of   the  
  work   and   shall   not   be   conclusive   as   to   copyright  
(K)  Drawings,  or  plastic  works  of  a  scientific  or   ownership  or  the  term  of  copyrights  or  the  rights  of  the  
technical  character;   copyright  owner,  including  neighboring  rights.    
   
(L)   Photographic   works   and   works   produced   SECTION  3.  EFFECT  OF  NON-­‐REGISTRATION  AND  D EPOSIT.  —    
by   a   process   analogous   to   photography;  
If,   within   three   (3)   weeks   after   receipt   by   the   copyright  
lantern  slides;  
owner  of  a  written  demand  from  TNL  and/or  SCL3  for  the  
  deposit  of  a  work  listed  in  Rule  5  Sec.  4  of  this  SAR,  the  
(M)   Cinematographic   works   and   works   required   copies   are   not   delivered   and   the   fee   for  
produced   by   a   process   analogous   to  
registration  and  deposit  is  not  paid,  the  copyright  owner,  
cinematography   or   any   process   for   making  
his   assignee,   or   his   agent   shall   be   liable   to   pay   a   fine  
audio-­‐visual  recordings;   equivalent  to  the  required  fee  per  month  of  delay  and  to  
  pay  to  TNL  and  SCL  the  amount  of  the  retail  price  of  the  
(N)  Computer  programs;   best  edition  of  the  work.  
 
 
(O)   Prints,   pictorial,   illustration,   advertising  
SECTION  4.  OTHER  LAWS.  —    
copies,  labels,  tags,  and  box  wraps;  
Upon   issuance   of   a   certificate   of   deposit,   the   copyright  
 
owner  shall  be  exempt  from  making  additional  deposits  
(P)   Dramatization,   translations,   adaptations,  
of  the  work  with  TNL  or  the  SCL  under  other  laws.  
abridgements,   arrangements   and   other  
 
alterations  of  literary,  musical  or  artistic  works  
or   of   works   of   the   Philippine   Government   as   4. Scope  of  Copyright  
herein   defined,   which   shall   be   protected   as    
provided  in  Section  8  of  this  Decree.   ARTICLE   2,   BERNE   CONVENTION   FOR   THE   PROTECTION   OF  
  LITERARY  AND  ARTISTIC  WORKS  
(Q)   Collection   of   literary,   scholarly,   or   artistic  
Protected  Works:  
works   or   of   works   referred   to   in   Section   9   of  
this   Decree   which   by   reason   of   the   selection   1.  “Literary  and  artistic  works”;    
and   arrangement   of   their   contents   constitute   2.  Possible  requirement  of  fixation;    
intellectual  creations,  the  same  to  be  protected   3.  Derivative  works;  
as   such   in   accordance   with   Section   8   of   this   4.  Official  texts;    
Decree.   5.  Collections;    
  6.   Obligation   to   protect;   beneficiaries   of  
(R)   Other   literary,   scholarly,   scientific   and   protection;  
artistic  works.   7.  Works  of  applied  art  and  industrial  designs;    
  8.  News  
 
ARTICLE   5(2),   BERNE   CONVENTION   FOR   THE   PROTECTION  
(1)   The   expression   “literary   and   artistic   works”   shall  
OF  LITERARY  AND  ARTISTIC  W ORKS   include   every   production   in   the   literary,   scientific   and  
RIGHTS  GUARANTEED   artistic   domain,   whatever   may   be   the   mode   or   form   of  
(2)  The  enjoyment  and  the  exercise  of  these  rights  shall  
not   be   subject   to   any   formality;   such   enjoyment   and                                                                                                                                                          
3
such   exercise   shall   be   independent   of   the   existence   of    TNL  –  The  National  Library;  SCL  –  Library  of  the  Supreme  Court  

o 11
Katrina Michelle Mancao
 
its   expression,   such   as   books,   pamphlets   and   other   (8)   The   protection   of   this   Convention   shall   not   apply   to  
writings;   lectures,   addresses,   sermons   and   other   works   news   of   the   day   or   to   miscellaneous   facts   having   the  
of   the   same   nature;   dramatic   or   dramatico-­‐musical   character  of  mere  items  of  press  information.  
works;   choreographic   works   and   entertainments   in    
dumb   show;   musical   compositions   with   or   without   SECTION  172  AND  172.2,  SUPRA  (ON  ORIGINAL  WORKS)  
words;   cinematographic   works   to   which   are   assimilated    
works   expressed   by   a   process   analogous   to  
SECTION  173.  DERIVATIVE  W ORKS.  –    
cinematography;   works   of   drawing,   painting,  
architecture,   sculpture,   engraving   and   lithography;   173.1.   The   following   derivative   works   shall   also   be  
photographic   works   to   which   are   assimilated   works   protected  by  copyright:  
expressed   by   a   process   analogous   to   photography;    
works  of  applied  art;  illustrations,  maps,  plans,  sketches   (a)   Dramatizations,   translations,   adaptations,  
and   three-­‐dimensional   works   relative   to   geography,   abridgments,   arrangements,   and   other  
topography,  architecture  or  science.   alterations  of  literary  or  artistic  works;  and  
   
(2)   It   shall,   however,   be   a   matter   for   legislation   in   the   (b)   Collections   of   literary,   scholarly   or   artistic  
countries  of  the  Union  to  prescribe  that  works  in  general   works,   and   compilations   of   data   and   other  
or   any   specified   categories   of   works   shall   not   be   materials   which   are   original   by   reason   of   the  
protected  unless  they  have  been  fixed  in  some  material   selection   or   coordination   or   arrangement   of  
form.   their  contents.  
   
(3)  Translations,  adaptations,  arrangements  of  music  and   173.2.   The  works  referred  to  in  paragraphs  (a)  and  (b)  
other   alterations   of   a   literary   or   artistic   work   shall   be   of   Subsection   173.1   shall   be   protected   as   new   works:  
protected   as   original   works   without   prejudice   to   the   Provided   however,   That   such   new   work   shall   not   affect  
copyright  in  the  original  work.   the   force   of   any   subsisting   copyright   upon   the   original  
  works  employed  or  any  part  thereof,  or  be  construed  to  
(4)  It  shall  be  a  matter  for  legislation  in  the  countries  of   imply   any   right   to   such   use   of   the   original   works,   or   to  
the  Union  to  determine  the  protection  to  be  granted  to   secure  or  extend  copyright  in  such  original  works.  
official   texts   of   a   legislative,   administrative   and   legal    
nature,  and  to  official  translations  of  such  texts.   SECTION  174.  PUBLISHED  EDITION  OF  WORK.  –    
  In   addition   to   the   right   to   publish   granted   by   the   author,  
(5)   Collections   of   literary   or   artistic   works   such   as   his  heirs,  or  assigns,  the  publisher  shall  have  a  copyright  
encyclopaedias  and  anthologies  which,  by  reason  of  the   consisting   merely   of   the   right   of   reproduction   of   the  
selection   and   arrangement   of   their   contents,   constitute   typographical   arrangement   of   the   published   edition   of  
intellectual  creations  shall  be  protected  as  such,  without   the  work.    
prejudice  to  the  copyright  in  each  of  the  works  forming    
part  of  such  collections.   SECTION  175.  UNPROTECTED  SUBJECT  MATTER.  –    
 
Notwithstanding   the   provisions   of   Sections   172   and   173,  
(6)   The   works   mentioned   in   this   Article   shall   enjoy  
no   protection   shall   extend,   under   this   law,   to   any   idea,  
protection   in   all   countries   of   the   Union.   This   protection  
procedure,   system,   method   or   operation,   concept,  
shall   operate   for   the   benefit   of   the   author   and   his  
principle,  discovery  or  mere  data  as  such,  even  if  they  are  
successors  in  title.  
expressed,  explained,  illustrated  or  embodied  in  a  work;  
 
news   of   the   day   and   other   miscellaneous   facts   having  
(7)   Subject   to   the   provisions   of   Article   7(4)   of   this  
the  character  of  mere  items  of  press  information;  or  any  
Convention,   it   shall   be   a   matter   for   legislation   in   the  
official  text  of  a  legislative,  administrative  or  legal  nature,  
countries   of   the   Union   to   determine   the   extent   of   the  
as  well  as  any  official  translation  thereof.  
application   of   their   laws   to   works   of   applied   art   and  
 
industrial   designs   and   models,   as   well   as   the   conditions  
under   which   such   works,   designs   and   models   shall   be   SECTION  176.  WORKS  OF  THE  GOVERNMENT.  –    
protected.   Works   protected   in   the   country   of   origin   176.1.   No   copyright   shall   subsist   in   any   work   of   the  
solely  as  designs  and  models  shall  be  entitled  in  another   Government  of  the  Philippines.  However,  prior  approval  
country  of  the  Union  only  to  such  special  protection  as  is   of  the  government  agency  or  office  wherein  the  work  is  
granted  in  that  country  to  designs  and  models;  however,   created  shall  be  necessary  for  exploitation  of  such  work  
if   no   such   special   protection   is   granted   in   that   country,   for   profit.   Such   agency   or   office   may,   among   other  
such  works  shall  be  protected  as  artistic  works.   things,   impose   as   a   condition   the   payment   of   royalties.  
  No  prior  approval  or  conditions  shall  be  required  for  the  
use   of   any   purpose   of   statutes,   rules   and   regulations,  
and   speeches,   lectures,   sermons,   addresses,   and  

12 z
Intellectual Property Law

dissertations,  pronounced,  read  or  rendered  in  courts  of   Copyright,   in   the   strict   sense   of   the   term,   is   purely   a  
justice,   before   administrative   agencies,   in   deliberative   statutory  right.  It  is  a  new  or  independent  right  granted  
assemblies  and  in  meetings  of  public  character.   by   the   statute,   and   not   simply   a   pre-­‐existing   right  
  regulated     by   the   statute.     Being   a   statutory   grant,   the  
176.2.   The   author   of   speeches,   lectures,   sermons,   rights   are   only   such   as   the   statute   confers,   and   may   be  
addresses,   and   dissertations   mentioned   in   the   preceding   obtained   and   enjoyed   only   with   respect   to   the   subjects  
paragraphs   shall   have   the   exclusive   right   of   making   a   and   by   the   persons,   and   on   terms   and   conditions  
collection  of  his  works.     specified  in  the  statute.  
   
176.3.   Notwithstanding   the   foregoing   provisions,   the   Since   copyright   in   published   works   is   purely   a   statutory  
Government   is   not   precluded   from   receiving   and   holding   creation,   a   copyright   may   be   obtained   only   for   a   work  
copyrights   transferred   to   it   by   assignment,   bequest   or   falling  within  the  statutory  enumeration  or  description.  
otherwise;   nor   shall   publication   or   republication   by   the    
Government  in  a  public  document  of  any  work  in  which   Regardless   of   the   historical   viewpoint,   it   is  
copyright  is  subsisting  be  taken  to  cause  any  abridgment   authoritatively   settled   in   the   United   States   that   there   is  
or   annulment   of   the   copyright   or   to   authorize   any   use   or   no  copyright  except    that    which    is    both    created    and  
appropriation   of   such   work   without   the   consent   of   the   secured  by  act  of  Congress.  
copyright  owner.      
  P.D.   No.   49,   §2,   in   enumerating   what   are   subject   to  
copyright,   refers   to   finished   works   and   not   to   concepts.    
JOAQUIN,  JR.  V.  DRILON  (1999)  
The   copyright   does   not   extend   to   an   idea,   procedure,  
Refresher:   process,  system,  method  of  operation,  concept,  principle,  
Petitioner   BJ   Productions,   Inc.   (BJPI)   is   the   or   discovery,   regardless   of   the   form   in   which   it   is  
holder/grantee   of   Certificate   of   Copyright   No.   M922,   described,   explained,   illustrated,   or   embodied   in   such  
dated  January  28,  1971,  of  Rhoda  and  Me,  a  dating  game   work.   In   fact,   Section   175   of   the   Intellectual   Property  
show  aired  from  1970  to  1977.     Code   expressly   excluded   these   as   protected   subject  
  matter.    
On   June   28,   1973,   petitioner   BJPI   submitted   to   the    
National   Library   an   addendum   to   its   certificate   of   What   then   is   the   subject   matter   of   petitioners’  
copyright   specifying   the   show’s   format   and   style   of   copyright?     This   Court   is   of   the   opinion   that   petitioner  
presentation.   BJPI’s   copyright   covers   audio-­‐visual   recordings   of   each  
  episode   of   Rhoda   and   Me,   as   falling   within   the   class   of  
On   July   14,   1991,   while   watching   television,   petitioner   works  mentioned  in  P.D.  49,  §2(M),  to  wit:  
Francisco   Joaquin,   Jr.,   president   of   BJPI,   saw   on   RPN    
Channel   9   an   episode   of  It’s   a   Date,   which   was   produced   Cinematographic   works   and   works   produced  
by  IXL  Productions,  Inc.  (IXL).  On  July  18,  1991,  he  wrote   by   a   process   analogous   to   cinematography   or  
a  letter  to  private  respondent  Gabriel  M.  Zosa,  president   any  process  for  making  audio-­‐visual  recordings;  
and   general   manager   of   IXL,   informing   Zosa   that   BJPI    
had   a   copyright   to   Rhoda   and   Me   and   demanding   that   The  copyright  does  not  extend  to  the  general  concept  or  
IXL   discontinue   airing   It’s   a   Date,   because   the   latter’s   format  of  its  dating  game  show.  Accordingly,  by  the  very  
game   format   was   substantially   the   same   as   the   former.   nature   of   the   subject   of   petitioner   BJPI’s   copyright,   the  
The   substance   of   the   television   productions   of   the   investigating  prosecutor  should  have  the  opportunity  to  
complainant’s   “RHODA   AND   ME”   and   Zosa’s   “IT’S   A   compare  the  videotapes  of  the  two  shows.  
DATE”   is   that   two   matches   are   made   between   a   male    
and   a   female,   both   single,   and   the   two   couples   are   Mere  description  by  words  of  the  general  format  of  the  
treated   to   a   night   or   two   of   dining   and/or   dancing   at   the   two  dating  game  shows  is  insufficient;  the  presentation  
expense   of   the   show.     The   major   concepts   of   both   of   the   master   videotape   in   evidence   was   indispensable  
shows   is   the   same.   Any   difference   appear   mere   to   the   determination   of   the   existence   of   probable   cause.    
variations  of  the  major  concepts.   As  aptly  observed  by  respondent  Secretary  of  Justice:  
   
Doctrine:   A   television   show   includes   more   than   mere  
To  begin  with,  the  format  of  a  show  is  not  copyrightable.   words   can   describe   because   it   involves   a   whole  
The   format   or   mechanics   of   a   television   show   is   not   spectrum   of   visuals   and   effects,   video   and  
included   in   the   list   of   protected   works   in   §2   of   P.D.   No.   audio,   such   that   no   similarity   or   dissimilarity  
49,  nor  in  §172  of  the  Intellectual  Property  Code.  For  this   may   be   found   by   merely   describing   the   general  
reason,   the   protection   afforded   by   the   law   cannot   be   copyright/format  of  both  dating  game  shows.  
extended  to  cover  them.    
 

o 13
Katrina Michelle Mancao
 
UNITED   FEATURE   SYNDICATE,   INC.   V.   MUNSINGWEAR   license   its   white   pages   listings   to   Feist   for   a   directory  
covering   11   different   telephone   service   areas,   Feist  
CREATION  MANUFACTURING  COMPANY  (1989)  
extracted   the   listings   it   needed   from   Rural's   directory  
Refresher:  
without   Rural's   consent.  Although  Feist  altered  many  of  
This   case   arose   from   petition   filed   by   petitioner   for   the  
Rural's   listings,   several   were   identical   to   listings   in  
cancellation   of   the   registration   of   trademark   CHARLIE  
Rural's   white   pages.   The   District   Court   granted   summary  
BROWN   (Registration   No.   SR.   4224)   in   the   name   of  
judgment   to   Rural   in   its   copyright   infringement   suit,  
respondent   MUNSINGWEAR   in   Inter   Partes   Case   No.  
holding   that   telephone   directories   are   copyrightable.  
1350   entitled   "United   Feature   Syndicate,   Inc.   v.  
The  Court  of  Appeals  affirmed.  
Munsingwear   Creation   Mfg.   Co.",   with   the   Philippine  
 
Patent  Office  alleging  that  petitioner  is  damaged  by  the  
Doctrine:  
registration   of   the   trademark   CHARLIE   BROWN   of   T-­‐
Rural's   white   pages   are   not   entitled   to   copyright,   and  
Shirts   under   Class   25   with   the   Registration   No.   SR-­‐4224  
therefore   Feist's   use   of   them   does   not   constitute  
dated   September   12,   1979   in   the   name   of   Munsingwear  
infringement.    
Creation   Manufacturing   Co.,   Inc.,   on   the   following  
 
grounds:   (1)   that   respondent   was   not   entitled   to   the  
Article   I,   §   8,   cl.   8,   of   the   Constitution   mandates  
registration   of   the   mark   CHARLIE   BROWN,   &   DEVICE   at  
originality   as   a   prerequisite   for   copyright   protection.   The  
the  time  of  application  for  registration;  (2)  that  CHARLIE  
constitutional   requirement   necessitates   independent  
BROWN  is  a  character  creation  or  a  pictorial  illustration,  
creation  plus  a  modicum  of  creativity.  Since  facts  do  not  
the   copyright   to   which   is   exclusively   owned   worldwide  
owe   their   origin   to   an   act   of   authorship,   they   are   not  
by   the   petitioner;   (3)   that   as   the   owner   of   the   pictorial  
original,   and   thus   are   not   copyrightable.   Although   a  
illustration   CHARLIE   BROWN,   petitioner   has   since   1950  
compilation   of   facts   may   possess   the   requisite   originality  
and   continuously   up   to   the   present,   used   and  
because   the   author   typically   chooses   which   facts   to  
reproduced   the   same   to   the   exclusion   of   others;   (4)   that  
include,  in  what  order  to  place  them,  and  how  to  arrange  
the   respondent-­‐registrant   has   no   bona   fide   use   of   the  
the   data   so   that   readers   may   use   them   effectively,  
trademark   in   commerce   in   the   Philippines   prior   to   its  
copyright   protection   extends   only   to   those   components  
application  for  registration.  
of   the   work   that   are   original   to   the   author,   not   to   the  
 
facts   themselves.   This   fact/expression   dichotomy  
Doctrine:  
severely   limits   the   scope   of   protection   in   fact-­‐based  
Since   the   name   "CHARLIE   BROWN"   and   its   pictorial  
works.    
representation  were  covered  by  a  copyright  registration  
 
way   back   in   1950,   the   same   are   entitled   to   protection  
The   Copyright   Act   of   1976   and   its   predecessor,   the  
under  PD  No.  49.  
Copyright   Act   of   1909,   leave   no   doubt   that   originality   is  
 
the  touchstone  of  copyright  protection  in  directories  and  
Aside   from   its   copyright   registration,   petitioner   is   also  
other   fact-­‐based   works.   The   1976   Act   explains   that  
the   owner   of   several   trademark   registrations   and  
copyright   extends   to   "original   works   of   authorship,"   17  
application   for   the   name   and   likeness   of   "CHARLIE  
U.S.C.   §   102(a),   and   that   there   can   be   no   copyright   in  
BROWN"   which   is   the   duly   registered   trademark   and  
facts,  §  102(b).  
copyright   of   petitioner   United   Feature   Syndicate   Inc.   as  
 
early   as   1957   and   additionally   also   as   TV   SPECIALS  
A   compilation   is   not   copyrightable   per   se,   but   is  
featuring  the  “PEANUTS”  characters  “CHARLIE  BROWN.”  
copyrightable   only   if   its   facts   have   been   "selected,  
 
coordinated,   or   arranged   in   such   a   way   that   the   resulting  
FEIST   PUBLICATIONS,   INC.   V.   RURAL   TELEPHONE   SERVICE   work   as   a   whole   constitutes   an   original   work   of  
CO.  (1991)   authorship.   Thus,   the   statute   envisions   that   some   ways  
Refresher:   of   selecting,   coordinating,   and   arranging   data   are   not  
Respondent   Rural   Telephone   Service   Company   is   a   sufficiently  original  to  trigger  copyright  protection.  Even  
certified   public   utility   providing   telephone   service   to   a  compilation  that  is  copyrightable  receives  only  limited  
several   communities   in   Kansas.   Pursuant   to   state   protection,   for   the   copyright   does   not   extend   to   facts  
regulation,  Rural  publishes  a  typical  telephone  directory,   contained  in  the  compilation.  Lower  courts  that  adopted  
consisting   of   white   pages   and   yellow   pages.   It   obtains   a   "sweat   of   the   brow"   or   "industrious   collection"   test   -­‐-­‐  
data   for   the   directory   from   subscribers,   who   must   which   extended   a   compilation's   copyright   protection  
provide   their   names   and   addresses   to   obtain   telephone   beyond   selection   and   arrangement   to   the   facts  
service.     themselves   –   misconstrued   the   1909   Act   and   eschewed  
  the  fundamental  axiom  of  copyright  law  that  no  one  may  
Petitioner   Feist   Publications,   Inc.,   is   a   publishing   copyright  facts  or  ideas.    
company   that   specializes   in   area-­‐wide   telephone    
directories   covering   a   much   larger   geographic   range   Rural's   white   pages   do   not   meet   the   constitutional   or  
than   directories   such   as   Rural's.   When   Rural   refused   to   statutory   requirements   for   copyright   protection.   While  

14 z
Intellectual Property Law

Rural   has   a   valid   copyright   in   the   directory   as   a   whole   the   person   who   so   commissioned   the   work   shall   have  
because  it  contains  some  forward  text  and  some  original   ownership   of   the   work,   but   the   copyright   thereto   shall  
material   in   the   yellow   pages,   there   is   nothing   original   in   remain   with   the   creator,   unless   there   is   a   written  
Rural's   white   pages.   The   raw   data   are   uncopyrightable   stipulation  to  the  contrary;  
facts,  and  the  way  in  which  Rural  selected,  coordinated,    
and   arranged   those   facts   is   not   original   in   any   way.   178.5.   In   the   case   of   audiovisual   work,   the   copyright   shall  
Rural's  selection  of  listings  -­‐-­‐   subscribers'  names,  towns,   belong   to   the   producer,   the   author   of   the   scenario,   the  
and   telephone   numbers   -­‐-­‐   could   not   be   more   obvious,   composer  of  the  music,  the  film  director,  and  the  author  
and   lacks   the   modicum   of   creativity   necessary   to   of   the   work   so   adapted.   However,   subject   to   contrary   or  
transform  mere  selection  into  copyrightable  expression.   other   stipulations   among   the   creators,   the   producer  
In  fact,  it  is  plausible  to  conclude  that  Rural  did  not  truly   shall  exercise  the  copyright  to  an  extent  required  for  the  
"select"  to  publish  its  subscribers'  names  and  telephone   exhibition   of   the   work   in   any   manner,   except   for   the  
numbers,   since   it   was   required   to   do   so   by   state   law.   right   to   collect   performing   license   fees   for   the  
Moreover,   there   is   nothing   remotely   creative   about   performance   of   musical   compositions,   with   or   without  
arranging   names   alphabetically   in   a   white   pages   words,  which  are  incorporated  into  the  work;  and  
directory.   It   is   an   age-­‐old   practice,   firmly   rooted   in    
tradition   and   so   commonplace   that   it   has   come   to   be   178.6.  In  respect  of  letters,  the  copyright  shall  belong  to  
expected  as  a  matter  of  course.     the  writer  subject  to  the  provisions  of  Article  723  of  the  
  Civil  Code.    
Remarks:  US  case.    
  SECTION   179.   ANONYMOUS   AND   PSEUDONYMOUS   WORKS.  
5. Who  owns  Copyright?  
–  
 
For  purposes  of  this  Act,  the  publishers  shall  be  deemed  
SECTION  178.  RULES  ON  COPYRIGHT  OWNERSHIP.  –     to   represent   the   authors   of   articles   and   other   writings  
Copyright  ownership  shall  be  governed  by  the  following   published   without   the   names   of   the   authors   or   under  
rules:   pseudonyms,   unless   the   contrary   appears,   or   the  
  pseudonyms   or   adopted   name   leaves   no   doubt   as   to   the  
178.1.  Subject  to  the  provisions  of  this  section,  in  the  case   author's   identity,   or   if   the   author   of   the   anonymous  
of   original   literary   and   artistic   works,   copyright   shall   works  discloses  his  identity.    
belong  to  the  author  of  the  work;    
  SECTION  174.  PUBLISHED  EDITION  OF  WORK.  –    
178.2.   In   the   case   of   works   of   joint   authorship,   the   co-­‐ In   addition   to   the   right   to   publish   granted   by   the   author,  
authors  shall  be  the  original  owners  of  the  copyright  and   his  heirs,  or  assigns,  the  publisher  shall  have  a  copyright  
in   the   absence   of   agreement,   their   rights   shall   be   consisting   merely   of   the   right   of   reproduction   of   the  
governed   by   the   rules   on   co-­‐ownership.   If,   however,   a   typographical   arrangement   of   the   published   edition   of  
work   of   joint   authorship   consists   of   parts   that   can   be   the  work.  
used   separately   and   the   author   of   each   part   can   be    
identified,   the   author   of   each   part   shall   be   the   original   ARTICLE  722,  CIVIL  C ODE.    
owner  of  the  copyright  in  the  part  that  he  has  created;  
The   author   and   the   composer,   mentioned   in   Nos.   1   and   2  
 
of   the   preceding   article,   shall   have   the   ownership   of  
178.3.   In   the   case   of   work   created   by   an   author   during  
their  creations  even  before  the  publication  of  the  same.  
and  in  the  course  of  his  employment,  the  copyright  shall   Once   their   works   are   published,   their   rights   are  
belong  to:   governed  by  the  Copyright  laws.  
   
(a)  The  employee,  if  the  creation  of  the  object   The  painter,  sculptor  or  other  artist  shall  have  dominion  
of   copyright   is   not   a   part   of   his   regular   duties   over  the  product  of  his  art  even  before  it  is  copyrighted.  
even   if   the   employee   uses   the   time,   facilities    
and  materials  of  the  employer.   The   scientist   or   technologist   has   the   ownership   of   his  
  discovery  or  invention  even  before  it  is  patented.    
(b)   The   employer,   if   the   work   is   the   result   of    
the   performance   of   his   regularly-­‐assigned  
ARTICLE  723,  CIVIL  CODE.    
duties,   unless   there   is   an   agreement,   express  
or  implied,  to  the  contrary.   Letters  and  other  private  communications  in  writing  are  
  owned   by   the   person   to   whom   they   are   addressed   and  
delivered,  but  they  cannot  be  published  or  disseminated  
178.4.   In   the   case   of   a   work   commissioned   by   a   person  
without  the  consent  of  the  writer  or  his  heirs.  However,  
other  than  an  employer  of  the  author  and  who  pays  for  
the   court   may   authorize   their   publication   or  
it   and   the   work   is   made   in   pursuance   of   the   commission,  

o 15
Katrina Michelle Mancao
 
dissemination   if   the   public   good   or   the   interest   of   justice    
so  requires.     (5)   The   term   of   protection   subsequent   to   the   death   of  
  the  author  and  the  terms  provided  by  paragraphs  (2),  (3)  
ARTICLE  520,  CIVIL  CODE.     and  (4)  shall  run  from  the  date  of  death  or  of  the  event  
A  trade-­‐mark  or  trade-­‐name  duly  registered  in  the  proper   referred   to   in   those   paragraphs,   but   such   terms   shall  
government  bureau  or  office  is  owned  by  and  pertains  to   always  be  deemed  to  begin  on  the  first  of  January  of  the  
the   person,   corporation,   or   firm   registering   the   same,   year  following  the  death  or  such  event.  
subject  to  the  provisions  of  special  laws.    
  (6)   The   countries   of   the   Union   may   grant   a   term   of  
protection  in  excess  of  those  provided  by  the  preceding  
6. Duration  of  Copyright  
paragraphs.  
 
 
ARTICLE   7,   BERNE   CONVENTION   FOR   THE   PROTECTION   OF   (7)  Those  countries  of  the  Union  bound  by  the  Rome  Act  
LITERARY  AND  ARTISTIC  WORKS   of   this   Convention   which   grant,   in   their   national  
TERM  OF  PROTECTION:   legislation   in   force   at   the   time   of   signature   of   the  
1.  Generally;     present   Act,   shorter   terms   of   protection   than   those  
2.  For  cinematographic  works;     provided  for  in  the  preceding  paragraphs  shall  have  the  
3.  For  anonymous  and  pseudonymous  works;   right  to  maintain  such  terms  when  ratifying  or  acceding  
4.  For  photographic  works  and  works  of  applied   to  the  present  Act.  
art;      
5.  Starting  date  of  computation;   (8)   In   any   case,   the   term   shall   be   governed   by   the  
6.  Longer  terms;     legislation   of   the   country   where   protection   is   claimed;  
7.  Shorter  terms;     however,  unless  the  legislation  of  that  country  otherwise  
8.  Applicable  law;  “comparison”  of  terms   provides,   the   term   shall   not   exceed   the   term   fixed   in   the  
  country  of  origin  of  the  work.  
(1)   The   term   of   protection   granted   by   this   Convention    
shall   be   the   life   of   the   author   and   fifty   years   after   his   ARTICLE   7BIS,   BERNE   CONVENTION   FOR   THE   PROTECTION  
death.   OF  LITERARY  AND  ARTISTIC  W ORKS  
  TERM  OF  PROTECTION  FOR  WORKS  OF  JOINT  AUTHORSHIP  
(2)  However,  in  the  case  of  cinematographic  works,  the  
 
countries   of   the   Union   may   provide   that   the   term   of  
The   provisions   of   the   preceding   Article   shall   also   apply   in  
protection   shall   expire   fifty   years   after   the   work   has  
the  case  of  a  work  of  joint  authorship,  provided  that  the  
been   made   available   to   the   public   with   the   consent   of  
terms   measured   from   the   death   of   the   author   shall   be  
the   author,   or,   failing   such   an   event   within   fifty   years  
calculated  from  the  death  of  the  last  surviving  author.  
from   the   making   of   such   a   work,   fifty   years   after   the  
 
making.  
SECTION  213.  TERM  OF  PROTECTION.  –    
 
(3)   In   the   case   of   anonymous   or   pseudonymous   works,   213.1.   Subject   to   the   provisions   of   Subsections   213.2   to  
the   term   of   protection   granted   by   this   Convention   shall   213.5,  the  copyright  in  works  under  Sections  172  and  173  
expire  fifty  years  after  the  work  has  been  lawfully  made   shall   be   protected   during   the   life   of   the   author   and   for  
available   to   the   public.   However,   when   the   pseudonym   fifty   (50)   years   after   his   death.   This   rule   also   applies   to  
adopted  by  the  author  leaves  no  doubt  as  to  his  identity,   posthumous  works.    
the   term   of   protection   shall   be   that   provided   in    
paragraph   (1).   If   the   author   of   an   anonymous   or   213.2.  In  case  of  works  of  joint  authorship,  the  economic  
pseudonymous   work   discloses   his   identity   during   the   rights   shall   be   protected   during   the   life   of   the   last  
above-­‐mentioned   period,   the   term   of   protection   surviving  author  and  for  fifty  (50)  years  after  his  death.    
applicable   shall   be   that   provided   in   paragraph   (1).   The    
countries   of   the   Union   shall   not   be   required   to   protect   213.3.   In   case   of   anonymous   or   pseudonymous   works,  
anonymous   or   pseudonymous   works   in   respect   of   which   the  copyright  shall  be  protected  for  fifty  (50)  years  from  
it   is   reasonable   to   presume   that   their   author   has   been   the  date  on  which  the  work  was  first  lawfully  published:  
dead  for  fifty  years.   Provided,   That   where,   before   the   expiration   of   the   said  
  period,   the   author's   identity   is   revealed   or   is   no   longer   in  
(4)  It  shall  be  a  matter  for  legislation  in  the  countries  of   doubt,  the  provisions  of  Subsections  213.1.  and  213.2  shall  
the   Union   to   determine   the   term   of   protection   of   apply,   as   the   case   may   be:   Provided,   further,   That   such  
photographic   works   and   that   of   works   of   applied   art   in   works   if   not   published   before   shall   be   protected   for   fifty  
so   far   as   they   are   protected   as   artistic   works;   however,   (50)  years  counted  from  the  making  of  the  work.  
this   term   shall   last   at   least   until   the   end   of   a   period   of    
twenty-­‐five  years  from  the  making  of  such  a  work.  

16 z
Intellectual Property Law

213.4.   In   case   of   works   of   applied   art   the   protection   shall   Government   or   any   of   its   subdivisions   and  
be  for  a  period  of  twenty-­‐five  (25)  years  from  the  date  of   instrumentalities,   including   government-­‐owned   or  
making.   controlled   corporations   as   a   part   of   his   regularly  
  prescribed  official  duties.  
213.5.  In  case  of  photographic  works,  the  protection  shall    
be  for  fifty  (50)  years  from  publication  of  the  work  and,   SECTION  184.  LIMITATIONS  ON  COPYRIGHT.  –    
if  unpublished,  fifty  (50)  years  from  the  making.     184.1.   Notwithstanding   the   provisions   of   Chapter   V,   the  
  following   acts   shall   not   constitute   infringement   of  
213.6.   In   case   of   audio-­‐visual   works   including   those   copyright:  
produced   by   process   analogous   to   photography   or   any    
process   for   making   audio-­‐visual   recordings,   the   term   (a)   The   recitation   or   performance   of   a   work,  
shall   be   fifty   (50)   years   from   date   of   publication   and,   if   once   it   has   been   lawfully   made   accessible   to  
unpublished,  from  the  date  of  making.   the  public,  if  done  privately  and  free  of  charge  
  or   if   made   strictly   for   a   charitable   or   religious  
SECTION  214.  CALCULATION  OF  TERM.  –     institution  or  society;    
The   term   of   protection   subsequent   to   the   death   of   the    
author  provided  in  the  preceding  Section  shall  run  from   (b)  The  making  of  quotations  from  a  published  
the   date   of   his   death   or   of   publication,   but   such   terms   work   if   they   are   compatible   with   fair   use   and  
shall   always   be   deemed   to   begin   on   the   first   day   of   only   to   the   extent   justified   for   the   purpose,  
January  of  the  year  following  the  event  which  gave  rise   including   quotations   from   newspaper   articles  
to  them.     and  periodicals  in  the  form  of  press  summaries:  
  Provided,  That  the  source  and  the  name  of  the  
author,   if   appearing   on   the   work,   are  
7. Limitations  on  Copyright  
mentioned;    
   
SECTION  176.  WORKS  OF  THE  GOVERNMENT.  –     (c)   The   reproduction   or   communication   to   the  
176.1.   No   copyright   shall   subsist   in   any   work   of   the   public   by   mass   media   of   articles   on   current  
Government  of  the  Philippines.  However,  prior  approval   political,   social,   economic,   scientific   or   religious  
of  the  government  agency  or  office  wherein  the  work  is   topic,   lectures,   addresses   and   other   works   of  
created  shall  be  necessary  for  exploitation  of  such  work   the   same   nature,   which   are   delivered   in   public  
for   profit.   Such   agency   or   office   may,   among   other   if  such  use  is  for  information  purposes  and  has  
things,   impose   as   a   condition   the   payment   of   royalties.   not   been   expressly   reserved:   Provided,   That  
No  prior  approval  or  conditions  shall  be  required  for  the   the  source  is  clearly  indicated;    
use   of   any   purpose   of   statutes,   rules   and   regulations,    
and   speeches,   lectures,   sermons,   addresses,   and   (d)   The   reproduction   and   communication   to  
dissertations,  pronounced,  read  or  rendered  in  courts  of   the  public  of  literary,  scientific  or  artistic  works  
justice,   before   administrative   agencies,   in   deliberative   as   part   of   reports   of   current   events   by   means  
assemblies  and  in  meetings  of  public  character.   of   photography,   cinematography   or  
  broadcasting   to   the   extent   necessary   for   the  
176.2.   The   author   of   speeches,   lectures,   sermons,   purpose;    
addresses,   and   dissertations   mentioned   in   the   preceding    
paragraphs   shall   have   the   exclusive   right   of   making   a   (e)   The   inclusion   of   a   work   in   a   publication,  
collection  of  his  works.     broadcast,   or   other   communication   to   the  
  public,  sound  recording  or  film,  if  such  inclusion  
176.3.   Notwithstanding   the   foregoing   provisions,   the   is   made   by   way   of   illustration   for   teaching  
purposes   and   is   compatible   with   fair   use:  
Government   is   not   precluded   from   receiving   and   holding  
Provided,   That   the   source   and   of   the   name   of  
copyrights   transferred   to   it   by   assignment,   bequest   or  
the   author,   if   appearing   in   the   work,   are  
otherwise;   nor   shall   publication   or   republication   by   the  
mentioned;  
Government  in  a  public  document  of  any  work  in  which  
 
copyright  is  subsisting  be  taken  to  cause  any  abridgment  
(f)  The  recording  made  in  schools,  universities,  
or   annulment   of   the   copyright   or   to  authorize   any   use   or  
or  educational  institutions  of  a  work  included  in  
appropriation   of   such   work   without   the   consent   of   the  
a   broadcast   for   the   use   of   such   schools,  
copyright  owner.  
universities   or   educational   institutions:  
 
Provided,  That  such  recording  must  be  deleted  
SECTION  171.11.     within   a   reasonable   period   after   they   were   first  
A   "WORK   OF   THE   GOVERNMENT   OF   THE   PHILIPPINES"   is   a   work   broadcast:   Provided,   further,   That   such  
created   by   an   officer   or   employee   of   the   Philippine   recording   may   not   be   made   from   audiovisual  

o 17
Katrina Michelle Mancao
 
works   which   are   part   of   the   general   cinema   whether   the   use   made   of   a   work   in   any   particular   case   is  
repertoire   of   feature   films   except   for   brief   fair  use,  the  factors  to  be  considered  shall  include:  
excerpts  of  the  work;    
  (a)   The   purpose   and   character   of   the   use,  
(g)   The   making   of   ephemeral   recordings   by   a   including   whether   such   use   is   of   a   commercial  
broadcasting  organization  by  means  of  its  own   nature   or   is   for   non-­‐profit   educational  
facilities  and  for  use  in  its  own  broadcast;   purposes;  
   
(h)   The   use   made   of   a   work   by   or   under   the   (b)  The  nature  of  the  copyrighted  work;  
direction  or  control  of  the  Government,  by  the    
National  Library  or  by  educational,  scientific  or   (c)   The   amount   and   substantiality   of   the  
professional   institutions   where   such   use   is   in   portion   used   in   relation   to   the   copyrighted  
the   public   interest   and   is   compatible   with   fair   work  as  a  whole;  and  
use;    
  (d)   The   effect   of   the   use   upon   the   potential  
(i)   The   public   performance   or   the   market  for  or  value  of  the  copyrighted  work.  
communication   to   the   public   of   a   work,   in   a    
place   where   no   admission   fee   is   charged   in   185.2.   The   fact   that   a   work   is   unpublished   shall   not   by  
respect   of   such   public   performance   or   itself  bar  a  finding  of  fair  use  if  such  finding  is  made  upon  
communication,   by   a   club   or   institution   for   consideration  of  all  the  above  factors.  
charitable   or   educational   purpose   only,   whose    
aim  is  not  profit  making,  subject  to  such  other   SECTION  186.  WORK  OF  ARCHITECTURE.  –    
limitations   as   may   be   provided   in   the  
Copyright  in  a  work  of  architecture  shall  include  the  right  
Regulations;  
to   control   the   erection   of   any   building   which   reproduces  
 
the   whole   or   a   substantial   part   of   the   work   either   in   its  
(j)   Public   display   of   the   original   or   a   copy   of   the  
original   form   or   in   any   form   recognizably   derived   from  
work   not   made   by   means   of   a   film,   slide,  
the   original:   Provided,   That   the   copyright   in   any   such  
television   image   or   otherwise   on   screen   or   by  
work   shall   not   include   the   right   to   control   the  
means  of  any  other  device  or  process:  Provided,  
reconstruction   or   rehabilitation   in   the   same   style   as   the  
That   either   the   work   has   been   published,   or,  
original  of  a  building  to  which  that  copyright  relates.    
that  the  original  or  the  copy  displayed  has  been  
 
sold,   given   away   or   otherwise   transferred   to  
another   person   by   the   author   or   his   successor   SECTION  187.  REPRODUCTION  OF  PUBLISHED  WORK.  –    
in  title;  and   187.1.   Notwithstanding   the   provision   of   Section   177,   and  
  subject   to   the   provisions   of   Subsection   187.2,   the   private  
(k)  Any  use  made  of  a  work  for  the  purpose  of   reproduction  of  a  published  work  in  a  single  copy,  where  
any   judicial   proceedings   or   for   the   giving   of   the   reproduction   is   made   by   a   natural   person   exclusively  
professional  advice  by  a  legal  practitioner.   for   research   and   private   study,   shall   be   permitted,  
  without   the   authorization   of   the   owner   of   copyright   in  
184.2.  The  provisions  of  this  section  shall  be  interpreted   the  work.  
in   such   a   way   as   to   allow   the   work   to   be   used   in   a    
manner   which   does   not   conflict   with   the   normal   187.2.   The   permission   granted   under   Subsection   187.1  
exploitation   of   the   work   and   does   not   unreasonably   shall  not  extend  to  the  reproduction  of:  
prejudice  the  right  holder's  legitimate  interests.4    
  (a)   A   work   of   architecture   in   the   form   of  
SECTION  185.  FAIR  USE  OF  A  COPYRIGHTED  WORK.  –     building  or  other  construction;  
185.1.   The   fair   use   of   a   copyrighted   work   for   criticism,    
(b)  An  entire  book,  or  a  substantial  part  thereof,  
comment,   news   reporting,   teaching   including   multiple  
or   of   a   musical   work   in   graphic   form   by  
copies   for   classroom   use,   scholarship,   research,   and  
reprographic  means;  
similar   purposes   is   not   an   infringement   of   copyright.  
 
Decompilation,   which   is   understood   here   to   be   the  
(c)  A  compilation  of  data  and  other  materials;  
reproduction   of   the   code   and   translation   of   the   forms   of  
 
the  computer  program  to  achieve  the  inter-­‐operability  of  
(d)  A  computer  program  except  as  provided  in  
an  independently  created  computer  program  with  other  
Section  189;  and  
programs   may   also   constitute   fair   use.   In   determining  
 
                                                                                                                                                        (e)   Any   work   in   cases   where   reproduction  
4
 IMPORTANT  NOTE:  184.1  must  be  read  with  184.2.   would   unreasonably   conflict   with   a   normal  

18 z
Intellectual Property Law

exploitation   of   the   work   or   would   otherwise   and   to   the   extent,   for   which   the   computer  
unreasonably  prejudice  the  legitimate  interests   program  has  been  obtained;  and  
of  the  author.      
  (b)  Archival  purposes,  and,  for  the  replacement  
SECTION  188.  REPROGRAPHIC  REPRODUCTION  BY  LIBRARIES.   of   the   lawfully   owned   copy   of   the   computer  
program  in  the  event  that  the  lawfully  obtained  
–  
copy   of   the   computer   program   is   lost,  
188.1.   Notwithstanding   the   provisions   of   Subsection   destroyed  or  rendered  unusable.  
177.6,   any   library   or   archive   whose   activities   are   not   for    
profit   may,   without   the   authorization   of   the   author   of   189.2.   No   copy   or   adaptation   mentioned   in   this  
copyright   owner,   make   a   single   copy   of   the   work   by  
Section   shall   be   used   for   any   purpose   other   than   the  
reprographic  reproduction:  
ones   determined   in   this   Section,   and   any   such   copy   or  
 
adaptation   shall   be   destroyed   in   the   event   that  
(a)   Where   the   work   by   reason   of   its   fragile  
continued   possession   of   the   copy   of   the   computer  
character  or  rarity  cannot  be  lent  to  user  in  its  
program  ceases  to  be  lawful.  
original  form;    
 
189.3.   This  provision  shall  be  without  prejudice  to  the  
(b)   Where   the   works   are   isolated   articles  
application  of  Section  185  whenever  appropriate.  
contained  in  composite  works  or  brief  portions  
 
of  other  published  works  and  the  reproduction  
is   necessary   to   supply   them,   when   this   is   SECTION  190.  IMPORTATION  FOR  PERSONAL  PURPOSES.  –    
considered   expedient,   to   persons   requesting   190.1.  Notwithstanding  the  provision  of  Subsection  177.6,  
their   loan   for   purposes   of   research   or   study   but  subject  to  the  limitation  under  the  Subsection  185.2,  
instead   of   lending   the   volumes   or   booklets   the  importation  of  a  copy  of  a  work  by  an  individual  for  
which  contain  them;  and   his   personal   purposes   shall   be   permitted   without   the  
  authorization   of   the   author   of,   or   other   owner   of  
(c)   Where   the   making   of   such   a   copy   is   in   order   copyright   in,   the   work   under   the   following  
to  preserve  and,  if  necessary  in  the  event  that  it   circumstances:  
is  lost,  destroyed  or  rendered  unusable,  replace    
a   copy,   or   to   replace,   in   the   permanent   (a)   When   copies   of   the   work   are   not   available  
collection  of  another  similar  library  or  archive,  a   in  the  Philippines  and:  
copy   which   has   been   lost,   destroyed   or    
rendered   unusable   and   copies   are   not   available   (i)  Not  more  than  one  (1)  copy  at  one  
with  the  publisher.   time  is  imported  for  strictly  individual  
  use  only;  or  
188.2.  Notwithstanding  the  above  provisions,  it  shall  not    
be  permissible  to  produce  a  volume  of  a  work  published   (ii)  The  importation  is  by  authority  of  
in  several  volumes  or  to  produce  missing  tomes  or  pages   and   for   the   use   of   the   Philippine  
of  magazines  or  similar  works,  unless  the  volume,  tome   Government;  or  
or  part  is  out  of  stock:  Provided,  That  every  library  which,    
by   law,   is   entitled   to   receive   copies   of   a   printed   work,   (iii)  The  importation,  consisting  of  not  
shall   be   entitled,   when   special   reasons   so   require,   to   more   than   three   (3)   such   copies   or  
reproduce   a   copy   of   a   published   work   which   is   likenesses   in   any   one   invoice,   is   not  
considered   necessary   for   the   collection   of   the   library   but   for   sale   but   for   the   use   only   of   any  
which  is  out  of  stock.   religious,   charitable,   or   educational  
  society   or   institution   duly  
SECTION  189.  REPRODUCTION  OF  COMPUTER  PROGRAM.  –     incorporated   or   registered,   or   is   for  
the   encouragement   of   the   fine   arts,  
189.1.  Notwithstanding  the  provisions  of  Section  177,  the  
or   for   any   state   school,   college,  
reproduction  in  one  (1)  back-­‐up  copy  or  adaptation  of  a  
university,  or  free  public  library  in  the  
computer   program   shall   be   permitted,   without   the  
Philippines.  
authorization   of   the   author   of,   or   other   owner   of  
 
copyright   in,   a   computer   program,   by   the   lawful   owner   (b)   When   such   copies   form   parts   of   libraries  
of   that   computer   program:   Provided,   That   the   copy   or  
and  personal  baggage  belonging  to  persons  or  
adaptation  is  necessary  for:  
families  arriving  from  foreign  countries  and  are  
 
not   intended   for   sale:   Provided,   That   such  
(a)   The   use   of   the   computer   program   in   copies  do  not  exceed  three  (3).  
conjunction   with   a   computer   for   the   purpose,  
 

o 19
Katrina Michelle Mancao
 
190.2.   Copies   imported   as   allowed   by   this   Section   may   SECTION   2.   USE   IN   JUDICIAL   PROCEEDINGS   OR   BY   LEGAL  
not   lawfully   be   used   in   any   way   to   violate   the   rights   of   PRACTITIONERS.  —    
owner   the   copyright   or   annul   or   limit   the   protection   Without   prejudice   to   other   exceptions   to   protection,   the  
secured   by   this   Act,   and   such   unlawful   use   shall   be   use   of   copyrighted   works   as   provided   in   the   IPC   “for   the  
deemed   an   infringement   and   shall   be   punishable   as   such   giving   of   professional   advise   by   a   legal   practitioner”   is  
without  prejudice  to  the  proprietor's  right  of  action.   limited  to  those  uses  directly  connected  to  the  rendering  
  of   legal   advice   with   respect   to   such   works,   or   to  
190.3.  Subject  to  the  approval  of  the  Secretary  of  Finance,   individuals,   institutions,   events   or   circumstances   to  
the   Commissioner   of   Customs   is   hereby   empowered   to   which   the   creation   of   exploitation   of   such   works   are  
make   rules   and   regulations   for   preventing   the   directly   related.   This   exception   does   not   apply   to   the  
importation   of   articles   the   importation   of   which   is   exercise   of   exclusive   rights   of   copyright   owners   in   legal  
prohibited   under   this   Section   and   under   treaties   and   research   materials,   legal-­‐related   computer   software,  
conventions   to   which   the   Philippines   may   be   a   party   and   legal-­‐related  online  material,  or  the  other  works  utilized  
for  seizing  and  condemning  and  disposing  of  the  same  in   in  the  practice  of  law  on  a  regular  basis.    
case  they  are  discovered  after  they  have  been  imported.    
  SECTION  3.  USES  COMPATIBLE  WITH  FAIR  USE.  —    
RULE   14   –   LIMITATIONS   ON   COPYRIGHT,   COPYRIGHT   To   determine   whether   use   of   a   copyrighted   work   is  
SAFEGUARDS  AND  REGULATIONS   “compatible   with   fair   use,”   the   criteria   set   forth   in   Sec.  
185  of  the  IPC  shall  be  applied.    
 
 
SECTION   1.   PUBLIC   PERFORMANCE   OR   COMMUNICATION   TO   THE  
SECTION   4.   FAIR   USE   AND   DECOMPILATION   OF   COMPUTER  
PUBLIC  OF  WORK  BY  NON-­‐PROFIT  INSTITUTION.  —    
SOFTWARE.  —    
The   public   performance   or   the   communication   to   the  
An   act   of   decompilation   of   software   may   qualify   for  
public   of   a   work,   in   a   place   where   no   admission   fee   is  
analysis  under  the  fair  use  provisions  if  it  meets  all  of  the  
charged   in   respect   of   such   public   performance   or  
criteria  set  out  in  the  IPC,  viz:  
communication,  by  a  club  or  institution  for  charitable  or  
 
educational   purposes   only,   whose   aim   is   not   profit-­‐
(a)   It   consist   only   of   the   reproduction   of   code  
making,   shall   not   constitute   infringement   of   copyright,  
and   translation   of   the   forms   of   a   computer  
if:  
program;  
 
 
(a)   The   works   are   limited   to   non-­‐dramatic  
(b)   The   reproduction   and   translation   are  
literary   works   and   non-­‐copyrighted   musical  
indispensable   to   obtain   information,   such   that  
compositions,   and   do   not   include   audiovisual  
information   can   be   obtained   in   no   other   way  
works  or  computer  programs;  
than  through  decompilation;  
 
 
(b)   The   clubs   or   institutions   to   which   this  
(c)   The   information   is   necessary   in   order   to  
subsection   applies   are   limited   to   those  
achieve   interoperability   (i.e.,   interoperability  
organized   exclusively   for   charitable   or  
cannot   be   achieved   without   obtaining   the  
educational   purposes;   and   if   the   following  
information   and   without   using   decompilation  
conditions  are  satisfied:  
to  obtain  it)  ;  and  
 
 
(i)   No   fee   or   compensation   is   paid   to  
(d)   The   interoperability   that   is   sought   is  
any   performers,   promoters   or  
between   the   decompiled   program   and   an  
organizers  of  the  public  performance;  
independently  created  computer  program  (i.e.,  
 
a  computer  program  that  was  created  prior  to  
(ii)   There   is   no   direct   or   indirect  
and  without  reference  to  the  decompilation).    
admission   charge   to   the   place   where  
 
the  performance  occurs;  and  
If   the   above   criteria   are   satisfied,   then   the   fair   use  
 
factors   set   forth   in   the   IPC   can   be   applied   in   order   to  
(iii)   Admission   to   the   performance   is  
decide   whether   or   not   the   decompilation,   in   the  
restricted  to  persons  who  are,  and  for  
particular  circumstances  involved,  constitutes  a  fair  use.  
at   least   the   preceding   thirty   days  
The   fact   that   a   use   is   decompilation   (as   defined   by   the  
have   been,   members   in   good  
statutory  criteria)  creates  no  presumption  that  the  use  is  
standing  of  the  club  or  the  institution  
fair.    
for   whose   charitable   or   education  
 
purposes   the   performance   is   being  
carried  out.    
   

20 z
Intellectual Property Law

SECTION  5.  EFFECT  ON  VALUE  OF  THE  WORK.  —     publication  of  the  article  "we  reserved  all  rights,"  which  
The  effect  of  the  use  upon  the  value  of  the  copyrighted   was  legally  equivalent  to  a  notice  "that  their  publication  
work   shall   also   constitute   a   limitation   on   fair   use   of   is   reserved."   To   give   that   paragraph   any   other  
works,   particularly   whenever   their   reproduction   will   construction  would  eliminate,  take  from  it,  and  wipe  out,  
affect   their   usefulness,   reliability,   and   validity   such   as   the   words   "that   their   publication   is   reserved,"   and   this  
psychological   tests   and   others   of   similar   nature.   court  has  no  legal  right  to  do  that.  It  was  contended  that  
Reproduction   of   these   works   shall,   therefore,   need   prior   this   construction   would   nullify   the   use   and   value   of   the  
authority  of  the  copyright  owner.     whole   Copyright   Law,   but   it   will   be   noted   that   this  
  exception   is   specifically   confined   and   limited   to   "news  
SECTION   6.   REPRODUCTION  OR   COMMUNICATION   TO   THE   PUBLIC   items   editorial   paragraphs,   and   articles   in   periodicals,"  
BY  MASS  M EDIA  OF  ARTICLES,  LECTURE,  ETC.  —    
and   hence   could   not   be   made   to   apply   to   any   other  
provisions   of   the   Copyright   Law.   It   will   also   be   noted  
The   reproduction   or   communication   to   the   public   by  
that  in  the  instant  case,  the  defendant  had  the  legal  right  
mass   media   of   articles,   such   as   those   publish   in  
to  publish  the  article  in  question  by  giving  "the  source  of  
newspapers   or   periodicals   on   current   political,   social,  
the  reproduction."  The  plaintiff  bought  and  paid  for  the  
economic,   scientific,   or   religious   topics,   as   well   as  
article  and  published  it  with  the  notice  that  "we  reserve  
lectures,  addresses,  and  other  works  of  the  same  nature,  
all   rights,"   and   the   defendant   published   the   article   in  
which   are   delivered   in   public,   shall   not   constitute  
question   without   citing   "the   source   of   the  
infringement   of   copyright   if   such   use   is   for   information  
reproduction,"   and   for   aught   that   appeared   in   his   paper,  
purposes  and  has  not  been  expressly  reserved:  Provided,  
the  article  was  purchase  and  paid  for  by  the  defendant.  
that   the   source   is   clearly   indicated   and   that   the   use   of  
 
the   work   should   not   unreasonably   prejudice   the   right  
We   are   clearly   of   the   opinion   that   the   language   in  
holder’s  legitimate  interests.  
question   in   the   Copyright   Law   of   the   Philippine   Islands,  
 
which   is   not   found   in   the   Copyright   Law   of   the   United  
FILIPINO   SOCIETY  OF   COMPOSERS  V.  BENJAMIN  TAN  (1987),   States,   was   inserted   for   a   specific   purpose,   and   it   was  
SUPRA   intended   to   prohibit   the   doing   of   the   very   thing   which  
  the   defendant   did   in   this   case;   otherwise,   the   use   of   all  
PHILIPPINE   EDUCATION   CO.   V.   SOTTO   AND   ALINDADA   of   those   words   is   a   nullity.   This   construction   does   not  
least  impair  the  Copyright  Law,  except  as  to  "news  items,  
(1929)  
editorial   paragraphs,   and   articles   in   periodicals,"   and   it  
Refresher:   protects   an   enterprising   newspaper   or   magazine   that  
The   True   Story   of   Mrs.   Rizal,   written   by   Prof.   Craig,   invests   its   money   and   pays   for   the   right   to   publish   an  
published   by   Petitioner   in   Philippine   Education   Magazine.   original   article,   and   that   was   the   reason   why   the  
It  was  alleged  that  Respondent  reproduced  the  same  in   Legislature  saw  fit  to  use  the  language  in  question.  
The   Independent,   without   the   Petitioner’s   permission.    
Applicable  law  is  Act  No.  3134.   Ma’am  comment  on  the  case:  This  is  erroneous!!!  
   
Doctrine:  
SONY   CORPORATION   OF   AMERICA   V.   UNIVERSAL   STUDIOS,  
Analyzing   the   language   used,   it   says,   first,   that   such  
news   items,   editorial   paragraphs,   and   articles   in   INC.  (1983)  
periodicals   may   be   reproduced,   unless   they   contain   a   Refresher:  
notice  that  their  publication  is  reserved,  or,  second,  that   Petitioner   Sony   Corp.   manufactures   home   video   tape  
may  also  be  reproduced,  unless  they  contain  a  notice  of   recorders   (VTR's),   and   markets   them   through   retail  
copyright.   But   in   either   event,   the   law   specifically   establishments,  some  of  which  are  also  petitioners.    
provides   that   "the   source   of   the   reproduction   or   original    
reproduced  shall  be  cited,"  and  is  not  confined  or  limited   Respondents   own   the   copyrights   on   some   of   the  
to   case   in   which   there   is   "a   notice   of   copyright,"   and   television   programs   that   are   broadcast   on   the   public  
specifically   says   that   in   either   event   "the   source   of   the   airwaves.    
reproduction   or   original   reproduced   shall   be   cited."   To    
give   this   section   any   other   construction   would   be   to   Respondents   brought   an   action   against   petitioners   in  
nullify,   eliminate   and   take   from   the   paragraph   the   words   Federal   District   Court,   alleging   that   VTR   consumers   had  
"they  contain  a  notice  that  their  publication  is  reserved,"   been  recording  some  of  respondents'  copyrighted  works  
and   to   say   that   the   Legislature   never   intended   to   say   that   had   been   exhibited   on   commercially   sponsored  
what   it   did   say.   This   court   must   construe   the   language   television   and   thereby   infringed   respondents'   copyrights,  
found  in  the  act.     and   further   that   petitioners   were   liable   for   such  
  copyright   infringement   because   of   their   marketing   of  
In  the  instant  case,  the  plaintiff  did  not  give  notice  of  its   the   VTR's.   Respondents   sought   money   damages,   an  
copyright,   for   the   simple   reason   that   it   did   not   have   a   equitable  accounting  of  profits,  and  an  injunction  against  
copyright,   but   it   did   notify   the   defendant   that   in   the   the  manufacture  and  marketing  of  the  VTR's.    

o 21
Katrina Michelle Mancao
 
  view  the  broadcast  so  that  it  can  be  watched  at  
The   District   Court   denied   respondents   all   relief,   holding   a  later  time);  and    
that   noncommercial   home   use   recording   of   material    
broadcast   over   the   public   airwaves   was   a   fair   use   of   (2)  that  there  is  no  likelihood  that  time-­‐shifting  
copyrighted   works   and   did   not   constitute   copyright   would  cause  nonminimal  harm  to  the  potential  
infringement,   and   that   petitioners   could   not   be   held   market   for,   or   the   value   of,   respondents'  
liable  as  contributory  infringers  even  if  the  home  use  of  a   copyrighted  works.    
VTR   was   considered   an   infringing   use.   The   Court   of    
Appeals   reversed,   holding   petitioners   liable   for   The   VTR's   are   therefore   capable   of   substantial  
contributory   infringement   and   ordering   the   District   noninfringing  uses.  Private,  noncommercial  time-­‐shifting  
Court  to  fashion  appropriate  relief.   in   the   home   satisfies   this   standard   of   noninfringing   uses  
  both   because   respondents   have   no   right   to   prevent   other  
Doctrine:   copyright  holders  from  authorizing  such  time-­‐shifting  for  
The   sale   of   the   VTR's   to   the   general   public   does   not   their  programs,  and  because  the  District  Court's  findings  
constitute   contributory   infringement   of   respondents'   reveal   that   even   the   unauthorized   home   time-­‐shifting   of  
copyrights.   respondents'  programs  is  legitimate  fair  use.  
   
The   protection   given   to   copyrights   is   wholly   statutory,   Remarks:  US  Case.  
and,  in  a  case  like  this,  in  which  Congress  has  not  plainly    
marked   the   course   to   be   followed   by   the   judiciary,   this   HARPER  &  ROW,  PUBLISHERS,  INC.  V.  NATION  ENTERPRISES  
Court   must   be   circumspect   in   construing   the   scope   of  
(1985)  
rights  created  by  a  statute  that  never  contemplated  such  
a   calculus   of   interests.   Any   individual   may   reproduce   a   Refresher:  
copyrighted  work  for  a  "fair  use";  the  copyright  owner   In   1977,   former   President   Ford   contracted   with  
does  not  possess  the  exclusive  right  to  such  a  use.     petitioners  to  publish  his  as  yet  unwritten  memoirs.  The  
  agreement  gave  petitioners  the  exclusive  first  serial  right  
Petitioner  is  not  a  “contributory  infringer.”  Kalem  Co.  v.   to   license   prepublication   excerpts.   Two   years   later,   as  
Harper   Brothers   does   not   support   respondents'   novel   the   memoirs   were   nearing   completion,   petitioners,   as  
theory   that   supplying   the   "means"   to   accomplish   an   the   copyright   holders,   negotiated   a   prepublication  
infringing  activity  and  encouraging  that  activity  through   licensing   agreement   with   Time   Magazine   under   which  
advertisement   are   sufficient   to   establish   liability   for   Time  agreed  to  pay  $25,000  ($12,500  in  advance  and  the  
copyright   infringement.   This   case   does   not   fall   in   the   balance   at   publication)   in   exchange   for   the   right   to  
category  of  those  in  which  it  is  manifestly  just  to  impose   excerpt   7,500   words   from   Mr.   Ford's   account   of   his  
vicarious   liability   because   the   "contributory"   infringer   pardon   of   former   President   Nixon.   Shortly   before   the  
was  in  a  position  to  control  the  use  of  copyrighted  works   Time  article's  scheduled  release,  an  unauthorized  source  
by   others   and   had   authorized   the   use   without   provided   The   Nation   Magazine   with   the   unpublished  
permission   from   the   copyright   owner.   Here,   the   only   Ford  manuscript.  Working  directly  from  this  manuscript,  
contact   between   petitioners   and   the   users   of   the   VTR's   an  editor  of  The  Nation  produced  a  2,250-­‐word  article,  at  
occurred   at   the   moment   of   sale.   And   there   is   no   least   300   to   400   words   of   which   consisted   of   verbatim  
precedent   for   imposing   vicarious   liability   on   the   theory   quotes   of   copyrighted   expression   taken   from   the  
that   petitioners   sold   the   VTR's   with   constructive   manuscript.  It  was  timed  to  "scoop"  the  Time  article.  As  
knowledge   that   their   customers   might   use   the   a   result   of   the   publication   of   The   Nation's   article,   Time  
equipment  to  make  unauthorized  copies  of  copyrighted   canceled   its   article   and   refused   to   pay   the   remaining  
material.  The   sale   of   copying   equipment,   like   the   sale   of   $12,500   to   petitioners.   Petitioners   then   brought   suit   in  
other   articles   of   commerce,   does   not   constitute   Federal   District   Court   against   respondent   publishers   of  
contributory  infringement  if  the  product  is  widely  used   The   Nation,   alleging,   inter   alia,   violations   of   the  
for   legitimate,   unobjectionable   purposes,   or,   indeed,   is   Copyright  Act  (Act).  The  District  Court  held   that   the   Ford  
merely  capable  of  substantial  noninfringing  uses.   memoirs   were   protected   by   copyright   at   the   time   of   The  
  Nation   publication   and   that   respondents'   use   of   the  
The  record  and  the  District  Court's  findings  show:     copyrighted   material   constituted   an   infringement   under  
  the   Act,   and   the   court   awarded   actual   damages   of  
(1)   that   there   is   a   significant   likelihood   that   $12,500.   The   Court   of   Appeals   reversed,   holding   that   The  
substantial   numbers   of   copyright   holders   who   Nation's  publication  of  the  300  to  400  words  it  identified  
license   their   works   for   broadcast   on   free   as   copyrightable   expression   was   sanctioned   as   a   "fair  
television   would   not   object   to   having   their   use"   of   the   copyrighted   material   under   107   of   the   Act.  
broadcast  time-­‐shifted  by  private  viewers  (i.  e.,   Section  107  provides  that  notwithstanding  the  provisions  
recorded  at  a  time  when  the  VTR  owner  cannot   of   106   giving   a   copyright   owner   the   exclusive   right   to  
reproduce   the   copyrighted   work   and   to   prepare  

22 z
Intellectual Property Law

derivative  works  based  on  the  copyrighted  work,  the  fair   separate   factor   tending   to   weigh   against   a  
use   of   a   copyrighted   work   for   purposes   such   as   finding   of   fair   use.   Fair   use   presupposes   good  
comment   and   news   reporting   is   not   an   infringement   of   faith.   The   Nation's   unauthorized   use   of   the  
copyright.   Section   107   further   provides   that   in   undisseminated   manuscript   had   not   merely   the  
determining   whether   the   use   was   fair   the   factors   to   be   incidental   effect   but   the   intended   purpose   of  
considered   shall   include:   (1)   the   purpose   and   character   supplanting   the   copyright   holders'  
of   the   use;   (2)   the   nature   of   the   copyrighted   work;   (3)   commercially  valuable  right  of  first  publication.    
the   substantiality   of   the   portion   used   in   relation   to   the    
copyrighted   work   as   a   whole;   and   (4)   the   effect   on   the   (ii)   While   there   may   be   a   greater   need   to  
potential  market  for  or  value  of  the  copyrighted  work.   disseminate   works   of   fact   than   works   of   fiction,  
  The   Nation's   taking   of   copyrighted   expression  
Doctrine:   exceeded   that   necessary   to   disseminate   the  
The   Nation's   article   was   not   a   "fair   use"   sanctioned   by   facts   and   infringed   the   copyright   holders'  
107.   interests  in  confidentiality  and  creative  control  
  over  the  first  public  appearance  of  the  work.    
In   using   generous   verbatim   excerpts   of   Mr.   Ford's    
unpublished   expression   to   lend   authenticity   to   its   (iii)   Although   the   verbatim   quotes   in   question  
account   of   the   forthcoming   memoirs,   The   Nation   were   an   insubstantial   portion   of   the   Ford  
effectively   arrogated   to   itself   the   right   of   first   manuscript,   they   qualitatively   embodied   Mr.  
publication,  an  important  marketable  subsidiary  right.   Ford's   distinctive   expression   and   played   a   key  
  role  in  the  infringing  article.    
Though   the   right   of   first   publication,   like   other   rights    
enumerated  in  106,  is  expressly  made  subject  to  the  fair   (iv)   As   to   the   effect   of   The   Nation's   article   on  
use   provisions   of   107,   fair   use   analysis   must   always   be   the   market   for   the   copyrighted   work,   Time's  
tailored   to   the   individual   case.   The   nature   of   the   interest   cancellation   of   its   projected   article   and   its  
at  stake  is  highly  relevant  to  whether  a  given  use  is  fair.   refusal  to  pay  $12,500  were  the  direct  effect  of  
The   unpublished   nature   of   a   work   is   a   key,   though   not   the   infringing   publication.   Once   a   copyright  
necessarily   determinative,   factor   tending   to   negate   a   holder   establishes   a   causal   connection  
defense   of   fair   use.   And   under   ordinary   circumstances,   between  the  infringement  and  loss  of  revenue,  
the   author's   right   to   control   the   first   public   appearance   the  burden  shifts  to  the  infringer  to  show  that  
of   his   undisseminated   expression   will   outweigh   a   claim   the   damage   would   have   occurred   had   there  
of  fair  use.   been  no  taking  of  copyrighted  expression.    
   
In  view  of  the  First  Amendment's  protections  embodied   Petitioners   established   a   prima   facie   case   of   actual  
in   the   Act's   distinction   between   copyrightable   damage   that   respondents   failed   to   rebut.   More  
expression  and  uncopyrightable  facts  and  ideas,  and  the   important,   to   negate   a   claim   of   fair   use   it   need   only   be  
latitude   for   scholarship   and   comment   traditionally   shown   that   if   the   challenged   use   should   become  
afforded   by   fair   use,   there   is   no   warrant   for   expanding,   widespread,   it   would   adversely   affect   the   potential  
as   respondents   contend   should   be   done,   the   fair   use   market   for   the   copyrighted   work.   Here,   The   Nation's  
doctrine  to  what  amounts  to  a  public  figure  exception  to   liberal   use   of   verbatim   excerpts   posed   substantial  
copyright.   Whether   verbatim   copying   from   a   public   potential   for   damage   to   the   marketability   of   first  
figure's   manuscript   in   a   given   case   is   or   is   not   fair   must   serialization  rights  in  the  copyrighted  work.  
be  judged  according  to  the  traditional  equities  of  fair  use.      
  Remarks:  US  Case  
Taking   into   account   the   FOUR   FACTORS   enumerated   in    
107  as  especially  relevant  in  determining  fair  use,  leads  to   ANN   BARTOW,   EDUCATIONAL   FAIR   USE   IN   COPYRIGHT:  
the  conclusion  that  the  use  in  question  here  was  not  fair.    
RECLAIMING  THE  RIGHT  TO  PHOTOCOPY  FREELY  (1998)  
 
(i)   The   fact   that   news   reporting   was   the   The   doctrine   of   fair   use   was   codified   in   §   107   of   the  
general   purpose   of   The   Nation's   use   is   simply   Copyright  Act  of  1976,  which  states  in  pertinent  part:  
one   factor.   While   The   Nation   had   every   right   to    
be  the  first  to  publish  the  information,  it  went   The  fair  use  of  a  copyrighted  work,  including  such  use  by  
beyond   simply   reporting   uncopyrightable   reproduction  in  copies  or  phonorecords  or  by  any  other  
information   and   actively   sought   to   exploit   the   means  specified  [in  §§  106  and  106A],  for  purposes  such  
headline   value   of   its   infringement,   making   a   as   criticism,   comment,   news   reporting,   teaching  
"news   event"   out   of   its   unauthorized   first   (including   multiple   copies   for   classroom   use),  
publication.   The   fact   that   the   publication   was   scholarship,   or   research,   is   not   an   infringement   of  
commercial   as   opposed   to   nonprofit   is   a   copyright.   In   determining   whether   the   use   made   of   a  

o 23
Katrina Michelle Mancao
 
work  in  any  particular  case  is  a  fair  use  the  factors  to  be   individual   copyright   owners.   Alternatively   expressed,  
considered  shall  include-­‐   "copyright   is   a   bargain   between   the   public   and  
  publishers,   in   which   the   public   consents   to   restrict   its  
(1)   the   purpose   and   character   of   the   use,   rights   as   a   kind   of   bribe   to   publishers."   To   effectuate  
including   whether   such   use   is   of   a   commercial   this   bargain,   copyrights   should   be   no   more   restrictive  
nature  or  is  for  nonprofit  educational  purposes;     than  is  necessary  to  create  incentives  for  the  promotion  
(2)  the  nature  of  the  copyrighted  work;     of   knowledge   and   learning.   Where   public   interests  
(3)   the   amount   and   substantiality   of   the   conflict   with   those   of   copyright   owners,   the   public  
portion   used   in   relation   to   the   copyrighted   interests  should  prevail.  
work  as  a  whole;  and      
(4)   the   effect   of   the   use   upon   the   potential   Fair  use  therefore  represents  an  amplitude  of  control  that  
market  for  or  value  of  the  copyrighted  work.   is   withheld   from   a   copyright   owner   when   a   grant   of  
  copyright   is   made,   rather   than   a   right   or   privilege  
The  omissions  from  §  107  are  immediately  apparent.  Fair   copyright   owners   earn   or   acquire   that   is   subsequently  
use   is   not   defined,   and   no   guidance   is   offered   with   "taken."   Fair   use   is   the   "only   limitation   on   copyright   that  
respect   to   an   ordering   of   priorities   in   the   application   of   applies   to   all   rights   of   the   copyright   owner   and   to   all  
the  four  articulated  factors  to  be  considered.   copyrighted  works."  
   
Fair  use  has  been  called  an  equitable  rule  of  reason  with   The  public's  statutorily  circumscribed  right  to  the  fair  use  
no  real  definition.  Fair  use  is,  however,  widely  recognized   of   copyrighted   works   can   be   justified   in   several   ways.  
as   a   doctrine   under   which   a   copyrighted   work   can   be   Fair   use   access   to   copyrighted   works   may   be   grounded  
used   without   permission   from,   or   even   against   the   in   recognition   that   no   newly   created,   copyrighted   work  
express   wishes   of   the   copyright   owner,   in   certain   can   be   truly   original.   All   authors   are   consciously   or  
circumstances.   Fair   use   doctrine   embodies   a   policy   unconsciously,   directly   or   indirectly   exposed   to,  
against  freighting  ideas  and  information  with  proprietary   informed   and   inspired   by   the   earlier   works   and   thoughts  
rights.   Although   one   of   the   bedrock   principles   of   of   others,   and   are   therefore   obligated   to   contribute  
copyright   law   is   that   only   the   expression   of   ideas   is   "free  samples"  from  their  own  works  to  the  marketplace  
copyrightable,   and   not   the   ideas   that   are   expressed,   as   a   of  ideas.  In  this  construction,  fair  use  functions  almost  as  
practical   matter   it   is   often   very   difficult   to   separate   the   a  tax  or  fee  that  is  levied  upon  individual  copyright  works  
two.   If,   for   example,   a   unique   idea   is   expressed   in   a   for   the   common   welfare,   and   justified   by   the   copyright  
highly   original,   copyrighted   book,   unless   one   is   prepared   owner's  consumption  of  common  or  public  knowledge.  
to   "re-­‐express"   the   idea,   thereby   rewriting   the   tome   in    
one's  own  words  (and  preferably  in  a  manner  that  does   Fair   use   may   also   be   perceived   as   a   tax   perhaps   paid   as  
not   induce   the   book's   author   to   sue   for   copyright   partial   reparation   for   the   costs   to   society   of   the  
infringement,   while   still   retaining   the   "essence"   of   "copyright   infrastructure"   comprised   of   the   Copyright  
whatever   makes   the   work   unique   and   valuable),   Office,   the   federal   court   system   (which   has   subject  
monopolistic  control  over  the  idea  contained  in  the  work   matter   jurisdiction   over   copyrights),   law   enforcement  
is   essentially   ceded   to   the   work's   copyright   owner.   agencies,   and   other   governmental   entities   dedicated   to  
Though  ideas  are  part  of  the  public  domain,  one  requires   delineating,   allocating,   and   protecting   copyrights.   Fair  
words   ("expression")   to   convey   them.   When   an   idea   is   use   thus   functions   as   a   burden   on   intellectual   property  
complex,  it  will  be  onerous,  if  not  impossible,  for  another   that   reciprocally   accompanies   the   benefits   provided   by  
person   to   communicate   using   other   words   without   the  copyright  infrastructure.  
slanting   or   shading   underlying   concepts.   Preservation    
and   accurate   transmission   of   an   elaborate   original   idea   Fair   use   can   also   be   viewed   as   somewhat   similar   to   an  
compel  use  of  intact,  original  expression.   easement.   The   recent   publisher-­‐propelled   rush   to  
  expand   the   scope   of   copyright   protections   has   been  
The   Copyright   Act   does   not   bestow   absolute   exclusive   aptly   analogized   to   "a   land   grab   the   likes   of   which   we  
rights   upon   a   copyright   owner,   but   rather   confers   a   have   not   seen   since   the   Government   opened   up   the  
limited  monopoly  with  respect  to  use  of  the  copyrighted   Northwest   Territory   for   settlement."   Fair   use   may   be  
work.   A   copyright   is   intangible   (or   "intellectual")   seen   as   society's   easement   across   "servient"   creative  
property   that   is   vested   with   a   public   interest,   intended   works.   Under   this   construction   authors   create   and  
to   achieve   an   "important   public   purpose."   There   is   a   publish   new   works   subject   to   the   burden   of   fair   use,   and  
societal   bargain   implicit   in   the   copyright   law.   Copyright   should  consider  the  possible  impact  of  the  doctrine  just  
owners   are   given   tools   in   the   form   of   exclusive   rights   as   a   landowner   would   factor   in   the   ramifications   of   an  
with  which  to  exploit  creative  endeavors  financially,  but   easement   when   deciding   whether   and   how   to   develop  
this   gift   is   conditioned   upon   an   understanding   that   the   land.   Unlike   the   typical   effect   of   an   easement   on   land,  
ultimate  goal  of  copyrights  is  to  maximize  the  number  of   however,   an   easement   doesn't   necessarily   burden   or  
creative  works  available  to  the  public,  and  not  to  benefit   devalue   intellectual   property.   A   book   can   be   equally  

24 z
Intellectual Property Law

valuable   to   each   in   a   series   of   new   readers   no   matter   doctrine  has  no  crisp  outlines,  no  precise  standards,  and  
how   many   times   it   is   read,   with   no   diminution   in   the   no  obvious  center  or  core.  Its  flexibility  allows  judges  to  
quality   of   the   work.   As   Thomas   Jefferson   stated:   "He   adjust   the   contours   of   copyright   protection   to   fit  
who   receives   an   idea   from   me,   receives   instructions   individual   circumstances   as   they   arise,   in   the   true  
himself   without   lessening   mine;   as   he   who   lights   his   common   law   tradition."   In   any   event,   though   §   107   has  
taper   at   mine,   receives   light   without   darkening   me."  Fair   not   been   legislatively   altered   with   respect   to   the   fair   use  
users   don't   erode   the   desirability   of   a   work,   and   may   of  published  works  since  it  went  into  effect  in  1978,  the  
actually   augment   royalty   generating   demand   for   it   by   scope   of   fair   use   that   it   delineates   has   been   steadily  
increasing  its  visibility  or  elevating  its  prominence.   shrinking.  
   
Fair   use   has   also   been   justified   as   a   response   to   market   SECTION  107'S  FOUR-­‐PART  TEST  
failure,   based   on   the   premise   that   the   public   ought   to    
have  free  access  to  information  and  ideas  manifested  in   Section   107   sets   out   a   non-­‐exhaustive   list   of   factors   a  
copyrighted   works   which   could   not   be   purchased   with   court   must   consider   when   deciding   whether   non-­‐
reasonable   ease   or   efficiency.   Under   this   notion   of   fair   permissive  utilization  of  a  copy-­‐righted  work  is  "fair  use."  
use,   the   availability   of   a   work   or   portion   thereof   has   an   In   the   context   of   educational   photocopying,   §   107's   four-­‐
inverse  relationship  with  the  prospective  scope  of  its  fair   part   test   was   intended   to   be   the   arbiter   of   fair   use   for  
usability.   The   scope   of   fair   use   would   therefore   be   photocopying.    
broader  with  respect  to  a  book  that  was  out  of  print,  and    
narrower   for   a   book   that   was   inexpensive   and   widely   1.   "The   purpose   and   character   of   the   use   including  
distributed.   Where   there   was   no   market   failure,   there   whether   such   use   is   of   a   commercial   nature   or   is   for  
could  be  no  fair  use.   nonprofit  educational  purposes:"    
  If   the   reproduction   is   made   for   profit,   with   commercial  
The   idea   that   there   was   a   fair   use   right   to   use   a   gain  as  its  primary  goal,  it  is  unlikely  to  fall  within  the  fair  
copyrighted   work   without   the   copyright   owner's   use  doctrine.  The  phrase  "including  whether  such  use  is  
permission   was   first   raised   in   1841   in   Folsom   v.   Marsh.   of   a   commercial   nature   or   is   for   nonprofit   educational  
The   plaintiffs   in   this   case   owned   a   copyright   in   a   purposes"   was   added   to   this   first   prong   of   §   107's   four-­‐
multivolume   autobiography   of   George   Washington,   part   fair   use   test   by   the   House   Judiciary   Committee  
which  defendants  were  accused  of  infringing  when  they   shortly   before   passage   of   the   Copyright   Act   of   1976   "in  
copied  353  pages  of  the  treatise.  Though  this  sounds  like   order   to   mollify   educators   who   had   lobbied  
an   appallingly   extensive   appropriation,   it   actually   unsuccessfully   for   an   across-­‐the-­‐board   exemption   for  
amounted   to   less   than   6%   of   the   original   lengthy   work.   nonprofit   educational   uses."   While   it   would   seem  
Defendant's   fair   use   claim   failed   because   the   court   obvious  that  the  purpose  and  character  of  photocopying  
concluded   that   the   most   important   parts   of   the   work   multiple   copies   for   classroom   use   is   educational   and  
had   been   copied   in   an   attempt   to   supplant   uses   (and   nonprofit   in   nature,   courts   have   held   that   educational  
presumably   purchases)   of   the   original   work.   In   Folsom,   copying   by   a   commercial   photocopy   business   was   a  
Justice   Story   articulated   the   underpinnings   of   the   commercial   undertaking   because   the   actual  
modern  test  of  fair  use,  subsequently  incorporated  into  §   photocopying   of   work   selected   by   educators   was  
107   of   the   Copyright   Act,   which   comprised   a   review   of   performed  by  for-­‐profit  vendors.  
"the   nature   and   objects   of   the   selections   made,   the    
quantity   and   value   of   the   materials   used,   and   the   degree   2.  "The  nature  of  the  copyrighted  work:"    
in  which  the  use  may  prejudice  the  sale,  or  diminish  the   In   applying   this   prong   of   the   fair   use   test,   courts   have  
profits,   or   supersede   the   objects,   of   the   original   work."   considered   the   originality,   informative   nature,   intended  
Justice   Story   was   most   concerned   about   the   market   use,   and   availability   of   the   work.   In   an   educational  
effect   of   the   accused   infringing   usage,   presaging   the   context,  courts  are  less  likely  to  find  fair  use  where  text  
extensive   (though   misdirected)   focus   by   modern   courts   books   and   other   materials   prepared   for   educational  
on   the   profitability   implications   of   nonpermissive   uses   markets  are  copied  than  when  works  created  for  general  
asserted  to  be  fair.   public  distribution  are  reproduced  for  classroom  use.  
   
It   has   been   argued   that   rather   than   being   "a   typical   Additionally,  in  part  because  facts  cannot  by  copyrighted,  
statutory   provision   representing   Congress's   creation   of   use   of   informational   works   is   more   likely   to   be  
new   law   and   establishment   of   rules   for   the   courts   to   considered   fair   than   use   of   creative   works.   Courts  
apply   [§   107]   is   instead   a   direction   to   the   courts   to   sometimes  have  difficulty  classifying  a  copyrighted  work  
continue  to  develop  the  common  law."  Under  this  view,   as   distinctly   informational   or   distinctly   creative.   For  
§  107  is  not  a  codification  of  the  doctrine  of  fair  use,  but   example,  on  one  occasion  the  Ninth  Circuit  decided  that  
rather   "statutory   recognition   of   a   judge-­‐made   rule   of   it  could  not  categorize  a  cake  decorating  booklet  which  
reason."  One  commentator  has  referred  to  fair  use  as  a   had   both   informational   and   creative   aspects.   As   one  
"paradigmatic   "catch-­‐all',"   and   asserted   that   "the   observer   uncharitably   put   it,   "If   cake   decorating  

o 25
Katrina Michelle Mancao
 
booklets  can  stump  the  Ninth  Circuit,  then  the  ability  of   eventually   became   the   working   draft   of   the   decision.      
courts   to   classify   more   complicated   materials   She   said   that,   for   most   parts,   she   did   her   research  
photocopied  by  educators  and  scholars  at  universities  is   electronically.     For   international   materials,   she   sourced  
doubtful."   these   mainly   from   Westlaw,   an   online   research   service  
  for   legal   and   law-­‐related   materials   to   which   the   Court  
3.  "The  amount  and  substantiality  of  the  portion  used  in   subscribes.    
relation  to  the  copyrighted  work  as  a  whole:"        
Photocopying   small   excerpts   of   a   copyrighted   work   for   In   the   old   days,   the   common   practice   was   that   after   a  
educational   purposes   is   more   likely   to   appear   fair   than   Justice   would   have   assigned   a   case   for   study   and   report,  
photocopying  large  portions  of  an  article  or  book.  Once   the   researcher   would   source   his   materials   mostly   from  
the   quantities   set   out   in   the   Guidelines   are   exceeded,   available   law   books   and   published   articles   on   print.    
there   is   no   bright   line   number   of   pages   or   percentage   of   When   he   found   a   relevant   item   in   a   book,   whether   for  
a   work   one   can   confidently   assume   will   be   deemed   fair   one  side  of  the  issue  or  for  the  other,  he  would  place  a  
or  unfair.  If  the  amount  of  copying  is  substantial  enough,   strip   of   paper   marker   on   the   appropriate   page,   pencil  
some   courts   will   not   find   fair   use   regardless   of   the   mark   the   item,   and   place   the   book   on   his   desk   where  
outcome  of  the  other  test  factors.   other   relevant   books   would   have   piled   up.     He   would  
  later  paraphrase  or  copy  the  marked  out  passages  from  
In   addition   to   doing   a   quantitative   assessment,   courts   some   of   these   books   as   he   typed   his   manuscript   on   a  
will  also  do  a  qualitative  assessment  of  the  portion  of  the   manual  typewriter.    This  occasion  would  give  him  a  clear  
work  that  was  reproduced.  Photocopying  the  "heart"  or   opportunity   to   attribute   the   materials   used   to   their  
critical   parts   of   a   work   may   be   deemed   unfair   even   if   the   authors  or  sources.    
number   of   pages   or   actual   percentage   of   the   work      
copied   is   insubstantial.   When   a   work   is   excerpted   for   With   the   advent   of   computers,   however,   as   Justice   Del  
educational  usage,  the  very  fact  that  certain  portions  of   Castillo’s   researcher   also   explained,   most   legal  
a  work  are  chosen  by  a  faculty  member  will  probably  be   references,   including   the   collection   of   decisions   of   the  
sufficient  to  implicate  them  as  "critical  parts,"  which  will   Court,   are   found   in   electronic   diskettes   or   in   internet  
weigh  against  a  finding  of  fair  use.   websites   that   offer   virtual   libraries   of   books   and   articles.    
  Here,   as   the   researcher   found   items   that   were   relevant  
4.   "The   effect   of   the   use   upon   the   potential   market   for   or   to  her  assignment,  she  downloaded  or  copied  them  into  
value  of  the  copyrighted  work:"     her   “main   manuscript,”   a   smorgasbord   plate   of  
This   factor   echoes   aspects   of   the   second   and   third   materials   that   she   thought   she   might   need.     The  
factors,  as  both  the  nature  of  the  work  and  the  amount   researcher’s  technique  in  this  case  is  not  too  far  different  
and   substantiality   copied   will   be   partially   determinative   from  that  employed  by  a  carpenter.    The  carpenter  first  
of   whether   a   copyright   owner   has   been   economically   gets   the   pieces   of   lumber   he   would   need,   choosing   the  
injured.  Market  effect  has  been  called  "undoubtedly  the   kinds  and  sizes  suitable  to  the  object  he  has  in  mind,  say  
single   most   important   element   of   fair   use"   by   the   a  table.    When  ready,  he  would  measure  out  the  portions  
Supreme   Court   in   a   non-­‐educational   context.   However,   he   needs,   cut   them   out   of   the   pieces   of   lumber   he   had  
the   exact   extent   of   its   preeminence   among   §   107's   four   collected,   and   construct   his   table.     He   would   get   rid   of  
factors  has  not  been  delineated.   the  scraps.      
     
IN   THE   MATTER   OF   THE   CHARGES   OF   PLAGIARISM,   ETC.   Here,  Justice  Del  Castillo’s  researcher  did  just  that.    She  
electronically   “cut”   relevant   materials   from   books   and  
AGAINST   ASSOCIATE   JUSTICE   MARIANO   C.   DEL   CASTILLO  
journals   in   the   Westlaw   website   and   “pasted”   these   to   a  
(2010)   “main   manuscript”   in   her   computer   that   contained   the  
Court  decision:   issues   for   discussion   in   her   proposed   report   to   the  
Petitioners   point   out   that   the   Vinuya   decision   lifted   Justice.     She   used   the   Microsoft   Word   program.   Later,  
passages   from   Tams’   book,   Enforcing   Erga   Omnes   after   she   decided   on   the   general   shape   that   her   report  
Obligations  in  International  Law  (2006)  and  used  them  in   would   take,   she   began   pruning   from   that   manuscript  
Footnote   69   with   what   the   author   thought   was   a   mere   those  materials  that  did  not  fit,  changing  the  positions  in  
generic  reference.    But,  although  Tams  himself  may  have   the  general  scheme  of  those  that  remained,  and  adding  
believed   that   the   footnoting   in   this   case   was   not   “an   and   deleting   paragraphs,   sentences,   and   words   as   her  
appropriate   form   of   referencing,”   Unless   amply   continuing  discussions  with  Justice  Del  Castillo,  her  chief  
explained,   the   above   lifting   from   the   works   of   Ellis   and   editor,   demanded.     Parenthetically,   this   is   the   standard  
Criddle-­‐Descent   could   be   construed   as   plagiarism.     But   scheme   that   computer-­‐literate   court   researchers   use  
one   of   Justice   Del   Castillo’s   researchers,   a   court-­‐ everyday  in  their  work.  
employed   attorney,   explained   how   she   accidentally      
deleted   the   attributions,   originally   planted   in   the   Justice   Del   Castillo’s   researcher   showed   the   Committee  
beginning   drafts   of   her   report   to   him,   which   report   the   early   drafts   of   her   report   in   the   Vinuya   case   and  

26 z
Intellectual Property Law

these   included   the   passages   lifted   from   the   separate   standards   on   plagiarism   in   the   academe   should   apply  
articles   of   Criddle-­‐Descent   and   of   Ellis   with   proper   with  more  force  to  the  judiciary.        
attributions   to   these   authors.     But,   as   it   happened,   in   the      
course   of   editing   and   cleaning   up   her   draft,   the   But  petitioners’  theory  ignores  the  fact  that  plagiarism  is  
researcher   accidentally   deleted   the   attributions.     She   and   essentially   a   form   of   fraud   where   intent   to   deceive   is  
petitioners   cannot   deny   that   the   decision   did   attribute   inherent.     Their   theory   provides   no   room   for   errors   in  
the   source   or   sources   of   such   passages.     Justice   Del   research,   an   unrealistic   position   considering   that   there   is  
Castillo   did   not   pass   off   Tams’   work   as   his   own.     The   hardly   any   substantial   written   work   in   any   field   of  
Justice   primarily   attributed   the   ideas   embodied   in   the   discipline  that  is  free  of  any  mistake.    The  theory  places  
passages  to  Bruno  Simma,  whom  Tams  himself  credited   an   automatic   universal   curse   even   on   errors   that,   as   in  
for   them.     Still,   Footnote   69   mentioned,   apart   from   this  case,  have  reasonable  and  logical  explanations.  
Simma,  Tams’  article  as  another  source  of  those  ideas.        
    Indeed,   the   8th   edition   of   Black’s   Law   Dictionary   defines  
The   Court   believes   that   whether   or   not   the   footnote   is   plagiarism   as   the   “deliberate   and   knowing   presentation  
sufficiently   detailed,   so   as   to   satisfy   the   footnoting   of  another  person's  original  ideas  or  creative  expressions  
standards   of   counsel   for   petitioners   is   not   an   ethical   as  one's  own.”  Thus,  plagiarism  presupposes  intent  and  
matter   but   one   concerning   clarity   of   writing.     The   a   deliberate,   conscious   effort   to   steal   another’s   work  
statement  “See  Tams,  Enforcing  Obligations  Erga  Omnes   and  pass  it  off  as  one’s  own.    
in  International  Law  (2005)”  in  the  Vinuya  decision  is  an      
attribution   no   matter   if   Tams   thought   that   it   gave   him   Besides,  the  Court  said  nothing  in  U.P.  Board  of  Regents  
somewhat  less  credit  than  he  deserved.    Such  attribution   that   would   indicate   that   an   intent   to   pass   off   another’s  
altogether   negates   the   idea   that   Justice   Del   Castillo   work   as   one’s   own   is   not   required   in   plagiarism.     The  
passed  off  the  challenged  passages  as  his  own.         Court   merely   affirmed   the   academic   freedom   of   a  
    university  to  withdraw  a  master’s  degree  that  a  student  
That   it   would   have   been   better   had   Justice   Del   Castillo   obtained   based   on   evidence   that   she   misappropriated  
used   the   introductory   phrase   “cited   in”   rather   than   the   the  work  of  others,  passing  them  off  as  her  own.    This  is  
phrase   “See”   would   make   a   case   of   mere   inadvertent   not   the   case   here   since,   as   already   stated,   Justice   Del  
slip   in   attribution   rather   than   a   case   of   “manifest   Castillo   actually   imputed   the   borrowed   passages   to  
intellectual   theft   and   outright   plagiarism.”   If   the   others.  
Justice’s   citations   were   imprecise,   it   would   just   be   a   case    
of  bad  footnoting  rather  than  one  of  theft  or  deceit.    If  it   IN   THE   MATTER   OF   THE   CHARGES   OF   PLAGIARISM,   ETC.  
were   otherwise,   many   would   be   target   of   abuse   for  
AGAINST   ASSOCIATE   JUSTICE   MARIANO   C.   DEL   CASTILLO  
every   editorial   error,   for   every   mistake   in   citing  
pagination,  and  for  every  technical  detail  of  form.   (2011)  
  Court  decision:  
Notably,   neither   Justice   Del   Castillo   nor   his   researcher   Mainly,   petitioners   claim   that   the   Court   has   by   its  
had   a   motive   or   reason   for   omitting   attribution   for   the   decision   legalized   or   approved   of   the   commission   of  
lifted   passages   to   Criddle-­‐Descent   or   to   Ellis.     The   latter   plagiarism   in   the   Philippines.     This   claim   is   absurd.     The  
authors  are  highly  respected  professors  of  international   Court,   like   everyone   else,   condemns   plagiarism   as   the  
law.     The   law   journals   that   published   their   works   have   world  in  general  understands  and  uses  the  term.    
exceptional   reputations.     It   did   not   make   sense   to      
intentionally  omit  attribution  to  these  authors  when  the   Plagiarism,  a  term  not  defined  by  statute,  has  a  popular  
decision   cites   an   abundance   of   other   sources.     Citing   or  common  definition.    To  plagiarize,  says  Webster,  is  “to  
these   authors   as   the   sources   of   the   lifted   passages   steal   and   pass   off   as   one’s   own”   the   ideas   or   words   of  
would   enhance   rather   than   diminish   their   informative   another.     Stealing   implies   malicious   taking.     Black’s   Law  
value.     Both   Justice   Del   Castillo   and   his   researcher   gain   Dictionary,   the   world’s   leading   English   law   dictionary  
nothing  from  the  omission.    Thus,  the  failure  to  mention   quoted  by  the  Court  in  its  decision,  defines  plagiarism  as  
the   works   of   Criddle-­‐Decent   and   Ellis   was   the   “deliberate   and   knowing   presentation   of   another  
unquestionably  due  to  inadvertence  or  pure  oversight.   person's   original   ideas   or   creative   expressions   as   one’s  
    own.”   The   presentation   of   another   person’s   ideas   as  
Petitioners   of   course   insist   that   intent   is   not   material   in   one’s  own  must  be  deliberate  or  premeditated—a  taking  
committing  plagiarism  since  all  that  a  writer  has  to  do,  to   with  ill  intent.  
avoid   the   charge,   is   to   enclose   lifted   portions   with      
quotation   marks   and   acknowledge   the   sources   from   There   is   no   commonly-­‐used   dictionary   in   the   world   that  
which   these   were   taken.   Petitioners   point   out   that   the   embraces   in   the   meaning   of   plagiarism   errors   in  
Court   should   apply   to   this   case   the   ruling   in   University   of   attribution  by  mere  accident  or  in  good  faith.    
the  Philippines  Board  of  Regents  v.  Court  of  Appeals  and      
Arokiaswamy   William   Margaret   Celine.   They   argue   that   Certain   educational   institutions   of   course   assume  
different   norms   in   its   application.     For   instance,   the  

o 27
Katrina Michelle Mancao
 
Loyola   Schools   Code   of   Academic   Integrity   ordains   that   not  that  they  are  originally  crafted  but  that  they  are  fair  
“plagiarism   is   identified   not   through   intent   but   through   and   correct   in   the   context   of   the   particular   disputes  
the   act   itself.     The   objective   act   of   falsely   attributing   to   involved.      Justice,  not  originality,  form,  and  style,  is  the  
one’s   self   what   is   not   one’s   work,   whether   intentional   or   object  of  every  decision  of  a  court  of  law.    
out   of   neglect,   is   sufficient   to   conclude   that   plagiarism      
has   occurred.     Students   who   plead   ignorance   or   appeal   There  is  a  basic  reason  for  individual  judges  of  whatever  
to  lack  of  malice  are  not  excused.”   level  of  courts,  including  the  Supreme  Court,  not  to  use  
    original   or   unique   language   when   reinstating   the   laws  
But   the   Court’s   decision   in   the   present   case   does   not   set   involved  in  the  cases  they  decide.    Their  duty  is  to  apply  
aside  such  norm.    The  decision  makes  this  clear,  thus:   the  laws  as  these  are  written.    But  laws  include,  under  the  
    doctrine   of   stare   decisis,   judicial   interpretations   of   such  
To   paraphrase   Bast   and   Samuels,   while   the   laws   as   are   applied   to   specific   situations.     Under   this  
academic   publishing   model   is   based   on   the   doctrine,   Courts   are   “to   stand   by   precedent   and   not   to  
originality   of   the   writer’s   thesis,   the   judicial   disturb  settled  point.”  Once  the  Court  has  “laid  down  a  
system  is  based  on  the  doctrine  of  stare  decisis,   principle  of  law  as  applicable  to  a  certain  state  of  facts,  it  
which   encourages   courts   to   cite   historical   legal   will   adhere   to   that   principle,   and   apply   it   to   all   future  
data,   precedents,   and   related   studies   in   their   cases,   where   facts   are   substantially   the   same;   regardless  
decisions.     The   judge   is   not   expected   to   of  whether  the  parties  or  property  are  the  same.”  
produce   original   scholarship   in   every   respect.        
The  strength  of  a  decision  lies  in  the  soundness   And   because   judicial   precedents   are   not   always   clearly  
and  general  acceptance  of  the  precedents  and   delineated,   they   are   quite   often   entangled   in   apparent  
long  held  legal  opinions  it  draws  from.   inconsistencies   or   even   in   contradictions,   prompting  
  experts   in   the   law   to   build   up   regarding   such   matters   a  
Original  scholarship  is  highly  valued  in  the  academe  and   large   body   of   commentaries   or   annotations   that,   in  
rightly  so.    A  college  thesis,  for  instance,  should  contain   themselves,   often   become   part   of   legal   writings   upon  
dissertations   embodying   results   of   original   research,   which   lawyers   and   judges   draw   materials   for   their  
substantiating  a  specific  view.  This  must  be  so  since  the   theories  or  solutions  in  particular  cases.    And,  because  of  
writing   is   intended   to   earn   for   the   student   an   academic   the   need   to   be   precise   and   correct,   judges   and  
degree,   honor,   or   distinction.     He   earns   no   credit   nor   practitioners   alike,   by   practice   and   tradition,   usually   lift  
deserves   it   who   takes   the   research   of   others,   copies   passages   from   such   precedents   and   writings,   at   times  
their   dissertations,   and   proclaims   these   as   his   own.     omitting,   without   malicious   intent,   attributions   to   the  
There   should   be   no   question   that   a   cheat   deserves   originators.  
neither  reward  nor  sympathy.    
    Is   this   dishonest?     No.     Duncan   Webb,   writing   for   the  
But   the   policy   adopted   by   schools   of   disregarding   the   International   Bar   Association   puts   it   succinctly.     When  
element   of   malicious   intent   found   in   dictionaries   is   practicing  lawyers  (which  include  judges)  write  about  the  
evidently   more   in   the   nature   of   establishing   what   law,  they  effectively  place  their  ideas,  their  language,  and  
evidence   is   sufficient   to   prove   the   commission   of   such   their   work   in   the   public   domain,   to   be   affirmed,   adopted,  
dishonest   conduct   than   in   rewriting   the   meaning   of   criticized,   or   rejected.     Being   in   the   public   domain,   other  
plagiarism.        Since  it  would  be  easy  enough  for  a  student   lawyers   can   thus   freely   use   these   without   fear   of  
to   plead   ignorance   or   lack   of   malice   even   as   he   has   committing  some  wrong  or  incurring  some  liability.    Thus:  
copied   the   work   of   others,   certain   schools   have   adopted      
the  policy  of  treating  the  mere  presence  of  such  copied   The   tendency   to   copy   in   law   is   readily  
work   in   his   paper   sufficient   objective   evidence   of   explicable.     In   law   accuracy   of   words   is  
plagiarism.    Surely,  however,  if  on  its  face  the  student’s   everything.    Legal  disputes  often  centre  round  
work   shows   as   a   whole   that   he   has   but   committed   an   the   way   in   which   obligations   have   been  
obvious  mistake  or  a  clerical  error  in  one  of  hundreds  of   expressed   in   legal   documents   and   how   the  
citations   in   his   thesis,   the   school   will   not   be   so   facts   of   the   real   world   fit   the   meaning   of   the  
unreasonable  as  to  cancel  his  diploma.   words   in   which   the   obligation   is   contained.    
    This,   in   conjunction   with   the   risk-­‐aversion   of  
In   contrast,   decisions   of   courts   are   not   written   to   earn   lawyers  means  that  refuge  will  often  be  sought  
merit,   accolade,   or   prize   as   an   original   piece   of   work   or   in  articulations  that  have  been  tried  and  tested.    
art.     Deciding   disputes   is   a   service   rendered   by   the   In  a  sense  therefore  the  community  of  lawyers  
government  for  the  public  good.    Judges  issue  decisions   have   together   contributed   to   this   body   of  
to   resolve   everyday   conflicts   involving   people   of   flesh   knowledge,   language,   and   expression   which   is  
and   blood   who  ache  for  speedy  justice  or  juridical  beings   common   property   and   may   be   utilized,  
which  have  rights  and  obligations  in  law  that  need  to  be   developed  and  bettered  by  anyone.  
protected.    The  interest  of  society  in  written  decisions  is      

28 z
Intellectual Property Law

The   implicit   right   of   judges   to   use   legal   materials   cited   in   the   charges   against   him.     He   compared   the  
regarded  as  belonging  to  the  public  domain  is  not  unique   divergent   views   these   present   as   they   developed   in  
to   the   Philippines.     As   Joyce   C.   George,   whom   Justice   history.     He   then   explained   why   the   Court   must   reject  
Maria   Lourdes   Sereno   cites   in   her   dissenting   opinion,   some  views  in  light  of  the  peculiar  facts  of  the  case  and  
observed  in  her  Judicial  Opinion  Writing  Handbook:   applied  those  that  suit  such  facts.  Finally,  he  drew  from  
    his  discussions  of  the  facts  and  the  law  the  right  solution  
A   judge   writing   to   resolve   a   dispute,   whether   to  the  dispute  in  the  case.    On  the  whole,  his  work  was  
trial  or  appellate,  is  exempted  from  a  charge  of   original.    He  had  but  done  an  honest  work.        
plagiarism  even  if  ideas,  words  or  phrases  from      
a   law   review   article,   novel   thoughts   published   The  Court  will  not,  therefore,  consistent  with  established  
in  a  legal  periodical  or  language  from  a  party’s   practice   in   the   Philippines   and   elsewhere,   dare   permit  
brief  are  used  without  giving  attribution.    Thus   the   filing   of   actions   to   annul   the   decisions   promulgated  
judges   are   free   to   use   whatever   sources   they   by  its  judges  or  expose  them  to  charges  of  plagiarism  for  
deem   appropriate   to   resolve   the   matter   before   honest  work  done.  
them,  without  fear  of  reprisal.    This  exemption      
applies   to   judicial   writings   intended   to   decide   This   rule   should   apply   to   practicing   lawyers   as   well.    
cases  for  two  reasons:  the  judge  is  not  writing   Counsels   for   the   petitioners,   like   all   lawyers   handling  
a   literary   work   and,   more   importantly,   the   cases  before  courts  and  administrative  tribunals,  cannot  
purpose   of   the   writing   is   to   resolve   a   dispute.     object   to   this.     Although   as   a   rule   they   receive  
As   a   result,   judges   adjudicating   cases   are   not   compensation   for   every   pleading   or   paper   they   file   in  
subject  to  a  claim  of  legal  plagiarism.   court  or  for  every  opinion  they  render  to  clients,  lawyers  
    also  need  to  strive  for  technical  accuracy  in  their  writings.    
If   the   Court   were   to   inquire   into   the   issue   of   plagiarism   They   should   not   be   exposed   to   charges   of   plagiarism   in  
respecting   its   past   decisions   from   the   time   of   Chief   what   they   write   so   long   as   they   do   not   depart,   as   officers  
Justice  Cayetano  S.  Arellano  to  the  present,  it  is  likely  to   of   the   court,   from   the   objective   of   assisting   the   Court   in  
discover   that   it   has   not   on   occasion   acknowledged   the   the  administration  of  justice.  
originators  of  passages  and  views  found  in  its  decisions.        
These  omissions  are  true  for  many  of  the  decisions  that   As  Duncan  Webb  said:  
have   been   penned   and   are   being   penned   daily   by      
magistrates   from   the   Court   of   Appeals,   the   In  presenting  legal  argument  most  lawyers  will  
Sandiganbayan,   the   Court   of   Tax   Appeals,   the   Regional   have   recourse   to   either   previous   decisions   of  
Trial   Courts   nationwide   and   with   them,   the   municipal   the  courts,  frequently  lifting  whole  sections  of  
trial   courts   and   other   first   level   courts.     Never   in   the   a   judge’s   words   to   lend   weight   to   a   particular  
judiciary’s  more  than  100  years  of  history  has  the  lack  of   point   either   with   or   without   attribution.     The  
attribution  been  regarded  and  demeaned  as  plagiarism.     words   of   scholars   are   also   sometimes   given  
    weight,   depending   on   reputation.     Some  
This  is  not  to  say  that  the  magistrates  of  our  courts  are   encyclopaedic   works   are   given   particular  
mere   copycats.     They   are   not.     Their   decisions   analyze   authority.     In   England   this   place   is   given   to  
the  often  conflicting  facts  of  each  case  and  sort  out  the   Halsbury’s   Laws   of   England   which   is   widely  
relevant  from  the  irrelevant.    They  identify  and  formulate   considered  authoritative.    A  lawyer  can  do  little  
the  issue  or  issues  that  need  to  be  resolved  and  evaluate   better   than   to   frame   an   argument   or   claim   to  
each   of   the   laws,   rulings,   principles,   or   authorities   that   fit  with  the  articulation  of  the  law  in  Halsbury’s.    
the  parties  to  the  case  invoke.    The  decisions  then  draw   While   in   many   cases   the   very   purpose   of   the  
their  apt  conclusions  regarding  whether  or  not  such  laws,   citation  is  to  claim  the  authority  of  the  author,  
rulings,   principles,   or   authorities   apply   to   the   particular   this   is   not   always   the   case.     Frequently  
cases  before  the  Court.    These  efforts,  reduced  in  writing,   commentary  or  dicta  of  lesser  standing  will  be  
are   the   product   of   the   judges’   creativity.     It   is   here— adopted   by   legal   authors,   largely   without  
actually   the   substance   of   their   decisions—that   their   attribution.  
genius,   originality,   and   honest   labor   can   be   found,   of      
which  they  should  be  proud.   x  x  x  x  
       
In  Vinuya,  Justice  Del  Castillo  examined  and  summarized   The  converse  point  is  that  originality  in  the  law  
the  facts  as  seen  by  the  opposing  sides  in  a  way  that  no   is   viewed   with   skepticism.     It   is   only   the  
one   has   ever   done.     He   identified   and   formulated   the   arrogant  fool  or  the  truly  gifted  who  will  depart  
core  of  the  issues  that  the  parties  raised.    And  when  he   entirely   from   the   established   template   and  
had   done   this,   he   discussed   the   state   of   the   law   relevant   reformulate  an  existing  idea  in  the  belief  that  in  
to   their   resolution.     It   was   here   that   he   drew   materials   doing  so  they  will  improve  it.    While  over  time  
from  various  sources,  including  the  three  foreign  authors   incremental   changes   occur,   the   wholesale  

o 29
Katrina Michelle Mancao
 
abandonment   of   established   expression   is   8. Transfer  of  Copyright  
generally  considered  foolhardy.    
   
SECTION  180.  RIGHTS  OF  ASSIGNEE.  –    
The   Court   probably   should   not   have   entertained   at   all  
the   charges   of   plagiarism   against   Justice   Del   Castillo,   180.1.  The  copyright  may  be  assigned  in  whole  or  in  part.  
coming   from   the   losing   party.     But   it   is   a   case   of   first   Within   the   scope   of   the   assignment,   the   assignee   is  
impression   and   petitioners,   joined   by   some   faculty   entitled  to  all  the  rights  and  remedies  which  the  assignor  
members   of   the   University   of   the   Philippines   school   of   had  with  respect  to  the  copyright.  
law,   have   unfairly   maligned   him   with   the   charges   of    
plagiarism,  twisting  of  cited  materials,  and  gross  neglect   180.2.   The   copyright   is   not   deemed   assigned   inter   vivos  
for  failing  to  attribute  lifted  passages  from  three  foreign   in  whole  or  in  part  unless  there  is  a  written  indication  of  
authors.     These   charges   as   already   stated   are   false,   such  intention.  
applying   the   meaning   of   plagiarism   as   the   world   in    
general  knows  it.     180.3.   The   submission   of   a   literary,   photographic   or  
    artistic  work  to  a  newspaper,  magazine  or  periodical  for  
True,  Justice  Del  Castillo  failed  to  attribute  to  the  foreign   publication   shall   constitute   only   a   license   to   make   a  
authors   materials   that   he   lifted   from   their   works   and   single   publication   unless   a   greater   right   is   expressly  
used   in   writing   the   decision   for   the   Court   in   the   Vinuya   granted.   If   two   (2)   or   more   persons   jointly   own   a  
case.    But,  as  the  Court  said,  the  evidence  as  found  by  its   copyright  or  any  part  thereof,  neither  of  the  owners  shall  
Ethics   Committee   shows   that   the   attribution   to   these   be   entitled   to   grant   licenses   without   the   prior   written  
authors  appeared  in  the  beginning  drafts  of  the  decision.     consent  of  the  other  owner  or  owners.  
Unfortunately,   as   testified   to   by   a   highly   qualified   and    
experienced  court-­‐employed  researcher,  she  accidentally   SECTION  181.  COPYRIGHT  AND  MATERIAL  OBJECT.  –    
deleted   the   same   at   the   time   she   was   cleaning   up   the   The   copyright   is   distinct   from   the   property   in   the  
final   draft.     The   Court   believed   her   since,   among   other   material   object   subject   to   it.   Consequently,   the   transfer  
reasons,  she  had  no  motive  for  omitting  the  attribution.     or  assignment  of  the  copyright  shall  not  itself  constitute  
The  foreign  authors  concerned,  like  the  dozens  of  other   a   transfer   of   the   material   object.   Nor   shall   a   transfer   or  
sources  she  cited  in  her  research,  had  high  reputations  in   assignment  of  the  sole  copy  or  of  one  or  several  copies  
international  law.   of   the   work   imply   transfer   or   assignment   of   the  
    copyright.    
Notably,   those   foreign   authors   expressly   attributed   the    
controversial   passages   found   in   their   works   to   earlier  
SECTION  182.  FILING  OF  ASSIGNMENT  OR  LICENSE.  –    
writings   by   others.     The   authors   concerned   were   not  
themselves   the   originators.     As   it   happened,   although   An   assignment   or   exclusive   license   may   be   filed   in  
the   ponencia   of   Justice   Del   Castillo   accidentally   deleted   duplicate  with  the  National  Library  upon  payment  of  the  
the   attribution   to   them,   there   remained   in   the   final   draft   prescribed   fee   for   registration   in   books   and   records   kept  
of  the  decision  attributions  of  the  same  passages  to  the   for   the   purpose.   Upon   recording,   a   copy   of   the  
earlier   writings   from   which   those   authors   borrowed   instrument   shall   be   returned   to   the   sender   with   a  
their  ideas  in  the  first  place.    In  short,  with  the  remaining   notation  of  the  fact  of  record.  Notice  of  the  record  shall  
attributions   after   the   erroneous   clean-­‐up,   the   passages   be  published  in  the  IPO  Gazette.    
as  it  finally  appeared  in  the  Vinuya  decision  still  showed    
on   their   face   that   the   lifted   ideas   did   not   belong   to   SECTION  183.  DESIGNATION  OF  SOCIETY.  –    
Justice  Del  Castillo  but  to  others.    He  did  not  pass  them   The   copyright   owners   or   their   heirs   may   designate   a  
off  as  his  own.       society   of   artists,   writers   or   composers   to   enforce   their  
    economic  rights  and  moral  rights  on  their  behalf.    
With   our   ruling,   the   Court   need   not   dwell   long   on    
petitioners’   allegations   that   Justice   Del   Castillo   had   also   SECTION  4.2.    
committed   plagiarism   in   writing   for   the   Court   his   The   term   "TECHNOLOGY   TRANSFER   ARRANGEMENTS"   refers   to  
decision   in   another   case,   Ang   Ladlad   v.   Commission   on  
contracts   or   agreements   involving   the   transfer   of  
Elections.   Petitioners   are   nit-­‐picking.     Upon   close  
systematic   knowledge   for   the   manufacture   of   a   product,  
examination   and   as   Justice   Del   Castillo   amply  
the   application   of   a   process,   or   rendering   of   a   service  
demonstrated   in   his   comment   to   the   motion   for  
including   management   contracts;   and   the   transfer,  
reconsideration,  he  in  fact  made  attributions  to  passages  
assignment   or   licensing   of   all   forms   of   intellectual  
in   such   decision   that   he   borrowed   from   his   sources  
property   rights,   including   licensing   of   computer  
although  they  at  times  suffered  in  formatting  lapses.  
software   except   computer   software   developed   for   mass  
 
market.  
 

30 z
Intellectual Property Law

SECTION  87.  PROHIBITED  CLAUSES.  –     conditions   or   to   initiate   research   and   development  


Except   in   cases   under   Section   91,   the   following   programs  in  connection  with  new  products,  processes  or  
provisions   shall   be   deemed   prima   facie   to   have   an   equipment;  
adverse  effect  on  competition  and  trade:    
  87.13.   Those   which   prevent   the   licensee   from  
87.1.   Those   which   impose   upon   the   licensee   the   adapting   the   imported   technology   to   local   conditions,   or  
obligation  to  acquire  from  a  specific  source  capital  goods,   introducing   innovation   to   it,   as   long   as   it   does   not   impair  
intermediate   products,   raw   materials,   and   other   the  quality  standards  prescribed  by  the  licensor;  
technologies,   or   of   permanently   employing   personnel    
indicated  by  the  licensor;   87.14.   Those  which  exempt  the  licensor  for  liability  for  
  non-­‐fulfilment   of   his   responsibilities   under   the  
87.2.   Those   pursuant   to   which   the   licensor   reserves   the   technology   transfer   arrangement   and/or   liability   arising  
right   to   fix   the   sale   or   resale   prices   of   the   products   from   third   party   suits   brought   about   by   the   use   of   the  
manufactured  on  the  basis  of  the  license;   licensed  product  or  the  licensed  technology;  and  
   
87.3.   Those   that   contain   restrictions   regarding   the   87.15.   Other  clauses  with  equivalent  effects.    
volume  and  structure  of  production;    
  SECTION  88.  MANDATORY  PROVISIONS.  –    
87.4.   Those   that   prohibit   the   use   of   competitive   The   following   provisions   shall   be   included   in   voluntary  
technologies   in   a   non-­‐exclusive   technology   transfer   license  contracts:  
agreement;    
  88.1.   That   the   laws   of   the   Philippines   shall   govern   the  
87.5.   Those   that   establish   a   full   or   partial   purchase   interpretation  of  the  same  and  in  the  event  of  litigation,  
option  in  favor  of  the  licensor;   the   venue   shall   be   the   proper   court   in   the   place   where  
  the  licensee  has  its  principal  office;  
87.6.  Those  that  obligate  the  licensee  to  transfer  for  free    
to   the   licensor   the   inventions   or   improvements   that   may   88.2.   Continued   access   to   improvements   in   techniques  
be  obtained  through  the  use  of  the  licensed  technology;   and   processes   related   to   the   technology   shall   be   made  
  available   during   the   period   of   the   technology   transfer  
87.7.   Those   that   require   payment   of   royalties   to   the   arrangement;  
owners  of  patents  for  patents  which  are  not  used;    
  88.3.   In   the   event   the   technology   transfer   arrangement  
87.8.   Those   that   prohibit   the   licensee   to   export   the   shall  provide  for  arbitration,  the  Procedure  of  Arbitration  
licensed  product  unless  justified  for  the  protection  of  the   of   the   Arbitration   Law   of   the   Philippines   or   the  
legitimate   interest   of   the   licensor   such   as   exports   to   Arbitration   Rules   of   the   United   Nations   Commission   on  
countries   where   exclusive   licenses   to   manufacture   International   Trade   Law   (UNCITRAL)   or   the   Rules   of  
and/or   distribute   the   licensed   product(s)   have   already   Conciliation   and   Arbitration   of   the   International  
been  granted;   Chamber  of  Commerce  (ICC)  shall  apply  and  the  venue  of  
  arbitration   shall   be   the   Philippines   or   any   neutral  
87.9.   Those   which   restrict   the   use   of   the   technology   country;  and  
 
supplied   after   the   expiration   of   the   technology   transfer  
88.4.  The  Philippine  taxes  on  all  payments  relating  to  the  
arrangement,  except  in  cases  of  early  termination  of  the  
technology   transfer   arrangement   due   to   reason(s)   technology   transfer   arrangement   shall   be   borne   by   the  
attributable  to  the  licensee;   licensor.    
   
87.10.   Those  which  require  payments  for  patents  and   SECTION   92.   NON-­‐REGISTRATION   WITH   THE  
other   industrial   property   rights   after   their   expiration,   DOCUMENTATION,   INFORMATION   AND   TECHNOLOGY  
termination  arrangement;   TRANSFER  BUREAU .  –    
  Technology   transfer   arrangements   that   conform   with  
87.11.   Those   which   require   that   the   technology   the   provisions   of   Sections   86   and   87   need   not   be  
recipient   shall   not   contest   the   validity   of   any   of   the   registered   with   the   Documentation,   Information   and  
patents  of  the  technology  supplier;   Technology  Transfer  Bureau.  Non-­‐conformance  with  any  
  of   the   provisions   of   Sections   87   and   88,   however,   shall  
87.12.   Those   which   restrict   the   research   and   automatically   render   the   technology   transfer  
development   activities   of   the   licensee   designed   to   arrangement   unenforceable,   unless   said   technology  
absorb   and   adapt   the   transferred   technology   to   local   transfer   arrangement   is   approved   and   registered   with  

o 31
Katrina Michelle Mancao
 
the   Documentation,   Information   and   Technology   SECTION  228.  PUBLIC  RECORDS.  –    
Transfer   Bureau   under   the   provisions   of   Section   91   on   The   section   or   division   of   the   National   Library   and   the  
exceptional  cases.   Supreme   Court   Library   charged   with   receiving   copies  
  and   instruments   deposited   and   with   keeping   records  
SECTION  237.  NOTIFICATION  ON  BERNE  APPENDIX.  –     required   under   this   Act   and   everything   in   it   shall   be  
The   Philippines   shall   by   proper   compliance   with   the   opened  to  public  inspection.  The  Director  of  the  National  
requirements  set  forth  under  the  Appendix  of  the  Berne   Library   is   empowered   to   issue   such   safeguards   and  
Convention   (Paris   Act,   1971)   avail   itself   of   the   special   regulations   as   may   be   necessary   to   implement   this  
provisions   regarding   developing   countries,   including   Section  and  other  provisions  of  this  Act.  
provisions   for   licenses   grantable   by   competent   authority    
under  the  Appendix.   SECTION  229.  COPYRIGHT  DIVISION;  FEES.  –    
  The   Copyright   Section   of   the   National   Library   shall   be  
9. Deposit   of   Copyrightable   classified   as   a   Division   upon   the   effectivity   of   this   Act.  
Materials   The  National  Library  shall  have  the  power  to  collect,  for  
  the  discharge  of  its  services  under  this  Act,  such  fees  as  
may   be   promulgated   by   it   from   time   to   time   subject   to  
SECTION   191.   REGISTRATION  AND   DEPOSIT  WITH   NATIONAL  
the  approval  of  the  Department  Head.    
LIBRARY  AND  THE  SUPREME  COURT  LIBRARY.  –      
After   the   first   public   dissemination   of   performance   by   RULE   5   -­‐   REGISTRATION   AND   DEPOSIT   OF   WORK,  
authority  of  the  copyright  owner  of  a  work  falling  under  
COPYRIGHT  SAFEGUARDS  AND  REGULATIONS  
Subsections  172.1,  172.2  and  172.3  of  this  Act,  there  shall,  
for   the   purpose   of   completing   the   records   of   the   SECTION  1.  WHO  MAY  APPLY.  —    
National   Library   and   the   Supreme   Court   Library,   within   The   owner   or   assignee   of   the   copyright   or   his   duly  
three  (3)  weeks,  be  registered  and  deposited  with  it,  by   authorized   agent   or   representative,   may   apply   for   a  
personal  delivery  or  by  registered  mail  two  (2)  complete   certificate   of   registration   and   deposit   of   the   work:  
copies  or  reproductions  of  the  work  in  such  form  as  the   Provided,  That  if  an  author  could  not  claim  the  benefit  of  
directors   of   said   libraries   may   prescribe.   A   certificate   of   copyright  protection,  his  assignee  or  agent  cannot  claim  
deposit   shall   be   issued   for   which   the   prescribed   fee   shall   it.   If   the   applicant   is   not   the   owner   or   author   or   assignee  
be   collected   and   the   copyright   owner   shall   be   exempt   of  the  work,  he  shall  be  required  to  submit  his  authority  
from   making   additional   deposit   of   the   works   with   the   to  apply.  
National   Library   and   the   Supreme   Court   Library   under    
other  laws.  If,  within  three  (3)  weeks  after  receipt  by  the   An   assignee   is   a   person   to   whom   an   author   may   assign  
copyright   owner   of   a   written   demand   from   the   directors   copyright  in  whole  or  in  part.  The  assignee  is  entitled  to  
for   such   deposit,   the   required   copies   or   reproductions   all   the   rights   and   remedies   which   the   assignor   has   with  
are   not   delivered   and   the   fee   is   not   paid,   the   copyright   respect  to  the  copyright.  
owner   shall   be   liable   to   pay   a   fine   equivalent   to   the    
required   fee   per   month   of   delay   and   to   pay   to   the   Although  no  copyright  should  subsist  in  any  work  of  the  
National   Library   and   the   Supreme   Court   Library   the   government,   any   employee   may   claim   it   by   submitting  
amount  of  the  retail  price  of  the  best  edition  of  the  work.   for   registration   any   work   that   has   been   created   during  
Only   the   above   mentioned   classes   of   work   shall   be   the  time  of  his  regularly  prescribed  official  duties.  
accepted   for   deposit   by   the   National   Library   and   the    
Supreme  Court  Library.   SECTION  2.  IDENTIFICATION  OF  AUTHOR  OR  AUTHORS.  —    
  An  application  for  copyright  certificate  shall  identify  the  
SECTION  192.  NOTICE  OF  COPYRIGHT.  –     author   or   authors,   as   far   as   practicable,   without  
Each   copy   of   a   work   published   or   offered   for   sale   may   prejudice   to   the   provisions   of   Sections   171.2   and   179   of  
contain   a   notice   bearing   the   name   of   the   copyright   the  IPC.  
owner,  and  the  year  of  its  first  publication,  and,  in  copies    
produced   after   the   creator's   death,   the   year   of   such   SECTION  3.  NON-­‐RESIDENT  APPLICANT.  —    
death.   A   non-­‐resident   applicant   shall   appoint   a   resident   agent,  
  by   special   power   of   attorney   (SPA),   who   shall   be  
SECTION   227.   OWNERSHIP  OF   DEPOSIT  AND   INSTRUMENTS.   authorized   to   pursue   the   copyright   application   for  
his/her/its  behalf  with  TNL  and/or  the  SCL  and  to  receive  
–    
service   of   notice   or   other   legal   process   relating   to   the  
All  copies  deposited  and  instruments  in  writing  filed  with   application   and   the   copyright.   In   the   event   of   death,  
the   National   Library   and   the   Supreme   Court   Library   in   absence   or   incapacity   of   the   resident   agent,   the  
accordance  with  the  provisions  of  this  Act  shall  become   applicant   shall   appoint   a   new   resident   agent,   by   SPA  
the  property  of  the  Government.    
 

32 z
Intellectual Property Law

with   revocation   of   the   prior   SPA,   and   file   notice   and   a   SECTION  7.  When  to  Register  and  Deposit.  —    
copy  thereof  with  TNL  and/or  the  SCL.   The   registration   and   deposit   of   copies   or   reproductions  
  of  the  work  or  works,  using  the  prescribed  form,  shall  be  
SECTION  4.  WORKS  THAT  SHALL  BE  REGISTERED  AND  DEPOSITED.   made   personally   or   by   registered   mail   within   three   (3)  
—     weeks  after  the  first  public  dissemination  or  publication  
Two  (2)  copies  or  reproductions  of  the  following  classes   as  authorized  by  the  author.  
of  works,  and  transfers  and  assignments  related  thereto,    
shall   be   registered   and   deposited   with   TNL   Copyright   08  and  15  December  2011  
Division  and  another  two  (2)  copies  with  the  SCL:   COPYRIGHT  
  What  is  copyright?  
• Books,  pamphlets,  articles  and  other  writings;   -­‐ It   is   a   bundle   of   rights   enumerated   in   Section  
• Periodicals  and  newspapers;   177.  
• Lectures,   sermons,   addresses,   dissertations    
prepared   for   oral   delivery   whether   or   not   Why  is  copyright  called  an  “economic  right”?    
reduced  in  writing  or  other  material  form;   -­‐ Because   the   purpose   of   the   establishment   of  
• Letters;   the  right  is  to  allow  the  author  or  originator  of  
• Musical  compositions  with  or  without  words.   the   work   to   maximize   the   value   that   can   be  
  derived  from  the  work.    
SECTION  5.  REPLICAS  AND  PICTURES.  —     -­‐ How   does   it   allow   the   copyright   owner   to  
maximize  the  value  of  the  work?  
For   practical   purposes,   only   replicas   and   pictures   of   the  
o By   allowing   him   to   assign   his   rights,  
following   classes   of   works,   shall   be   registered   and  
grant  licenses  for  the  use  of  the  work,  
deposited  with  TNL  Copyright  Division:  
etc.,   thus   allowing   him   to   get  
 
royalties  from  the  work.  
• Works   of   drawing,   painting,   architecture,  
 
sculpture,   engraving,   lithography   or   other  
Beauty   of   copyright:   Duration.   It   protects   the   owner  
works   of   art,   models   or   designs   for   works   of  
during  his  lifetime  +  50  years  after  death.  
art;  
 
• Original   ornamental   designs   or   models   for  
Standard  of  copyright  protection:  Originality.  
articles   of   manufacture,   whether   or   not  
-­‐ “Original”   means   it   is   the   product   of   one’s   skill,  
registerable   as   an   industrial   design,   and   other  
labor/employment  
works  of  applied  art;  
-­‐ Work   does   not   need   to   be   groundbreaking   or  
• Illustrations,   maps,   plans,   sketches,   charts   and  
revolutionary.  
three-­‐dimensional  works  relative  to  geography,   -­‐ What   it   merely   requires   is   that   the   work   is   your  
topography,  architecture  or  science;   own.  This  is  an  incredibly  low  standard!  
• Drawings   or   plastic   works   of   a   scientific   or    
technical  character.     Why  is  the  standard  so  low?    
  -­‐ To   remain   very   objective.   Having   subjective  
SECTION   6.   WORKS   THAT   MAY   BE   REGISTERED   AND   DEPOSITED.   standards   will   erode   the   IP   system   =   if   every  
—     new   work   that   challenges   the   status   quo   will  
The  following  works  may  be  registered  and  deposited:   not   be   protected,   no   one   might   challenge   the  
• Dramatic   or   dramatic-­‐musical   compositions,   status  quo.  
choreographic   works   or   entertainment   in   -­‐ Thus,   IP   should   not   look   into   the   substance   of  
shows;   the  work.  (See  Section  172.2)  
• Photographic   works   including   works   produced   -­‐ Consequence  of  the  low  standard:  It  is  hard  to  
by   a   process   analogous   to   photography,   prove   infringement.   To   prove   infringement,  
lantern  slides;   you  have  to  prove  ACCESS  and  COPYING.  
• Audiovisual   works   and   cinematographic   works    
and   works   produced   by   a   process   analogous   to   3  things  covered  by  IP:  
cinematography   or   any   process   for   making   1. Traditional  works  –  literary  and  artistic  works  
audio-­‐visual  recordings;   2. Derivative  works  –  translation  
• Pictorial  illustrations  and  advertisements;   3. Published  works  
• Computer  programs;    
• Other   literary,   scholarly,   scientific   and   artistic   Issue:   Can   a   work   that   was   transformed   without   the  
works;   consent  of  the  original  creator  be  considered  as  original  
• Sound  recordings;   and  be  protected?  
• Broadcast  recordings.     -­‐ It   can   be   understood   that   the   derivative   work  
  should  be  AUTHORIZED.  (See  173.2)  

o 33
Katrina Michelle Mancao
 
-­‐ BUT:  Ideas  are  too  important  to  be  kept  in  the   -­‐If   not   registered,   buy   only  
private  domain.  If  done,  you  are  increasing  the   the  thing.  
cost   for   the   creation   of   the   work.   Thus,   the   la   b. Post-­‐PD49   à   Were   all   the   rights  
protects   NOT   the   underlying   idea   by   the   assigned  to  one  person?  
EXPRESSION   of   the   idea.   Ideas   are   basic    
building   blocks.   If   you   allow   the   monopoly   of   Ownership  of  copyright  by  the  government:  
idea  through  copyright,  you  would  increase  the   -­‐ GR:  Government  cannot  acquire  copyright.    
cost   for   further   work   and   this   may   lead   to   -­‐ However,  if  somebody  would  exploit  the  work  
stagnation.   Remember,   the   primary   purpose   of   for   profit,   there   must   be   prior   approval   from  
IP   is   really   the   advancement   of   art   and   the   government   agency   or   office   where   the  
literature,   and   this   won’t   be   achieved   if   you   work  is  created.  
monopolized   the   fundamental   building   blocks   -­‐ Exception   to   the   prior   approval   requirement  
of  these,  i.e.  the  ideas.   exception:  176.3  
o Reconcile   this   with   Section   173.1(b)   à    
data   is   NOT   protected,   but   the   Ma’am  is  a  government  employee.  Are  her  oral  lectures  
COMPILATIONS   of   data   are   protected  by  copyright?  
protected?   -­‐ Yes.  Copyright  protects  the  work  regardless  of  
o Note  that  in  Feist  (which  is  a  US  case,   its  form.  
thus   merely   persuasive),   the   standard   -­‐ What  right  does  she  have?  
of   protection   for   compilation   à   o 176.2.   She   has   the   exclusive   right   to  
modicum  of  creativity.  It  required  the   reproduces,  etc.  
compilation   to   be   original   by   reason    
of   their   selection   or   arrangement.   Ownership  of  copyright:  
There   must   be   something   unusual   or   -­‐ GR:  Author  of  the  work  
exceptional   with   the   collection   of   -­‐ If  more  than  1  author:    
data.   o Apply  their  contract  if  there  is  any.  
  o If   no   contract:   apply   the   rules   on   co-­‐
When  does  copyright  vests?   ownership  
-­‐ Under   the   IP   Code:   from   the   moment   of    
creation   (we   follow   the   principle   of   automatic   If  the  work  is  made  by  an  employee:  
protection  under  the  Berne  Convention)   -­‐ Test:   Whether   the   work   is   part   of   the  
-­‐ The   principle   of   automatic   protection   was   employee’s  regularly-­‐assigned  duties:  
followed  since  1972  under  PD49.   o If   Yes:   the   work   is   owned   by   the  
o Under   PD   49,   the   copyright   owner   employer.  
can   still   sue   the   infringer   even   if   the   o If   No:   the   work   is   owned   by   the  
former  failed  to  deposit  the  material.   employee.  
The   only   effect   of   failure   to   register   is   -­‐ Note:  See  178.3    
that   the   copyright   owner   won’t   be   o 178.3(b)   expressly   allows   the  
allowed   to   obtain   damages   from   the   employer   and   the   employee   to   have  
defendant.   an  agreement  to  the  contrary    
-­‐ Prior   to   that:   Under   Act   3134,   registration   and   o 178.3(a)   does   not   have   a   similar  
deposit  were  needed  to  vest  copyright.   proviso  
o Reglamentary   period   for   registration:   o the   difference   in   treatment   implies  
30  days  from  creation.   that   an   agreement   to   make   the  
o Effect  of  failure  to  register  within  that   employer   own   the   work   in   instances  
period:   it   may   be   the   basis   for   where   the   employee   created  
questioning  the  copyright   something   and   such   is   not   a   part   of  
o Santos  v.  McCullough  Printing:  failure   his   regularly   assigned   duties   is  
to   deposit   the   copyrighted   material   PROHIBITED.    
made   the   material   part   of   the   public   § But   Ma’am   also   said   that  
domain.   you  can  still  argue  the  other  
-­‐ PD   49   was   actually   a   revolutionary   piece   of   way.  
legislation!    
  If  the  work  is  created  by  commission:  
So,  for  due  diligence  investigation:   -­‐ Work:   owned   by   the   person   who  
1. Ask  when  the  work  was  created.   commissioned  it  
2. If:   -­‐ Copyright:  owned  by  the  creator  
a. Pre-­‐PD49  à  Was  it  registered  or  not?   o UNLESS   there   is   a   contract   to   the  

34 z
Intellectual Property Law

contrary   184.1(i)  v.  184.1(a)  


  184.1(i)   184.1(a)  
If  the  work  is  a  letter:   “The   public   performance   “The   recitation   or  
-­‐ Letter:  owned  by  the  recipient   or   the   communication   to   performance   of   a   work,  
-­‐ Copyright:  owned  by  the  writer   the  public  of  a  work,  in  a   once   it   has   been   lawfully  
  place   where   no   made   accessible   to   the  
Duration  of  copyright:  lifetime  +  50  years  after  death   admission   fee   is   charged   public,   if   done   privately  
-­‐ Copyright  is  best  for  works  with  long  shelf  life.   in   respect   of   such   public   and   free   of   charge   or   if  
-­‐ Note   calculation   for   the   period   after   death:   performance   or   made   strictly   for   a  
begins   on   January   1   of   the   year   following   the   communication,  by  a  club   charitable   or   religious  
death     or   institution   for   institution  or  society”  
o Why?  For  easier  computation   charitable   or   educational  
-­‐ After   the   expiration   of   the   period,   the   work   purpose  only,  whose  aim  
becomes   part   of   the   public   domain   à   why   is   not   profit   making,  
classic  works  are  significantly  cheaper!   subject   to   such   other  
  limitations   as   may   be  
05  January  2012   provided   in   the  
Limitations   on   copyright   (exception   to   the   exclusive   Regulations”  
right  given  to  holder):   BOTH  must  be  present:   Only   EITHER   of   the   2   is  
1. Section   184   (limitations   of   copyright),   176   1. Educational/   required:  
(works  of  the  government)   charitable  purpose   1. Religious/   charitable  
2. Exceptions  for  certain  works:  library   2. Free  of  charge   institution  
3. Fair  use   2. Free   of   charge   and  
  private  
Q:   Showing   of   Harry   Potter,   done   by   the   Religious    
Society   of   Mary,   for   P50.   Is   the   Religious   Society   of   Mary   Filipino   Society   of   Composers   v.   Tan:   Standard   used   by  
liable?   the   court   to   determine   whether   the   performance   was  
-­‐ No!  184.1(a)  embodies  2  exceptions:   done   free   or   charge   or   not   à   whether   or   not   it   is  
1. Privately  and  free  of  charge;  OR   designed   to   induce   more   business.   Plus   costs   already  
2. Charitable   or   religious   included  in  other  costs.  
institution/society    
-­‐ BUT   the   showing   must   be   done   AFTER   the   first   Philippine  Educ.  Co.  v.  Sotto:  Erroneous  case!  There  was  
publication  of  the  work.   no   copyright   on   the   material   but   with   an   “all   rights  
-­‐ Follow-­‐up   question:   What   if   they   show   the   reserved”  clause.    
movie   all   over   the   Philippines,   are   they   now   -­‐ Ma’am:   If   no   copyright,   what   rights   were  
liable?   reserved?   “All   rights   reserved”   applicable   only  
o Yes!   See   184.2.   The   act   must   not   for  copyrighted  works.  
unreasonably   prejudice   the   right    
holder’s  legitimate  interests.   Del  Castillo  cases:  
o ALWAYS   READ   184.1   IN   -­‐ 1st  Court:  No  intent  on  the  part  of  Del  Castillo  to  
CONJUNCTION  WITH  184.2   plagiarize.  
  o Ma’am:   Intent   shouldn't   be   required  
184.1  (b)  –  drawing  from  work  of  others   in  special  laws!  (malum  prohibitum)  
-­‐ Conditions:   -­‐ 2nd   Court:   Court   made   a   distinction   between  
o Work  has  been  published   the   standards   of   an   academic   institution   and  
o Compatible  with  fair  use   the  standards  for  members  of  the  bar.  
o Only   to   the   extent   justified   for   the   o Acad:  Higher  standards  
purpose   o Bar:  Lower  standards  –  stare  decisis  
o Attribution!   o Ma’am:   184.1(k)   was   extended!   They  
  rewrote   the   IP   Coe!   Court,   in   effect,  
184.1(c)  –  exception  for  mass  media   said   lawyers   and   justices   were  
-­‐ Conditions:   exempted   from   the   application   of   the  
o Delivered  in  public   IP   Code.   They   did   not   even   bother  
o For  information  purposes   with  184.2!  
o No  “all  rights  reserved”  in  the  article    
o Attribution   Other  exceptions:  
  1. Work  of  architecture  
  2. Reproduction  of  published  work  

o 35
Katrina Michelle Mancao
 
o Old  law  (PD49)  =  complete  exception   rights  are  assigned.  
granted  to  natural  person    
o Now  =  subject  to  certain  conditions.   12  January  2012  
o IP  Code  abolished  PD  285    
3. For  libraries   Deposit  
4. Importation   -­‐ Not  a  requirement  for  the  vesting  of  copyright.  
-­‐ No   express   prohibition   of   parallel   -­‐ Where  made:  TNL  and  SCL  
importation   o Note   that   the   IPO   and   the   SCL   now  
  has  a  Memorandum  of  Agreement.  
FAIR  USE:   -­‐ Office   is   exercising   a   purely   ministerial  
-­‐ Drawn  from  US  jurisprudence   function.    
-­‐ Harper  &  Row  case:   -­‐ Certificate  of  deposit  
o Factors  of  fair  use:   o Legally  does  not  mean  anything.  
§ Purpose   of   use   –   for   o It   is   merely   a   proof   of   the   fact   of  
commercial   purpose   à   deposit  
presumption   that   use   is   not   o Can   establish   that   as   of   the   date   of  
fair   deposit,   work   was   already   created.  
§ Nature   of   work   –   Thus,   may   help   establish   that   the  
unpublished  à  unfair  use   depositor   is   not   an   appropriator   but  
§ Amount  and  substantiality  –   the  creator  of  the  work.  
measured   by   quality,   not   o With  respect  to  third  person  who  may  
quantity  (heart  of  work)   want  to  deal  with  the  depositor:  may  
§ Effect   on   the   market   –   impress   upon   the   third   person   that  
MOST  IMPORTANT  factor   the   depositor   is   the   right   person   to  
-­‐ Q:  Did  we  completely  adopt  the  US  concept  of   deal  with.  
fair  use?   § Philippines   is   a   formalistic  
o Not  really.   society.  
§ 185.2   –   fact   that   work   is   § This  may  reduce  transaction  
unpublished…   costs.  
§ 185.1   –   did   not   establish   a   § Removes   lingering   doubt  
hierarchy   among   the   4   about  copyright.  
factors   -­‐ Fine  for  failure  to  deposit  –  kicks  in  only  AFTER  
§ commercial…   a  demand  
-­‐ Sony  Corp.  v.  Universal  Studios:  very  important   -­‐ Why  require  a  deposit?    
case   o Public   interest   =   promote   availability  
o The  ruling  in  this  case  is  still  standard.   of   information   to   the  
§ Standard:   capable   of   single,   public   by   making   it  
non-­‐infringing   use   =   no   available   in   the   National  
infringement.   Library   (so   that   there   is   a  
§ In  this  case,  time  shifting  is  a   public   repository   of   these  
substantial,   non-­‐infringing   works  of  art)    
use.    
o No  contributory  infringement.   10. Infringement  
 
 
Transfer  of  copyright:  
a. Definition  
-­‐ Must  be  written.  Oral  assignment  is  invalid.  
-­‐ Purpose  of  the  requirement  =  protect  rights  of    
the   creator.   To   ensure   that   the   copyright   Ballantine’s   Law   Dictionary:   It   is   a   violation   of   a   right;   an  
owner   knows   exactly   what   he/she   is   giving   encroachment.  
away.    
  b. Remedies  
Assignment  v.  License    
Assignment   License   SECTION   221.   POINTS  OF   ATTACHMENT  FOR   WORKS  UNDER  
Transfer   of   ownership   of   Limited.   Only   permission   SECTIONS  172  AND  173.  –    
right  =  ceding  right   to   use.   Right   remains  
221.1.   The   protection   afforded   by   this   Act   to  
with  the  copyright  owner  
who   may   declare   copyrightable   works   under   Sections   172   and   173   shall  
conditions   with   which   apply  to:  

36 z
Intellectual Property Law

  223.2.   Sound   recordings   that   were   first  


(a)   Works   of   authors   who   are   nationals   of,   or   published  in  the  Philippines.    
have   their   habitual   residence   in,   the    
Philippines;   SECTION   224.   POINTS   OF   ATTACHMENT   FOR   BROADCASTS.  
 
(b)   Audio-­‐visual   works   the   producer   of   which  
–    
has   his   headquarters   or   habitual   residence   in   224.1.   The   provisions   of   this   Act   on   the   protection   of  
the  Philippines;   broadcasts  shall  apply  to:  
(c)   Works   of   architecture   erected   in   the    
Philippines   or   other   artistic   works   incorporated   (a)   Broadcasts   of   broadcasting   organizations  
in   a   building   or   other   structure   located   in   the   the   headquarters   of   which   are   situated   in   the  
Philippines;   Philippines;  and  
   
(d)  Works  first  published  in  the  Philippines;  and   (b)   Broadcasts   transmitted   from   transmitters  
  situated  in  the  Philippines.  
(e)   Works   first   published   in   another   country    
but   also   published   in   the   Philippines   within   224.2.   The   provisions   of   this   Act   shall   also   apply   to  
thirty   days,   irrespective   of   the   nationality   or   performers   who,   and   to   producers   of   sound   recordings  
residence  of  the  authors.   and   broadcasting   organizations   which,   are   to   be  
  protected   by   virtue   of   and   in   accordance   with   any  
221.2.  The  provisions  of  this  Act  shall  also  apply  to  works   international   convention   or   other   international  
that   are   to   be   protected   by   virtue   of   and   in   accordance   agreement  to  which  the  Philippines  is  a  party.  
with  any  international  convention  or  other  international    
agreement  to  which  the  Philippines  is  a  party.   SECTION   3.   INTERNATIONAL   CONVENTIONS   AND  
  RECIPROCITY.  –    
SECTION   222.   POINTS   OF   ATTACHMENT   FOR   PERFORMERS.   Any  person  who  is  a  national  or  who  is  domiciled  or  has  a  
–     real   and   effective   industrial   establishment   in   a   country  
The   provisions   of   this   Act   on   the   protection   of   which  is  a  party  to  any  convention,  treaty  or  agreement  
performers  shall  apply  to:   relating   to   intellectual   property   rights   or   the   repression  
  of   unfair   competition,   to   which   the   Philippines   is   also   a  
222.1.   Performers   who   are   nationals   of   the   party,   or   extends   reciprocal   rights   to   nationals   of   the  
Philippines;   Philippines   by   law,   shall   be   entitled   to   benefits   to   the  
  extent  necessary  to  give  effect  to  any  provision  of  such  
222.2.  Performers  who  are  not  nationals  of  the   convention,   treaty   or   reciprocal   law,   in   addition   to   the  
rights   to   which   any   owner   of   an   intellectual   property  
Philippines  but  whose  performances:  
  right  is  otherwise  entitled  by  this  Act.    
 
(a)  Take  place  in  the  Philippines;  or  
  SECTION  10.2.    
(b)   Are   incorporated   in   sound   (a)   Exercise   original   jurisdiction   in   administrative  
recordings   that   are   protected   under   complaints   for   violations   of   laws   involving   intellectual  
this  Act;  or   property   rights:   Provided,   That   its   jurisdiction   is   limited  
  to  complaints  where  the  total  damages  claimed  are  not  
(c)   Which   has   not   been   fixed   in   sound   less   than   Two   hundred   thousand   pesos   (P200,000):  
recording   but   are   carried   by   Provided   further,   That   availment   of   the   provisional  
broadcast   qualifying   for   protection   remedies   may   be   granted   in   accordance   with   the   Rules  
under  this  Act.     of   Court.   The   Director   of   Legal   Affairs   shall   have   the  
  power   to   hold   and   punish   for   contempt   all   those   who  
SECTION   223.   POINTS   OF   ATTACHMENT   FOR   SOUND   disregard   orders   or   writs   issued   in   the   course   of   the  
proceedings.    
RECORDINGS.  –    
 
The   provisions   of   this   Act   on   the   protection   of   sound   (b)   After   formal   investigation,   the   Director   for   Legal  
recordings  shall  apply  to:   Affairs   may   impose   one   (1)   or   more   of   the   following  
  administrative  penalties:  
223.1.  Sound  recordings  the  producers  of  which    
are  nationals  of  the  Philippines;  and   (i)   The   issuance   of   a   cease   and   desist   order  
  which   shall   specify   the   acts   that   the  
respondent   shall   cease   and   desist   from   and  
shall  require  him  to  submit  a  compliance  report  

o 37
Katrina Michelle Mancao
 
within   a   reasonable   time   which   shall   be   fixed   in   (vi)   The   cancellation   of   any   permit,   license,  
the  order;   authority,  or  registration  which  may  have  been  
  granted  by  the  Office,  or  the  suspension  of  the  
(ii)  The  acceptance  of  a  voluntary  assurance  of   validity   thereof   for   such   period   of   time   as   the  
compliance   or   discontinuance   as   may   be   Director  of  Legal  Affairs  may  deem  reasonable  
imposed.   Such   voluntary   assurance   may   which  shall  not  exceed  one  (1)  year;  
include  one  or  more  of  the  following:    
  (vii)   The   withholding   of   any   permit,   license,  
(1)   An   assurance   to   comply   with   the   authority,   or   registration   which   is   being  
provisions   of   the   intellectual   property   secured  by  the  respondent  from  the  Office;  
law  violated;    
  (viii)  The  assessment  of  damages;  
(2)   An   assurance   to   refrain   from    
engaging   in   unlawful   and   unfair   acts   (ix)  Censure;  and  
and   practices   subject   of   the   formal    
investigation;   (x)  Other  analogous  penalties  or  sanctions.  
   
(3)   An   assurance   to   recall,   replace,   SECTION  216.  REMEDIES  FOR  INFRINGEMENT.  –    
repair,   or   refund   the   money   value   of   216.1.  Any  person  infringing  a  right  protected  under  this  
defective   goods   distributed   in  
law  shall  be  liable:  
commerce;  and  
 
 
(a)   To   an   injunction   restraining   such  
(4)   An   assurance   to   reimburse   the  
infringement.   The   court   may   also   order   the  
complainant   the   expenses   and   costs  
defendant   to   desist   from   an   infringement,  
incurred   in   prosecuting   the   case   in  
among   others,   to   prevent   the   entry   into   the  
the  Bureau  of  Legal  Affairs.  
channels  of  commerce  of  imported  goods  that  
 
involve   an   infringement,   immediately   after  
The  Director  of  Legal  Affairs  may  also  
customs  clearance  of  such  goods.  
require   the   respondent   to   submit  
 
periodic  compliance  reports  and  file  a  
(b)   Pay   to   the   copyright   proprietor   or   his  
bond   to   guarantee   compliance   of   his  
assigns  or  heirs  such  actual  damages,  including  
undertaking;  
legal  costs  and  other  expenses,  as  he  may  have  
 
incurred  due  to  the  infringement  as  well  as  the  
(iii)   The   condemnation   or   seizure   of   products  
profits   the   infringer   may   have   made   due   to  
which   are   subject   of   the   offense.   The   goods  
such   infringement,   and   in   proving   profits   the  
seized   hereunder   shall   be   disposed   of   in   such  
plaintiff   shall   be   required   to   prove   sales   only  
manner  as  may  be  deemed  appropriate  by  the  
and   the   defendant   shall   be   required   to   prove  
Director   of   Legal   Affairs,   such   as   by   sale,  
every   element   of   cost   which   he   claims,   or,   in  
donation  to  distressed  local  governments  or  to  
lieu   of   actual   damages   and   profits,   such  
charitable   or   relief   institutions,   exportation,  
damages  which  to  the  court  shall  appear  to  be  
recycling  into  other  goods,  or  any  combination  
just  and  shall  not  be  regarded  as  penalty.  
thereof,   under   such   guidelines   as   he   may  
 
provide;  
(c)   Deliver   under   oath,   for   impounding   during  
 
the   pendency   of   the   action,   upon   such   terms  
(iv)  The  forfeiture  of  paraphernalia  and  all  real  
and   conditions   as   the   court   may   prescribe,  
and  personal  properties  which  have  been  used  
sales  invoices  and  other  documents  evidencing  
in  the  commission  of  the  offense;  
sales,  all  articles  and  their  packaging  alleged  to  
 
infringe  a  copyright  and  implements  for  making  
(v)   The   imposition   of   administrative   fines   in  
them.  
such   amount   as   deemed   reasonable   by   the  
 
Director  of  Legal  Affairs,  which  shall  in  no  case  
(d)   Deliver   under   oath   for   destruction   without  
be   less   than   Five   thousand   pesos   (P5,000)   nor  
any   compensation   all   infringing   copies   or  
more   than   One   hundred   fifty   thousand   pesos  
devices,   as   well   as   all   plates,   molds,   or   other  
(P150,000).   In   addition,   an   additional   fine   of  
means  for  making  such  infringing  copies  as  the  
not   more   than   One   thousand   pesos   (P1,000)  
court  may  order.  
shall   be   imposed   for   each   day   of   continuing  
 
violation;  
(e)   Such   other   terms   and   conditions,   including  
 
the  payment  of  moral  and  exemplary  damages,  

38 z
Intellectual Property Law

which   the   court   may   deem   proper,   wise   and   (c)  Trade  exhibit  of  the  article  in  public,  shall  be  
equitable   and   the   destruction   of   infringing   guilty   of   an   offense   and   shall   be   liable   on  
copies   of   the   work   even   in   the   event   of   conviction   to   imprisonment   and   fine   as   above  
acquittal  in  a  criminal  case.   mentioned.  
   
216.2.  In  an  infringement  action,  the  court   shall   also   have   SECTION  218.  AFFIDAVIT  EVIDENCE.  –    
the   power   to   order   the   seizure   and   impounding   of   any   218.1.   In   an   action   under   this   Chapter,   an   affidavit   made  
article   which   may   serve   as   evidence   in   the   court   before   a   notary   public   by   or   on   behalf   of   the   owner   of  
proceedings.     the   copyright   in   any   work   or   other   subject   matter   and  
  stating  that:  
SECTION  217.  CRIMINAL  PENALTIES.  –      
217.1.   Any   person   infringing   any   right   secured   by   (a)   At   the   time   specified   therein,   copyright  
provisions  of  Part  IV  of  this  Act  or  aiding  or  abetting  such   subsisted  in  the  work  or  other  subject  matter;  
infringement  shall  be  guilty  of  a  crime  punishable  by:    
  (b)   He   or   the   person   named   therein   is   the  
(a)   Imprisonment   of   one   (1)   year   to   three   (3)   owner  of  the  copyright;  and  
years   plus   a   fine   ranging   from   Fifty   thousand    
pesos  (P50,000)  to  One  hundred  fifty  thousand   (c)   The   copy   of   the   work   or   other   subject  
pesos  (P150,000)  for  the  first  offense.   matter  annexed  thereto  is  a  true  copy  thereof,  
  shall   be   admitted   in   evidence   in   any  
(b)  Imprisonment  of  three  (3)  years  and  one  (1)   proceedings   for   an   offense   under   this   Chapter  
day   to   six   (6)   years   plus   a   fine   ranging   from   and   shall   be   prima   facie   proof   of   the   matters  
One   hundred   fifty   thousand   pesos   (P150,000)   therein  stated  until  the  contrary  is  proved,  and  
to  Five  hundred  thousand  pesos  (P500,000)  for   the   court   before   which   such   affidavit   is  
the  second  offense.   produced   shall   assume   that   the   affidavit   was  
  made   by   or   on   behalf   of   the   owner   of   the  
(c)   Imprisonment   of   six   (6)   years   and   one   (1)   copyright.  
day   to   nine   (9)   years   plus   a   fine   ranging   from    
five   hundred   thousand   pesos   (P500,000)   to   218.2.  In  an  action  under  this  Chapter:  
One   million   five   hundred   thousand   pesos    
(P1,500,000)   for   the   third   and   subsequent   (a)   Copyright   shall   be   presumed   to   subsist   in  
offenses.   the  work  or  other  subject  matter  to  which  the  
  action  relates  if  the  defendant  does  not  put  in  
(d)   In   all   cases,   subsidiary   imprisonment   in   issue   the   question   whether   copyright   subsists  
cases  of  insolvency.   in  the  work  or  other  subject  matter;  and  
   
217.2.   In   determining   the   number   of   years   of   (b)   Where   the   subsistence   of   the   copyright   is  
imprisonment   and   the   amount   of   fine,   the   court   shall   established,   the   plaintiff   shall   be   presumed   to  
consider   the   value   of   the   infringing   materials   that   the   be  the  owner  of  the  copyright  if  he  claims  to  be  
defendant   has   produced   or   manufactured   and   the   the  owner  of  the  copyright  and  the  defendant  
damage  that  the  copyright  owner  has  suffered  by  reason   does   not   put   in   issue   the   question   of   his  
of  the  infringement.   ownership.  
   
217.3.   Any   person   who   at   the   time   when   copyright   (c)   Where   the   defendant,   without   good   faith,  
subsists  in  a  work  has  in  his  possession  an  article  which   puts   in   issue   the   questions   of   whether  
he  knows,  or  ought  to  know,  to  be  an  infringing  copy  of   copyright   subsists   in   a   work   or   other   subject  
the  work  for  the  purpose  of:   matter   to   which   the   action   relates,   or   the  
  ownership   of   copyright   in   such   work   or   subject  
(a)   Selling,   letting   for   hire,   or   by   way   of   trade   matter,  thereby  occasioning  unnecessary  costs  
offering   or   exposing   for   sale,   or   hire,   the   or   delay   in   the   proceedings,   the   court   may  
article;   direct   that   any   costs   to   the   defendant   in  
  respect   of   the   action   shall   not   be   allowed   by  
(b)  Distributing  the  article  for  purpose  of  trade,   him   and   that   any   costs   occasioned   by   the  
or  for  any  other  purpose  to  an  extent  that  will   defendant  to  other  parties  shall  be  paid  by  him  
prejudice   the   rights   of   the   copyright   owner   in   to  such  other  parties.    
the  work;  or    
 

o 39
Katrina Michelle Mancao
 
SECTION  219.  PRESUMPTION  OF  AUTHORSHIP.  –     the   Philippines,   Article   50   of   the   Agreement   on   Trade  
219.1.   The   natural   person   whose   name   is   indicated   on   a   Related   Aspects   of   Intellectual   Property   Rights,  
otherwise   known   as   TRIPS   and   other   related   laws   and  
work   in   the   usual   manner   as   the   author   shall,   in   the  
international  conventions.  
absence  of  proof  to  the  contrary,  be  presumed  to  be  the  
   
author   of   the   work.   This   provision   shall   be   applicable  
SECTION  2.  THE  WRIT  OF  SEARCH  AND  SEIZURE.  –    
even   if   the   name   is   a   pseudonym,   where   the   pseudonym  
leaves  no  doubt  as  to  the  identity  of  the  author.   Where  any  delay  is  likely  to  cause  irreparable  harm  to  the  
  intellectual   property   right   holder   or   where   there   is  
219.2.   The   person   or   body   corporate   whose   name   demonstrable   risk   of   evidence   being   destroyed,   the  
intellectual   property   right   holder   or   his   duly   authorized  
appears  on  a  audio-­‐visual  work  in  the  usual  manner  shall,  
representative  in  a  pending  civil  action  for  infringement  
in  the  absence  of  proof  to  the  contrary,  be  presumed  to  
or   who   intends   to   commence   such   an   action   may   apply  
be  the  maker  of  said  work.    
ex  parte  for  the  issuance  of  a  writ  of  search  and  seizure  
 
directing   the   alleged   infringing   defendant   or   expected  
SECTION  220.  INTERNATIONAL  REGISTRATION  OF  WORKS.  –     adverse   party   to   admit   into   his   premises   the   persons  
A   statement   concerning   a   work,   recorded   in   an   named  in  the  order  and  to  allow  the  search,  inspection,  
international  register  in  accordance  with  an  international   copying,  photographing,  audio  and  audiovisual  recording  
treaty  to  which  the  Philippines  is  or  may  become  a  party,   or   seizure   of   any   document   and   article   specified   in   the  
shall   be   construed   as   true   until   the   contrary   is   proved   order.    
except:    
  SECTION  3.  WHERE  APPLICATION  FILED.  –    
220.1.   Where   the   statement   cannot   be   valid   The   application   shall   be   filed   with   any   of   the   Regional  
under   this   Act   or   any   other   law   concerning   Trial   Courts   of   the   judicial   region   designated   to   try  
intellectual  property.   violations  of  intellectual  property  rights  stationed  at  the  
  place  where  the  alleged  violation  occurred  or  is  to  occur,  
220.2.   Where   the   statement   is   contradicted   by   or   the   place   to   be   searched,   at   the   election   of   the  
another   statement   recorded   in   the   applicant:  Provided,  however,  that  where  the  complaint  
international  register.   for   infringement   has   already   been   filed,   the   application  
  shall  be  made  in  the  court  where  the  case  is  pending.    
SECTION  225.  JURISDICTION.  –      
Without  prejudice  to  the  provisions  of  Subsection  7.1(c),   SECTION  4.  VERIFIED  APPLICATION  AND  AFFIDAVITS.  –    
actions   under   this   Act   shall   be   cognizable   by   the   courts   The  applicant  shall  file  a  verified  application  alleging  the  
with  appropriate  jurisdiction  under  existing  law.   ground   upon   which   it   is   based   and   the   specific  
  description   and   location   of   the   documents   and   articles  
SECTION  226.  DAMAGES.  –     to   be   searched,   inspected,   copied   or   seized   and   their  
No  damages  may  be  recovered  under  this  Act  after  four   value.   It   shall   also   state   the   name   applicant,   his  
(4)  years  from  the  time  the  cause  of  action  arose.     representative,   witnesses   and   counsel   who   will   attend  
  the   search   in   the   event   that   the   application   is   granted.  
The   application   shall   be   supported   by   affidavits   of  
SECTION  231.  REVERSE  RECIPROCITY  OF  FOREIGN  LAWS.  –    
witnesses   who   personally   know   the   facts   and   by   the  
Any   condition,   restriction,   limitation,   diminution,   authenticated  or  certified  documents.    
requirement,   penalty   or   any   similar   burden   imposed   by    
the   law   of   a   foreign   country   on   a   Philippine   national   The   application   shall   contain   a   certification   against  
seeking  protection  of  intellectual  property  rights  in  that   forum   shopping   as   prescribed   by   Section   5,   Rule   7   of   the  
country,  shall  reciprocally  be  enforceable  upon  nationals   1997  Rules  of  Civil  Procedure.    
of  said  country,  within  Philippine  jurisdiction.      
  The   applicant   shall   undertake   in   his   application   that   he  
RULE   ON   SEARCH   AND   SEIZURE   IN   CIVIL   ACTIONS   FOR   will   not   use   any   of   the   documents,   articles   or  
INFRINGEMENT  OF  INTELLECTUAL  PROPERTY  RIGHTS   information   obtained   by   reason   of   the   search   and  
  seizure   for   any   purpose   other   than   in   the   action   in   which  
SECTION  1.  COVERAGE.  –     the  writ  is  issued.    
 
This   Rule   shall   govern   the   provisional   seizure   and  
impounding   of   documents   and   articles   in   pending   and   SECTION  5.  EXAMINATION  OF  APPLICANT;  RECORD;  CONFIDENTIALITY  
intended   civil   actions   for   the   purpose   of   preventing   OF  PROCEEDINGS.  –    
infringement  and  preserving  relevant  evidence  in  regard   The   application   shall   be   acted   upon   within   twenty-­‐four  
to   alleged   infringement   under   Republic   Act   No.   8293,   (24)  hours  from  its  filing.  The  judge  must,  before  issuing  
otherwise   known   as   the   Intellectual   Property   Code   of   the  writ,  examine  in  the  form  of  searching  questions  and  

40 z
Intellectual Property Law

answers,   in   writing   an   under   oath   or   affirmation,   the   them  into  his  custody  subject  to  the  control  of  the  court.  
applicant   and   the   witnesses   he   may   produce   on   facts   The   enforcement   of   the   writ   shall   be   supervised   by   an  
personally   known   to   them.   The   examination   of   the   independent  Commissioner  to  be  appointed  by  the  court.    
applicant   and   his   witnesses   shall   be   recorded.   Their    
sworn  statements  and  their  affidavits  shall  form  part  of   SECTION  8.  CONTENTS  OF  THE  WRIT.  –    
the  record  of  the  case.     The  writ  shall  contain  the  following:    
   
The  hearing  on  the  application  for  the  writ  shall  be  held   (a)  an  order  to  the  alleged  infringing  defendant,  
in   the   chambers   of   the   judge.   Court   personnel   shall   expected   adverse   party   or   to   the   person   who  
maintain   the   confidentiality   of   the   application   appears   to   be   in   charge   or   in   control   of   the  
proceeding.     premises   or   residing   or   working   therein   to  
  permit  the  persons  named  in  the  writ  to  enter  
The   court   may   require   the   applicant   to   give   other   into   the   premises   for   the   purpose   of   searching,  
information   necessary   for   the   identification   of   the   inspecting,   copying,   or   removing   from   the  
articles   and   documents   to   be   searched,   inspected,   premises  and  transferring  to  the  custody  of  the  
copied   or   seized   and   the   premises   to   be   searched.   sheriff   and   subject   to   the   control   of   the   court  
Where   feasible,   it   may   direct   the   applicant   to   submit   the  subject  documents  and  articles;    
copies  and  photographs  of  the  documents  or  articles  to    
be  seized  and  impounded.     (b)  an  order  to  the  alleged  infringing  defendant,  
  expected   adverse   party   or   to   the   person   in  
SECTION  6.  GROUNDS  FOR  THE  ISSUANCE  OF  THE  ORDER.  –     charge   or   in   control   of   the   premises   to   disclose  
Before   the   Order   can   be   issued,   the   evidence   proffered   to   the   sheriff   serving   the   writ   the   location   of  
by   the   applicant   and   personally   evaluated   by   the   judge   the  documents  and  articles  subject  of  the  writ.    
must  show  that:      
  (c)  the  period  when  the  writ  shall  be  enforced  
(a)   the   applicant   is   the   right   holder   or   his   duly   which   in   no   case   shall   be   more   than   ten   (10)  
authorized  representative;     days  from  the  date  of  issuance  by  the  court.    
   
(b)   there   is   probable   cause   to   believe   that   the   (d)   the   names   of   the   applicant   or   his   agent   or  
applicant’s  right  is  being  infringed  or  that  such   representative  and  the  Commissioner  who  shall  
infringement   is   imminent   and   there   is   a   prima   supervise  the  enforcement  of  the  writ;  and    
facie   case   for   final   relief   against   the   alleged    
infringing   defendant   or   expected   adverse   (e)   other   terms   and   conditions   that   will   insure  
party;     the   proper   execution   of   the   writ   with   due  
  regard   to   the   rights   of   the   alleged   infringing  
(c)   damage,   potential   or   actual,   likely   to   be   defendant  or  expected  adverse  party.  
caused  to  the  applicant  is  irreparable;      
  It  shall  also  contain  a  warning  that  violation  of  any  of  the  
(d)  there  is  demonstrable  risk  of  evidence  that   terms   and   conditions   of   the   writ   shall   constitute  
the   alleged   infringing   defendant   or   expected   contempt  of  court.    
adverse  party  may  destroy,  hide  or  remove  the    
documents   or   articles   before   any   application   SECTION  9.  BOND  AND  ITS  CONDITIONS.  –    
inter  partes  can  be  made;  and     The   applicant   shall   be   required   to   post   a   cash   bond,  
  surety   bond   or   other   equivalent   security   executed   in  
(e)   the   documents   and   articles   to   be   seized   favor   of   the   defendant   or   expected   adverse   party   in   a  
constitute   evidence   of   the   alleged   infringing   reasonable   amount   to   be   fixed   by   the   court   in   its   order  
defendant’s   or   expected   adverse   party’s   granting  the  issuance  of  a  writ  of  search  and  seizure.  The  
infringing  activity  or  that  they  infringe  upon  the   bond   shall   be   conditioned   on   the   undertaking   of   the  
intellectual   property   right   of   the   applicant   or   applicant   that   he   will   pay   all   the   costs   which   may   be  
that   they   are   used   or   intended   to   be   used   as   adjudged  to  defendant  or  expected  adverse  party  and  all  
means   of   infringing   the   applicant’s   intellectual   damages   which   the   latter   may   sustain   by   reason   of   the  
property  right.   issuance  of  the  writ.    
   
SECTION  7.  WHEN  WRIT  MAY  ISSUE.  –     SECTION  10.  WHEN  WRIT  SHALL  BE  SERVED .  –    
If   the   judge   is   satisfied   with   the   proof   of   facts   upon   The   writ   shall   be   served   only   on   weekdays   and   from   8  
which   the   application   is   based,   he   shall   issue   the   writ   o'clock   in   the   morning   to   5   o'clock   in   the   afternoon.  
requiring  the  search,  inspection  or  copying  of  the  subject   However,  the  court  may  direct  that  the  writ  be  served  on  
documents   or   articles   or   commanding   the   sheriff   to   take  

o 41
Katrina Michelle Mancao
 
any   day   and   any   time   for   compelling   reasons   stated   in   charge   or   in   control   of   the   premises   or   residing   or  
the  application  and  duly  proved.     working   therein   who   shall   be   given   the   opportunity   to  
  read   the   writ   before   its   enforcement   and   seek   its  
SECTION  11.  TO  WHOM  WRIT  SHALL  BE  SERVED.  –     interpretation   from   the   Commissioner.   In   the   absence   of  
The   writ   shall   be   served   on   the   alleged   infringing   the   latter,   two   persons   of   sufficient   age   and   discretion  
defendant   or   expected   adverse   party   in   the   place   to   be   residing  in  the  nearest  locality.    
searched.      
  SECTION   14.   MANNER   OF   SEARCH   AND   SEIZURE;   DUTIES   OF   THE  
If   the   alleged   infringing   defendant   or   expected   adverse   SHERIFF.  –    
party  cannot  be  found  in  the  premises,  the  writ  shall  be   Upon   service   of   the   writ   in   accordance   with   section   11  
served  on  his  agent  or  representative.  In  the  absence  of   hereof,   the   sheriff,   under   the   supervision   of   the  
an   agent   or   representative,   it   shall   be   served   on   the   Commissioner,   shall   search   for   the   documents   and  
person   in   charge   or   in   control   of   the   premises,   or   articles   specified   in   the   writ,   and   take   them   in   his  
residing  or  working  therein  who  is  of  sufficient  age  and   custody  subject  to  the  control  of  the  court.    
discretion.  If  such  person  is  absent,  the  sheriff  or  proper    
officer   shall   post   the   papers   on   the   premises   and   If   the   subject   articles   are   not   capable   of   manual   delivery,  
proceed  with  the  enforcement  of  the  writ.     the  sheriff  shall  attach  to  them  a  tag  or  label  stating  the  
  fact   of   seizure   and   warning   all   persons   from   tampering  
SECTION  12.  COMMISSIONER,  DUTIES,  QUALIFICATIONS  AND  FEES.  –     with  them.    
The   enforcement   of   the   writ   shall   be   supervised   by   the    
independent   Commissioner   appointed   by   the   court   in   The   sheriff   shall,   in   the   presence   of   the   applicant   or   his  
the  performance  of  his  duty,  the  Commissioner  shall:     representative,   and   under   the   supervision   of   the  
  Commissioner,   prepare   a   detailed   list   of   the   seized  
(a)   give   impartial   advice   to   the   alleged   documents  and  articles.  He  shall  give  an  accurate  copy  of  
infringing   defendant,   expected   adverse   party   the   same   to   the   alleged   infringing   defendant,   expected  
or   to   the   person   in   charge   of   the   premises   to   adverse   party,   his   agent   or   representative,   to   the   person  
be  searched  as  to  the  meaning  and  coverage  of   in   charge   or   in   control   of   the   premises   or   residing   or  
the  writ.     working   therein   in   whose   presence   the   search   and  
(b)  attempt  to  achieve  agreement  on  a  suitable   seizure   were   made.   In   the   absence   of   the   person   in  
search  procedure     charge   or   in   control   of   the   premises   or   residing   or  
  working   therein,   the   sheriff   must,   in   the   presence   of   at  
(c)   assess   what   documents   or   articles   come   least   two   witnesses   of   sufficient   age   and   discretion  
within  the  terms  of  the  writ     residing  in  the  same  locality,  leave  a  copy  of  the  receipt  
  in   the   place   in   which   he   found   the   seized   property.  
(d)   ensure   the   accuracy   of   the   list   of   Where   no   witnesses   are   available   in   the   same   locality,  
documents   and   articles   searched,   inspected,   the  copy  of  the  receipt  shall  be  left  by  the  sheriff  in  the  
copied  or  seized  by  the  sheriff;     presence   of   two   witnesses   residing   in   the   nearest  
  locality   The   applicant   or   his   representative   and   the  
(e)   prepare   his   own   report   on   the   search   and   Commissioner  shall  also  be  given  a  copy  of  the  receipt.    
seizure  and  verify  and  sign  the  return  prepared    
by  the  sheriff,  and     After  the  sheriff  has  taken  possession  of  the  documents  
  and   articles,   he   shall   deliver   them   to   a   bonded  
(f)   generally,   assist   in   the   proper   execution   of   warehouse   or   government   warehouse   for   safekeeping.  
the  writ.   The   applicant   or   his   representative   shall   be   allowed  
  access   to   said   materials   for   the   purpose   of   examining  
The   Commissioner   shall   be   a   member   of   the   Philippine   them.    
Bar   and   of   proven   competence,   integrity   and   probity.   He    
shall   receive   such   reasonable   compensation   as   may   be   The   applicant   shall   be   responsible   for   the   necessary  
determined  by  the  court  which  can  be  charged  as  cost  of   expenses  incurred  in  the  seizure  and  safekeeping  of  the  
suit.     documents   and   articles   in   a   bonded   warehouse   or  
  government  warehouse.      
SECTION   13.   SEARCH   TO   BE   CONDUCTED   IN   THE   PRESENCE   OF    
DEFENDANT,   HIS   REPRESENTATIVE,   PERSON   IN   CHARGE   OF   THE   SECTION  15.  USE  OF  REASONABLE  FORCE  TO  EFFECT  WRIT.  –    
PREMISES,  OR  WITNESSES.  –     The   sheriff,   if   refused   admittance   to   the   premises   after  
The   premises   may   not   be   searched   except   in   the   giving   notice   of   his   purpose   and   authority   or   in   the  
presence   of   the   alleged   infringing   defendant,   expected   absence  of  the  alleged  infringing  defendant  or  expected  
adverse   party   or   his   representative   or   the   person   in   adverse   party,   his   agent   or   representative,   or   person   in  
charge   or   in   control   of   the   premises   or   residing   or  

42 z
Intellectual Property Law

working   therein   who   is   of   sufficient   age   and   discretion,    


may   use   reasonable   force   to   gain   entry   to   the   premises   (b)  that  the  bond  is  insufficient;    
or   any   part   of   the   building   or   anything   therein,   to    
enforce   the   writ   or   to   liberate   himself   or   any   person   (c)   that   the   safeguards   provided   in   the   writ  
lawfully  aiding  him  when  unlawfully  detained  therein.     have   been   violated   by   the   applicant   or   the  
  sheriff;  or    
SECTION   16.   SEIZURE   OF   COMPUTER   DISKS   OR   OTHER   STORAGE    
DEVICES.  –     (d)   that   the   documents   and   articles   seized   are  
The   seizure   of   a   computer   disk   or   any   storage   device   not   infringing   copies   or   means   for   making   the  
may  be  executed  in  any  of  the  following  manner:     materials   alleged   to   infringe   the   intellectual  
  property  right  of  the  applicant.  
(a)  by  the  physical  taking  thereof,      
  The   writ   may   be   discharged   in   a   summary   hearing   by   the  
(b)   by   copying   its   contents   in   suitable   device   or   court   after   notice   to   the   applicant,   the   sheriff   and   the  
disk  provided  by  the  applicant;  or     Commissioner.    
   
(c)   by   printing   out   the   contents   of   the   disk   or   If   the   court   finds   that   the   bond   is   insufficient,   it   shall  
device  with  the  use  of  a  printer.   order   a   new   bond   to   be   filed   by   the   applicant   within   a  
  reasonable  time.  The  discharge  of  the  writ  based  on  the  
When   the   computer   disks   or   storage   device   cannot   be   insufficiency   of   the   bond   may   only   be   made   if   the  
readily   removed   from   the   computer   to   which   they   are   applicant   fails   to   post   the   new   bond   within   the   period  
fitted,   the   sheriff   may   take   the   subject   computer   from   fixed  by  the  court.    
the   custody   of   the   alleged   infringing   defendant,    
expected   adverse   party   or   person   in   charge   or   in   control   SECTION  19.  PROCEEDINGS  ON  RETURN.  –    
of  the  premises  or  residing  or  working  therein.       Five  (5)  days  after  issuance  of  the  writ,  the  issuing  judge  
  shall   ascertain   if   the   writ   has   not   been   served   or   the  
SECTION  17.  SHERIFF’S  RETURN.  –     return  has  been  made  by  the  sheriff.  If  the  writ  was  not  
The  sheriff  who  executed  the  writ  shall,  within  three  (3)   served   or   no   return   was   made,   it   shall   summon   the  
days   from   its   enforcement,   make   a   verified   return   to   the   sheriff   and   the   applicant   to   whom   the   writ   was   issued  
court   which   issued   the   writ.   The   return   shall   contain   a   and  require  them  to  explain  why  the  writ  was  not  served  
full   statement   of   the   proceedings   under   the   writ   and   a   or  why  no  return  has  been  filed  as  the  case  may  be.  If  the  
complete   inventory   of   the   documents   and   articles   return  has  been  made,  the  judge  shall,  after  notice  to  the  
searched,   inspected   or   copied   or   seized   and   impounded,   applicant,   the   alleged   infringing   defendant   or   expected  
with   copies   served   on   the   applicant,   the   defendant   or   adverse   party,   the   sheriff   and   the   Commissioner,  
expected  adverse  party  and  the  Commissioner.     ascertain   whether   the   provisions   of   this   Rule   and  
  applicable  laws  have  been  complied  with.    
If   not   all   of   the   documents   and   articles   enumerated   in    
the  order  and  writ  were  seized,  the  sheriff  shall  so  report   SECTION  20.  FAILURE  TO  FILE  COMPLAINT.  –    
to   the   court   and   state   the   reasons   therefor.   All   The  writ  shall  also,  upon  motion  of  the  expected  adverse  
objections   of   the   defendant   expected   adverse   party   or   party,  be  set  aside  and  the  seized  documents  and  articles  
person   in   charge   of   the   premises   on   the   manner   and   returned   to   the   expected   adverse   party   if   no   case   is   filed  
regularity  of  the  service  of  the  writ  shall  be  included  by   with  the  appropriate  court  or  authority  within  thirty-­‐one  
the  sheriff  in  his  return.       (31)  calendar  days  from  the  date  of  issuance  of  the  writ.    
   
SECTION   18.   DISCHARGE  OF  WRIT  BY   THE  DEFENDANT   OR  EXPECTED   SECTION  21.  CLAIM  FOR  DAMAGES.  –    
ADVERSE  PARTY.  –     When   the   writ   is   discharged   on   any   of   the   grounds  
Without  waiting  for  the  return  to  be  filed  by  the  sheriff,   provided  in  this  Rule,  or  where  it  is  found  after  trial  that  
the   defendant,   expected   adverse   party   or   the   party   there   has   been   no   infringement   or   threat   of  
whose   property   has   been   searched,   inspected,   copied   or   infringement   of   an   intellectual   property   right,   the   court  
seized  may  file  a  motion  with  the  court  which  issued  the   upon   motion   of   the   alleged   infringing   defendant   or  
writ   for   its   discharge   with   prayer   for   the   return   of   the   expected  adverse  and  after  due  hearing,  shall  order  the  
documents  and  articles  seized.     applicant   to   compensate   the   defendant   or   expected  
  adverse  party  upon  the  cash  bond,  surety  bond  or  other  
The   writ   may   be   discharged   on   any   of   the   following   equivalent   security   for   any   injury   or   damage   the   latter  
grounds:     suffered   by   the   issuance   and   enforcement   of   the   writ.  
  Should   the   damages   exceed   the   amount   of   the   bond,  
(a)   that   the   writ   was   improperly   or   irregularly   the   applicant   shall   be   liable   for   the   payment   of   the  
issued,  or  excessively  enforced;     excess.    

o 43
Katrina Michelle Mancao
 
   
When   a   complaint   is   already   filed   in   court,   the   motion   SECTION  24.  SEPARATE  LOGBOOK.  –    
shall   be   filed   with   the   same   court   during   the   trial   or   In   every   court,   there   shall   be   a   logbook   under   the  
before  appeal  is  perfected  or  before  judgment  becomes   custody  of  the  Clerk  of  Court  wherein  shall  be  docketed  
executory,  with  due  notice  to  the  applicant  setting  forth   and   entered   within   twenty-­‐four   (24)   hours   after   the  
the  facts  showing  the  defendant’s  right  to  damages  and   issuance   or   denial   of   the   writ   search   and   seizure,   the  
amount  thereof.  The  award  of  damages  shall  be  included   filing   of   such   application   and   other   particulars   thereof.  
in  the  judgment  in  the  main  case.     All   the   subsequent   proceedings   concerning   the   writ   of  
  search   and   seizure   shall   be   faithfully   recorded   in   the  
Where   no   complaint   is   filed   against   the   expected   separate  logbook.    
adverse   party,   the   motion   shall   be   filed   with   the   court    
which  issued  the  writ.  In  such  a  case,  the  court  shall  set   SECTION  25.  EFFECT  OF  VIOLATION.  –    
the   motion   for   summary   hearing   and   immediately   A   violation   of   any   of   the   terms   and   conditions   of   the  
determine   the   expected   adverse   party’s   right   to   order   and   the   writ   of   search   and   seizure   or   any   provision  
damages.     of  the  Rule  shall  constitute  contempt  of  court.    
   
A   judgment   in   favor   of   the   applicant   in   its   principal   claim   SECTION  26.  WRIT  NOT  A  BAR  TO  OTHER  MEASURES.  –    
should   not   necessarily   bar   the   alleged   infringing  
The   availment   of   the   writ   of   search   and   seizure   under  
defendant   from   recovering   damages   where   he   suffered  
this   Rule   shall   not   prevent   the   applicant   from   resorting  
losses   by   reasons   of   the   wrongful   issuance   or  
to   other   provisional   measures   or   remedies   provided   in  
enforcement  of  the  writ.    
existing  laws  and  procedural  rules.    
 
 
The   damages   provided   f   or   in   this   section   shall   be  
SECTION  27.  EFFECTIVITY.  –    
independent   from   the   damages   claimed   by   the  
defendant  in  his  counterclaim     This   Rule   shall   take   effect   on   February   15,   2002   after   its  
  publication  in  two  (2)  newspapers  of  general  circulation  
SECTION  22.  JUDGMENT.  –     not  later  than  January  30,  2002.      
 
If   it   appears   after   trial   that   the   seized   documents   and  
articles   are   bound   to   infringe   the   intellectual   property   RE:   CONSOLIDATION   OF   INTELLECTUAL   PROPERTY   COURTS  
right  of  the  applicant  or  that  they  constitute  the  means   WITH   COMMERCIAL   COURTS   [A.M.   NO .   03-­‐03-­‐03-­‐SC  
the   means   for   the   production   of   infringing   goods,   the   2003-­‐06-­‐17]  
court   shall   order   their   destruction   or   donation   to    
charitable,   educational   or   religious   institutions   with   the   RESOLUTION  
prohibition  against  bringing  the  same  in  to  the  channels    
of  commerce.  In  the  latter  case,  infringing  trademarks  or   WHEREAS,  to  implement  the  provisions  of  Section  5.2  of  
trade  names  found  on  labels,  tags  and  other  portions  of   Republic  Act  No.  8799  (The  Securities  Regulation  Code),  
the   infringing   materials   shall   be   removed   or   defaced   and   in   the   interest   of   a   speedy   and   efficient  
before   the   donation.   In   no   case   shall   the   infringing   administration  of  justice,  the  Supreme  Court  en  banc,  in  
materials  be  returned  to  the  defendant.     the  (a)  Resolution  dated  21  November  2000  (Annex  1),  4  
  July  2001  (Annex  1-­‐a),  12  November  2002  (Annex  1-­‐b),  and  
If   the   court   finds   no   infringement,   the   seized   material   9  July  2002  (Annex  1-­‐c),  all  issued  in  A.M.  No.  00-­‐11-­‐03-­‐SC;  
shall  be  immediately  returned  to  the  defendant.     (b)   Resolution   dated   27   August   2001   in   A.M.   No.   01-­‐5-­‐298  
  RTC   (Annex   2);   and   (c)   Resolution   dated   8   July   2002   in  
SECTION   23.   DIRECT   FILING,   PROVISIONAL   DOCKETING  AND  DEPOSIT   A.M.  No.  01-­‐12-­‐656-­‐RTC  (Annex  3),  resolved  to  designate  
OF  PRESCRIBED  FILING  FEE.  –     certain   branches   of   the   Regional   Trial   Courts   to   try   and  
The   Regional   Trial   Courts   specially   designated   to   try   decide   cases   formerly   cognizable   by   the   Securities   and  
violations   of   intellectual   property   rights   shall   keep   a   Exchange  Commission;  
distinct   and   separate   logbook   for   writs   of   search   and    
seizure.   The   application   for   a   writ   of   search   and   seizure   WHEREAS,   pursuant   to   the   same   Resolution,   sixty-­‐five  
filed   directly   with   the   said   courts   shall   be   given   a   (65)   Regional   Trial   Courts,   distributed   in   all   regions  
provisional   docket   number.   The   prescribed   filing   fee   (NCJR  and  Regions  1-­‐12),  were  designated  as  SEC  courts  
shall   be   deposited   with   the   branch   clerk   of   court   and   ("SEC   Courts"),   which   courts   have   presently   a   total   of  
properly   receipted   for   and   transmitted   to   the   Clerk   of   812  pending  SEC  cases  (see  Annex  6,  Table);  
Court  within  twenty-­‐four  (24)  hours  from  issuance  of  the    
order   granting   or   denying   the   application   for   said   writ.   If   WHEREAS,   in   A.O   No.   113-­‐95,   dated   2   October   1995,   as  
a  formal  complaint  is  filed  thereafter,  the  Clerk  of  Court   amended   by   A.O.   No.   104-­‐96,   dated   21   October   1996,   the  
may  make  a  reassessment  of  the  filing  fee.     Regional   Trial   Courts   in   the   National   Capital   Region   and  
  Regions   3,   4,   6,   7,   9,   10   and   11,   with-­‐twenty   seven   (27)  

44 z
Intellectual Property Law

judges,  were  specially  designated  to  try  and  decide  cases   the   pretrial   stage   in   civil   cases   or   those   where   any   of   the  
for   violations   of   Intellectual   Property   Rights   (Annex   4),   accused  has  been  arraigned  in  criminal  cases  which  shall  
and   to   ensure   the   speedy   disposition   of   cases   involving   be  retained  by  the  court  previously  assigned  to  try  them;  
violations   of   intellectual   property   rights   under   the    
Intellectual   Property   Code   (Rep.   Act   No.   8293),   the   4. The   Special   Commercial   Courts   shall   have  
Supreme   Court   en   banc,   in   A.M.   No.   02-­‐1-­‐11-­‐   SC,   dated   jurisdiction   over   cases   arising   within   their   respective  
February   19,   2002,   designated   the   Regional   Trial   Courts   territorial  jurisdiction  with  respect  to  the  National  Capital  
in   Regions   1,   2,   5,   8   and   12,   with   a   total   of   seven   (7)   Judicial  Region  and  within  the  respective  provinces  with  
judges,   and   Branch   24   of   the   Regional   Trial   Court   of   respect   to   the   First   to   Twelfth   Judicial   Regions.   Thus,  
Manila  with  one  (1)  judge,  as  Special  Intellectual  Property   cases  shall  be  filed  in  the  Office  of  the  Clerk  of  Court  in  
Courts  ("Special  IP  Courts")  (Annex  5)   the  official  station  of  the  designated  Special  Commercial  
  Court;  
WHEREAS,  pursuant  to  A.M.  No.  02-­‐1-­‐11  SC  and  A.O.  No.    
113-­‐95,   these   Special   IP   Courts   have   a   total   caseload   of   5. In   the   event   of   inhibition   of   the   judge   of   a  
503  cases.  Of  this  number  434  IP  cases  are  pending  in  the   designated   Special   Commercial   Court,   the   following  
NCJR  (Annex  6,  Table);   guidelines  shall  be  observed:  (a)  where  there  is  only  one  
  (1)   Special   Commercial   Court,   the   case   shall   be   raffled  
WHEREAS,   since   the   establishment   of   Special   IP   Courts   among   the   other   judges   in   the   station;   (b)   where   there  
(except   for   the   Special   IP   Courts   in   Manila),   15   are   two   (2)   Special   Commercial   Courts   in   the   station,   the  
designated  courts,  in  Regions  1,  2,  3,  4,  5,  6,  7,  8,  9  and  12   Executive   Judge   shall   immediately   assign   the   case   to   the  
have   zero   (0)   IP   cases,   and   do   not   warrant   their   other   Special   Commercial   Court;   and   (c)   in   case   of  
continued   designations   as   Intellectual   Property   Courts   inhibition   of   both   judges   of   the   Special   Commercial  
(Annex  7,  Table);   Courts,   the   Executive   Judge   shall   raffle   the   case   among  
  the  judges  in  the  station;  and  
WHEREAS,  intellectual  property  cases  are  commercial  in    
nature;   6. In   order   to   ensure   a   just   and   equitable  
  distribution  of  cases,  the  designated  Special  Commercial  
WHEREAS,   to   streamline   the   court   structure   and   to   Courts  shall  continue  to  participate  in  the  raffles  of  other  
promote   expediency   and   efficiency   in   handling   such   cases.   Provided,   however,   that   the   Executive   Judge  
special  cases,  the  jurisdiction  to  hear  and  decide  IPC  and   concerned  shall  adopt  a  procedure  whereby  every  IP  and  
SEC  cases  are  best  consolidated  in  one  court;   SEC  case  assigned  to  a  Special  Commercial  Court  should  
  be   considered   a   case   raffled   to   it   and   duly   credited   to  
NOW,  THEREFORE,  the  Court  Resolves:   such  court.  
     
1. The   Regional   Courts   previously   designated   as   This  Resolution  shall  take  effect  on  1  July  2003  and  shall  
SEC   Courts   through   the:   (a)   Resolutions   of   this   Court   be   published   in   two   (2)   newspapers   of   general  
dated  21  November  2000,  4  July  2001,  12  November  2002,   circulation.  
and   9   July   2002,   all   issued   in   A.M.   No.   00-­‐11-­‐03-­‐SC,   (b)    
Resolution   dated   27   August   2001   in   A.M.   No.   01-­‐5-­‐298-­‐ 17  June  2003  
RTC;   and   (c)   Resolution   dated   8   July   2002   in   A.M.   No.   01-­‐  
12-­‐656-­‐RTC   are   hereby   DESIGNATED   and   shall   be   CALLED   20TH  CENTURY  FOX  FILM  CORP.  V.  CA,  BARRETO,  SAGULLO,  
as   Special   Commercial   Courts   to   try   and   decide   cases  
LEDESMA  (1988)  
involving  violations  of  Intellectual  Property  Rights  which  
fall   within   their   jurisdiction   and   those   cases   formerly   Refresher:  
In   a   letter-­‐complaint   dated   August   26,   1985,   petitioner  
cognizable  by  the  Securities  and  Exchange  Commission;  
20th   Century   Fox   Film   Corporation   through   counsel  
 
sought   the   NBI’s   assistance   in   the   conduct   of   searches  
2. The  designation  of  Intellectual  Property  Courts  
under   Administrative   Order   No.   113-­‐95   dated   2   October   and   seizures   in   connection   with   the   latter's   anti-­‐film  
piracy   campaign.   Specifically,   the   letter-­‐complaint  
1995,   as   amended   by   Administrative   Order   No.   104-­‐96  
alleged   that   certain   videotape   outlets   all   over   Metro  
dated   21   October   1996   and   Resolution   dated   19   February  
Manila   are   engaged   in   the   unauthorized   sale   and   renting  
2002  in  A.M.  No.  02-­‐1-­‐11-­‐SC,  is  hereby  revoked.  However,  
out   of   copyrighted   films   in   videotape   form   which  
the   Regional   Trial   Court,   Branch   24,   Manila   is   hereby  
designated  as  an  additional  Special  Commercial  Court  in   constitute  a  flagrant  violation  of  PD  49.  
 
the  City  of  Manila;  
Acting   on   the   letter-­‐complaint,   the   NBI   conducted  
 
surveillance   and   investigation   of   the   outlets   pinpointed  
3. Upon   the   effectivity   of   this   Resolution,   all   IP  
cases   shall   be   transferred   to   the   designated   Special   by   the   petitioner   and   subsequently   filed   3   applications  
for   search   warrants   against   the   video   outlets   owned   by  
Commercial  Courts  except  those  which  have  undergone  
the  private  respondents.    

o 45
Katrina Michelle Mancao
 
   
On   September   4,   1985,   the   lower   court   issued   the   Acting   on   the   applications,   then   RTC   Judge   Maria   Alicia  
desired  search  warrants.   M.   Austria   conducted   a   joint   hearing   during   which   she  
  made   a   personal   examination   of   the   applicant   and   his  
Armed   with   the   search   warrants,   the   NBI   accompanied   witnesses.   Finding   just   and   probable   cause   for   granting  
by   the   petitioner's   agents,   raided   the   video   outlets   and   the   application   at   the   time,   Judge   Austria   issued   the  
seized   the   items   described   therein.   An   inventory   of   the   corresponding   Search   Warrants   numbered   95,   96,   and  
items   seized   was   made   and   left   with   the   private   97.  
respondents.   However,   the   lower   court   subsequently    
lifted  the  search  warrants  issued.   Doctrine:  
  It  is  not  correct  to  say  that  "the  basic  fact"  to  be  proven  
Issue:  whether  or  not  the  judge  properly  lifted  the  search   to  establish  probable  cause  in  the  instant  cases  is  not  the  
warrants   he   issued   earlier   upon   the   application   of   the   "unauthorized   transfer"   of   a   motion   picture   that   has  
National   Bureau   of   Investigation   on   the   basis   of   the   been   recorded   but   the   "sale,   lease,   or   distribution   of  
complaint  filed  by  the  petitioner.   pirated  video  tapes  of  copyrighted  films."  
   
Held:     In   applying   for   the   search   warrants   the   NBI   charged  
The   presentation   of   the   master   tapes   of   the   copyrighted   violation  of  the  entire  provisions  of  Section  56  of  P.D.  No.  
films  from  which  the  pirated  films  were  allegedly  copied,   49  as  amended  by  P.D.  No.  1988.  This  included  not  only  
was   necessary   for   the   validity   of   search   warrants   against   the   sale,   lease   or   distribution   of   pirated   tapes   but   also  
those  who  have  in  their  possession  the  pirated  films.  The   the   transfer   or   causing   to   be   transferred   of   any   sound  
petitioner's  argument  to  the  effect  that  the  presentation   recording  or  motion  picture  or  other  audio  visual  work.  
of  the  master  tapes  at  the  time  of  application  may  not  be    
necessary  as  these  would  be  merely  evidentiary  in  nature   But   even   assuming,   as   appellants   argue,   that   only   the  
and   not   determinative   of   whether   or   not   a   probable   sale,   lease,   or   distribution   of   pirated   video   tapes   is  
cause   exists   to   justify   the   issuance   of   the   search   involved,  the  fact  remains  that  there  is  need  to  establish  
warrants   is   not   meritorious.   The   court   cannot   presume   probable   cause   that   the   tapes   being   sold,   leased   or  
that   duplicate   or   copied   tapes   were   necessarily   distributed   are   pirated   tapes,   hence   the   issue   reverts  
reproduced  from  master  tapes  that  it  owns.   back  to  the  question  of  whether  there  was  unauthorized  
  transfer,   directly   or   indirectly,   of   a   sound   recording   or  
The   application   for   search   warrants   was   directed   against   motion  picture  or  other  audio  visual  work  that  has  been  
video   tape   outlets   which   allegedly   were   engaged   in   the   recorded.  
unauthorized   sale   and   renting   out   of   copyrighted   films    
belonging  to  the  petitioner  pursuant  to  P.D.  49.   COLUMBIA  PICTURES,  INC.,  ET  AL.  V.  CA  (1996)  
  Refresher:  
The  essence  of  a  copyright  infringement  is  the  similarity   Complainants   thru   counsel   lodged   a   formal   complaint  
or  at  least  substantial  similarity  of  the  purported  pirated   with   the   NBI   for   violation   of   PD   49,   as   amended,   and  
works   to   the   copyrighted   work.   Hence,   the   applicant   sought   its   assistance   in   their   anti-­‐film   piracy   drive.    
must   present   to   the   court   the   copyrighted   films   to   Agents   of   the   NBI   and   private   researchers   made   discreet  
compare  them  with  the  purchased  evidence  of  the  video   surveillance   on   various   video   establishments   in   Metro  
tapes   allegedly   pirated   to   determine   whether   the   latter   Manila  including  Sunshine  Home  Video  Inc.  (Sunshine  for  
is   an   unauthorized   reproduction   of   the   former.   This   brevity),  owned  and  operated  by  Danilo  A.  Pelindario.  
linkage  of  the  copyrighted  films  to  the  pirated  films  must    
be   established   to   satisfy   the   requirements   of   probable   NBI   Senior   Agent   Lauro   C.   Reyes   applied   for   a   search  
cause.   Mere   allegations   as   to   the   existence   of   the   warrant   with   the   court   a   quo   against   Sunshine   seeking  
copyrighted  films  cannot  serve  as  basis  for  the  issuance   the   seizure,   among   others,   of   pirated   video   tapes   of  
of  a  search  warrant.   copyrighted  films  all  of  which  were  enumerated  in  a  list  
  attached   to   the   application;   and,   television   sets,   video  
COLUMBIA  PICTURES,  INC.,  ET  AL.  V.  CA  (1994)   cassettes   and/or   laser   disc   recordings   equipment   and  
Refresher:   other   machines   and   paraphernalia   used   or   intended   to  
On   07   April   1988,   the   NBI,   through   its   Agent   Lauro   C.   be   used   in   the   unlawful   exhibition,   showing,  
Reyes,  filed  with  the  Pasig  RTC  3  applications  for  search   reproduction,   sale,   lease   or   disposition   of   videograms  
warrant   against   private   respondents   Tube   Video   tapes  in  the  premises  above  described.    
Enterprises   and   Edward   C.   Cham,   the   Blooming   Rose    
Tape  Center  and  Ma.  Jajorie  T.  Uy,  and  the  Video  Channel   Main   issue:   Retroactive   application   to   the   present  
and   Lydia   Nabong,   charging   said   respondents   with   controversy   of   the   ruling   in   20th   Century   Fox   Film  
violation   of   Section   56   of   PD   49,   as   amended   by   P.D.   No.   Corporation  vs.  Court  of  Appeals,  et  al.,  promulgated  on  
1988.   August  19,  1988,  that  for  the  determination  of  probable  

46 z
Intellectual Property Law

cause   to   support   the   issuance   of   a   search   warrant   in   done  after  the  ten-­‐year   period   has   lapsed   —   since   1960  
copyright   infringement   cases   involving   videograms,   the   and   1964   were   both   leap   years   and   the   case   was   thus  
production   of   the   master   tape   for   comparison   with   the   filed   two   (2)   days   too   late.   Since   this   case   was   filed   on  
allegedly  pirated  copies  is  necessary.   September   3,   1965,   it   was   filed   one   day   too   late;  
  considering   that   the   730th   day   fell   on   September   2,   1965  
Doctrine:   —  the  year  1964  being  a  leap  year.  
Mindful  as  we  are  of  the  ramifications  of  the  doctrine  of    
stare   decisis   and   the   rudiments   of   fair   play,   it   is   our   LAKTAW  V.  PAGLINAWAN  (1918)  
considered  view  that  the  20th  Century  Fox  ruling  cannot   Refresher:  
be  retroactively  applied  to  the  instant  case  to  justify  the   In  the  complaint  presented  in  the  Court  of  First  Instance  
quashal  of  Search  Warrant  No.  87-­‐053.    Herein  petitioners’   of   the   City   of   Manila   on   February   20,   1915,   it   was   alleged:  
consistent   position   that   the   order   of   the   lower   court   of   (1)   That   the   plaintiff   was,   according   to   the   laws  
September   5,   1988   denying   therein   defendants’   motion   regulating   literary   properties,   the   registered   owner   and  
to   lift   the   order   of   search   warrant   was   properly   issued,   author   of   a   literary   work   entitled   Diccionario   Hispano-­‐
there  having  been  satisfactory  compliance  with  the  then   Tagalog   (Spanish-­‐Tagalog   Dictionary)   published   in   the  
prevailing  standards  under  the  law  for  determination  of   City   of   Manila   in   1889   by   the   printing   establishment   La  
probable   cause,   is   indeed   well   taken.     The   lower   court   Opinion,   and   a   copy   of   which   was   attached   to   the  
could   not   possibly   have   expected   more   evidence   from   complaint,   as   Exhibit   A;   (2)   that   the   defendant,   without  
petitioners  in  their  application  for  a  search  warrant  other   the   consent   of   the   plaintiff,   reproduced   said   literary  
than   what   the   law   and   jurisprudence,   then   existing   and   work,  improperly  copied  the  greater  part  thereof  in  the  
judicially   accepted,   required   with   respect   to   the   finding   work   published   by   him   and   entitled   Diccionariong  
of  probable  cause.   Kastila-­‐Tagalog   (Spanish-­‐Tagalog   Dictionary),   a   copy   of  
  which   was   also   attached   to   the   complaint   as   Exhibit   B;  
PEOPLE  V.  RAMOS  (1978)   (3)  that  said  act  of  the  defendant,  which  is  a  violation  of  
Refresher:   article   7   of   the   Law   of   January   10,   1879,   on   Intellectual  
Soccorro   Ramos,   as   the   proprietor   aid   general   manager   Property,   caused   irreparable   injuries   to   the   plaintiff,   who  
of   the   National   Book   Store,   as   enterprise   engaged   in   the   was  surprised  when,  on  publishing  his  new  work  entitled  
business   of   publishing,   selling   and   distributing   books,   Diccionario   Tagalog-­‐Hispano   (Tagalog-­‐Spanish  
was   charged   with   copyright   infringement   for   willfully   Dictionary)   he   learned   of   the   fact,   and   (4)   that   the  
and  illegally  selling  and  distributing  spurious  and  pirated   damages  occasioned  to  the  plaintiff  by  the  publication  of  
copies   of   the   high   school   textbook,   entitled   General   defendant's   work   amounted   to   $10,000.   The   plaintiff  
Science  Today  for  Philippine  School,  First  Year,  by  Gilam,   therefore   prayed   the   court   to   order   the   defendant   to  
Van  Houten  and  Cornista,  which  was  duly  copyrighted  by   withdraw   from   sale   all   stock   of   the   work   herein  
the   Phoenix   Publishing   House,   Inc.,   and   was   being   identified   as   Exhibit   B   and   to   pay   the   plaintiff   the   sum   of  
distributed  exclusively  by  its  sister  corporation,  Alemar's   $10,000,  with  costs.  
or  Sibal  and  Sons,  Inc.  The  alleged  act  happened  on  July    
–   September   1963.   The   criminal   cases   were   filed   on   Doctrine:  
September  1965.   In   the   judgment   appealed   from,   the   court   gives   one   to  
  understand  that  the  reproduction  of  another's  dictionary  
Issue:  Whether  or  not  the  extra  day  in  the  leap  year,  1964   without   the   owner's   consent   does   not   constitute   a  
should  be  taken  into  consideration  in  the  computation  of   violation   of   the   Law   of   Intellectual   Property   for   the  
the   two-­‐year   period   of   prescription   provided   in   Section   court's   idea   of   a   dictionary   is   stated   in   the   decision   itself,  
24  of  the  copyright  law.   as  follows:  
   
Held:   Dictionaries   have   to   be   made   with   the   aid   of  
Resolution   of   this   issue   hinges,   in   turn,   on   whether   others,   and   they   are   improved   by   the   increase  
February   28,   and   29   of   a   leap   year,   1964,   should   be   of   words.   What   may   be   said   of   a   pasture  
counted  as  one  day,  as  proposed  by  the  prosecution;  or   ground   may   be   said   also   of   a   dictionary,   i.   e.,  
as  separate  days,  as  alleged  by  the  defense.   that  it  should  be  common  property  for  all  who  
  may   desire   to   write   a   new   dictionary,   and   the  
This  issue  which  was  in  1965  still  undetermined  is  now  a   defendant   has   come   to   this   pasture   ground  
settled  matter.  It  was  held  in  1969  in  Namarco  vs.  Tuazon     and   taken   whatever   he   needed   from   it   in   the  
that  February  28  and  29  of  a  leap  year  should  be  counted   exercise  of  a  perfect  right.  
as   separate   days   in   computing   periods   of   prescription.    
Thus,   this   Court,   speaking   thru   former   Chief   Justice   Such   idea   is   very   erroneous,   especially   in   relation   to   the  
Roberto   Concepcion,   held   that   where   the   prescriptive   Law   of   Intellectual   Property.   Danvilla   y   Collado   the  
period   was   supposed   to   commence   on   December   21,   author   of   the   Law   of   January   10,   1879,   on   Intellectual  
1955,  the  filing  of  the  action  on  December  21,  1965,  was   Property,   which   was   discussed   and   approved   in   the  

o 47
Katrina Michelle Mancao
 
Spanish   Cortes,   in   his   work   entitled   La   Propiedad   thereof.   If   the   author   of   a   book,   after   its  
Intelectual   (page   362,   1st   ed.)   states   with   respect   to   publication,   cannot   prevent   its   reproduction   by  
dictionaries  and  in  relation  to  article  7  of  said  law:   any   person   who   may   want   to   reproduce   it,  
  then  the  property  right  granted  him  is  reduced  
The  protection  of  the  law  cannot  be  denied  to   to   a   very   insignificant   thing   and   the   effort  
the   author   of   a   dictionary,   for   although   words   made   in   the   production   of   the   book   is   no   way  
are   not   the   property   of   anybody,   their   rewarded.  
definitions,   the   example   that   explain   their    
sense,   and   the   manner   of   expressing   their   Indeed   the   property   right   recognized   and   protected   by  
different   meanings,   may   constitute   a   special   the   Law   of   January   10,   1879,   on   Intellectual   Property,  
work.   On   this   point,   the   correctional   court   of   would  be  illusory  if,  by  reason  of  the  fact  that  said  law  is  
the   Seine   held,   on   August   16,   1864,   that   a   no   longer   in   force   as   a   consequence   of   the   change   of  
dictionary   constitutes   property,   although   some   sovereignty   in   these   Islands,   the   author   of   a   work,   who  
of   the   words   therein   are   explained   by   mere   has  the  exclusive  right  to  reproduce  it,  could  not  prevent  
definitions   expressed   in   a   few   lines   and   another   person   from   so   doing   without   his   consent,   and  
sanctioned   by   usage,   provided   that   the   greater   could   not   enforce   this   right   through   the   courts   of   justice  
part  of  the  other  words  contain  new  meanings;   in  order  to  prosecute  the  violator  of  this  legal  provision  
new   meanings   which   evidently   may   only   and   the   defrauder   or   usurper   of   his   right,   for   he   could  
belonged   to   the   first   person   who   published   not   obtain   the   full   enjoyment   of   the   book   or   other   work,  
them.   and   his   property   right   thereto,   which   is   recognized   by  
  law,   would   be   reduced,   as   Manresa   says,   to   an  
In   addition   to   what   has   been   said,   according   to   article   insignificant  thing,  if  he  should  have  no  more  right  than  
428   of   the   Civil   Code,   the   author   of   a   literary,   scientific,   that  of  selling  his  work.  
or   artistic   work,   has   the   right   to   exploit   it   and   dispose    
thereof   at   will.   In   relation   to   this   right,   there   exists   the   HABANA,   CINCO   AND   FERNANDO   V.   ROBLES   AND  
exclusive  right  of  the  author,  who  is  the  absolute  owner  
GOODWILL  TRADING  CO.,  INC.  (1999)  
of  his  own  work,  to  produce  it,  according  to  article  2  of  
the   Law   of   January   10,   1879,   and   consequently,   nobody   Refresher:  
Petitioners   are   authors   and   copyright   owners   of   duly  
may   reproduce   it,   without   his   permission,   not   even   to  
issued   certificates   of   copyright   registration   covering  
annotate   or   add   something   to   it,   or   to   improve   any  
their  published  works,  produced  through  their  combined  
edition   thereof,   according   to   article   7   of   said   law.  
resources   and   efforts,   entitled   COLLEGE   ENGLISH   FOR  
Manresa,   in   his   commentaries   on   article   429   of   the   Civil  
Code   (vol.   3,   p.   633,   3d   ed.)   says   that   the   concrete   TODAY  (CET  for  brevity),  Books  1  and  2,  and  WORKBOOK  
FOR  COLLEGE  FRESHMAN  ENGLISH,  Series  1.  
statement   of   the   right   to   literary   properties   is   found   in  
 
the   legal   doctrine   according   to   which   nobody   may  
Respondent   Felicidad   Robles   and   Goodwill   Trading   Co.,  
reproduce   another   person's   work,   without   the   consent  
of   his   owner,   or   even   to   annotate   or   add   something   to   it   Inc.   are   the   author/publisher   and   distributor/seller   of  
another  published  work  entitled  “DEVELOPING  ENGLISH  
or   to   improve   any   edition   thereof.   And   on   page   616   of  
PROFICIENCY”   (DEP   for   brevity),   Books   1   and   2   (1985  
said  volume,  Manresa  says  the  following:  
edition)  which  book  was  covered  by  copyrights  issued  to  
 
them.  
He   who   writes   a   book,   or   carves   a   statue,   or  
 
makes   an   invention,   has   the   absolute   right   to  
In   the   course   of   revising   their   published   works,  
reproduce   or   sell   it,   just   as   the   owner   of   land  
petitioners   scouted   and   looked   around   various  
has  the  absolute  right  to  sell  it  or  its  fruits.  But  
bookstores  to  check  on  other  textbooks  dealing  with  the  
while   the   owner   of   land,   by   selling   it   and   its  
fruits,   perhaps   fully   realizes   all   its   economic   same   subject   matter.   By   chance   they   came   upon   the  
book   of   respondent   Robles   and   upon   perusal   of   said  
value,   by   receiving   its   benefits   and   utilities,  
book   they   were   surprised   to   see   that   the   book   was  
which  are  presented,  for  example,  by  the  price,  
strikingly  similar  to  the  contents,  scheme  of  presentation,  
on  the  other  hand  the  author  of   a  book,  statue  
illustrations  and  illustrative  examples  in  their  own  book,  
or  invention,  does  not  reap  all  the  benefits  and  
CET.  
advantages  of  his  own  property  by  disposing  of  
 
it,  for  the  most  important  form  of  realizing  the  
After   an   itemized   examination   and   comparison   of   the  
economic   advantages   of   a   book,   statue   or  
two   books   (CET   and   DEP),   petitioners   found   that   several  
invention,   consists   in   the   right   to   reproduce   it  
in   similar   or   like   copies,   everyone   of   which   pages   of   the   respondent’s   book   are   similar,   if   not   all  
together   a   copy   of   petitioners’   book,   which   is   a   case   of  
serves  to  give  to  the  person  reproducing  them  
plagiarism  and  copyright  infringement.  
all  the  conditions  which  the  original  requires  in  
 
order   to   give   the   author   the   full   enjoyment  

48 z
Intellectual Property Law

Respondent   Robles   denied   the   allegations   of   plagiarism   may  be  true.    However,  in  this  jurisdiction  under  Sec  184  
and   copying   that   petitioners   claimed.     Respondent   of  Republic  Act  8293  it  is  provided  that:  
stressed   that   (1)   the   book   DEP   is   the   product   of   her    
independent   researches,   studies   and   experiences,   and   Limitations   on   Copyright.   Notwithstanding   the  
was   not   a   copy   of   any   existing   valid   copyrighted   book;   provisions  of  Chapter  V,  the  following  shall  not  
(2)   DEP   followed   the   scope   and   sequence   or   syllabus   constitute  infringement  of  copyright:  
which   are   common   to   all   English   grammar   writers   as    
recommended   by   the   Association   of   Philippine   Colleges   x  x  x  x    
of  Arts  and  Sciences  (APCAS),  so  any  similarity  between    
the   respondents   book   and   that   of   the   petitioners   was   (c)  The  making  of  quotations  from  a  published  
due  to  the  orientation  of  the  authors  to  both  works  and   work   if   they   are   compatible   with   fair   use   and  
standards   and   syllabus;   and   (3)   the   similarities   may   be   only   to   the   extent   justified   for   the   purpose,  
due   to   the   authors’   exercise   of   the   “right   to   fair   use   of   including   quotations   from   newspaper   articles  
copyrighted  materials,  as  guides.”   and  periodicals  in  the  form  of  press  summaries:    
  Provided,  That  the  source  and  the  name  of  the  
Doctrine:   author,   if   appearing   on   the   work,   are  
We   believe   that   respondent   Robles’   act   of   lifting   from   mentioned.  
the   book   of   petitioners   substantial   portions   of    
discussions   and   examples,   and   her   failure   to   A  copy  of  a  piracy  is  an  infringement  of  the  original,  and  
acknowledge  the  same  in  her  book  is  an  infringement  of   it   is   no   defense   that   the   pirate,   in   such   cases,   did   not  
petitioners’  copyrights.   know  whether  or  not  he  was  infringing  any  copyright;  he  
  at   least   knew   that   what   he   was   copying   was   not   his,   and  
When   is   there   a   substantial   reproduction   of   a   book?     It   he  copied  at  his  peril.  
does   not   necessarily   require   that   the   entire   copyrighted    
work,  or  even  a  large  portion  of  it,  be  copied.    If  so  much   The   next   question   to   resolve   is   to   what   extent   can  
is   taken   that   the   value   of   the   original   work   is   copying   be   injurious   to   the   author   of   the   book   being  
substantially   diminished,   there   is   an   infringement   of   copied.   Is   it   enough   that   there   are   similarities   in   some  
copyright   and   to   an   injurious   extent,   the   work   is   sections  of  the  books  or  large  segments  of  the  books  are  
appropriated.   the  same?  
   
In  determining  the  question  of  infringement,  the  amount   In   cases   of   infringement,   copying   alone   is   not   what   is  
of   matter   copied   from   the   copyrighted   work   is   an   prohibited.     The   copying   must   produce   an     “injurious  
important   consideration.     To   constitute   infringement,   it   effect”.     Here,   the   injury   consists   in   that   respondent  
is   not   necessary   that   the   whole   or   even   a   large   portion   Robles  lifted  from  petitioners’  book  materials  that  were  
of  the  work  shall  have  been  copied.    If  so  much  is  taken   the   result   of   the   latter’s   research   work   and   compilation  
that   the   value   of   the   original   is   sensibly   diminished,   or   and  misrepresented  them  as  her  own.    She  circulated  the  
the  labors  of  the  original  author  are  substantially  and  to   book   DEP   for   commercial   use   and   did   not   acknowledge  
an   injurious   extent   appropriated   by   another,   that   is   petitioners  as  her  source.  
sufficient  in  point  of  law  to  constitute  piracy.    
  Hence,   there   is   a   clear   case   of   appropriation   of  
The   essence   of   intellectual   piracy   should   be   essayed   in   copyrighted  work  for  her  benefit  that  respondent  Robles  
conceptual  terms  in  order  to  underscore  its  gravity  by  an   committed.     Petitioners’   work   as   authors   is   the   product  
appropriate   understanding   thereof.     Infringement   of   a   of  their  long  and  assiduous  research  and  for  another  to  
copyright   is   a   trespass   on   a   private   domain   owned   and   represent   it   as   her   own   is   injury   enough.     In   copyrighting  
occupied  by  the  owner  of  the  copyright,  and,  therefore,   books   the   purpose   is   to   give   protection   to   the  
protected   by   law,   and   infringement   of   copyright,   or   intellectual  product  of  an  author.    This  is  precisely  what  
piracy,   which   is   a   synonymous   term   in   this   connection,   the  law  on  copyright  protected,  under  Section  184.1  (b).    
consists   in   the   doing   by   any   person,   without   the   consent   Quotations  from  a  published  work  if  they  are  compatible  
of  the  owner  of  the  copyright,  of  anything  the  sole  right   with   fair   use   and   only   to   the   extent   justified   by   the  
to  do  which  is  conferred  by  statute  on  the  owner  of  the   purpose,   including   quotations   from   newspaper   articles  
copyright.   and   periodicals   in   the   form   of   press   summaries   are  
  allowed   provided   that   the   source   and   the   name   of   the  
The   respondents’   claim   that   the   copied   portions   of   the   author,  if  appearing  on  the  work,  are  mentioned.  
book   CET   are   also   found   in   foreign   books   and   other    
grammar   books,   and   that   the   similarity   between   her   In   the   case   at   bar,   the   least   that   respondent   Robles  
style   and   that   of   petitioners   can   not   be   avoided   since   could   have   done   was   to   acknowledge   petitioners  
they   come   from   the   same   background   and   orientation   Habana  et.  al.  as  the  source  of  the  portions  of  DEP.    The  
final   product   of   an   author’s   toil   is   her   book.     To   allow  

o 49
Katrina Michelle Mancao
 
another   to   copy   the   book   without   appropriate   the  terms  thereof.    In  his  reply  dated  February  17,  1986,  
acknowledgment  is  injury  enough.   Vergara   protested   the   unilateral   action   of   SMI,   saying   it  
  was   without   basis.     In   the   same   letter,   he   pushed   for   the  
JOAQUIN   AND   BJ   PRODUCTIONS,   INC.   V.   DRILON,   ET   AL.   signing  of  the  contract  for  SM  Cubao.  
 
(1999),  SUPRA  
Two  years  later,  Metro  Industrial  Services,  the  company  
Refresher:  
formerly   contracted   by   Pearl   and   Dean   to   fabricate   its  
Rhoda  and  Me  and  It’s  a  Date  dating  game  show.  
display   units,   offered   to   construct   light   boxes   for  
 
Shoemart’s  chain  of  stores.    SMI  approved  the  proposal  
Doctrine:  
and  ten  (10)  light  boxes  were  subsequently  fabricated  by  
P.D.   No.   49,   2,   in   enumerating   what   are   subject   to  
Metro   Industrial   for   SMI.     After   its   contract   with   Metro  
copyright,  refers  to  finished  works  and  not  to  concepts.  
Industrial   was   terminated,   SMI   engaged   the   services   of  
The   copyright   does   not   extend   to   an   idea,   procedure,  
EYD  Rainbow  Advertising  Corporation  to  make  the  light  
process,  system,  method  of  operation,  concept,  principle,  
boxes.     Some   300   units   were   fabricated   in   1991.     These  
or   discovery,   regardless   of   the   form   in   which   it   is   were  delivered  on  a  staggered  basis  and  installed  at  SM  
described,   explained,   illustrated,   or   embodied   in   such  
Megamall  and  SM  City.  
work.  
 
 
Sometime  in  1989,  Pearl  and  Dean,  received  reports  that  
PEARL  &  DEAN  (PHIL),  INC.  V.  SHOEMART,  INC.  AND  NORTH   exact   copies   of   its   light   boxes   were   installed   at   SM   City  
EDSA  MARKETING,  INC.  (2003)   and   in   the   fastfood   section   of   SM   Cubao.     Upon  
Refresher:   investigation,  Pearl  and  Dean  found  out  that  aside  from  
Plaintiff-­‐appellant   Pearl   and   Dean   (Phil.),   Inc.   is   a   the  two  (2)  reported  SM  branches,  light  boxes  similar  to  
corporation   engaged   in   the   manufacture   of   advertising   those   it   manufactures   were   also   installed   in   two   (2)  
display   units   simply   referred   to   as   light   boxes.     These   other   SM   stores.     It   further   discovered   that   defendant-­‐
units   utilize   specially   printed   posters   sandwiched   appellant  North  Edsa  Marketing  Inc.  (NEMI),  through  its  
between  plastic  sheets  and  illuminated  with  back  lights.     marketing  arm,  Prime  Spots  Marketing  Services,  was  set  
Pearl   and   Dean   was   able   to   secure   a   Certificate   of   up   primarily   to   sell   advertising   space   in   lighted   display  
Copyright  Registration  dated  January  20,  1981  over  these   units  located  in  SMI’s  different  branches.    Pearl  and  Dean  
illuminated   display   units.     The   advertising   light   boxes   noted  that  NEMI  is  a  sister  company  of  SMI.  
were  marketed  under  the  trademark  “Poster  Ads”.    The    
application   for   registration   of   the   trademark   was   filed   Doctrine:  
with  the  Bureau  of  Patents,  Trademarks  and  Technology   On  copyright  infringement:  
Transfer   on   June   20,   1983,   but   was   approved   only   on   Petitioner’s  application  for  a  copyright  certificate  clearly  
September   12,   1988,   per   Registration   No.   41165.     From   stated   that   it   was   for   a   class   “O”   work   under   Section  
1981   to   about   1988,   Pearl   and   Dean   employed   the   2(O)   of   PD   49   (The   Intellectual   Property   Decree)   which  
services   of   Metro   Industrial   Services   to   manufacture   its   was   the   statute   then   prevailing.     Said   Section   2   expressly  
advertising  displays.   enumerated  the  works  subject  to  copyright:  
   
Sometime   in   1985,   Pearl   and   Dean   negotiated   with   (O)   Prints,   pictorial   illustrations,   advertising  
defendant-­‐appellant   Shoemart,   Inc.   (SMI)   for   the   lease   copies,  labels,  tags,  and  box  wraps;  
and  installation  of  the  light  boxes  in  SM  City  North  Edsa.      
Since   SM   City   North   Edsa   was   under   construction   at   that   Although   petitioner’s   copyright   certificate   was   entitled  
time,   SMI   offered   as   an   alternative,   SM   Makati   and   SM   “Advertising   Display   Units”   (which   depicted   the   box-­‐
Cubao,  to  which  Pearl  and  Dean  agreed.    On  September   type   electrical   devices),   its   claim   of   copyright  
11,   1985,   Pearl   and   Dean’s   General   Manager,   Rodolfo   infringement  cannot  be  sustained.  
Vergara,   submitted   for   signature   the   contracts   covering    
SM   Cubao   and   SM   Makati   to   SMI’s   Advertising   Copyright,   in   the   strict   sense   of   the   term,   is   purely   a  
Promotions   and   Publicity   Division   Manager,   Ramonlito   statutory  right.    Being  a  mere  statutory  grant,  the  rights  
Abano.    Only  the  contract  for  SM  Makati,  however,  was   are   limited   to   what   the   statute   confers.     It   may   be  
returned   signed.     On   October   4,   1985,   Vergara   wrote   obtained   and   enjoyed   only   with   respect   to   the   subjects  
Abano  inquiring  about  the  other  contract  and  reminding   and   by   the   persons,   and   on   terms   and   conditions  
him   that   their   agreement   for   installation   of   light   boxes   specified   in   the   statute.   Accordingly,   it   can   cover   only   the  
was   not   only   for   its   SM   Makati   branch,   but   also   for   SM   works   falling   within   the   statutory   enumeration   or  
Cubao.    SMI  did  not  bother  to  reply.   description.  
   
Instead,   in   a   letter   dated   January   14,   1986,   SMI’s   house   P  &  D  secured  its  copyright  under  the  classification  class  
counsel   informed   Pearl   and   Dean   that   it   was   rescinding   “O”   work.     This   being   so,   petitioner’s   copyright  
the   contract   for   SM   Makati   due   to   non-­‐performance   of   protection   extended   only   to   the   technical   drawings   and  

50 z
Intellectual Property Law

not  to  the  light  box  itself  because  the  latter  was  not  at  all   Issue:  Validity  of  the  rule?    
in   the   category   of   “prints,   pictorial   illustrations,   -­‐ Q:   If   you   were   a   commercial   court   judge  
advertising   copies,   labels,   tags   and   box   wraps.”     Stated   hearing   a   criminal   case   (1st   offense)   and   a  
otherwise,   even   as   we   find   that   P   &   D   indeed   owned   a   motion   to   quash   was   filed   by,   how   would   you  
valid  copyright,  the  same  could  have  referred  only  to  the   uphold  your  jurisdiction?  
technical   drawings   within   the   category   of   “pictorial   o Note:   BP   129,   jurisdiction   for   the   1st  
illustrations.”    It  could  not  have  possibly  stretched  out  to   offense  is  lodged  in  the  MTC  
include  the  underlying  light  box.    The  strict  application  of    
the   law’s   enumeration   in   Section   2   prevents   us   from   HW  Answer  (Marie  Estrella)  
giving   petitioner   even   a   little   leeway,   that   is,   even   if   its   Jurisdiction  over  the  subject  matter  of  a  case  is  
copyright   certificate   was   entitled   “Advertising   Display   conferred   by   law   and   is   determined   by   the  
Units.”    What  the  law  does  not  include,  it  excludes,  and   allegations   of   the   complaint,   irrespective   of  
for   the   good   reason:   the   light   box   was   not   a   literary   or   whether   the   plaintiff   is   entitled   to   all   or   some  
artistic   piece   which   could   be   copyrighted   under   the   of  the  claims  asserted  therein.  (Intestate  Estate  
copyright   law.     And   no   less   clearly,   neither   could   the   lack   of  Alexander  Ty  v.  Court  of  Appeals,  356  SCRA  
of   statutory   authority   to   make   the   light   box   661)   What   determines   the   jurisdiction   of   the  
copyrightable   be   remedied   by   the   simplistic   act   of   court   in   criminal   cases   is   the   extent   of   the  
entitling  the  copyright  certificate  issued  by  the  National   penalty   which   the   law   imposes   for   the  
Library  as  “Advertising  Display  Units.”   misdemeanor,  crime  or  violation  charged  in  the  
  complaint.   (People   vs.   Fajardo,   49   Phil.,   206,  
On  unfair  competition:   210)  
By   the   nature   of   things,   there   can   be   no   unfair    
competition   under   the   law   on   copyrights   although   it   is   With   the   penal   provisions   of   the   Intellectual  
applicable  to  disputes  over  the  use  of  trademarks.  Even   Property   Code,   we   are   baffled   with   issue,  
a   name   or   phrase   incapable   of   appropriation   as   a   “ Which  court  has  jurisdiction.”  The  provision  
trademark  or  tradename  may,  by  long  and  exclusive  use   of  the  Code  on  jurisdiction  is  not  any  much  help  
by   a   business   (such   that   the   name   or   phrase   becomes   either.  It  simply  provides  that  without  prejudice  
associated   with   the   business   or   product   in   the   mind   of   to   the   provisions   of   Subsection   7.1(c),   actions  
the  purchasing  public),  be  entitled  to  protection  against   under  this  Act  shall  be  cognizable  by  the  courts  
unfair   competition.   In   this   case,   there   was   no   evidence   with   appropriate   jurisdiction   under   existing  
that  P  &  D’s  use  of  “Poster  Ads”  was  distinctive  or  well-­‐ law.   The   existing   law   conferring   jurisdiction   to  
known.   As   noted   by   the   Court   of   Appeals,   petitioner’s   our  courts  is  Batas  Pambansa  Blg.  129  (BP  129),  
expert  witnesses  himself  had  testified  that  “  ‘Poster  Ads’   or   the   Judiciary   Reorganization   Act   of   1980.  
was  too  generic  a  name.  So  it  was  difficult  to  identify  it   Under   Section   32   of   BP   129,   Municipal   Trial  
with   any   company,   honestly   speaking.”   This   crucial   Courts   (MTC)   shall   have   exclusive   original  
admission   by   its   own   expert   witness   that   “Poster   Ads”   jurisdiction   over   all   offenses   punishable   with  
could  not  be  associated  with  P  &  D  showed  that,  in  the   imprisonment   not   exceeding   six   (6)   years  
mind   of   the   public,   the   goods   and   services   carrying   the   irrespective   of   the   amount   of   fine,   and  
trademark   “Poster   Ads”   could   not   be   distinguished   from   regardless   of   all   other   impossible   of   accessory  
the  goods  and  services  of  other  entities.   penalties.   Taking   only   this   into   consideration,  
  we   are   led   to   conclude   that   with   the   first   and  
12  January  2012   second   offenses,   the   MTC   has   jurisdiction.  
  However,   we   also   know   that   all   IP   cases   are  
Infringement   heard   by   the   Regional   Trial   Court   (RTC)   acting  
-­‐ Violation   of   the   rights   exclusively   granted   by   as   Special   Commercial   Court.   (A.M.   N0.   03-­‐03-­‐
law  to  the  copyright  owner.   03-­‐SC,  JULY  01,  2003)  
-­‐ Exception:    
o Fair  use   To   reconcile   the   seeming   conflict,   I   think   it   is  
o Limitations  to  copyright   also   useful   to   refer   to   Section   23   of   BP   129  
  which  enables  the  Supreme  Court  to  designate  
Remedies:   certain   branches   of   the   Regional   Trial   Courts   to  
1. Civil   handle   exclusively   criminal   cases,   juvenile   and  
2. Criminal   domestic   cases,   agrarian   cases,   urban   land  
3. Administrative   reform   cases   which   do   not   fall   under   the  
  jurisdiction   of   quasi-­‐judicial   bodies   and  
NOTE:   SC   issued   a   circular   à   IP   case   shall   be   filed   in   agencies  and/or  such  other  special  cases  as  the  
courts  designated  as  commercial  courts   Supreme   Court   may   determine   in   the   interest  
  of   a   speedy   and   efficient   administration   of  

o 51
Katrina Michelle Mancao
 
justice.   With   this   provision,   the   Supreme   Court,   nationwide  
to   streamline   and   promote   expediency   and    
efficiency,  designated  the  Regional  Trial  Courts,   Criminal  penalties  
as   Special   Commercial   Courts   to   exclusively   -­‐ No  accessories/  accomplices  
hear  IP  cases.   o It  is  a  special  law  
  -­‐ Infringer  and  those  who  aids  and  abets  =  same  
I  think,  in  the  construction  of  laws  such  as  the   penalty  
one   before   us,   we   are   enjoined   to   read   and   -­‐ Note:   the   law   does   not   have   a   definition   of  
construe   them   together   and   try   to   reconcile   “aiding  or  abetting”  =  quite  broad  
them.  In  this  case,  I  think  we  can  reconcile  the    
IPC   and   BP   129   by   construing   that   IPC   is   a   Issue:  Can  a  consumer  be  liable  for  aiding  or  abetting?  
special  law  and,  as  such,  is  an  exception  to  the    
general   rule   that   jurisdiction   is   determined   by   HW  Answer  (Katrina  Mancao):  
the  extent  of  the  penalty  the  law  imposes.  We   The   buyer   of   products   that   infringe   on   the  
can  also  use  the  interpretation  that  the  general   copyright  of  another  may  not  be  held  liable  for  
law  yields  to  the  special  law.   aiding  and  abetting.  
   
Lastly,  taking  into  consideration  the  purpose  of   Section   217.1   of   the   Intellectual   Property   Code  
BP   129,   which   I   think   is   to   ultimately   promote   makes   any   person   aiding   or   abetting   an  
justice,   expediency   and   efficiency,   the   infringement   criminally   liable.   The   law,  
designation   of   the   RTC   as   Special   Commercial   however,  does  not  define  what  actions  may  be  
Courts  works  to  towards  this  end.       considered   as   aiding   or   abetting.   Unlike   the  
  other   acts   of   infringement,   this   provision   is  
Civil  Remedies:   vague,   and   thus   calls   for   an   application   of   the  
1. Injunction  –  this  is  a  critical  provisional  remedy   rule   on   statutory   construction.   On   this   issue,   I  
so  that  the  infringing  activity  stops   propose   that   the   provision   be   construed  
2. Damages   –   plaintiff   is   only   required   to   prove   liberally  in  favor  of  the  buyers.  
gross  sales    
o If  gross  sales  was  not  proved  –  court   The  act  of  aiding  and  abetting  an  infringement  
is  given  leeway   imposes   a   criminal   liability   upon   the   person  
3. Impounding   charged   with   the   offense.   The   first   offense  
4. Destruction   alone   merits   an   imprisonment   of   1   year   to   3  
5. Other  damages   years,   plus   a   fine   ranging   from   P50,000.00   to  
  P150,000.00.   I   believe   that   the   gravity   the   law  
Criminal  remedy:   placed   on   the   offense,   and   the   excessiveness  
-­‐ Why   is   this   more   effective   than   civil   and   of  the  penalty  and  fines  imposed  on  it  calls  for  
administrative  remedies?   the   application   of   the   legal   maxim   nullum  
o Because   of   the   notoriety   of   criminal   crimen,  nulla  poena  sine  lege.  There  is  no  crime  
cases   à   defendant   has   reputation   when   there   is   no   law   punishing   it.   To   be  
concerns;   moreover,   people   value   punishable,   the   act   must   be   clearly   defined   by  
liberty   the  legislature  as  a  criminal  act.  The  Intellectual  
  Property  Code  does  not  explicitly  define  aiding  
How  do  you  best  initiate  case  against  infringers?   or  abetting.  Neither  does  it  punish  mere  buying  
-­‐ By  obtaining  a  search  warrant.   or   possession   of   an   infringing   material.   The  
o Timeliness     absence   of   any   intention   by   the   legislature   to  
o Ex  parte  (no  notice)   punish   the   buyers   of   an   infringing   material  
o No  premature  disclosure  of  the  fact     makes  it  not  punishable  under  our  jurisdiction.  
-­‐ Search  and  seizure  in  civil  actions    
o Ma’am:   Dangerous   to   constitutional   Moreover,  punishing  the  act  violates  Article  III,  
rights.   It   is   very   much   similar   to   a   Section  14,  paragraph  1  of  the  1987  Constitution  
criminal  search  warrant.   which   states,   “No   person   shall   be   held   to  
§ Differences:     answer   for   a   criminal   offense   without   due  
• Bond  requirement   process  of  law.”  The  constitutional  right  to  due  
• Independent   process   requires   that   the   law   accord   people  
commissioner   fair   notice   of   which   conduct   to   avoid.  
o Where  do  you  apply?   Otherwise,   the   law   may   be   said   to   undermine  
§ Metro  Manila   due   process   in   two   respects:   (1)   in   failing   to  
§ Writs   are   imposable   accord   persons   fair   notice   of   the   conduct   to  

52 z
Intellectual Property Law

avoid,  and  (2)  in  giving  law  enforcers  unbridled   imprisonment   in   cases   of  
discretion   in   implementing   the   provisions   of   insolvency.  
the   law.   The   provision   on   aiding   and   abetting    
does   just   that.   In   failing   to   define   or   describe   217.2.   In   determining   the   number   of  
acts   that   may   be   considered   as   falling   within   years   of   imprisonment   and   the  
the  provision,  it  does  not  reasonably  notify  the   amount   of   fine,   the   court   shall  
public  that  mere  buying  of  infringing  material  is   consider   the   value   of   the   infringing  
criminally   punishable.   It   does   not,   therefore,   materials   that   the   defendant   has  
afford  them  with  the  opportunity  to  stop  doing   produced   or   manufactured   and   the  
the  act  to  avoid  criminal  liability.   damage   that   the   copyright   owner   has  
  suffered   by   reason   of   the  
HW  Answer  (Marie  Estrella):   infringement.  
It   is   an   elementary   rule   of   statutory    
construction   that   penal   laws   are   strictly   217.3.   Any   person   who   at   the   time  
construed   against   the   State   and   liberally   in   when  copyright  subsists  in  a  work  has  
favor   of   the   accused.   (People   vs   Subido,   66   in   his   possession   an   article   which   he  
SCRA   545)     Penal   statutes   cannot   be   enlarged   knows,   or   ought   to   know,   to   be   an  
or   extended   by   intendment,   implication,   or   any   infringing   copy   of   the   work   for   the  
equitable   consideration.   (People   vs   Garcia,   85   purpose  of:  
Phil.  651)    
  (a)   Selling,   letting   for   hire,  
Section   217   of   the   Intellectual   property   Code   or   by   way   of   trade,   offering  
(IPC)  of  the  Philippines  provides:   or  exposing  for  sale,  or  hire,  
  the  article;  
Art.   217-­‐217.1.   Any   person   infringing    
any   right   secured   by   provisions   of   (b)   Distributing   the   article  
Part   IV   of   this   Act   or   aiding   or   for   purpose   of   trade   or   for  
abetting   such   infringement   shall   be   any   other   purpose   to   an  
guilty  of  a  crime  punishable  by:   extent   that   will   prejudice  
  the   rights   of   the   copyright  
(a)   Imprisonment   of   one   (1)   owner  in  the  work;  or  
year  to  three  (3)  years  plus  a    
fine   ranging   from   fifty   (c)   Trade   exhibit   of   the  
thousand   pesos   (P50,000)   article   in   public,   shall   be  
to   one   hundred   and   fifty   guilty   of   an   offense   and  
thousand   pesos   (P150,000)   shall   be   liable   on   conviction  
for  the  first  offense.   to  imprisonment  and  fine  as  
  above  mentioned.  
(b)   Imprisonment   of   three    
(3)   years   and   one   (1)   day   to   Certainly,   Section   217   punishes   acts   that  
six   (6)   years   plus   a   fine   infringe   or   aids   or   abets   the   infringement   of  
ranging   from   one   hundred   copyright   as   provided   in   the   chapter   on  
and   fifty   thousand   pesos   copyright.    There   is   no   dispute   as   to   the  
(P150,000)   to   five   hundred   definition   of   infringement,   that   is,   to   directly  
thousand   pesos   (P500,000)   violate   the   rights   of   the   copyright   owner   as  
for  the  second  offense.   provided   in   the   law.     Anyone   violating   any   of  
  the   rights   provided   in   Section   177,   and   other  
(c)   Imprisonment   of   six   (6)   sections,   of   the   IPC   is   liable   for  
years  and  one  (1)  day  to  nine   infringement.     There   is   no   question   as   to   the  
(9)  years  plus  a  fine  ranging   culpability  of  direct  infringers.    However,  there  
from  five  hundred  thousand   is   a   quandary   as   to   what   constitutes   “ aiding  
pesos   (P500,000)   to   one   and  abetting” .  
million   five   hundred    
thousand   pesos   There   seems   no   established   legal   definition   of  
(P1,500,000)   for   the   third   aiding   and   abetting.     The   phrase,   commonly  
and  subsequent  offenses.   used   in   criminal   prosecution,   connotes   the  
  commission   of   a   crime   without   direct  
(d)   In   all   cases,   subsidiary   involvement.     The  phrase  was  first  used  during  

o 53
Katrina Michelle Mancao
 
the   Nuremberg   trial   in   pinning   liability   on   the   cannot   interpret   the   law   to   include  
officers   and   directors   of   a   company   that   acts  which  it  does  not  penalize.  
manufactured  the  gas  used  by  the  Nazis  in  the    
5 Copyright  ALLOWS  parallel  creation.  Standard  
annihilation   of   the   Jews .      In   aiding   and  
abetting,   a   person   does   not   actually   and   only  is  originality.  To  prove  infringement,  you  
directly   performs   the   acts   constitutive   of   the   must  prove  ACCESS  +  COPYING.    
crime   but   however   is   instrumental   in   its    
commission   because   without   him,   the   crime   C. Related  Rights  
would  not  have  been  possible.      
 
1. Moral  Rights  
In   the   field   of   Intellectual   Property,   we   learn   of  
the  concept  “ contributory  infringement” .     In    
the   case   of   Sony   Corp.   vs   Universal   City   ARTICLE  6BIS.    
Studios,  contributory  infringement  was  used  in   Moral  Rights:    
referring   to   the   act   of   selling   a   machine   which   1.   To   claim   authorship;   to   object   to   certain  
for   the   absolute   purpose   of   committing   modifications  and  other  derogatory  actions;    
infringing   acts.     In   this   sense,   contributory   2.  After  the  author's  death;    
infringement   reflects   the   same   premise   in   3.  Means  of  redress  
aiding   and   abetting.     The   person   who   sells   the    
machine,   although   he   does   not   do   the   actual   (1)   Independently   of   the   author's   economic   rights,   and  
reproduction  of  copyrighted  materials,  is  guilty   even  after  the  transfer  of  the  said  rights,  the  author  shall  
of  contributory  infringement.     have   the   right   to   claim   authorship   of   the   work   and   to  
  object  to  any  distortion,  mutilation  or  other  modification  
Aiding   and   abetting   presupposes   that   the   of,   or   other   derogatory   action   in   relation   to,   the   said  
person   charged   has   actual   or   constructive   work,   which   would   be   prejudicial   to   his   honor   or  
knowledge   that   his   or   her   actions   are   likely   to   reputation.  
facilitate  infringement  by  another,  and  that  he    
6 (2)   The   rights   granted   to   the   author   in   accordance   with  
materially  contributes  to  the  infringement .    
the   preceding   paragraph   shall,   after   his   death,   be  
 
maintained,   at   least   until   the   expiry   of   the   economic  
In   all   the   above   definitions   of   aiding   and  
rights,   and   shall   be   exercisable   by   the   persons   or  
abetting,  and  contributory  negligence,  there  is  
institutions   authorized   by   the   legislation   of   the   country  
a   positive   act,   although   not   directly   infringing,  
where   protection   is   claimed.   However,   those   countries  
that   contributes   to   the   ultimate   act   of  
whose  legislation,  at  the  moment  of  their  ratification  of  
infringement.     This   positive   act   makes   the  
or   accession   to   this   Act,   does   not   provide   for   the  
infringement   possible.     Conversely,   without   the  
protection  after  the  death  of  the  author  of  all  the  rights  
positive   act   of   the   contributory   infringer,   the  
set   out   in   the   preceding   paragraph   may   provide   that  
infringement   would   not   have   been  
some   of   these   rights   may,   after   his   death,   cease   to   be  
committed.     This   is   not   the   case   with   the  
maintained.  
purchasers   of   pirated   DVDs.     The   act   of   the  
 
purchasers   in   buying   the   DVDs   is   not  
(3)   The   means   of   redress   for   safeguarding   the   rights  
necessary.     The   infringement   would   be  
granted   by   this   Article   shall   be   governed   by   the  
committed   even   without   them   and   their  
legislation  of  the  country  where  protection  is  claimed.  
patronage.    Having  said  this,  the  purchasers  of  
 
pirated   DVDs   or   other   copyrighted   materials  
cannot   be   held   criminally   liable.     The   IPC   SECTION  193.  SCOPE  OF  MORAL  RIGHTS.  –    
provisions   are   clear.     It   criminalizes   The   author   of   a   work   shall,   independently   of   the  
infringement   and   the   acts   of   aiding   and   economic   rights   in   Section   177   or   the   grant   of   an  
abetting   its   commission.     It   does   not   penalize   assignment   or   license   with   respect   to   such   right,   have  
the   purchase   of   infringing   materials.     We   the  right:  
 
                                                                                                                                                        193.1.   To   require   that   the   authorship   of   the  
5
 Prof.  Harry  Roque.  “Ampatuan  Victims  vs  Arroyo,”  blog  entry  
from   http://harryroque.com/tag/aiding-­‐and-­‐abetting/,   last   works   be   attributed   to   him,   in   particular,   the  
accessed  January  20,  2012.   right   that   his   name,   as   far   as   practicable,   be  
6
indicated   in   a   prominent   way   on   the   copies,  
 Mark   Bartholomew.   “Contributory   Infringers   and   Good   and   in   connection   with   the   public   use   of   his  
Samaritans,”   electronic   copy   from  
work;  
http://ssrn.com/abstract=1276286,   last   accessed   January   20,  
2012.    

54 z
Intellectual Property Law

193.2.   To   make   any   alterations   of   his   work   prior   SECTION  198.  TERM  OF  MORAL  RIGHTS.  –    
to,  or  to  withhold  it  from  publication;   198.1.  The  rights  of  an  author  under  this  chapter  shall  last  
  during  the  lifetime  of  the  author  and  for  fifty  (50)  years  
193.3.  To  object  to  any  distortion,  mutilation  or   after  his  death  and  shall  not  be  assignable  or  subject  to  
other   modification   of,   or   other   derogatory   license.   The   person   or   persons   to   be   charged   with   the  
action   in   relation   to,   his   work   which   would   be   posthumous  enforcement  of  these  rights  shall  be  named  
prejudicial  to  his  honor  or  reputation;  and   in   writing   to   be   filed   with   the   National   Library.   In   default  
  of   such   person   or   persons,   such   enforcement   shall  
193.4.   To   restrain   the   use   of   his   name   with   devolve  upon  either  the  author's  heirs,  and  in  default  of  
respect  to  any  work  not  of  his  own  creation  or   the  heirs,  the  Director  of  the  National  Library.  
in  a  distorted  version  of  his  work.      
  198.2.  For  purposes  of  this  Section,  "Person"  shall  mean  
SECTION  194.  BREACH  OF  CONTRACT.  –     any   individual,   partnership,   corporation,   association,   or  
An  author  cannot  be  compelled  to  perform  his  contract   society.   The   Director   of   the   National   Library   may  
to   create   a   work   or   for   the   publication   of   his   work   prescribe  reasonable  fees  to  be  charged  for  his  services  
already  in  existence.  However,  he  may  be  held  liable  for   in  the  application  of  provisions  of  this  Section.    
damages  for  breach  of  such  contract.      
  SECTION  199.  ENFORCEMENT  REMEDIES.  –    
SECTION  195.  WAIVER  OF  MORAL  RIGHTS.  –     Violation   of   any   of   the   rights   conferred   by   this   Chapter  
An   author   may   waive   his   rights   mentioned   in   Section   193   shall  entitle  those  charged  with  their  enforcement  to  the  
by  a  written  instrument,  but  no  such  waiver  shall  be  valid   same  rights  and  remedies  available  to  a  copyright  owner.  
where  its  effects  is  to  permit  another:   In  addition,  damages  which  may  be  availed  of  under  the  
  Civil  Code  may  also  be  recovered.  Any  damage  recovered  
195.1.   To   use   the   name   of   the   author,   or   the   after   the   creator's   death   shall   be   held   in   trust   for   and  
title   of   his   work,   or   otherwise   to   make   use   of   remitted   to   his   heirs,   and   in   default   of   the   heirs,   shall  
his   reputation   with   respect   to   any   version   or   belong  to  the  government.    
adaptation   of   his   work   which,   because   of    
alterations  therein,  would  substantially  tend  to   2. Right   to   Proceeds   in  
injure   the   literary   or   artistic   reputation   of   Subsequent   Transfers    
another  author;  or   (Droit   De   Suite   or   Follow  
 
195.2.   To   use   the   name   of   the   author   with   Up  Rights)  
respect  to  a  work  he  did  not  create.      
  SECTION  200.  SALE  OR  LEASE  OF  W ORK.  –    
SECTION  196.  CONTRIBUTION  TO  COLLECTIVE  WORK.  –     In   every   sale   or   lease   of   an   original   work   of   painting   or  
When   an   author   contributes   to   a   collective   work,   his   sculpture   or   of   the   original   manuscript   of   a   writer   or  
right   to   have   his   contribution   attributed   to   him   is   composer,   subsequent   to   the   first   disposition   thereof   by  
deemed  waived  unless  he  expressly  reserves  it.     the   author,   the   author   or   his   heirs   shall   have   an  
  inalienable   right   to   participate   in   the   gross   proceeds   of  
the  sale  or  lease  to  the  extent  of  five  percent  (5%).  This  
SECTION   197.   EDITING,   ARRANGING   AND   ADAPTATION   OF  
right  shall  exist  during  the  lifetime  of  the  author  and  for  
WORK.  –     fifty  (50)  years  after  his  death.    
In   the   absence   of   a   contrary   stipulation   at   the   time   an    
author   licenses   or   permits   another   to   use   his   work,   the   SECTION  201.  WORKS  NOT  COVERED.  –    
necessary  editing,  arranging  or  adaptation  of  such  work,   The   provisions   of   this   Chapter   shall   not   apply   to   prints,  
for   publication,   broadcast,   use   in   a   motion   picture,   etchings,   engravings,   works   of   applied   art,   or   works   of  
dramatization,   or   mechanical   or   electrical   reproduction   similar   kind   wherein   the   author   primarily   derives   gain  
in   accordance   with   the   reasonable   and   customary   from  the  proceeds  of  reproductions.    
standards   or   requirements   of   the   medium   in   which   the    
work   is   to   be   used,   shall   not   be   deemed   to   contravene  
3. Neighboring  Rights  
the   author's   rights   secured   by   this   chapter.   Nor   shall  
complete   destruction   of   a   work   unconditionally    
transferred   by   the   author   be   deemed   to   violate   such   SECTION  202.  DEFINITIONS.  –    
rights.   For   the   purpose   of   this   Act,   the   following   terms   shall  
  have  the  following  meanings:  
 

o 55
Katrina Michelle Mancao
 
202.1.   "PERFORMERS"   are   actors,   singers,   musicians,   Sections   203,   208   and   209   shall   not   apply   where   the   acts  
dancers,   and   other   persons   who   act,   sing,   declaim,   play   referred  to  in  those  Sections  are  related  to:  
in,   interpret,   or   otherwise   perform   literary   and   artistic    
work;   212.1.   The   use   by   a   natural   person   exclusively  
  for  his  own  personal  purposes;  
202.2.   "SOUND   RECORDING"   means   the   fixation   of   the    
sounds   of   a   performance   or   of   other   sounds,   or   212.2.   Using   short   excerpts   for   reporting  
representation   of   sound,   other   than   in   the   form   of   a   current  events;  
fixation   incorporated   in   a   cinematographic   or   other    
audiovisual  work;   212.3.  Use  solely  for  the  purpose  of  teaching  or  
  for  scientific  research;  and  
202.3.   An   "AUDIOVISUAL   WORK   OR   FIXATION"   is   a   work   that    
consists   of   a   series   of   related   images   which   impart   the   212.4.   Fair   use   of   the   broadcast   subject   to   the  
impression   of   motion,   with   or   without   accompanying   conditions  under  Section  185.    
sounds,   susceptible   of   being   made   visible   and,   where    
accompanied   by   sounds,   susceptible   of   being   made   a. Rights  of  Performers  
audible;  
 
 
SECTION  203.  SCOPE  OF  PERFORMERS'  RIGHTS.  –    
202.4.   "FIXATION"   means   the   embodiment   of   sounds,   or  
of   the   representations   thereof,   from   which   they   can   be   Subject   to   the   provisions   of   Section   212,   performers   shall  
enjoy  the  following  exclusive  rights:  
perceived,   reproduced   or   communicated   through   a  
 
device;  
  203.1.   As   regards   their   performances,   the   right  
202.5.   "PRODUCER  OF  A  SOUND  RECORDING"  means  the  person,   of  authorizing:  
 
or  the  legal  entity,  who  or  which  takes  the  initiative  and  
(a)   The   broadcasting   and   other   communication  
has  the  responsibility  for  the  first  fixation  of  the  sounds  
to  the  public  of  their  performance;  and  
of   a   performance   or   other   sounds,   or   the   representation  
of  sounds;    
(b)  The  fixation  of  their  unfixed  performance.  
 
 
202.6.   "PUBLICATION   OF   A   FIXED   PERFORMANCE   OR   A   SOUND  
203.2.   The   right   of   authorizing   the   direct   or  
RECORDING"   means   the   offering   of   copies   of   the   fixed  
indirect   reproduction   of   their   performances  
performance   or   the   sound   recording   to   the   public,   with  
fixed   in   sound   recordings,   in   any   manner   or  
the   consent   of   the   right   holder:   Provided,   That   copies  
form;  
are  offered  to  the  public  in  reasonable  quality;  
 
 
203.3.  Subject  to  the  provisions  of  Section  206,  
202.7.   "BROADCASTING"   means   the   transmission   by  
the   right   of   authorizing   the   first   public  
wireless   means   for   the   public   reception   of   sounds   or   of  
distribution   of   the   original   and   copies   of   their  
images  or  of  representations  thereof;  such  transmission  
performance   fixed   in   the   sound   recording  
by   satellite   is   also   "broadcasting"   where   the   means   for  
through   sale   or   rental   or   other   forms   of  
decrypting   are   provided   to   the   public   by   the  
transfer  of  ownership;  
broadcasting  organization  or  with  its  consent;  
 
 
203.4.   The   right   of   authorizing   the   commercial  
202.8.   "BROADCASTING  ORGANIZATION"  shall  include  a  natural  
rental  to  the  public  of  the  original  and  copies  of  
person  or  a  juridical  entity  duly  authorized  to  engage  in  
their   performances   fixed   in   sound   recordings,  
broadcasting;  and  
even  after  distribution  of  them  by,  or  pursuant  
 
to  the  authorization  by  the  performer;  and  
202.9   "COMMUNICATION  TO  THE  PUBLIC  OF  A  PERFORMANCE  OR  A    
SOUND  RECORDING"   means   the   transmission   to   the   public,   203.5.   The   right   of   authorizing   the   making  
by   any   medium,   otherwise   than   by   broadcasting,   of   available   to   the   public   of   their   performances  
sounds   of   a   performance   or   the   representations   of   fixed   in   sound   recordings,   by   wire   or   wireless  
sounds   fixed   in   a   sound   recording.   For   purposes   of   means,   in   such   a   way   that   members   of   the  
Section   209,   "communication   to   the   public"   includes   public  may  access  them  from  a  place  and  time  
making  the  sounds  or  representations  of  sounds  fixed  in   individually  chosen  by  them.    
a  sound  recording  audible  to  the  public.    
  SECTION  204.  MORAL  RIGHTS  OF  PERFORMERS.  –    
SECTION  212.  LIMITATIONS  ON  RIGHTS.  –    

56 z
Intellectual Property Law

204.1.   Independently   of   a   performer's   economic   rights,   215.2.   In   case   of   broadcasts,   the   term   shall   be   twenty  
the   performer,   shall,   as   regards   his   live   aural   (20)   years   from   the   date   the   broadcast   took   place.   The  
performances  or  performances  fixed  in  sound  recordings,   extended   term   shall   be   applied   only   to   old   works   with  
have  the  right  to  claim  to  be  identified  as  the  performer   subsisting  protection  under  the  prior  law.    
of   his   performances,   except   where   the   omission   is    
dictated   by   the   manner   of   the   use   of   the   performance,   b. Rights   of   Producers  
and   to   object   to   any   distortion,   mutilation   or   other  
of  Sound  Recording  
modification   of   his   performances   that   would   be  
 
prejudicial  to  his  reputation.  
  SECTION  208.  SCOPE  OF  RIGHT.  –    
204.2.   The   rights   granted   to   a   performer   in   accordance   Subject   to   the   provisions   of   Section   212,   producers   of  
with  Subsection  203.1  shall  be  maintained  and  exercised   sound   recordings   shall   enjoy   the   following   exclusive  
fifty  (50)  years  after  his  death,  by  his  heirs,  and  in  default   rights:  
of  heirs,  the  government,  where  protection  is  claimed.      
  208.1.   The   right   to   authorize   the   direct   or  
SECTION  205.  LIMITATION  ON  RIGHT.  –     indirect   reproduction   of   their   sound   recordings,  
in   any   manner   or   form;   the   placing   of   these  
205.1.  Subject  to  the  provisions  of  Section  206,  once  the  
reproductions   in   the   market   and   the   right   of  
performer  has  authorized  the  broadcasting  or  fixation  of  
rental  or  lending;  
his   performance,   the   provisions   of   Sections   203   shall  
 
have  no  further  application.  
208.2.   The   right   to   authorize   the   first   public  
 
distribution   of   the   original   and   copies   of   their  
205.2.  The  provisions  of  Section  184  and  Section  185  shall  
sound   recordings   through   sale   or   rental   or  
apply  mutatis  mutandis  to  performers.    
other  forms  of  transferring  ownership;  and  
 
 
SECTION   206.   ADDITIONAL   REMUNERATION   FOR   208.3.   The   right   to   authorize   the   commercial  
SUBSEQUENT  COMMUNICATIONS  OR  BROADCASTS.  –     rental  to  the  public  of  the  original  and  copies  of  
Unless   otherwise   provided   in   the   contract,   in   every   their   sound   recordings,   even   after   distribution  
communication   to   the   public   or   broadcast   of   a   by   them   by   or   pursuant   to   authorization   by   the  
performance   subsequent   to   the   first   communication   or   producer.  
broadcast  thereof  by  the  broadcasting  organization,  the    
performer   shall   be   entitled   to   an   additional   SECTION  209.  COMMUNICATION  TO  THE  PUBLIC.  –    
remuneration   equivalent   to   at   least   five   percent   (5%)   of   If   a   sound   recording   published   for   commercial   purposes,  
the  original  compensation  he  or  she  received  for  the  first   or   a   reproduction   of   such   sound   recording,   is   used  
communication  or  broadcast.   directly  for  broadcasting  or  for  other  communication  to  
  the  public,  or  is  publicly  performed  with  the  intention  of  
SECTION  207.  CONTRACT  TERMS.  –     making   and   enhancing   profit,   a   single   equitable  
Nothing   in   this   Chapter   shall   be   construed   to   deprive   remuneration   for   the   performer   or   performers,   and   the  
performers   of   the   right   to   agree   by   contracts   on   terms   producer   of   the   sound   recording   shall   be   paid   by   the  
and  conditions  more  favorable  for  them  in  respect  of  any   user   to   both   the   performers   and   the   producer,   who,   in  
use  of  their  performance.   the  absence  of  any  agreement  shall  share  equally.  
   
SECTION   215.   TERM   OF   PROTECTION   FOR   PERFORMERS,   SECTION  210.  LIMITATION  OF  RIGHT.  –    
PRODUCERS  AND  BROADCASTING  ORGANIZATIONS.  –     Sections  184  and  185  shall  apply  mutatis  mutandis  to  the  
215.1.  The  rights  granted  to  performers  and  producers  of   producer  of  sound  recordings.    
 
sound  recordings  under  this  law  shall  expire:  
  SECTION   215.   TERM   OF   PROTECTION   FOR   PERFORMERS,  
(a)   For   performances   not   incorporated   in   PRODUCERS  AND  BROADCASTING  ORGANIZATIONS.  –    
recordings,  fifty  (50)  years  from  the  end  of  the   215.1.  The  rights  granted  to  performers  and  producers  of  
year  in  which  the  performance  took  place;  and   sound  recordings  under  this  law  shall  expire:  
   
(b)   For   sound   or   image   and   sound   recordings   (a)   For   performances   not   incorporated   in  
and   for   performances   incorporated   therein,   recordings,  fifty  (50)  years  from  the  end  of  the  
fifty   (50)   years   from   the   end   of   the   year   in   year  in  which  the  performance  took  place;  and  
which  the  recording  took  place.    
  (b)   For   sound   or   image   and   sound  
recordings   and   for   performances   incorporated  

o 57
Katrina Michelle Mancao
 
therein,   fifty   (50)   years   from   the   end   of   the   digital   direct-­‐to-­‐home   (DTH)   television   via   satellite   to   its  
year  in  which  the  recording  took  place.   subscribers   all   over   the   Philippines.   Herein   individual  
  respondents  are  members  of  PMSI’s  Board  of  Directors.  
215.2.   In  case  of  broadcasts,  the  term  shall  be  twenty    
(20)   years   from   the   date   the   broadcast   took   place.   The   PMSI  was  granted  a  legislative  franchise  under  RA  86305  
extended   term   shall   be   applied   only   to   old   works   with   on  May  7,  1998  and  was  given  a  Provisional  Authority  by  
subsisting  protection  under  the  prior  law.   the   National   Telecommunications   Commission   (NTC)   on  
  February   1,   2000   to   install,   operate   and   maintain   a  
c. Rights   of   nationwide   DTH   satellite   service.   When   it   commenced  
operations,  it  offered  as  part  of  its  program  line-­‐up  ABS-­‐
Broadcasting   CBN  Channels  2  and  23,  NBN,  Channel  4,  ABC  Channel  5,  
Organizations   GMA   Channel   7,   RPN   Channel   9,   and   IBC   Channel   13,  
  together  with  other  paid  premium  program  channels.  
SECTION  211.  SCOPE  OF  RIGHT.  –      
Subject   to   the   provisions   of   Section   212,   broadcasting   However,  on  April  25,  2001,  ABS-­‐CBN  demanded  for  PMSI  
organizations   shall   enjoy   the   exclusive   right   to   carry   out,   to  cease  and  desist  from  rebroadcasting  Channels  2  and  
authorize  or  prevent  any  of  the  following  acts:   23.   On   April   27,   2001,   PMSI   replied   that   the  
  rebroadcasting   was   in   accordance   with   the   authority  
211.1.   The   rebroadcasting   of   their   granted   it   by   NTC   and   its   obligation   under   NTC  
Memorandum  Circular  No.  4-­‐08-­‐88,  Section  6.2  of  which  
broadcasts;  
requires   all   cable   television   system   operators   operating  
 
in   a   community   within   Grade   “A”   or   “B”   contours   to  
211.2.   The   recording   in   any   manner,  
carry   the   television   signals   of   the   authorized   television  
including   the   making   of   films   or   the   use   of   broadcast  stations.  
video  tape,  of  their  broadcasts  for  the  purpose    
of   communication   to   the   public   of   television   ABS-­‐CBN   contends   that   PMSI’s   unauthorized  
broadcasts  of  the  same;  and   rebroadcasting   of   Channels   2   and   23   is   an   infringement  
  of   its   broadcasting   rights   and   copyright   under   the  
211.3.   The   use   of   such   records   for   fresh   Intellectual  Property  Code  (IP  Code);  that  Memorandum  
transmissions  or  for  fresh  recording.   Circular   No.   04-­‐08-­‐88   excludes   DTH   satellite   television  
  operators;   that   the   Court   of   Appeals’   interpretation   of  
SUBSECTION  215.2.     the  must-­‐carry  rule  violates  Section  9  of  Article  III  of  the  
In   case   of   broadcasts,   the   term   shall   be   twenty   (20)   Constitution  because  it  allows  the  taking  of  property  for  
years   from   the   date   the   broadcast   took   place.   The   public  use  without  payment  of  just  compensation.  
extended   term   shall   be   applied   only   to   old   works   with    
subsisting  protection  under  the  prior  law.   Respondents,   on   the   other   hand,   argue   that   PMSI’s  
  rebroadcasting   of   Channels   2   and   23   is   sanctioned   by  
ABS-­‐CBN   BROADCASTING   COMMUNICATION  V.   PHILIPPINE   Memorandum  Circular  No.  04-­‐08-­‐88;  that  the  must-­‐carry  
rule   under   the   Memorandum   Circular   is   a   valid   exercise  
MULTIMEDIA  SYSTEM,  ET  AL.  (2009)  
of  police  power;  and  that  the  Court  of  Appeals  correctly  
Refresher:   dismissed  CA-­‐G.R.  SP  No.  90762  since  it  found  no  need  to  
ABS-­‐CBN   is   licensed   to   engage   in   television   and   radio   exercise  its  power  of  contempt.  
broadcasting.   It   broadcasts   television   programs   by    
wireless   means   to   Metro   Manila   and   nearby   provinces,   Doctrine:  
and  by  satellite  to  provincial  stations  through  Channel  2   Section  202.7  of  the  IP  Code  defines  broadcasting  as  “the  
on   Very   High   Frequency   (VHF)   and   Channel   23   on   Ultra   transmission  by  wireless  means  for  the  public  reception  
High   Frequency   (UHF).   The   programs   aired   over   of   sounds   or   of   images   or   of   representations   thereof;  
Channels   2   and   23   are   either   produced   by   ABS-­‐CBN   or   such   transmission   by   satellite   is   also   ‘broadcasting’  
purchased  from  or  licensed  by  other  producers.   where   the   means   for   decrypting   are   provided   to   the  
  public   by   the   broadcasting   organization   or   with   its  
ABS-­‐CBN   also   owns   regional   television   stations   which   consent.”  
pattern  their  programming  in  accordance  with  perceived    
demands  of  the  region.  Thus,  television  programs  shown   On   the   other   hand,   rebroadcasting   as   defined   in   Article  
in  Metro  Manila  and  nearby  provinces  are  not  necessarily   3(g)   of   the   International   Convention   for   the   Protection  
shown  in  other  provinces.   of   Performers,   Producers   of   Phonograms   and  
  Broadcasting   Organizations,   otherwise   known   as   the  
Respondent   Philippine   Multi-­‐Media   System,   Inc.   (PMSI)   1961   Rome   Convention,   of   which   the   Republic   of   the  
is  the  operator  of  Dream  Broadcasting  System.  It  delivers   Philippines   is   a   signatory,   is   “the   simultaneous  

58 z
Intellectual Property Law

broadcasting   by   one   broadcasting   organization   of   the   commercials,   etc.   In   general,   however,   the  
broadcast  of  another  broadcasting  organization.”   term   “retransmission”   seems   to   be   reserved  
  for   such   transmissions   which   are   both  
Under   the   Rome   Convention,   rebroadcasting   is   “the   simultaneous  and  unaltered.  
simultaneous   broadcasting   by   one   broadcasting    
organization   of   the   broadcast   of   another   broadcasting   49.  The  Rome  Convention  does  not  grant  rights  
organization.”   The   Working   Paper   prepared   by   the   against   unauthorized   cable   retransmission.  
Secretariat  of  the  Standing  Committee  on  Copyright  and   Without   such   a   right,   cable   operators   can  
Related   Rights   defines   broadcasting   organizations   as   retransmit  both  domestic  and  foreign  over  the  
“entities   that   take   the   financial   and   editorial   air   broadcasts   simultaneously   to   their  
responsibility  for  the  selection  and  arrangement  of,  and   subscribers   without   permission   from   the  
 
investment  in,  the  transmitted  content.” Evidently,  PMSI   broadcasting   organizations   or   other  
would  not  qualify  as  a  broadcasting  organization  because   rightholders   and   without   obligation   to   pay  
it   does   not   have   the   aforementioned   responsibilities   remuneration.  
imposed   upon   broadcasting   organizations,   such   as   ABS-­‐  
CBN.   Thus,   while   the   Rome   Convention   gives   broadcasting  
  organizations   the   right   to   authorize   or   prohibit   the  
ABS-­‐CBN   creates   and   transmits   its   own   signals;   PMSI   rebroadcasting   of   its   broadcast,   however,   this  
merely   carries   such   signals   which   the   viewers   receive   in   protection  does  not  extend  to  cable  retransmission.  The  
its   unaltered   form.   PMSI   does   not   produce,   select,   or   retransmission   of   ABS-­‐CBN’s   signals   by   PMSI   –   which  
determine  the  programs  to  be  shown  in  Channels  2  and  23.   functions   essentially   as   a   cable   television   –   does   not  
Likewise,   it   does   not   pass   itself   off   as   the   origin   or   author   therefore   constitute   rebroadcasting   in   violation   of   the  
of   such   programs.   Insofar   as   Channels   2   and   23   are   former’s  intellectual  property  rights  under  the  IP  Code.  
concerned,   PMSI   merely   retransmits   the   same   in    
accordance   with   Memorandum   Circular   04-­‐08-­‐88.   With   It   must   be   emphasized   that   the   law   on   copyright   is   not  
regard   to   its   premium   channels,   it   buys   the   channels   absolute.  The  IP  Code  provides  that:  
from  content  providers  and  transmits  on  an  as-­‐is  basis  to    
its  viewers.  Clearly,  PMSI  does  not  perform  the  functions   Sec.  184.  Limitations  on  Copyright.  -­‐  
of   a   broadcasting   organization;   thus,   it   cannot   be   said   184.1.   Notwithstanding   the   provisions   of  
that  it  is  engaged  in  rebroadcasting  Channels  2  and  23.   Chapter   V,   the   following   acts   shall   not  
  constitute  infringement  of  copyright:  
The  Director-­‐General  of  the  IPO  and  the  Court  of  Appeals   x  x  x  x  
also  correctly  found  that  PMSI’s  services  are  similar  to  a   (h)   The   use   made   of   a   work   by   or   under   the  
cable  television  system  because  the  services  it  renders  fall   direction  or  control  of  the  Government,  by  the  
under   cable   “retransmission,”   as   described   in   the   National  Library  or  by  educational,  scientific  or  
Working  Paper,  to  wit:   professional   institutions   where   such   use   is   in  
  the   public   interest   and   is   compatible   with   fair  
(G)  Cable  Retransmission   use;  
   
47.   When   a   radio   or   television   program   is   being   The  carriage  of  ABS-­‐CBN’s  signals  by  virtue  of  the  must-­‐
broadcast,   it   can   be   retransmitted   to   new   carry  rule  in  Memorandum  Circular  No.  04-­‐08-­‐88  is  under  
audiences   by   means   of   cable   or   wire.   In   the   the  direction  and  control  of  the  government  though  the  
early   days   of   cable   television,   it   was   mainly   NTC   which   is   vested   with   exclusive   jurisdiction   to  
used  to  improve  signal  reception,  particularly  in   supervise,  regulate  and  control  telecommunications  and  
so-­‐called   “shadow   zones,”   or   to   distribute   the   broadcast   services/facilities   in   the   Philippines.   The  
signals  in  large  buildings  or  building  complexes.   imposition   of   the   must-­‐carry   rule   is   within   the   NTC’s  
With   improvements   in   technology,   cable   power   to   promulgate   rules   and   regulations,   as   public  
operators   now   often   receive   signals   from   safety   and   interest   may   require,   to   encourage   a   larger  
satellites   before   retransmitting   them   in   an   and   more   effective   use   of   communications,   radio   and  
unaltered   form   to   their   subscribers   through   television   broadcasting   facilities,   and   to   maintain  
cable.   effective   competition   among   private   entities   in   these  
  activities   whenever   the   Commission   finds   it   reasonably  
48.   In   principle,   cable   retransmission   can   be   feasible.    
either   simultaneous   with   the   broadcast   over-­‐  
the-­‐air   or   delayed   (deferred   transmission)   on   PD   285.   Authorizing   the   Compulsory   Licensing   or  
the   basis   of   a   fixation   or   a   reproduction   of   a   Reprinting   of   Educational,   Scientific   or   Cultural   Books  
fixation.  Furthermore,  they  might  be  unaltered   and   Materials   as   a   Temporary   or   Emergency   Measure  
or  altered,  for  example  through  replacement  of  

o 59
Katrina Michelle Mancao
 
Whenever  the  Prices  thereof  become  so  Exorbitant  as  to   personally   in   the   Philippines   or   their   respective  
be  Detrimental  to  the  National  Interest   representative  or  branch  offices  in  the  Philippines.  
   
WHEREAS,   the   spiralling   of   prices   of   educational,   SECTION   4.   Any   person,   natural   or   judicial,   who   shall  
scientific   or   cultural   books   and   materials   has   reached   violate   the   provisions   of   this   Decree   shall,   upon  
very  serious  proportion;   conviction  thereof,  be  punished  by  imprisonment  of  not  
  less   than   Five   Thousand   Pesos   nor   more   than   two   Ten  
WHEREAS,   it   is   in   the   national   interest   that   such   books   Thousand  Pesos.  If  the  violation  is  committed  by  a  firm,  
and   materials   be   made   available   to   everybody   at   the   company   or   corporation,   the   manager   or   person   in  
least  cost;  and   charge  of  the  management  of  the  business  thereof  shall  
  be   responsible   thereof.   Books   and   materials   printed   or  
WHEREAS,   this   can   be   achieved   by   the   compulsory   published   or   exported   in   violation   of   this   Decree   shall   be  
licensing   and   reprinting   of   both   domestic   and   foreign   immediately   confiscated   and   the   establishment   that  
educational,   scientific   or   cultural   books   and   materials,   as   printed  or  published  or  exported  them  shall  forthwith  be  
a   temporary   or   emergency   measure,   whenever   their   closed  and  its  operation  discontinued.  
prices  become  exorbitant;    
  SECTION  5.  The  Committee  herein  named  shall  promulgate  
NOW,  THEREFORE,  I,  FERDINAND  E.  MARCOS,  President   rules   and   regulations   for   the   implementation   of   this  
of  the  Philippines,  by  virtue  of  the  powers  vested  in  me   Degree.  
by   the   Constitution   as   Commander-­‐in-­‐Chief   of   all   the    
Armed   Forces   of   the   Philippines,   and   pursuant   to   SECTION  6.   This   Decree   shall   take   effect   immediately,   the  
Proclamation   No.   1081   dated   September   21,   1972,   and  
provisions  of  any  law,  decree,  executive  order,  treaty  or  
General  Order  No.  1  dated  September  22,  1972,  do  hereby  
executive  agreement  to  the  contrary  notwithstanding.  
order  and  decree  as  follows:  
 
 
Done  in  the  City  of  Manila,  this  3rd  day  of  September,  in  
SECTION   1.   Whenever   the   price   of   any   educational,  
the   year   of   Our   Lord,   nineteen   hundred   and   seventy-­‐
scientific   or   cultural   book,   pamphlet   and   other   written   three.  
materials,   whether   of   domestic   or   foreign   origin,   has    
become   so   exorbitant   as   to   be   detrimental   to   the   Remarks:  Repealed  by  the  IP  Code.  
national   interest,   as   determined   and   declared   by   a  
 
committee  composed  of  the  Secretary  of  Education  and  
Culture,  such  book,  pamphlet  or  written  material  may  be  
SECTION  239.1.    
reprinted  by  the  Government  or  by  any  private  printer  or   All   Acts   and   parts   of   Acts   inconsistent   herewith,   more  
printers  for  a  limited  period  and  only  for  the  purpose  of   particularly   Republic   Act   No.   165,   as   amended;   Republic  
making   the   same   available   to   the   people   at   reasonable   Act   No.   166,   as   amended;   and   Articles   188   and   189   of   the  
cost.   Revised   Penal   Code;   Presidential   Decree   No.   49,  
  including   Presidential   Decree   No.   285,   as   amended,   are  
SECTION  2.  The  reprinted  books,  pamphlets  and  materials   hereby  repealed.  
 
shall   bear   proper   acknowledgment   of   the   source,  
authorship,   copyright   proprietors   and   past   printers,   if   26  January  2012  
known,   as   well   as   the   names   and   addresses   of   the    
reprinter   or   reprinters.   If   abbreviated   or   edited   in   any   RELATED  RIGHTS  
-­‐ Separate  and  independent  from  copyright  
manner,   such   fact   shall   be   stated   be   stated.   The  
following   inscription   shall   also   appear   on   the   covers    
thereof:   "Export   of   this   Book   or   material   from   the   MORAL  RIGHTS    
Philippines   is   punishable   by   law."   The   export   of   such   -­‐ Consists  of  (193.1,  193.2,  193.3,  193.4)  
o Right  to  attribution  
reprints  under  any  circumstance  is  hereby  prohibited.  
o Right  to  integrity  of  work  
 
-­‐ Separate  from  economic  rights  
SECTION  3.  The  reprinting  of  the  above  books,  pamphlets  
-­‐ Personal  to  the  author  
and   materials   shall   be   subject   to   the   condition   that   the  
-­‐ Retained   by   the   author   despite   assignment   of  
reprinter   shall   pay,   in   local   currency,   a   royalty   of   three  
the  copyright.  
per   centum   (3%)   of   the   gross   selling   price,   if   so  
-­‐ Not  assignable/  subject  to  license  (198.1)  
demanded   by   the   authors,   publishers   or   copyright  
o BUT  the  same  may  be  waived!  
proprietors   concerned,   whoever   is   legally   entitled  
§ Subject  to  exceptions  
thereto:   Provided,   That   in   the   case   of   non-­‐resident  
-­‐ Illustration:  Producer  obtained  the  copyright  of  
authors,   publishers   or   copyright   proprietors,   the  
the   work.   Such   does   not   include   the  
payment   of   the   royalties   shall   be   made   only   to   them  
assignment  of  moral  rights.  

60 z
Intellectual Property Law

  § Right   of   all   performers   in  


What  is  the  point  of  giving  moral  right?   general   –   Section   203.1  
-­‐ To   protect   the   author   à   preserve   how   the   ONLY  
author  is  represented  to  the  public.   § Right   of   those   whose  
-­‐ The  work  is  the  representation  of  the  author  –   performances   are   fixed   in  
how  the  author  is  represented  to  the  world.   sound   recordings   –   Section  
  203.1   +   203.2,   203.3,   203.4,  
Can  you  compel  the  author  to  write?   203.5  
-­‐ No.  See  Section  194.  Your  remedy  is  to  ask  for   o Why  not  include  other  works?  
damages.   § Ma’am:   It   is   not   financially  
  feasible   to   apportion   all  
Is  destruction  of  the  work  allowed?   rights   to   all   performers   in  
-­‐ Yes,  only  if  the  destruction  is  COMPLETE.   audiovisual   works   because  
-­‐ Incomplete  destruction  is  not  allowed.   these   are   collaborative  
-­‐ Why?   In   complete   destruction,   there   would   be   works.  
no   misrepresentation   of   the   author   to   the   o But  can  other  performers  demand  the  
public.   The   purpose   of   the   law   is   to   prevent   rights  in  203.2-­‐203.5?  
misrepresentation   of   the   author   to   the   public.   § YES.   Section   207   allows   the  
If   purpose   of   the   law   ceases   to   exist,   no   need   parties   to   agree   for   better  
for  the  application  of  the  law.   terms.   The   law   does   not  
  provide  for  ceilings.  
Term   of   moral   right:   Lifetime   of   the   author   +   50   years   -­‐ Producers   of   sound   recording   and  
after  death  (same  as  copyright)   Broadcasting  organization  
-­‐ Note   the   persons   charged   with   posthumous   o Can  they  also  claim  copyright?    Yes.  
enforcement   of   the   right   is   enumerated   in   o When  can  they  claim  copyright?  
Section  198.1   § The   work   is   original,   they  
o Those  named  by  the  author  in  writing,   produced   content   itself,   and  
o In   default   of   the   former,   author’s   such   is   created   by   their  
heirs   employees  in  the  exercise  of  
o In  default  of  the  heirs,  Director  of  TNL   their  functions.  OR  
  § Commissioned   work   AND  
FOLLOW-­‐UP  RIGHTS  (SECTION  200)   the   copyright   is   vested   in  
-­‐ Of   French   origin,   adopted   because   of   the   them   by   the   contract   with  
Convention   the  author  of  work.  
-­‐ Applicable  to  specific  class  only:   o Independent   of   copyright,   they   also  
o Painting   have  other  rights.  
o Sculpture   o Why   give   them   copyright   and   other  
o Original   manuscript   (of   a   writer   or   rights?  
composer)   § Because   it   is   possible   that  
-­‐ Ratio:   These   works   appreciate   in   value.   Thus,   they   do   not   own   the  
the   purpose   of   the   law   is   to   allow   the   creator   copyright   of   the   work   that  
to   participate   in   the   appreciation   in   value   of   they  produced.  Thus,  if  they  
their  works.   do   not   own   the   copyrights,  
-­‐ Basis:  Gross  proceeds.   they   can   at   least   invoke  
-­‐ Problem   with   these   rights:   Enforcement   (how   other  rights.  
do  you  enforce  these?)    
o We  have  no  registry  system  for  these   Issue:  Why  would  copyright  infringement  be  the  better  
works;  thus,  no  way  to  monitor  it.   right   to   be   invoked   by   a   broadcasting   organization  
  against  a  cable  operator  based  in  Lanao  rather  than  the  
NEIGHBORING  RIGHTS:   rights  of  a  broadcasting  organization  under  Chapter  XIV  
-­‐ Granted  to:   of   the   IP   Code?   What   is   the   LEGAL   risk   if   the  
o Performers   broadcasting   organization   applies   for   a   search   warrant  
o Producers  of  sound  recording   on  the  basis  of  the  rights  of  a  broadcasting  organization  
o Broadcasting  corporations   under  chapter  XIV?    
-­‐ Performers:    
o Defined  in  Section  202.1   HW  Answer  (Di  Bonilla):  
o Some   performers   are   better   off   than   The   search   warrant   is   susceptible   to   being  
others.  How?   quashed  for  failure  to  establish  probable  cause  

o 61
Katrina Michelle Mancao
 
for   violation   of   the   rights   of   a   broadcasting   organization   entitles   the   latter   to   the   same  
organization.   First,   as   emphasized   in   the   case   criminal   remedies,   among   others,   in   case   of  
of   ABS-­‐CBN   vs.   PMMI,   the   Rome   Convention   infringement   of   copyright,   the   same   rights  
defines   rebroadcasting   as   the   “the   must   be   construed   strictly   against   the   State.  
simultaneous   broadcasting   by   one   Thus,   these   rights   and   the   corresponding  
broadcasting   organization   of   the   broadcast   of   criminal  violations  thereof  must  not  be  deemed  
another   broadcasting   organization.”   For   there   to   include   acts   which   do   not   clearly   and  
to   be   rebroadcasting,   the   alleged   explicitly   fall   under   them.   Criminal   acts   must  
rebroadcaster   must   be   a   broadcasting   not  be  deemed  to  exist  by  implication.  
organization.   However,   the   Lanao   cable    
operator   does   not   meet   the   definition   of  
broadcasting  organizations,  which  according  to  
the  foregoing  case,    are  “entities  that  take  the  
financial   and   editorial   responsibility   for   the  
selection   and   arrangement   of,   and   investment  
in,   the   transmitted   content.”   The   Lanao   cable  
operator   does   not   select,   arrange   or   invest   in  
the   content   it   transmits.   It   only   transmits   but  
does   not   take   financial   and   editorial  
responsibility   for   the   content.     It   is   HBO   that  
creates,  selects,  arranges  and  transmits  its  own  
channel   to   its   distributors   in   whole   -­‐   including  
movies   it   has   been   licensed   to   show   and   its  
own   originally   created   miniseries.   It   is   part   of  
the   package   it   offers   and   paid   for   by   its  
distributors.   On   the   other   hand,   the   Lanao  
cable   operator   only   carries   the   signals   of   HBO  
in  its  unchanged  form.  it  does  not  perform  the  
functions   of   a   broadcasting   organization   so  
that   it   cannot   be   said   to   be   engaged   in  
rebroadcasting  of  HBO's  broadcasts.  
 
Second,   the   Lanao   cable   operator   is   not  
engaged   in   broadcasting.     As   a   cable   television,  
it  is  engaged  in  mere  cable  retransmission.  The  
right   of   broadcasting   organizations   against  
rebroadcasting  without  their  consent  does  not  
cover  cable  retransmission  which,  according  to  
the   same   ABSCBN   case,   is   a   mere   simultaneous  
and   unaltered   transmission   by   a   cable  
television   through   wire   or   cable.   Sec.   202.7   of  
the   IP   Code   defines   broadcasting   as     "the  
transmission   by   wireless   means   for   the   public  
reception   of   sounds   or   of   images   or   of  
representations   thereof;   such   transmission   by  
satellite   is   also   "broadcasting"   where   the  
means   for   decrypting   are   provided   to   the  
public  by  the  broadcasting  organization  or  with  
its   consent."   Thus,   the   Lanao   cable   television  
operator   which   transmits   through   wire   or  
cables  is  not  broadcasting  because  the  latter  is  
limited   to   transmission   by   wireless   and   by  
satellite.   Also,   Lanao's   transmission   is   not   for  
the   public   reception   in   general   but  
only  for  its  subscribers.  
 
Finally,   since   any   violation   of   the  
rights   of   a   broadcasting  

62 z
Intellectual Property Law

VI. Law   on   Trademarks,   Tradenames   and   industrial   designs,   marks   and   works,   acquired   in   good  
faith  prior  to  the  effective  date  of  this  Act.  
Service  Marks  
 
 
SECTION  241.  EFFECTIVITY.  –    
A. Legislative  History  
This  Act  shall  take  effect  on  1  January  1998.  
 
 
REPUBLIC  ACT  166   B. Definition  of  Trademarks  
An  Act  to  Provide  for  the  Registration  and  Protection  of  
 
Trade-­‐marks,   Trade-­‐names   and   Service-­‐marks,   defining  
SECTION   121.1.   "MARK"  means  any  visible  sign  capable  of  
Unfair   Competition   and   False   Marking   and   Providing  
Remedies  against  the  Same,  and  for  Other  Purposes   distinguishing  the  goods  (trademark)  or  services  (service  
  mark)   of   an   enterprise   and   shall   include   a   stamped   or  
-­‐ Took  effect  20  June  1947   marked  container  of  goods  
   
SECTION  239.  REPEALS.  –     DISTILLERIA  WASHINGTON,  INC.  V.  CA  (1996)  
239.1.   All   Acts   and   parts   of   Acts   inconsistent   herewith,   Refresher:  
La  Tondeña  Distillers,  Inc.  (LTDI)  filed  a  case  for  manual  
more   particularly   Republic   Act   No.   165,   as   amended;  
delivery   with   dmages   against   Distilleria   Washington.  
Republic   Act   No.   166,   as   amended;   and   Articles   188   and  
LTDI,   under   a   claim   of   ownership,   sought   to   seize   from  
189   of   the   Revised   Penal   Code;   Presidential   Decree   No.  
Distilleria   Washington   18,157   empty   "350   c.c.   white   flint  
49,   including   Presidential   Decree   No.   285,   as   amended,  
bottles"   bearing   the   blown-­‐in   marks   of   "La   Tondeña  
are  hereby  repealed.  
Inc."  and  "Ginebra  San  Miguel."    
 
 
239.2.  Marks  registered  under  Republic  Act  No.  166  shall  
LTDI   asserted   that,   being   the   owner   and   registrant   of  
remain   in   force   but   shall   be   deemed   to   have   been   the  bottles,  it  was  entitled  to  the  protection  so  extended  
granted   under   this   Act   and   shall   be   due   for   renewal   by   Republic   Act   ("R.A.")   No.   623,   as   amended,  
within  the  period  provided  for  under  this  Act  and,  upon   notwithstanding   its   sale   of   the   Ginebra   San   Miguel   gin  
renewal   shall   be   reclassified   in   accordance   with   the   product  contained  in  said  bottles.  
International   Classification.   Trade   names   and   marks    
registered   in   the   Supplemental   Register   under   Republic   Washington   countered   that   R.A.   No.   623,   invoked   by  
Act   No.   166   shall   remain   in   force   but   shall   no   longer   be   LTDI,   should   not   apply   to   gin,   an   alcoholic   beverage  
subject  to  renewal.   which   is   unlike   that   of   "soda   water,   mineral   or   aerated  
  water,   ciders,   milks,   cream,   or   other   lawful   beverages"  
239.3.   The   provisions   of   this   Act   shall   apply   to   works   in   mentioned  in  the  law,  and  that,  in  any  case,  ownership  of  
which   copyright   protection   obtained   prior   to   the   the   bottles   should,   considering   the   attendant   facts   and  
effectivity   of   this   Act   is   subsisting:   Provided,   That   the   circumstances,  be  held  lawfully  transferred  to  the  buyers  
application   of   this   Act   shall   not   result   in   the   diminution   upon  the  sale  of  the  gin  and  containers  at  a  single  price.  
of  such  protection.    
  Doctrine:  
SECTION  235.2.     The   case   before   us   goes   beyond   just   seeking   to   have  
All  applications  for  registration  of  marks  or  trade  names   such  use  stopped  but  it  so  takes  on  even  the  ownership  
pending   in   the   Bureau   of   Patents,   Trademarks   and   issue   as   well.  Parenthetically,  petitioner  is  not  here  being  
Technology  Transfer  at  the  effective  date  of  this  Act  may   charged   with   a   violation   of   Section   2   of   R.A.   No.   623   or   of  
be   amended,   if   practicable   to   bring   them   under   the   the   Trademark   Law.   The   instant   suit   is   one   for   replevin  
provisions   of   this   Act.   The   prosecution   of   such   (manual   delivery)   where   the   claimant   must   be   able   to  
applications   so   amended   and   the   grant   of   registrations   show  convincingly  that  he  is  either  the  owner  or  clearly  
thereon  shall  be  proceeded  with  in  accordance  with  the   entitled   to   the   possession   of   the   object   sought   to   be  
provisions  of  this  Act.  If  such  amendments  are  not  made,   recovered.   Replevin   is   a   possessory   action   the   gist   of  
the   prosecution   of   said   applications   shall   be   proceeded   which   focuses   on   the   right   of   possession   that,   in   turn,   is  
with   and   registrations   thereon   granted   in   accordance   dependent  on  a  legal  basis  that,  not  infrequently,  looks  to  
with   the   Acts   under   which   said   applications   were   filed,   the  ownership  of  the  object  sought  to  be  replevied.  
and   said   Acts   are   hereby   continued   in   force   to   this    
extent   for   this   purpose   only,   notwithstanding   the   It  is  to  be  pointed  out  that  a  trademark  refers  to  a   word,  
foregoing  general  repeal  thereof.   name,   symbol,   emblem,   sign   or   device   or   any  
  combination  thereof  adopted  and  used  by  a  merchant  to  
SECTION  236.  PRESERVATION  OF  EXISTING  RIGHTS.  –     identify,   and   distinguish   from   others,   his   goods   of  
Nothing   herein   shall   adversely   affect   the   rights   on   the   commerce.   It   is   basically   an   intellectual   creation   that   is  
enforcement   of   rights   in   patents,   utility   models,   susceptible   to   ownership   and,   consistently   therewith,  

o 63
Katrina Michelle Mancao
 
gives   rise   to   its   own   elements   of   jus   posidendi,   jus   utendi,   C. Functions  of  Trademarks  
jus   fruendi,   jus   disponendi,   and   jus   abutendi,   along   with    
the   applicable   jus   lex,   comprising   that   ownership.   The  
ANG  V.  TEODORO  (1942)  
incorporeal  right,  however,  is  distinct  from  the  property  
Refresher:  
in   the   material   object   subject   to   it.   Ownership   in   one  
Trademark  involved:  “Ang  Tibay”  
does   not   necessarily   vest   ownership   in   the   other.   Thus,  
Toribo  Teodoro  –  used  it  for  slippers,  shoes,  and  indoor  
the   transfer   or   assignment   of   the   intellectual   property  
baseballs   since   1910.   Registered   it   as   trademark   in   1915,  
will   not   necessarily   constitute   a   conveyance   of   the   thing  
as  trade-­‐name  in  1933.  
it  covers,  nor  would  a  conveyance  of  the  latter  imply  the  
 
transfer  or  assignment  of  the  intellectual  right.    
Ana   L.   Ang   –   registered   the   trademark   for   pants   and  
 
shirts  in  1932.  
R.A.   No.   623   evidently   does   not   disallow   the   sale   or  
 
transfer   of   ownership   of   the   marked   bottles   or  
Doctrine:  
containers.  In  fact,  the  contrary  is  implicit  in  the  law;  thus  
We   find   it   necessary   to   go   into   the   etymology   and  
—  
meaning  of  the  Tagalog  words  "Ang  Tibay"  to  determine  
 
whether   they   are   a   descriptive   term,   i.e.,   whether   they  
Sec.   5.   No   action   shall   be   brought   under   this  
relate  to  the  quality  or  description  of  the  merchandise  to  
Act   against   any   person   to   whom   the   registered  
which  respondent  has  applied  them  as  a  trademark.    
manufacturer,  bottler  or  seller,  has  transferred  
 
by   way   of   sale,   any   of   the   containers   herein  
The   word   "ang"   is   a   definite   article   meaning   "the"   in  
referred   to,   but   the   sale   of   the   beverage  
English.  It  is  also  used  as  an  adverb,  a  contraction  of  the  
contained   in   the   said   containers   shall   not  
word   "anong"   (what   or   how).   For   instance,   instead   of  
include   the   sale   of   the   containers   unless  
saying,  "Anong  ganda!"  ("How  beautiful!"),  we  ordinarily  
specifically  so  provided.  
say,   "Ang   ganda!"   Tibay   is   a   root   word   from   which   are  
 
derived   the   verb   magpatibay   (to   strenghten;   the   nouns  
Sec.   6.   The   provisions   of   this   Act   shall   not   be  
pagkamatibay   (strength,   durability),   katibayan   (proof,  
interpreted  as  prohibiting  the  use  of  bottles  as  
support,   strength),   katibay-­‐tibayan   (superior   strength);  
containers   for   "sisi,"   "bagoong,"   "patis,"   and  
and   the   adjectives   matibay   (strong,   durable,   lasting),  
similar  native  products.  
napakatibay   (very   strong),   kasintibay   or   magkasintibay  
 
(as   strong   as,   or   of   equal   strength).   The   phrase   "Ang  
Scarcely   disputed   are   certain   and   specific   industry  
Tibay"   is   an   exclamation   denoting   administration   of  
practices   in   the   sale   of   gin:   The   manufacturer   sells   the  
strength   or   durability.   For   instance,   one   who   tries   hard  
product   in   marked   containers,   through   dealers,   to   the  
but  fails  to  break  an  object  exclaims,  "Ang  tibay!"  (How  
public   in   supermarkets,   grocery   shops,   retail   stores   and  
strong!")   It   may   also   be   used   in   a   sentence   thus,   "Ang  
other   sales   outlets.   The   buyer   takes   the   item;   he   is  
tibay   ng   sapatos   mo!"   (How   durable   your   shoes   are!")  
neither   required   to   return   the   bottle   nor   required   to  
The   phrase   "ang   tibay"   is   never   used   adjectively   to  
make  a  deposit  to  assure  its  return  to  the  seller.  He  could  
define   or   describe   an   object.   One   does   not   say,   "ang  
return  the  bottle  and  get  a  refund.  A  number  of  bottles  
tibay   sapatos"   or   "sapatos   ang   tibay"   is   never   used  
at   times   find   their   way   to   commercial   users.   It   cannot   be  
adjectively   to   define   or   describe   an   object.   One   does   not  
gainsaid   that   ownership   of   the   containers   does   pass   on  
say,  "ang  tibay  sapatos"  or  "sapatos  ang  tibay"  to  mean  
to  the  consumer  albeit  subject  to  the  statutory  limitation  
"durable   shoes,"   but   "matibay   na   sapatos"   or   "sapatos  
on   the   use   of   the   registered   containers   and   to   the  
na  matibay."  
trademark   right   of   the   registrant.   The   statement   in  
 
Section   5   of   R.A.   623   to   the   effect   that   the   "sale   of  
From   all   of   this   we   deduce   that   "Ang   Tibay"   is   not   a  
beverage   contained   in   the   said   containers   shall   not  
descriptive   term   within   the   meaning   of   the   Trade-­‐Mark  
include   the   sale   of   the   containers   unless   specifically   so  
Law   but   rather   a   fanciful   or   coined   phrase   which   may  
provided"   is   not   a   rule   of   proscription.   It   is   a   rule   of  
properly   and   legally   be   appropriated   as   a   trademark   or  
construction  that,  in  keeping  with  the  spirit  and  intent  of  
trade-­‐name.   In   this   connection   we   do   not   fail   to   note  
the   law,   establishes   at   best   a   presumption   (of   non-­‐
that   when   the   petitioner   herself   took   the   trouble   and  
conveyance   of   the   container)   and   which   by   no   means  
expense   of   securing   the   registration   of   these   same  
can   be   taken   to   be   either   interdictive   or   conclusive   in  
words   as   a   trademark   of   her   products   she   or   her  
character.   Upon   the   other   hand,   LTDI's   sales   invoice,  
attorney   as   well   as   the   Director   of   Commerce   was  
stipulating   that   the   "sale   does   not   include   the   bottles  
undoubtedly   convinced   that   said   words   (Ang   Tibay)  
with   the   blown-­‐in   marks   of   ownership   of   La   Tondeña  
were   not   a   descriptive   term   and   hence   could   be   legally  
Distillers,"   cannot   affect   those   who   are   not   privies  
used   and   validly   registered   as   a   trade-­‐mark.   It   seems  
thereto.  
stultifying  and  puerile  for  her  now  to  contend  otherwise,  
 
suggestive   of   the   story   of   sour   grapes.   Counsel   for   the  
 
petitioner   says   that   the   function   of   a   trademark   is   to  

64 z
Intellectual Property Law

point   distinctively,   either   by   its   own   meaning   or   by    


association,   to   the   origin   or   ownership   of   the   wares   to   Barbizon   Corporation   –   NY   corp.   =   opposed   the  
which   it   is   applied.   That   is   correct,   and   we   find   that   "Ang   application.  
Tibay,"  as  used  by  the  respondent  to  designate  his  wares,    
had   exactly   performed   that   function   for   twenty-­‐two   Escobar  later  assigned  all  her  rights  and  interest  over  the  
years  before  the  petitioner  adopted  it  as  a  trade-­‐mark  in   trademark   to   petitioner   Pribhdas   J.   Mirpuri   who,   under  
her   own   business.   Ang   Tibay   shoes   and   slippers   are,   by   his   firm   name   then,   the   "Bonito   Enterprises,"   was   the  
association,   known   throughout   the   Philippines   as   sole   and   exclusive   distributor   of   Escobar's   "Barbizon"  
products   of   the   Ang   Tibay   factory   owned   and   operated   products.  
by  the  respondent  Toribio  Teodoro.    
  Doctrine:  
ETEPHA,   A.G.   V.   DIRECTOR   OF   PATENTS   AND   WESTMONT   In   Philippine   jurisprudence,   the   function   of   a   trademark  
is   to   point   out   distinctly   the   origin   or   ownership   of   the  
PHARMACEUTICALS,  INC.  (1966)  
goods   to   which   it   is   affixed;   to   secure   to   him,   who   has  
Refresher:  
been  instrumental  in  bringing  into  the  market  a  superior  
Westmont   Pharmaceuticals,   Inc.   –   NY   Corp.   =   sought   to  
article  of  merchandise,  the  fruit  of  his  industry  and  skill;  
register   “Atussin”   in   1959   on   its   "medicinal   preparation   to  assure  the  public  that  they  are  procuring  the  genuine  
of   expectorant   antihistaminic,   bronchodilator   sedative,  
article;   to   prevent   fraud   and   imposition;   and   to   protect  
ascorbic   acid   (Vitamin   C)   used   in   the   treatment   of  
the   manufacturer   against   substitution   and   sale   of   an  
cough".  
inferior  and  different  article  as  his  product.  
 
 
Etepha,   A.   G.   –   Liechtenstein   corp.   =   objected   to   the  
Modern   authorities   on   trademark   law   view   trademarks  
registration  on  the  ground  that  “Atussin”  is  similar  to  its  
as  performing  three  distinct  functions:    
trademark   “Pertussin”   that   they   used   on   a   preparation  
(1)   they   indicate   origin   or   ownership   of   the  
for  the  treatment  of  coughs.  
articles  to  which  they  are  attached;    
  (2)  they  guarantee  that  those  articles  come  up  
Doctrine:  
to  a  certain  standard  of  quality;  and    
The   objects   of   a   trademark   are   "to   point   out   distinctly  
(3)  they  advertise  the  articles  they  symbolize.  
the   origin   or   ownership   of   the   articles   to   which   it   is  
 
affixed,   to   secure   to   him   who   has   been   instrumental   in  
Symbols   have   been   used   to   identify   the   ownership   or  
bringing   into   market   a   superior   article   or   merchandise  
origin   of   articles   for   several   centuries.   As   early   as   5,000  
the   fruit   of   his   industry   and   skill,   and   to   prevent   fraud  
B.C.,   markings   on   pottery   have   been   found   by  
and  imposition."  
archaeologists.   Cave   drawings   in   southwestern   Europe  
 
show  bison  with  symbols  on  their  flanks.  Archaeological  
Confusion  is  likely  between  trademarks,  however,  only  if   discoveries   of   ancient   Greek   and   Roman   inscriptions   on  
their   over-­‐all   presentations   in   any   of   the   particulars   of   sculptural   works,   paintings,   vases,   precious   stones,  
sound,   appearance,   or   meaning   are   such   as   would   lead  
glassworks,   bricks,   etc.   reveal   some   features   which   are  
the  purchasing  public  into  believing  that  the  products  to  
thought   to   be   marks   or   symbols.   These   marks   were  
which   the   marks   are   applied   emanated   from   the   same  
affixed   by   the   creator   or   maker   of   the   article,   or   by  
source.   In   testing   this   issue,   fixed   legal   rules   exist   —   if   public   authorities   as   indicators   for   the   payment   of   tax,  
not   in   harmony,   certainly   in   abundance   —   but,   in   the   for   disclosing   state   monopoly,   or   devices   for   the  
final  analysis,  the  application  of  these  rules  in  any  given  
settlement   of   accounts   between   an   entrepreneur   and  
situation   necessarily   reflects   a   matter   of   individual  
his  workmen.  
judgment   largely   predicated   on   opinion.   There   is,    
however,   and   can   be   no   disagreement   with   the   rule   that   In  the  Middle  Ages,  the  use  of  many  kinds  of  marks  on  a  
the   purchaser   is   confused,   if   at   all,   by   the   marks   as   a   variety   of   goods   was   commonplace.   Fifteenth   century  
whole.  
England  saw  the  compulsory  use  of  identifying  marks  in  
  certain   trades.   There   were   the   baker's   mark   on   bread,  
MIRPURI   V.   CA,   DIRECTOR   OF   PATENTS   AND   BARBIZON   bottlemaker's   marks,   smith's   marks,   tanner's   marks,  
CORP.  (1999)   watermarks  on  paper,  etc.  Every  guild  had  its  own  mark  
Refresher:   and   every   master   belonging   to   it   had   a   special   mark   of  
Trademark  in  issue  =  “Barbizon”   his   own.   The   marks   were   not   trademarks   but   police  
  marks  compulsorily  imposed  by  the  sovereign  to  let  the  
Lolita   Escobar   (predecessor-­‐in-­‐interest   of   Mirpuiri)   =   public   know   that   the   goods   were   not   "foreign"   goods  
sought   to   register   “Barbizon”   for   use   in   brassieres   and   smuggled  into  an  area  where  the  guild  had  a  monopoly,  
ladies   undergarments.   Escobar   alleged   that   she   had   as   well   as   to   aid   in   tracing   defective   work   or   poor  
been   manufacturing   and   selling   these   products   under   craftsmanship   to   the   artisan.   For   a   similar   reason,  
the  firm  name  "L  &  BM  Commercial"  since  March  3,  1970.     merchants  also  used  merchants'  marks.  Merchants  dealt  

o 65
Katrina Michelle Mancao
 
in   goods   acquired   from   many   sources   and   the   marks    
enabled   them   to   identify   and   reclaim   their   goods   upon   D. How  are  trademarks  acquired?  
recovery  after  shipwreck  or  piracy.    
 
SECTION  122.  HOW  MARKS  ARE  ACQUIRED.  –    
With   constant   use,   the   mark   acquired   popularity   and  
became   voluntarily   adopted.   It   was   not   intended   to   The   rights   in   a   mark   shall   be   acquired   through  
create   or   continue   monopoly   but   to   give   the   customer   registration   made   validly   in   accordance   with   the  
an   index   or   guarantee   of   quality.   It   was   in   the   late   18th   provisions  of  this  law.  
 
century  when  the  industrial  revolution  gave  rise  to  mass  
production  and  distribution  of  consumer  goods  that  the   NOTE:  
mark  became  an  important  instrumentality  of  trade  and   RA   166,   SECTION   2-­‐A.   OWNERSHIP   OF   TRADE-­‐MARKS,  
commerce.   By   this   time,   trademarks   did   not   merely   TRADE-­‐NAMES  AND  SERVICE-­‐MARKS,  HOW  ACQUIRED.  –    
identify   the   goods;   they   also   indicated   the   goods   to   be   Anyone   who   lawfully   produces   or   deals   in   merchandise  
of   satisfactory   quality,   and   thereby   stimulated   further   of   any   kind   or   who   engages   in   any   lawful   business,   or  
purchases   by   the   consuming   public.   Eventually,   they   who   renders   an   lawful   service   in   commerce,   by   actual  
came  to  symbolize  the  goodwill  and  business  reputation   use  thereof  in  manufacture  or  trade,  in  business,  and  in  
of  the  owner  of  the  product  and  became  a  property  right   the   service   rendered,   may   appropriate   to   his   exclusive  
protected   by   law.   The   common   law   developed   the   use   a   trade-­‐mark,   trade-­‐name,   or   a   service-­‐mark   not   so  
doctrine   of   trademarks   and   tradenames   "to   prevent   a   appropriated   by   another,   to   distinguish   his   merchandise,  
person   from   palming   off   his   goods   as   another's,   from   business   or   service   from   the   merchandise,   business   or  
getting   another's   business   or   injuring   his   reputation   by   service   of   others.   The   ownership   or   possession   of   a  
unfair   means,   and,   from   defrauding   the   public."   trade-­‐mark,   trade-­‐name,   service-­‐mark,   heretofore   or  
Subsequently,   England   and   the   United   States   enacted   hereafter  appropriated,  as  in  this  section  provided,  shall  
national   legislation   on   trademarks   as   part   of   the   law   be   recognized   and   protected   in   the   same   manner   and   to  
regulating   unfair   trade.   It   became   the   right   of   the   the   same   extent   as   are   other   property   rights   known   to  
trademark   owner   to   exclude   others   from   the   use   of   his   the  laws.  
mark,   or   of   a   confusingly   similar   mark   where   confusion    
resulted   in   diversion   of   trade   or   financial   injury.   At   the  
UNNO   COMMERCIAL   ENTERPRISES   V.   GENERAL   MILLING  
same   time,   the   trademark   served   as   a   warning   against  
the   imitation   or   faking   of   products   to   prevent   the   CORPORATION  (1983)  
imposition  of  fraud  upon  the  public.   Refresher:  
  Trademark  in  issue:  "All  Montana"  
Today,   the   trademark   is   not   merely   a   symbol   of   origin   General  Milling  =  used  it  on  wheat  flour  in  the  Philippines  
and  goodwill;  it  is  often  the   most   effective   agent   for   the   since  1955  
actual   creation   and   protection   of   goodwill.   It   imprints   Unno   Commercial   =   (prior)   registrant   of   the   mark   since  
upon   the   public   mind   an   anonymous   and   impersonal   1962,  allegedly  started  using  the  mark  since  1956  
guaranty   of   satisfaction,   creating   a   desire   for   further    
satisfaction.   In   other   words,   the   mark   actually   sells   the   Doctrine:  
goods.  The  mark  has  become  the  "silent  salesman,"  the   The  right  to  register  trademark  is  based  on  ownership.  
conduit   through   which   direct   contact   between   the   When   the   applicant   is   not   the   owner   of   the   trademark  
trademark   owner   and   the   consumer   is   assured.   It   has   being   applied   for,   he   has   no   right   to   apply   for   the  
invaded   popular   culture   in   ways   never   anticipated   that   it   registration  of  the  same.  Under  the  Trademark  Law  only  
has   become   a   more   convincing   selling   point   than   even   the   owner   of   the   trademark,   trade   name   or   service   mark  
the  quality  of  the  article  to  which  it  refers.  In  the  last  half   used   to   distinguish   his   goods,   business   or   service   from  
century,   the   unparalleled   growth   of   industry   and   the   the   goods,   business   or   service   of   others   is   entitled   to  
rapid   development   of   communications   technology   have   register  the  same.  
enabled   trademarks,   tradenames   and   other   distinctive    
signs  of  a  product  to  penetrate  regions  where  the  owner   The   term   owner   does   NOT   include   the   importer   of   the  
does  not  actually  manufacture  or  sell  the  product  itself.   goods  bearing  the  trademark,  trade  name,  service  mark,  
Goodwill  is  no  longer  confined  to  the  territory  of  actual   or   other   mark   of   ownership,   UNLESS   such   importer   is  
market   penetration;   it   extends   to   zones   where   the   actually   the   owner   thereof   in   the   country   from   which   the  
marked   article   has   been   fixed   in   the   public   mind   through   goods   are   imported.   A   LOCAL   IMPORTER,   however,   may  
advertising.  Whether  in  the  print,  broadcast  or  electronic   make   application   for   the   registration   of   a   foreign  
communications   medium,   particularly   on   the   Internet,   trademark,   trade   name   or   service   mark   if   he   is   duly  
advertising  has  paved  the  way  for  growth  and  expansion   authorized   by   the   actual   owner   of   the   name   or   other  
of  the  product  by  creating  and  earning  a  reputation  that   mark   of   ownership.   A   mere   distributor   of   the  
crosses   over   borders,   virtually   turning   the   whole   world   merchandise   covered   by   the   TM   cannot   apply   for   the  
into  one  vast  marketplace.   registration  of  the  TM.  

66 z
Intellectual Property Law

  of  the  name  in  advertisements,  circulars,  price  lists,  and  


Thus,   this   Court,   has   on   several   occasions   ruled   that   on  signs  and  stationery.  
where  the  applicant's  alleged  ownership  is  not  shown  in    
any   notarial   document   and   the   applicant   appears   to   be   The   Paris   Convention   for   the   Protection   of   Industrial  
merely   an   importer   or   distributor   of   the   merchandise   Property  does  not  automatically  exclude  all  countries  of  
covered   by   said   trademark,   its   application   cannot   be   the   world   which   have   signed   it   from   using   a   tradename  
granted.   which  happens  to  be  used  in  one  country.  To  illustrate  -­‐  
  If   a   taxicab   or   bus   company   in   a   town   in   the   United  
Ownership   of   a   trademark   is   not   acquired   by   the   mere   Kingdom  or  India  happens  to  use  the  tradename  "Rapid  
fact  of  registration  alone.     Registration   merely   creates   a   Transportation",   it   does   not   necessarily   follow   that  
prima   facie   presumption   of   the   validity   of   the   "Rapid"   can   no   longer   be   registered   in   Uganda,   Fiji,   or  
registration,   of   the   registrant's   ownership   of   the   the  Philippines.  
trademark  and  of  the  exclusive  right  to  the  use  thereof.    
Registration   does   not   perfect   a   trademark   right.   As   Indeed,   the   Philippines   is   a   signatory   to   this   Treaty   and,  
conceded   itself   by   petitioner,   evidence   may   be   hence,   we   must   honor   our   obligation   thereunder   on  
presented   to   overcome   the   presumption.   Prior   use   by   matters  concerning  internationally  known  or  well  known  
one   will   controvert   a   claim   of   legal   appropriation,   by   marks.   However,   this   Treaty   provision   clearly   indicated  
subsequent  users.   the   conditions   which   must   exist   before   any   trademark  
  owner   can   claim   and   be   afforded   rights   such   as   the  
KABUSHI  KAISHA   ISETAN  V.   IAC  AND  ISETANN  DEPT.  STORE   Petitioner  herein  seeks  and  those  conditions  are  that:  
 
(1991)  
a)   the   mark   must   be   internationally   known   or  
Refresher:  
well  known;  
Trademark  in  issue:  “Isetan”  
 
Kabushi   Kaisha   Isetan   –   Japan   Corp.,   sought   to   cancel  
b)  the  subject  of  the  right  must  be  a  trademark,  
Isetann’s  registration  
not  a  patent  or  copyright  or  anything  else;  
Isetann   Dept.   Store   –   domestic   corp.,   registrant   of  
 
“Isetann  Dept.  Store”  
c)   the   mark   must   be   for   use   in   the   same   or  
 
similar  kinds  of  goods;  and  
Doctrine:  
 
A   fundamental   principle   of   Philippine   Trademark   Law   is  
d)   the   person   claiming   must   be   the   owner   of  
that   actual   use   in   commerce   in   the   Philippines   is   a   pre-­‐
the  mark  
requisite   to   the   acquisition   of   ownership   over   a  
 
trademark   or   a   tradename.   Adoption   alone   of   a  
PHILIP   MORRIS   V.   CA   AND   FORTUNE   TOBACCO   CORP.  
trademark   would   not   give   exclusive   right   thereto.   Such  
right  grows  out  of  their  actual  use.  Adoption  is  not  use.   (1993)  
One   way   make   advertisements,   issue   circulars,   give   out   Refresher:  
price   lists   on   certain   goods;   but   these   alone   would   not   Trademark  in  issue  =  “Mark”  
give   exclusive   right   of   use.   For   trademark   is   a   creation   of   Philip   Morris   =   Virginia   corp.   not   doing   business   in   the  
use.   The   underlying   reason   for   all   these   is   that   Philippines.  
purchasers   have   come   to   understand   the   mark   as   Fortune  Tobacco  =  domestic  corp.  
indicating   the   origin   of   the   wares.   Flowing   from   this   is    
the  trader's  right  to  protection  in  the  trade  he  has  built   Doctrine:  
up   and   the   goodwill   he   has   accumulated   from   use   of   the   A  foreign  corporation  not  doing  business  in  the  PH  may  
trademark.   have   the   right   to   sue   before   PH   courts   but   existing  
  adjective   axioms   require   that   qualifying   circumstances  
The   records   show   that   the   petitioner   has   never   necessary   for   the   assertion   of   such   right   should   first   be  
conducted   any   business   in   the   Philippines.   It   has   never   affirmatively  pleaded.    
promoted  its  tradename  or  trademark  in  the  Philippines.    
It  has  absolutely  no  business  goodwill  in  the  Philippines.   Foreign   corporation   not   engaged   in   business   in   the   PH  
It  is  unknown  to  Filipinos  except  the  very  few  who  may   may   maintain   a   cause   of   action   for   infringement  
have  noticed  it  while  travelling  abroad.  It  has  never  paid   primarily  because  of  Sec.  21-­‐A  of  TM  Law  when  the  legal  
a   single   centavo   of   tax   to   the   Philippine   government.   standing  to  sue  is  alleged.    
Under  the  law,  it  has  no  right  to  the  remedy  it  seeks.    
  Our   municipal   law   on   TM   regarding   the   requirement   of  
The   mere   origination   or   adoption   of   a   particular   actual   use   in   the   PH   must   subordinate   an   international  
tradename   without   actual   use   thereof   in   the   market   is   agreement   inasmuch   as   the   apparent   clash   is   being  
insufficient   to   give   any   exclusive   right   to   its   use,   even   decided   by   a   municipal   tribunal.   Withal,   the   fact   that  
though  such  adoption  is  publicly  declared,  such  as  by  use   international  law  has  been  made  part  of  law  of  the  land  

o 67
Katrina Michelle Mancao
 
does   not   by   any   means   imply   the   primacy   of   Developers  Group  of  Companies,  Inc.  =  claim  ownership  
international   law   over   national   law   in   the   municipal   of   the   logo   and   the   mark   that   they   used   for   their  
sphere.  Under  the  doctrine  of  incorporation  as  applied  in   restaurant   business.   They   had   the   mark   registered   in  
most   countries,   rules   of   international   law   are   given   a   1983.  
standing   equal,   not   superior   to   the   national   legislative    
enactments.     Kuok   family   owns   and   operates   a   chain   of   hotels   since  
  1962.  
A  foreign  corporation  may  have  the  personal  to  file  a  suit    
for   infringement   BUT   it   may   not   necessarily   be   entitled   Doctrine:  
to   protection   due   to   absence   of   actual   use   of   the   Under   the   provisions   of   the   former   trademark   law,   RA  
emblem  in  the  local  market.     166,  as  amended,  which  was  in  effect  up  to  December  31,  
  1997,   hence,   the   law   in   force   at   the   time   of   respondent's  
PHILIP  MORRIS  V.  FORTUNE  TOBACCO  CORP.  (2006)   application   for   registration   of   trademark,   the   root   of  
Doctrine:   ownership   of   a   trademark   is   actual   use   in   commerce.  
Foreign   corporation   may   not   successfully   sue   on   the   Section   2   of   said   law   requires   that   before   a   trademark  
basis  alone  of  their  respective  certificates  of  registration   can   be   registered,   it   must   have   been   actually   used   in  
of   TM,   for   as   far   as   a   condition   to   the   availment   of   the   commerce   and   service   for   not   less   than   two   months   in  
rights   and   privileges   vis-­‐à-­‐vis   their   TM   in   this   country,   the  Philippines  prior  to  the  filing  of  an  application  for  its  
they   ought   to   show   proof   that,   on   top   of   the   PH   registration.  
registration,   their   country   grants   substantially   similar    
rights  and  privileges  to  Filipino  citizens  pursuant  to  Sec.   Registration,   without   more,   does   not   confer   upon   the  
21-­‐A  of  RA  166.     registrant   an   absolute   right   to   the   registered   mark.   The  
  certificate   of   registration   is   merely   a   prima   facie   proof  
Members   of   the   Paris   Union   are   not   automatically   that  the  registrant  is  the  owner  of  the  registered  mark  
entitled   protection   of   TM   in   this   country,   absent   actual   or   trade   name.  Evidence  of  prior  and  continuous  use  of  
use  of  the  marks  in  the  local  commerce  and  trade.  True,   the   mark   or   trade   name   by   another   can   overcome   the  
the   PH   adherence   to   Paris   Convention   effectively   presumptive   ownership   of   the   registrant   and   may   very  
obligates  the  country  to  honor  and  enforce  its  provisions   well   entitle   the   former   to   be   declared   owner   in   an  
as   regards   the   protection   of   industrial   property   of   appropriate  case.  
foreign   nationals   in   this   country.   However,   any    
protection   accorded   has   to   be   made   subject   to   the   Among   the   effects   of   registration   of   a   mark,   as  
limitations  of  PH  laws.     catalogued  by  the  Court  in  Lorenzana  v.  Macagba,  are:  
   
The   registration   of   TM   cannot   be   deemed   conclusive   as   1.   Registration   in   the   Principal   Register   gives  
to   the   actual   use   of   such   TM   in   local   commerce.   As   it   rise   to   a   presumption   of   the   validity   of   the  
were,   registration   does   not   confer   upon   the   registrant   registration,   the   registrant's   ownership   of   the  
an  absolute  right  to  the  registered  mark.  The  certificate   mark,  and  his  right  to  the  exclusive  use  thereof.  
of   registration   merely   constitutes   prima   facie   evidence   x  x  x  
that   the   registrant   is   the   owner   of   the   registered   mark.    
Evidence   of   non-­‐usage   of   the   mark   rebuts   the   2.   Registration   in   the   Principal   Register   is  
presumption   of   TM   ownership,   as   what   happened   here   limited   to   the   actual   owner   of   the   trademark  
when  petitioners  no  less  admitted  not  doing  business  in   and   proceedings   therein   pass   on   the   issue   of  
this  country.     ownership,   which   may   be   contested   through  
  opposition   or   interference   proceedings,   or,  
Registration  in  the  PH  of  TM  does  not  ipso  facto  convey   after  registration,  in  a  petition  for  cancellation.  
an   absolute   right   or   exclusive   ownership   thereof.   The   xxx  
registration  of  TM  unaccompanied  by  actual  use  thereof    
in  the  country  accords  the  registrant  only  the  standing  to   Ownership  of  a  mark  or  trade  name  may  be  acquired  not  
sue   for   infringement   in   the   PH   courts.   Entitlement   to   necessarily   by   registration   but   by   adoption   and   use   in  
protection   of   such   TM   in   the   country   is   entirely   a   trade   or   commerce.   As   between   actual   use   of   a   mark  
different  matter.     without   registration,   and   registration   of   the   mark  
  without   actual   use   thereof,   the   former   prevails   over   the  
SHANGRI-­‐LA  INTERNATIONAL  HOTEL  MANAGEMENT,  LTD.  V.   latter.  For  a  rule  widely  accepted  and  firmly  entrenched,  
because   it   has   come   down   through   the   years,   is   that  
DEVELOPERS  GROUP  OF  COMPANIES,  INC.  (2006)   actual   use   in   commerce   or   business   is   a   pre-­‐requisite   to  
Refresher:   the  acquisition  of  the  right  of  ownership.  
Trademark  in  issue:  “Shangri-­‐la”  and  the  “S”  logo    

68 z
Intellectual Property Law

While  the  present  law  on  trademarks  has  dispensed  with   or   the   name,   signature,   or   portrait   of   a   deceased  
the   requirement   of   prior   actual   use   at   the   time   of   President   of   the   Philippines,   during   the   life   of   his   widow,  
registration,   the   law   in   force   at   the   time   of   registration   if  any,  except  by  written  consent  of  the  widow;  
must   be   applied,   and   thereunder   it   was   held   that   as   a    
condition   precedent   to   registration   of   trademark,   trade   (d)   Is   identical   with   a   registered   mark   belonging   to   a  
name   or   service   mark,   the   same   must   have   been   in   different   proprietor   or   a   mark   with   an   earlier   filing   or  
actual   use   in   the   Philippines   before   the   filing   of   the   priority  date,  in  respect  of:  
application   for   registration.   Trademark   is   a   creation   of    
use   and   therefore   actual   use   is   a   pre-­‐requisite   to   (i)  The  same  goods  or  services,  or  
exclusive   ownership   and   its   registration   with   the    
Philippine   Patent   Office   is   a   mere   administrative   (ii)  Closely  related  goods  or  services,  or  
confirmation  of  the  existence  of  such  right.    
  (iii)  If  it  nearly  resembles  such  a  mark  as  to  be  
By   itself,   registration   is   not   a   mode   of   acquiring   likely  to  deceive  or  cause  confusion;  
ownership.   When   the   applicant   is   not   the   owner   of   the    
trademark   being   applied   for,   he   has   no   right   to   apply   for   (e)   Is   identical   with,   or   confusingly   similar   to,   or  
registration   of   the   same.   Registration   merely   creates   a   constitutes   a   translation   of   a   mark   which   is   considered  
prima  facie  presumption  of  the  validity  of  the  registration,   by   the   competent   authority   of   the   Philippines   to   be   well-­‐
of  the  registrant's  ownership  of  the  trademark  and  of  the   known  internationally  and  in  the  Philippines,  whether  or  
exclusive   right   to   the   use   thereof.   Such   presumption,   just   not   it   is   registered   here,   as   being   already   the   mark   of   a  
like   the   presumptive   regularity   in   the   performance   of   person   other   than   the   applicant   for   registration,   and  
official   functions,   is   rebuttable   and   must   give   way   to   used  for  identical  or  similar  goods  or  services:  Provided,  
evidence  to  the  contrary.   That   in   determining   whether   a   mark   is   well-­‐known,  
  account  shall  be  taken  of  the  knowledge  of  the  relevant  
SHANGRI-­‐LA  INTERNATIONAL  HOTEL  MANAGEMENT,  LTD.  V.   sector   of   the   public,   rather   than   of   the   public   at   large,  
including   knowledge   in   the   Philippines   which   has   been  
DEVELOPERS  GROUP  OF  COMPANIES,  INC.  (2007)  
obtained  as  a  result  of  the  promotion  of  the  mark;  
Court  decision:    
The   interpretation   of   Republic   Act   No.   166   in   the   (f)   Is   identical   with,   or   confusingly   similar   to,   or  
Decision   does   not   in   any   way   diminish   the   protection   constitutes   a   translation   of   a   mark   considered   well-­‐
afforded   to   valid   trademark   registrations   made   under   known   in   accordance   with   the   preceding   paragraph,  
said   law.   It   was   glaringly   obvious,   however,   from   the   which   is   registered   in   the   Philippines   with   respect   to  
testimony   of   movant’s   own   witness   that   DGCI’s   goods   or   services   which   are   not   similar   to   those   with  
registration   of   the   subject   mark   and   logo   was   void   due   respect   to   which   registration   is   applied   for:   Provided,  
to   the   existence   of   bad   faith   and   the   absence   of   the   That   use   of   the   mark   in   relation   to   those   goods   or  
requisite  2-­‐month  prior  use.   services   would   indicate   a   connection   between   those  
  goods  or  services,  and  the  owner  of  the  registered  mark:  
E. Standard  for  registrability   Provided  further,  That  the  interests  of  the  owner  of  the  
  registered  mark  are  likely  to  be  damaged  by  such  use;  
Distinctiveness.    
  (g)   Is   likely   to   mislead   the   public,   particularly   as   to   the  
F. What  marks  may  be  registered?   nature,   quality,   characteristics   or   geographical   origin   of  
  the  goods  or  services;  
SECTION  123.  REGISTRABILITY.  –      
(h)  Consists  exclusively  of  signs  that  are  generic  for  the  
123.1.  A  mark  cannot  be  registered  if  it:  
goods  or  services  that  they  seek  to  identify;  
   
(a)  Consists  of  immoral,  deceptive  or  scandalous  matter,   (i)   Consists   exclusively   of   signs   or   of   indications   that  
or   matter   which   may   disparage   or   falsely   suggest   a   have  become  customary  or  usual  to  designate  the  goods  
connection   with   persons,   living   or   dead,   institutions,   or   services   in   everyday   language   or   in   bona   fide   and  
beliefs,  or  national  symbols,  or  bring  them  into  contempt   established  trade  practice;  
or  disrepute;    
  (j)  Consists  exclusively  of  signs  or  of  indications  that  may  
(b)  Consists  of  the  flag  or  coat  of  arms  or  other  insignia   serve   in   trade   to   designate   the   kind,   quality,   quantity,  
of  the  Philippines  or  any  of  its  political  subdivisions,  or  of   intended   purpose,   value,   geographical   origin,   time   or  
any  foreign  nation,  or  any  simulation  thereof;   production  of  the  goods  or  rendering  of  the  services,  or  
  other  characteristics  of  the  goods  or  services;  
(c)   Consists   of   a   name,   portrait   or   signature   identifying   a    
particular  living  individual  except  by  his  written  consent,  

o 69
Katrina Michelle Mancao
 
(k)   Consists   of   shapes   that   may   be   necessitated   by   CIA  GENERAL  DE  TOBACCO  V.  AHLAMBRA  CIGAR  (1916)  
technical   factors   or   by   the   nature   of   the   goods   Court  descision:  
themselves  or  factors  that  affect  their  intrinsic  value;   Where  plaintiff’s  registered  TN  is  “La  Flor  de  la  Isabela”  
  which   it   uses   by   way   of   advertisements,   signs   over   the  
(l)   Consists   of   color   alone,   unless   defined   by   a   given   place  of  business,  upon  letter  heads  and  in  other  ways  to  
form;  or   furnish   the   public   a   method   of   distinguishing   the  
  plaintiff’s  business,  and  sends  its  manufactured  products  
(m)  Is  contrary  to  public  order  or  morality.   into   the   market   with   such   TN   upon   them,   the   plaintiff  
  acquire  no  trade-­‐name  rights  in  the  single  word  “Isabela”  
123.2.   As   regards   signs   or   devices   mentioned   in   where   it   is   not   claimed   that   the   use   of   the   word   “Isabela”  
paragraphs   (j),   (k),   and   (l),   nothing   shall   prevent   the   is   an   infringement   of   the   trade-­‐name   “La   Flor   de   la  
registration  of  any  such  sign  or  device  which  has  become   Isabela.”  
distinctive   in   relation   to   the   goods   for   which   registration    
is  requested  as  a  result  of  the  use  that  have  been  made   “Isabela”   is   a   name,   quality   or   description   of   the  
of   it   in   commerce   in   the   Philippines.   The   Office   may   merchandise   in   connectin   with   which   the   word   is   used,  
accept   as   prima   facie   evidence   that   the   mark   has   or   the   geographical   place   of   its   production   and   origin,  
become   distinctive,   as   used   in   connection   with   the   and   the   statute   provides   that   such   a   name   cannot   be  
applicant's   goods   or   services   in   commerce,   proof   of   appropriated  as  a  trademark  or  a  trade-­‐name.  
substantially   exclusive   and   continuous   use   thereof   by    
the  applicant  in  commerce  in  the  Philippines  for  five  (5)   ANG  TIBAY  V.  TEODORO,  SUPRA  
years   before   the   date   on   which   the   claim   of  
Court  decision:  
distinctiveness  is  made.  
"Ang   Tibay"   is   not   a   descriptive   term   within   the   meaning  
 
of   the   Trade-­‐Mark   Law   but   rather   a   fanciful   or   coined  
123.3.   The   nature   of   the   goods   to   which   the   mark   is   phrase   which   may   properly   and   legally   be   appropriated  
applied  will  not  constitute  an  obstacle  to  registration.   as  a  trademark  or  trade-­‐name.  
   
ETEPHA  V.  DIRECTOR  OF  PATENTS,  SUPRA   ARCE  SONS  V.  SELECTA  BISCUITS  (1961)  
Court  decision:   Court  decision:  
Tussin   is   merely   descriptive;   it   is   generic;   it   furnishes   to   The   word   “Selecta”   may   be   an   ordinary   or   common  
the   buyer   no   indication   of   the   origin   of   the   goods;   it   is   word   in   the   sense   that   it   may   be   used   or   employed   by  
open   for   appropriation   by   anyone.   It   is   accordingly   anyone  in  promoting  his  business  or  enterprise  BUT  once  
barred   from   registration   as   a   trademark.   But   while   adopted   or   coined   in   connection   with   one’s   business   as  
“Tussin”   by   itself   cannot   thus   be   used   exclusively   to   an   emblem   or   sign   to   characterize   its   products,   or   as   a  
identify  one’s  goods,  it  may  properly  become  the  subject   badge   of   authenticity,   it   may   acquire   a   secondary  
of   a   trademark   by   “combination   with   another   word   or   meaning  as  to  be  exclusively  associated  with  its  products  
phrase."   and  business.    
   
BAXTER  V.  ZUASUA  (1905)   “DOCTRINE   OF   SECONDARY   MEANING”   a   word   or  
Refresher:   phrase   originally   incapable   of   exclusive   appropriation  
“Agua   de   Kananga”   and   “Kananga   Superior”   are   toilet   with   reference   to   an   article   on   the   market   because  
waters   produced   by   two   competing   companies.   Baxter   geographically   or   otherwise   descriptive,   might  
acquired  right  to  use  TM  of  Agua  de  Kananga.  Kananga  is   nevertheless   have   been   used   so   long   and   so   exclusively  
a  flower.     by  one  producer  with  reference  to  his  article  that,  in  that  
  trade   and   to   that   branch   of   the   purchasing   public,   the  
Doctrine:   word   or   phrase   has   come   to   mean   that   the   article   was  
IF   the   goods   in   question   had   really   nothing   to   do   with   his  product.    
the  said  flower,  then  it  was  not  lawful  for  the  plaintiffs  to    
sell   them   to   the   public   under   the   name   of   “Agua   de   The  word  “Selecta”  may  be  placed  at  par  with  the  words  
Kananga”   because   the   people   might   be   deceived   as   to   “Ang  Tibay”  which  this  Court  has  considered  not  merely  
the  nature  of  the  goods,  taking  for  “Kananga”  an  article   as   descriptive   term   within   the   meaning   of   the   TM   Law  
which   as   a   matter   of   fact,   had   nothing   to   do   with   the   but  as  “fanciful”  or  “coined  phrase”  or  a  trademark.  
said  flower.      
  KABUSHI  KAISHA  ISETAN  V.  IAC,  SUPRA  
The   names   and   denominations   generally   used   in   Court  decision:  
commerce   to   designate   the   class   of   goods   or   It  might  be  pertinent  at  this  point  to  stress  that  what  is  
merchandise  cannot  be  the  subject  of  a  trademark.   involved   in   this   case   is   not   so   much   a   trademark   as   a  
  tradename.   Isetann   Department   Store,   Inc.   is   the   name  

70 z
Intellectual Property Law

of   a   store   and   not   of   product   sold   in   various   parts   of   the   this   ground   alone,   Fredco’s   registration   of   the   mark  
country.   This   case   must   be   differentiated   from   cases   "Harvard"  should  have  been  disallowed.    
involving   products   bearing   such   familiar   names   as    
"colgate",   "Singer".   "Toyota",   or   "Sony"   where   the   “Harvard”   is   a   well-­‐known   name   and   mark   not   only   in  
products   are   marketed   widely   in   the   Philippines.   There   is   the   United   States   but   also   internationally,   including   the  
not   product   with   the   name   "Isetann"   popularized   with   Philippines.   The   mark   "Harvard"   is   rated   as   one   of   the  
that   brand   name   in   the   Philippines.   Unless   one   goes   to   most  famous  marks  in  the  world.  It  has  been  registered  
the  store  called  Isetann  in  Manila,  he  would  never  know   in  at  least  50  countries.  It  has  been  used  and  promoted  
what   the   name   means.   Similarly,   until   a   Filipino   buyer   extensively   in   numerous   publications   worldwide.   It   has  
steps  inside  a  store  called  "Isetan"  in  Tokyo  or  Hongkong,   established  a  considerable  goodwill  worldwide  since  the  
that  name  would  be  completely  alien  to  him.  The  records   founding  of  Harvard  University  more  than  350  years  ago.  
show  that  among  Filipinos,  the  name  cannot  claim  to  be   It   is   easily   recognizable   as   the   trade   name   and   mark   of  
internationally  well-­‐known.   Harvard  University  of  Cambridge,  Massachusetts,  U.S.A.,  
  internationally  known  as  one  of  the  leading  educational  
ASIA  BREWERY  V.  CA  (1933)   institutions   in   the   world.   As   such,   even   before   Harvard  
Court  decision:   University   applied   for   registration   of   the   mark   "Harvard"  
The   fact   that   the   words   “pale   pilsen”   are   part   of   ABI’s   in   the   Philippines,   the   mark   was   already   protected   under  
trademark   does   not   constitute   an   infringement   of   SMC’s   Article  6bis  and  Article  8  of  the  Paris  Convention.  Again,  
trademark  (San  Miguel  Pale  Pilsen)  for  “pale  pilsen”  are   even   without   applying   the   Paris   Convention,   Harvard  
generic   words   descriptive   of   the   color   (pale),   of   a   strong   University  can  invoke  Section  4(a)  of  R.A.  No.  166  which  
type   of   beer   (pilsen)   which   is   a   light   bohemian   beer   with   prohibits   the   registration   of   a   mark   "which   may  
a  strong  hops  flavor  that  originated  in  the  City  of  Pilsen   disparage   or   falsely   suggest   a   connection   with   persons,  
in   Czechoslovakia   and   became   famous   in   the   Middle   living  or  dead,  institutions,  beliefs  x  x  x."  
Ages.   “Pilsen”   is   primarily   geographically   descriptive    
word,   hence,   non-­‐registrabel   and   not   appropriable   by   02  February  2012  
any  beer  manufacturer.    
  DEFINITION  OF  “TRADEMARK”    
EMERALD  GARMENT   MANUFACTURING  V.   CA  AND  H.D.  LEE   -­‐ Scent  and  aural  cannot  be  trademarks  because  
Phil  law  on  visible  signs  
CO.,  INC.  (1995)    
Refresher:   FUNCTIONS  OF  A  TRADEMARK    
H.D.   Lee   Co.   =   a   Delaware   corp.,   owner   of   the   “Lee”   -­‐ Traditional   purpose   –   Point   out   the   source   of  
trademark   goods  or  services.    
  o This   began   when   artisans   decided   to  
Emerald   Garment   Manufacturing   =   domestic   corp.,   put   mark   on   their   goods   to   indicate  
registrant   of   “Stylistic   Mr.   Lee”   trademark   used   on   skirts,   who   made   the   goods,   where   the  
jeans,   blouses,   socks,   briefs,   jackets,   jogging   suits,   good  were  made.    
dresses,  shorts,  shirts  and  lingerie  under  Class  25.   -­‐ Why   should   trademarks   be   recognized   and  
  protected?    
Court  decision:   o To   encourage   the   enterprise   to   invest  
"LEE"   is   primarily   a   surname.   Private   respondent   cannot,   in   the   creation   of   durable   goods  
therefore,   acquire   exclusive   ownership   over   and   singular   through   the   guaranty   that   if   they  
use  of  said  term.  It  has  been  held  that  a  personal  name   continue   to   bring   goods   in   such  
or   surname   may   not   be   monopolized   as   a   trademark   or   market,   their   identity   is   protected,  
tradename   as   against   others   of   the   same   name   or   their   market   is   secured   because   no  
surname.   For   in   the   absence   of   contract,   fraud,   or   other   person   can   appropriate   their  
estoppel,   any   man   may   use   his   name   or   surname   in   all   marks.  If  the  enterprise  invest  in  their  
legitimate   ways.   Thus,   "Wellington"   is   a   surname,   and   its   services   and   it   acquires   goodwill,   the  
first  user  has  no  cause  of  action  against  the  junior  user  of   goodwill   will   be   preserved   because  
"Wellington"  as  it  is  incapable  of  exclusive  appropriation.   no  other  person  allowed  to  make  use  
  of   its   mark   that   would   undermine  
FREDCO   MANUFACTURING   CORP.   V.   PRESIDENT   AND   your  integrity  in  the  market.    
FELLOWS  OF  HARVARD  COLLEGE  (2011)   o In   turn,   the   public   will   benefit   from  
the  creation  of  superior  goods.  
Court  decision:  
-­‐ Current   purpose   changed   –   Trademark   has  
Fredco’s   use   of   the   mark   “Harvard,”   coupled   with   its  
evolved   with   centuries   of   use.   Because   the  
claimed   origin   in   Cambridge,   Massachusetts,   obviously  
marketplace   has   become   global,   trademarks  
suggests  a  false  connection  with  Harvard  University.  On  
has   evolved   from   mere   source   indicators,   to  

o 71
Katrina Michelle Mancao
 
serving   very   critical   functions   especially   in   a   Royce  but  not  call  it  as  such.  
crowded  marketplace.     Later,   BMW   acquired   the  
o Associative  quality  for  luxury  brands  –   mark   from   Aero   Engine.  
because   of   a   particular   trademark,   BMW   gave   Volkswagen   a  
consumers   would   choose   to   buy   the   year  to  use  its  mark.  
product  because  they  want  to  belong    
to  a  particular  class.   How  are  marks  acquired?  
§ Mark   is   associated   with   a   -­‐ Now  (under  the  IPC):  Through  registration.  
particular   class   or   lifestyle.   -­‐ Previously:  Through  actual  use.  
People   wearing   a   brand   (if   -­‐ Because   of   this   shift/change   in   the   basis   of  
genuine)   belong   to   a   special   ownership   over   trademarks,   it   presents   a   host  
class  of  people.   of  interesting  transitional  problems:    
§ When   a   mark   creates   a   certain   o Under   old   law,   the   owner   of   the   mark  
kind   of   image,   it   can   make   was  prior  user  in  Philippine  commerce  
consumers  want  it.   (Unno   Commercial   Enterprises   vs.  
§ It   creates   a   segment   of   the   General   Milling).   First   user/entity   who  
population.   Without   the   used  it  in  commerce  owns  the  mark.    
consumers   realizing   it,   they   buy   o NOTE:   Under   the   old   law,   local  
not  just  the  product  but  also  the   applicants   were   required   to   use   the  
lifestyle.   mark   for   at   least   2   months   prior   to  
o Bearer  of  information  –  it  is  the  heart   the   filing   of   the   application.   If   you  
of  a  business.   want   to   file,   you   must   have   used   the  
§ It   bears   information   that   such   mark   2   months   prior   to   filing   of  
thing   was   created   by   the   application.    
manufacturer.   § It’s  a  very  risky  undertaking  à  To  
o It  is  the  hear  of  the  business.   own   the   mark,   one   has   to   build  
§ It   has   a   direct   effect   on   the   the   business   and   spend   for  
company’s   profits;   whether   the   investments,   put   up   store,  
company   that   will   flourish   or   incorporate,  hire  accountant  and  
become  bankrupt.     signage   maker,   and   do   your  
§ Strength   of   the   mark   dictates   marketing  materials.  Since  one  is  
the  premium  of  the  price.   required   to   have   used   mark   for  
§ Trademark   has   become   an   asset   at   least   2   month   prior   to   the  
of   the   company.   It   is   hard   to   filing   of   application,   for   2  
distinguish   between   it   and   a   months,  you’re  taking  a  risk  that  
business.  If  the  company  loses  its   somebody   else   might   use   your  
right   to   a   trademark,   it   also   loses   mark.   In   the   meantime,   you  
the  business.   already   have   spent   your   money  
§ Note:   As   an   asset,   you   can   sell   and  sunk  in  the  cost.    
the   trademark   independently   of   § That   was   the   requirement  
the  business.   because   under   the   law,   actual  
• Ex:   Volkswagen   and   BMW   use  was  the  basis  of  ownership.    
bid   for   the   trademark   of   o Why  change  the  law?    
Rolls-­‐Royce.   Volkswagen   § Because   the   Philippines   became  
won   the   bidding.   Why   did   a   party   to   the   Paris   Convention.  
they   want   to   acquire   the   It   required   us   to   adhere   to   the  
trademark?   Because   it   is   “national  treatment”  principle.    
very   difficult   to   build   a    
brand.   Unfortunately,   the   Kabushi  Kaisha  Isetan  v  IAC    
car   companies   did   not   do   a   -­‐ Case   between   a   local   registrant   and   a   foreign  
due   diligence   investigation.   non-­‐user.  
They   did   not   realize   that   the   -­‐ Local  user  prevailed  because  the  old  law  based  
trademark   did   not   belong   to   ownership   of   the   trademark   on   actual   use.  
the   car   company.   The   Since   there   was   no   actual   use   for   Kabushi  
trademark  belonged  to  Aero   Kaisha   in   the   PH,   there   was   no   goodwill   to   be  
Engine.   What   Volkswagen   protected.  
bought   was   the   factory.   -­‐ BUT   the   Court   recognized   an   exception   à  
Thus,   it   can   build   Rolls-­‐ Under   the   Paris   Convention,   States   are  

72 z
Intellectual Property Law

required  to  recognize  well-­‐known  trademark.   o ONLY   EXCEPTION:   internationally  


o Exception  has  following  elements:   well-­‐known  mark  
1. the  mark  must  be  internationally   • The   only   way   a   prior   registrant   user   will   not  
known  or  well  known;   prevail  is  if  use  was  subsequent  to  the  filing  of  
2. the   subject   of   the   mark   must   be   the  application.    
a  trademark;    
3. the   mark   must   be   for   use   in   the   NOW  à  Registration  is  the  basis  of  ownership.  
same  or  similar  kinds  of  goods;   -­‐ Under   IPC,   are   you   required   to   make   use   of  
4. the  person  claiming  must  be  the   mark  prior  to  filing  of  application?  No.    
owner  of  the  mark.    
-­‐ Kabushi  Kaisha  was  not  able  to  prove  the     Standard  of  registerability:  Distinctiveness.  
   
RECAP:   WHAT  MARKS  MAY  NOT  BE  REGISTERED?  
GR:  First  user  (Established  in  the  law  and  Unno)   -­‐ Basically,   Section   123   IPC   is   an   enumeration   of  
EXCEPTION:   First   user   who   did   not   use   in   PH   marks  that  are  not  distinctive;  i.e.  Incapable  of  
commerce     pointing  our  source  of  goods  or  services.    
EXCEPTION   TO   THE   EXCEPTION:   First    
user,   unregistered   and   not   used   in   PH   Deceptive  
Commerce   AND   internationally   well-­‐ -­‐ Cannot   be   distinctive   because   suggests   a   false  
known   (e.g.   Kabushi   Kaisha)   –   connection.    
Applying  Paris  Convention   -­‐ E.g.  Jose  Rizal  Shoes  
   
Philip  Morris  cases   Flag/  Coat  of  arms  
-­‐ Between   a   registrant   non-­‐user   vs.   non-­‐ -­‐ Paris   Convention   makes   obligation   not   to   allow  
registrant   user   of   the   mark   in   the   Philippine   registration   of   flags   or   national   symbols.   They  
commerce?     are  outside  commerce  of  men.    
o The   user   prevails   because   ownership   o Everyone   can   make   use   of   the   mark  
of  trademark  is  on  the  basis  of  use  in   to   indicate   origin   but   it   cannot   be   a  
the  Philippine  commerce.  While  Philip   trademark.    It  cannot  be  appropriated  
Morris   was   registrant,   it   did   not   make   by   manufacturer   at   the   expense   of  
use  of  the  same.     the  rest.    
  -­‐ BUT:   There   are   certain   countries   that   have  
Shangrila  v.  DGCI   acquired   a   certain   reputation   for   producing  
-­‐ Case  between  a  registrant  user  (the  restaurant)   goods   of   superior   quality.   Companies   based  
and  the  hotel  that  wants  to  come  in.   here   could   use   the   country’s   flag,   not   as  
-­‐ The   Hotel   prevailed   and   restaurant’s   trademark,   but   as   an   indicia   of   the   country   of  
registration  was  cancelled.  Why?   origin  of  the  goods.  
o Evidence   showed   that   there   was   no   o E.g.   Switzerland.   Swiss   have   strong  
actual   use   prior   to   the   filing   of   laws  about  who  can  use  the  mark.  To  
application.   There   was   a   violation   of   be   able   to   use   the   flag,   the   Swiss  
the  requirements  the  RA  166,  thus  the   company   must   be   able   to   meet   a  
registration   was   not   a   valid   certain   quality,   and   undergo   quality  
registration.     control.  The  Swiss  government  wants  
o You   may   be   the   registrant   and   prior   to   ensure   that   those   who   use   Swiss  
user  BUT  if  you  failed  to  comply  with   flags  would  maintain  the  standard  so  
requirement   of   actual   use   before   that   their   reputation   of   producing  
filing   of   application,   you   will   lose   the   superior  goods  will  be  maintained.    
registration   because   the   registration    
will  be  invalid.     Name,  portrait  or  signature  identifying  a  particular  living  
  individual  
The  foregoing  were  cases  decided  under  a  different  law   -­‐ If   a   famous   doctor’s   name   is   being   used   in  
(RA  166)  –  PRIOR  to  the  passage  of  the  IP  Code.    Thus:   relation   to   cosmetic   products,   will   the  
• Prior   use   was   the   basis   of   ownership   of   the   trademark  be  allowed?    
trademark.     o Yes,   if   done   with   the   consent   of   the  
• Prior  user  in  Philippine  commerce  shall  own  the   doctor.  
trademark.   -­‐ If   the   person   has   died,   what   will   you   do   to  
• Prior  user  in  Philippine  commerce  will  prevail  as   make  use  of  the  name,  etc?  
against  a  registrant  non-­‐user.     o Get  heirs’  consent.  

o 73
Katrina Michelle Mancao
 
   
Identical   with   a   registered   mark,   or   a   mark   with   an   Generic  
earlier  filing/priority  date   -­‐ E.g.  shoes,  bags,  damit,  blusa,  pantalon,  radyo.    
-­‐ Very  important  because  this  indicated  the  shift.     -­‐ They   refer   to   the   goods/services   themselves.  
-­‐ “Likely   to   deceive/cause   confusion”   means   you   Therefore,   nobody   can   monopolize   or  
don’t   even   have   to   prove   actual   confusion   for   appropriate   them,   it   cannot   be   acquired   by  
as  long  as  there  is  likelihood  of  confusion.     anyone.    
-­‐ This   shows   to   you   that   our   TM   regime   has   o If   you   allow   somebody   to   exclusively  
shifted  to  a  first-­‐to-­‐file   system  because  a  mere   use   them,   it   would   be   to   the  
application   with   an   earlier   filing   date   can   detriment   of   everyone   else.   Imagine,  
prevent   subsequent   application   for   the   same   you   can   no   longer   sell   bags   because  
or  confusingly  similar  mark  subsequently  filed.     bags  have  become  a  trademark!  
o It’s   very   critical   now   to   file   an    
application   ASAP   because   owner   of   Become   customary   or   usual   to   designate   the   goods   or  
mark   with   earlier   filing   date   would   services  in  everyday  language  
prevail  as  against  the  owner  of  a  mark   -­‐ This   refers   to   the   phenomena   called  
with  a  later  filing  date.  Time  is  critical   “genericide”   à   marks   that   used   to   be  
and  essential.     distinctive  lose  their  distinctiveness  because  of  
  popularity  of  the  mark.    
Well-­‐known  mark  (123.1e  and  123.1f)   -­‐ The   downside   of   being   a   strong   mark   that  
-­‐ E   and   F   both   deal   with   WELL-­‐KNOWN   MARK   people   stop   referring   to   the   generic   term   of  
locally  and  internationally.     good/services   but   actually   use   the   trademark  
-­‐ What’s  the  difference  between  the  two?     as  generic  term.    
o Under   E:   speaks   about   well-­‐known   -­‐ E.g.  “Kodak”  in  Ph;  Xerox;  Sunkist  
mark  WON  registered   -­‐ Xerox  and  Kodak  are  arbitrary   trademarks;  i.e.  
o Under   F:   speaks   about   registered   they   were   just   coined   by   the   manufacturers/  
well-­‐known  mark.     they  never  existed  before.  
-­‐ What’s   the   difference   in   terms   of   scope   of   o In   the   realm   of   trademarks,   arbitrary  
protection?     marks   should   be   entitled   to   the  
o E:   protected   only   against   use   for   strongest  protection.  
identical  or  similar  goods  or  services.   o The   Philippines   seem   to   defy   that.  
o F:   protected   also   as   against   use   for   The   more   arbitrary   the   trademark   is,  
goods  or  services  not  similar  to  those   the  more  we  think  we  have  license  to  
with   respect   to   which   registration   is   use  them.  (ex.  Colgate,  Frigidaire)  
applied  for.   -­‐ Other  trademarks  that  have  fallen  in  the  public  
-­‐ The   foregoing   illustrated   why   registration   in   domain  –  escalator;  elevator;  nylon;  thermos  
the   Philippines   of   a   well-­‐known   mark   is   still   o Owners   of   these   didn’t   work   hard  
practical   à   scope   of   protection   after   enough   to   prevent   the   falling   into  
registration  would  be  broader:     public  domain  of  their  trademarks.    
o They   could   oppose   the   filing   of   -­‐ What’s   the   best   evidence   to   prove   that   the  
applications   for   the   same   or   mark   has   become   generic,   a   victim   of  
confusingly   similar   mark   even   for   genericide,   has   lost   its   distinctiveness   and  
goods/services   that   are   dissimilar   if   therefore,  may  be  cancelled?    
you  had  registered  in  the  PH.   o Existence   of   the   word   in   the  
-­‐ In   short,   it   appears   that   under   the   IPC,   only   dictionary.  
registered   internationally   well-­‐known   mark   in   § Existence   in   the   dictionary  
the   Philippines   can   oppose   on   the   ground   of   means   it   forms   part   of   the  
dilution.     lexicon.    
o On   the   other   hand,   an   unregistered   § Companies   spend   a   great   deal  
well-­‐known   mark   cannot   raise   for   service   watch   to   check   US  
dilution.   The   scope   of   protection   for   dictionaries   to   make   sure   that  
an   unregistered   well-­‐known   mark   is   well-­‐known  marks  have  not  been  
limited  to  similar  or  identical  goods  or   incorporated  in  the  dictionary    
services.      
  Characteristics  of  the  goods  or  services  
Can  mislead  the  public   -­‐ Also  known  as  “descriptive  marks”  
-­‐ Because  it’s  misleading,  therefore,  it  cannot  be   -­‐ Being   merely   descriptive,   they   do   not   help   in  
descriptive.     pointing  out  the  source  or  origin  of  the  good.  

74 z
Intellectual Property Law

  G. Application  for  registration  


Contrary  to  public  order  or  morality    
  1. Application  
RECAP:  
 
GR:   The   foregoing   cannot   be   registered   because   they   do  
not  meet  the  standards  of  distinctiveness.     SECTION   3.   INTERNATIONAL   CONVENTIONS   AND  
  RECIPROCITY.  –  
EXCEPTION:   Certain   marks   may   be   registered   Any  person  who  is  a  national  or  who  is  domiciled  or  has  
provided  they  acquired  secondary  meaning.     a  real  and  effective  industrial  establishment  in  a  country  
  which  is  a  party  to  any  convention,  treaty  or  agreement  
What  are  these  marks  that  may  be  registered?     relating   to   intellectual   property   rights   or   the   repression  
1. Descriptive  marks   of   unfair   competition,   to   which   the   Philippines   is   also   a  
2. Shapes   party,   or   extends   reciprocal   rights   to   nationals   of   the  
3. Colors   Philippines   by   law,   shall   be   entitled   to   benefits   to   the  
  extent  necessary  to  give  effect  to  any  provision  of  such  
While   not   registrable   at   first   instance   because   they   are   convention,   treaty   or   reciprocal   law,   IN   ADDITION   to  
deemed   non-­‐distinctive,   they   may   be   registered   the   rights   to   which   any   owner   of   an   intellectual   property  
provided  they  have  acquired  secondary  meaning.     right  is  otherwise  entitled  by  this  Act.  
   
What  is  “SECONDARY  MEANING”?     SECTION  124.  REQUIREMENTS  OF  APPLICATION.  –    
-­‐ Originally  not  distinctive  but  through  its  use  by  
124.1.   The   application   for   the   registration   of   the   mark  
relevant  public  has  become  distinctive.  
-­‐ Under   the   old   law   (RA   166)   à   proprietor   can   shall   be   in   Filipino   or   in   English   and   shall   contain   the  
use   the   supplemental   register   to   support   the   following:  
claim   that   the   term   has   acquired   secondary    
meaning.     (a)  A  request  for  registration;  
o Now,   there   is   no   supplemental    
register.   Propritor   would   have   to   (b)  The  name  and  address  of  the  applicant;  
convince   IPO   that   while   your   mark    
may   be   descriptive,   shapes,   colors,   he   (c)   The   name   of   a   State   of   which   the   applicant   is   a  
national   or   where   he   has   domicile;   and   the   name   of   a  
has   exclusively   and   continuously  
State   in   which   the   applicant   has   a   real   and   effective  
used  it  for  at  5  years  à  the  mark  has  
industrial  or  commercial  establishment,  if  any;  
acquired   a   secondary   meaning   that  
would  now  entitle  him  to  register.      
  (d)  Where  the  applicant  is  a  juridical  entity,  the  law  under  
Choice  of  trademark  is  very  critical  to  any  business.  If  the   which  it  is  organized  and  existing;  
 
proprietor   makes   a   mistake   and   uses   a   non-­‐registrable  
(e)  The  appointment  of  an  agent  or  representative,  if  the  
TM,  there  is  really  an  economic  impact  on  the  business.    
-­‐ Proprietor  should  be  careful  not  to  use  generic   applicant  is  not  domiciled  in  the  Philippines;  
terms  as  the  same  cannot  be  registered.  If  the    
proprietor   has   acquired   goodwill   for   a   generic   (f)   Where   the   applicant   claims   the   priority   of   an   earlier  
application,  an  indication  of:  
term,   he   cannot   prevent   other   people   from  
using   it   à   generic   term   is   always   in   the   public    
domain.   i)   The   name   of   the   State   with   whose   national  
o “Kamiseta”   and   “Bayo”   (which   is   office   the   earlier   application   was   filed   or   if   filed  
with  an  office  other  than  a  national  office,  the  
Ilonggo   for   clothes)   are  
name  of  that  office,  
NOT   registrable   under   the  
IPC.      
§ They   are,   however,   ii)   The   date   on   which   the   earlier   application  
registrable   under   the   was  filed,  and  
 
old  law.  
iii)   Where   available,   the   application   number   of  
 
the  earlier  application;  
 
(g)   Where   the   applicant   claims   color   as   a   distinctive  
feature  of  the  mark,  a  statement  to  that  effect  as  well  as  
the   name   or   names   of   the   color   or   colors   claimed   and   an  
indication,   in   respect   of   each   color,   of   the   principal   parts  
of  the  mark  which  are  in  that  color;  

o 75
Katrina Michelle Mancao
 
  became   distinctive   of   the   applicant's   or   owner's   goods,  
(h)   Where   the   mark   is   a   three-­‐dimensional   mark,   a   business  or  services.    
statement  to  that  effect;    
  SECTION   128.   SINGLE   REGISTRATION   FOR   GOODS   AND/OR  
(i)  One  or  more  reproductions  of  the  mark,  as  prescribed  
SERVICES.  –    
in  the  Regulations;  
Where   goods   and/or   services   belonging   to   several  
 
classes   of   the   Nice   Classification   have   been   included   in  
(j)  A  transliteration  or  translation  of  the  mark  or  of  some  
one  (1)  application,  such  an  application  shall  result  in  one  
parts  of  the  mark,  as  prescribed  in  the  Regulations;  
registration.  
 
 
(k)   The   names   of   the   goods   or   services   for   which   the  
registration   is   sought,   grouped   according   to   the   classes   SECTION   130.   SIGNATURE   AND   OTHER   MEANS   OF   SELF-­‐
of   the   Nice   Classification,   together   with   the   number   of   IDENTIFICATION.  –    
the  class  of  the  said  Classification  to  which  each  group  of   130.1.   Where   a   signature   is   required,   the   Office   shall  
goods  or  services  belongs;  and   accept:  
   
(l)   A   signature   by,   or   other   self-­‐identification   of,   the   (a)  A  hand-­‐written  signature;  or  
applicant  or  his  representative.    
  (b)  The  use  of  other  forms  of  signature,  such  as  a  printed  
124.2.   The   applicant   or   the   registrant   shall   file   a   or   stamped   signature,   or   the   use   of   a   seal   instead   of   a  
declaration   of   actual   use   of   the   mark   with   evidence   to   hand-­‐written   signature:   Provided,   That   where   a   seal   is  
that  effect,  as  prescribed  by  the  Regulations  within  three   used,   it   should   be   accompanied   by   an   indication   in  
(3)   years   from   the   filing   date   of   the   application.   letters  of  the  name  of  the  signatory.  
Otherwise,   the   application   shall   be   refused   or   the   mark    
shall  be  removed  from  the  Register  by  the  Director.   130.2.   The   Office   shall   accept   communications   to   it   by  
  telecopier,   or   by   electronic   means   subject   to   the  
124.3.   One   (1)   application   may   relate   to   several   goods   conditions  or  requirements  that  will  be  prescribed  by  the  
and/or  services,  whether  they  belong  to  one  (1)  class  or   Regulations.   When   communications   are   made   by  
to  several  classes  of  the  Nice  Classification.   telefacsimile,   the   reproduction   of   the   signature,   or   the  
  reproduction  of  the  seal  together  with,  where  required,  
124.4.   If   during   the   examination   of   the   application,   the   the   indication   in   letters   of   the   name   of   the   natural  
Office   finds   factual   basis   to   reasonably   doubt   the   person   whose   seal   is   used,   appears.   The   original  
veracity  of  any  indication  or  element  in  the  application,  it   communications   must   be   received   by   the   Office   within  
may   require   the   applicant   to   submit   sufficient   evidence   thirty  (30)  days  from  date  of  receipt  of  the  telefacsimile.  
to  remove  the  doubt.      
  130.3.   No   attestation,   notarization,   authentication,  
SECTION  125.  REPRESENTATION;  ADDRESS  FOR  SERVICE.  –     legalization   or   other   certification   of   any   signature   or  
If   the   applicant   is   not   domiciled   or   has   no   real   and   other   means   of   self-­‐identification   referred   to   in   the  
effective  commercial  establishment  in  the  Philippines,  he   preceding   paragraphs,   will   be   required,   except,   where  
shall   designate   by   a   written   document   filed   in   the   Office,   the  signature  concerns  the  surrender  of  a  registration.  
the  name  and  address  of  a  Philippine  resident  who  may    
be   served   notices   or   process   in   proceedings   affecting   SECTION  131.  PRIORITY  RIGHT.  –    
the   mark.   Such   notices   or   services   may   be   served   upon   131.1.   An   application   for   registration   of   a   mark   filed   in   the  
the   person   so   designated   by   leaving   a   copy   thereof   at   Philippines   by   a   person   referred   to   in   Section   3,   and   who  
the  address  specified  in  the  last  designation  filed.  If  the   previously  duly  filed  an  application  for  registration  of  the  
person   so   designated   cannot   be   found   at   the   address   same  mark  in  one  of  those  countries,  shall  be  considered  
given   in   the   last   designation,   such   notice   or   process   may   as   filed   as   of   the   day   the   application   was   first   filed   in   the  
be  served  upon  the  Director.     foreign  country.  
   
SECTION  126.  DISCLAIMERS.  –     131.2.   No   registration   of   a   mark   in   the   Philippines   by   a  
The   Office   may   allow   or   require   the   applicant   to   disclaim   person   described   in   this   section   shall   be   granted   until  
an  unregistrable  component  of  an  otherwise  registrable   such  mark  has  been  registered  in  the  country  of  origin  of  
mark  but  such  disclaimer  shall  not  prejudice  or  affect  the   the  applicant.  
applicant's   or   owner's   rights   then   existing   or   thereafter    
arising  in  the  disclaimed  matter,  nor  such  shall  disclaimer   131.3.  Nothing  in  this  section  shall  entitle  the  owner  of  a  
prejudice   or   affect   the   applicant's   or   owner's   right   on   registration   granted   under   this   section   to   sue   for   acts  
another  application  of  later  date  if  the  disclaimed  matter   committed   prior   to   the   date   on   which   his   mark   was  

76 z
Intellectual Property Law

registered   in   this   country:   Provided,   That,   SECTION  132.  APPLICATION  NUMBER  AND  FILING  DATE.  –    
notwithstanding   the   foregoing,   the   owner   of   a   well-­‐ 132.1.   The   Office   shall   examine   whether   the   application  
known   mark   as   defined   in   Section   123.1(e)   of   this   Act,  
satisfies  the  requirements  for  the  grant  of  a  filing  date  as  
that  is  not  registered  in  the  Philippines,  may,  against  an  
provided   in   Section   127   and   Regulations   relating   thereto.  
identical   or   confusingly   similar   mark,   oppose   its  
If  the  application  does  not  satisfy  the  filing  requirements,  
registration,   or   petition   the   cancellation   of   its  
the   Office   shall   notify   the   applicant   who   shall   within   a  
registration   or   sue   for   unfair   competition,   without  
period  fixed  by  the  Regulations  complete  or  correct  the  
prejudice  to  availing  himself  of  other  remedies  provided  
application   as   required,   otherwise,   the   application   shall  
for  under  the  law.  
be  considered  withdrawn.  
 
 
131.4.  In  like  manner  and  subject  to  the  same  conditions  
132.2.  Once  an  application  meets  the  filing  requirements  
and  requirements,  the  right  provided  in  this  section  may  
of   Section   127,   it   shall   be   numbered   in   the   sequential  
be  based  upon  a  subsequent  regularly  filed  application  in  
order,   and   the   applicant   shall   be   informed   of   the  
the   same   foreign   country:   Provided,   That   any   foreign  
application  number  and  the  filing  date  of  the  application  
application  filed  prior  to  such  subsequent  application  has  
will  be  deemed  to  have  been  abandoned.  
been   withdrawn,   abandoned,   or   otherwise   disposed   of,  
 
without   having   been   laid   open   to   public   inspection   and  
What’s   the   difference   between   “filing   date”   and   a  
without   leaving   any   rights   outstanding,   and   has   not  
“priority  date”?    
served,  nor  thereafter  shall  serve,  as  a  basis  for  claiming  
 
a  right  of  priority.  
FILING   DATE   -­‐   date   you   actually   filed   the   application,  
 
PROVIDED   you   met   all   the   minimum   requirements   under  
SECTION  239.2.  Marks  registered  under  Republic  Act  No.   Sec.  127.    
166   shall   remain   in   force   but   shall   be   deemed   to   have   The   IPO   enforces   this   strictly,   if   inadvertently   the   duty  
been  granted  under  this  Act  and  shall  be  due  for  renewal   officer   of   the   day   of   IPO   gave   you   a   filing   date   but   it  
within  the  period  provided  for  under  this  Act  and,  upon   turns   out   that   you   did   not   submit   the   list   of   goods   and  
renewal   shall   be   reclassified   in   accordance   with   the   services,   when   the   application   is   referred   to   trademark  
International   Classification.   Trade   names   and   marks   examiner,  the  trademark  examiner  will  examine  and  if  he  
registered   in   the   Supplemental   Register   under   Republic   discovers   that   you   did   not   comply   with   the   minimum  
Act   No.   166   shall   remain   in   force   but   shall   no   longer   be   requirement,   the   TE   will   revoke   your   filing   date.   Pag  
subject  to  renewal.   nirevoke   niya,   he   will   send   communication   that   you  
  forgot   one   requirement.   Only   when   you   submit   list   of  
2. Assignment   of   Application   goods   and   services   that   you   will   be   given   another   filing  
Number  and  Filing  Date   date.   Pati   application   number   mo   mawawala.   In   the  
  meantime,   if   somebody   else   files   and   application   for  
same   or   confusingly   similar   mark,   lagot   ka   na.   Because  
SECTION  127.  FILING  DATE.  –    
your   filing   date   is   only   when   you   complied   with   the  
127.1.  Requirements.  –     minimum   requirement.   It   takes   about   6   to   8   months  
The   filing   date   of   an   application   shall   be   the   date   on   before   it   is   assigned   to   an   examiner.   So   there   is   a   high  
which   the   Office   received   the   following   indications   and   probability   that   within   those   8   months   somebody   else  
elements  in  English  or  Filipino:   filed   an   application   and   because   that   application   meets  
  the  minimum  requirement,  it  would  have  an  earlier  filing  
(a)   An   express   or   implicit   indication   that   the   date.   As   between   you   with   revoked   filing   date   and  
registration  of  a  mark  is  sought;   somebody   earlier   filing   date,   the   latter   will   prevail.   Yours  
  has   been   revoked,   so   critical.   Filing   date   is   date   you  
(b)  The  identity  of  the  applicant;   actually   filed   provided   you   met   the   minimum  
  requirements  under  127.  
(c)   Indications   sufficient   to   contact   the    
applicant  or  his  representative,  if  any;   PRIORITY   DATE   -­‐   date   when   you   filed   a   foreign  
  application  for  the  same  mark  in  another  country,  a  Paris  
(d)   A   reproduction   of   the   mark   whose   Convention  country  or  TRIPS  country.  Then  you  filed  the  
registration  is  sought;  and   application  in  the  Ph  within  6  months,  you  can  claim  the  
  earlier  foreign  filing  date  as  your  filing  date  in  the  Ph.  So  
(e)   The   list   of   the   goods   or   services   for   which   the  law  will  deem  your  application  filed  as  of  that  date  of  
the  registration  is  sought.   filing  of  the  foreign  application.  BUT  in  order  for  you  to  
  avail   of   foreign   priority   date,   you   must   have   filed   the  
127.2.  No  filing  date  shall  be  accorded  until  the  required   Philippine  application  within  6  months  from  the  time  you  
fee  is  paid.   filed  the  foreign  application.    
   

o 77
Katrina Michelle Mancao
 
NB   Under   IP   Code   Sec   131,   there   is   no   6   months   the   mark   as   defined   in   Section   121   is   registrable   under  
limitation.  This  is  an  error  on  the  part  of  the  drafters  of   Section  123.  
IPC   because   priority   date   is   actually   a   convention    
obligation   (Paris   Convention).   Under   which   there   is   6   133.2.  Where  the  Office  finds  that  the  conditions  referred  
mos  limitation  to  avail  of  priority  date.  Since  every  other   to  in  Subsection  133.1  are  fulfilled,  it  shall  upon  payment  
country  in  the  world  subjects  6-­‐mo  period,  and  because   of  the  prescribed  fee,  forthwith  cause  the  application,  as  
of   reciprocity,   why   would   we   adapt   a   longer   period   to   filed,  to  be  published  in  the  prescribed  manner.  
claim  priority  date?  That  seems  to  be  the  effect  because    
of  131  which  failed  to  reproduce  the  6  months  limitation   133.3.   If   after   the   examination,   the   applicant   is   not  
under  the  Paris  Convention.     entitled   to   registration   for   any   reason,   the   Office   shall  
-­‐ But   what   the   IPO   did   was   to   issue   an   advise   the   applicant   thereof   and   the   reasons   therefor.  
administrative   circular   that   clarifies   priority   The   applicant   shall   have   a   period   of   four   (4)   months   in  
right   can   only   be   claimed   if   the   Philippine   which  to  reply  or  amend  his  application,  which  shall  then  
application  is  filed  within  6  months.     be   re-­‐examined.   The   Regulations   shall   determine   the  
-­‐ Basis  for  that:     procedure   for   the   re-­‐examination   or   revival   of   an  
a. Follow   basic   principles   of   reciprocity:   application   as   well   as   the   appeal   to   the   Director   of  
Why   should   we   give   foreign   countries   Trademarks  from  any  final  action  by  the  Examiner.  
who   only   limit   to   6mos   the   right   to    
claim   priority   date   more   rights?   Why   133.4.   An   abandoned   application   may   be   revived   as   a  
construe   IPC   to   be   limitless?   That  
pending   application   within   three   (3)   months   from   the  
would   be   contrary   to   national  
date  of  abandonment,  upon  good  cause  shown  and  the  
interest.    
payment  of  the  required  fee.  
b. Convention   obligation:   The   domestic  
 
law   provision   that   implements   the  
133.5.   The   final   decision   of   refusal   of   the   Director   of  
international   treaty   obligation   should  
be   read   in   conjunction   with   Trademarks   shall   be   appealable   to   the   Director   General  
international   treaty.   So   only   within   6   in   accordance   with   the   procedure   fixed   by   the  
months.     Regulations.  
   
NOTE:  Priority  dates  are  important  because  as   between   SECTION  126.  DISCLAIMERS.  –    
an   earlier   filing   date   and   an   earlier   priority   date,   the   The   Office   may   allow   or   require   the   applicant   to   disclaim  
priority  date  prevails.     an  unregistrable  component  of  an  otherwise  registrable  
E.g.  Jances  files  application  in  the  PH  on  01  Mar  2012,  to   mark  but  such  disclaimer  shall  not  prejudice  or  affect  the  
comply   with   minimum   requirements   of   Sec.   127.   Earla   applicant's   or   owner's   rights   then   existing   or   thereafter  
filed   application   for   the   same   mark   or   similar   arising  in  the  disclaimed  matter,  nor  such  shall  disclaimer  
goods/services   and   she   filed   it   on   12   Mar   2012   BUT   she   prejudice   or   affect   the   applicant's   or   owner's   right   on  
earlier   filed   in   SG   an   application   dated   15   Feb   2012.   When   another  application  of  later  date  if  the  disclaimed  matter  
Earla  made  a  claim  on  the  basis  of  that  foreign  application,   became   distinctive   of   the   applicant's   or   owner's   goods,  
a  priority  date  of  15  Feb  2012,  who  would  prevail?  Jances  or   business  or  services.    
Earla?      
Earla!   Although   filing   date   is   later,   she   has   a   claim   of   SECTION  129.  DIVISION  OF  APPLICATION.  –    
priority.   The   law   will   deem   Earla’s   application   to   have   Any   application   referring   to   several   goods   or   services,  
been   filed   as   of   15   Feb   2012   (date   she   filed   SG   hereafter  referred  to  as  the  "initial  application,"  may  be  
application)   for   as   long   as   she   complies   with   the   divided   by   the   applicant   into   two   (2)   or   more  
requirements  for  a  claim  of  priority,  namely,     applications,   hereafter   referred   to   as   the   "divisional  
• Within  6  months  from  application     applications,"  by  distributing  among  the  latter  the  goods  
• Certified  copy  of  the  application  and   or   services   referred   to   in   the   initial   application.   The  
English  translation   divisional   applications   shall   preserve   the   filing   date   of  
• Submission  of  foreign  registration   the   initial   application   or   the   benefit   of   the   right   of  
priority.  
   
3. Examination   Examination   consists   of   a   determination   of  
  compliance   with   the   formal   and   substantive  
SECTION  133.  EXAMINATION  AND  PUBLICATION.  –     (WON   the   mark   is   registrable)   requirements  
of  registration.    
133.1.  Once  the  application  meets  the  filing  requirements  
 
of   Section   127,   the   Office   shall   examine   whether   the  
application   meets   the   requirements   of   Section   124   and  

78 z
Intellectual Property Law

4. Publication   Evidence   received   by   the   Philippine   Patent   Office  


  showed   that   Bata   shoes   made   by   Gerbec   and   Hrdina   of  
Czechoslovakia   were   sold   in   the   Philippines   prior   to  
SECTION   133.2.   Where   the   Office   finds   that   the  
World  War  II.  Some  shoes  made  by  Bata  of  Canada  were  
conditions  referred  to  in  Subsection  133.1  are  fulfilled,  it  
perhaps  also  sold  in  the  Philippines  until  1948.  However,  
shall   upon   payment   of   the   prescribed   fee,   forthwith  
the   trademark   BATA   was   never   registered   in   the  
cause   the   application,   as   filed,   to   be   published   in   the  
Philippines   by   any   foreign   entity.   Under   the  
prescribed  manner.   circumstances,  it  was  concluded  that  opposer  has,  to  all  
 
intents   and   purposes,   technically   abandoned   its  
5. Opposition   trademark  BATA  in  the  Philippines.  
   
SECTION  134.  OPPOSITION.  –     Upon  the  other  hand,  the  Philippine  Patent  Office  found  
Any  person  who  believes  that  he  would  be  damaged  by   that   New   Olympian   Rubber   Products   Co.,   Inc.   has  
the   registration   of   a   mark   may,   upon   payment   of   the   overwhelmingly  and  convincingly  established  its  right  to  
required   fee   and   within   thirty   (30)   days   after   the   the   trademark   BATA   and   consequently,   its   use   and  
publication   referred   to   in   Subsection   133.2,   file   with   the   registration  in  its  favor.  There  is  no  gainsaying  the  truth  
Office  an  opposition  to  the  application.  Such  opposition   that   the   respondent   has   spent   a   considerable   amount   of  
shall  be  in  writing  and  verified  by  the  oppositor  or  by  any   money   and   effort   in   popularizing   the   trademark   BATA  
person   on   his   behalf   who   knows   the   facts,   and   shall   for   shoes   in   the   Philippines   through   the   advertising  
specify   the   grounds   on   which   it   is   based   and   include   a   media   since   it   was   lawfully   used   in   commerce   on   July   1,  
statement  of  the  facts  relied  upon.  Copies  of  certificates   1970.   It   can   not   be   denied,   therefore,   that   it   is   the  
of  registration  of  marks  registered  in  other  countries  or   respondent-­‐applicant's   expense   that   created   the  
other   supporting   documents   mentioned   in   the   enormous   goodwill   of   the   trademark   BATA   in   the  
opposition   shall   be   filed   therewith,   together   with   the   Philippines   and   not   the   opposer   as   claimed   in   its  
translation  in  English,  if  not  in  the  English  language.  For   opposition   to   the   registration   of   the   BATA   mark   by   the  
good   cause   shown   and   upon   payment   of   the   required   respondent.  
surcharge,   the   time   for   filing   an   opposition   may   be    
extended   by   the   Director   of   Legal   Affairs,   who   shall   PPO   dismissed   the   opposition   and   ordered   the  
notify   the   applicant   of   such   extension.   The   Regulations   registration   of   Bata   in   favor   of   New   Olympian   Rubber  
shall   fix   the   maximum   period   of   time   within   which   to   file   Products  Co.  
the  opposition.    
  Doctrine:  
SECTION  135.  NOTICE  AND  HEARING.  –     The  Court  was  satisfied  from  the  evidence  that  any  slight  
goodwill   generated   by   the   Czechoslovakian   product  
Upon   the   filing   of   an   opposition,   the   Office   shall   serve  
during   the   Commonwealth   years   was   completely  
notice   of   the   filing   on   the   applicant,   and   of   the   date   of  
abandoned  and  lost  in  the  more  than  35  years  that  have  
the   hearing   thereof   upon   the   applicant   and   the  
passed  since  the  liberation  of  Manila  from  the  Japanese  
oppositor  and  all  other  persons  having  any  right,  title  or  
troops.  
interest   in   the   mark   covered   by   the   application,   as  
 
appear  of  record  in  the  Office.  
New   Olympian   has   reproduced   excerpts   from   the  
 
testimonies   of   the   opposer-­‐appellant's   witnesses   to  
BATA   INDUSTRIES   V.   CA   AND   NEW   OLYMPIAN   RUBBER   prove  that  the  opposer-­‐appellant  was  never  a  user  of  the  
PRODUCTS  CO.  (1982)   trademark  BATA  either  before  or  after  the  war,  that  the  
Refresher:   appellant   is   not   the   successor-­‐in-­‐interest   of   Gerbec   and  
New   Olympian   Rubber   Products   Co.   wanted   to   register   Hrdina   who   were   not   is   representatives   or   agents,   and  
BATA   for   casual   rubber   shoes   that   it   has   allegedly   used   could   not   have   passed   any   rights   to   the   appellant,   that  
since   1   July   1970.   Bata   Industries,   Ltd.,   a   Canadian   there   was   no   privity   of   interest   between   the  
corporation,   opposed   the   same   on   the   ground   that   it   Czechoslovakian   owner   and   the   Canadian   appellant   and  
owns,  and  has  not  abandoned,  the  trademark  BATA.   that   the   Czechoslovakian   trademark   has   been  
  abandoned  in  Czechoslovakia.  
The  parties  stipulated  to  the  following:    
1.   Bata   Industries,   Ltd.   has   no   license   to   do   business   in   MIRPURI  V.  CA  (1999),  SUPRA  
the  Philippines;   Refresher:  
2.   It   is   not   presently   selling   footwear   under   the   Trademark  involved  in  this  case:  "Barbizon"  
trademark  BATA  in  the  Philippines;  and    
3.  It  has  no  licensing  agreement  with  any  local  entity  or   Relevant   issue:   Res   judicata   in   2   oppositions   filed   by   the  
firm  to  sell  its  products  in  the  Philippines.   Private  respondent.  
  Opposition  No.  1  à  IPC  No.  686:  

o 79
Katrina Michelle Mancao
 
  IPC   No.   2049   raised   the   issue   of   ownership   of   the  
(a)  "confusing  similarity"  of  its  trademark  with   trademark,   the   first   registration   and   use   of   the  
that  of  Escobar's;   trademark  in  the  United  States  and  other  countries,  and  
  the   international   recognition   and   reputation   of   the  
(b)   that   the   registration   of   Escobar's   similar   trademark   established   by   extensive   use   and  
trademark   will   cause   damage   to   private   advertisement   of   private   respondent's   products   for   over  
respondent's  business  reputation  and  goodwill;   forty   years   here   and   abroad.   These   are   different   from  
and   the  issues  of  confusing  similarity  and  damage  in  IPC  No.  
  686.  The  issue  of  prior  use  may  have  been  raised  in  IPC  No.  
(c)   that   Escobar's   use   of   the   trademark   686   but   this   claim   was   limited   to   prior   use   in   the  
amounts   to   an   unlawful   appropriation   of   a   Philippines   only.   Prior   use   in   IPC   No.   2049   stems   from  
mark   previously   used   in   the   Philippines   which   private  respondent's  claim  as  originator  of  the  word  and  
act   is   penalized   under   Section   4   (d)   of   the   symbol  "Barbizon,"  as  the  first  and  registered  user  of  the  
Trademark  Law.   mark  attached  to  its  products  which  have  been  sold  and  
  advertised   worldwide   for   a   considerable   number   of   years  
Opposition  No.  2  à  IPC  No.  2049:   prior   to   petitioner's   first   application   for   registration   of  
  her   trademark   in   the   Philippines.   Indeed,   these   are  
(a)   as   early   as   1933,   it   adopted   the   word   substantial   allegations   that   raised   new   issues   and  
"BARBIZON"  as  trademark  on  its  products  such   necessarily   gave   private   respondent   a   new   cause   of  
as   robes,   pajamas,   lingerie,   nightgowns   and   action.   Res   judicata   does   not   apply   to   rights,   claims   or  
slips;   demands,   although   growing   out   of   the   same   subject  
  matter,   which   constitute   separate   or   distinct   causes   of  
(b)   that   the   trademark   "BARBIZON"   was   action  and  were  not  put  in  issue  in  the  former  action.  
registered   with   the   United   States   Patent   Office    
in   1934   and   1949;   and   that   variations   of   the   Respondent   corporation   also   introduced   in   the   second  
same   trademark,   i.e.,   "BARBIZON"   with   Bee   case   a   fact   that   did   not   exist   at   the   time   the   first   case  
design   and   "BARBIZON"   with   the   was  filed  and  terminated.  The  cancellation  of  petitioner's  
representation   of   a   woman   were   also   certificate  of  registration  for  failure  to  file  the  affidavit  of  
registered   with   the   U.S.   Patent   Office   in   1961   use  arose  only  after  IPC  No.  686.  It  did  not  and  could  not  
and  1976;   have   occurred   in   the   first   case,   and   this   gave   respondent  
  another   cause   to   oppose   the   second   application.   Res  
(c)   that   these   marks   have   been   in   use   in   the   judicata   extends   only   to   facts   and   conditions   as   they  
Philippines   and   in   many   countries   all   over   the   existed   at   the   time   judgment   was   rendered   and   to   the  
world  for  over  forty  years.  "Barbizon"  products   legal   rights   and   relations   of   the   parties   fixed   by   the   facts  
have   been   advertised   in   international   so  determined.  When  new  facts  or  conditions  intervene  
publications   and   the   marks   registered   in   36   before   the   second   suit,   furnishing   a   new   basis   for   the  
countries  worldwide;   claims   and   defenses   of   the   parties,   the   issues   are   no  
  longer   the   same,   and   the   former   judgment   cannot   be  
(d)   Escobar's   registration   of   the   similar   pleaded  as  a  bar  to  the  subsequent  action.  
trademark   "BARBIZON"   in   1974   was   based   on    
fraud;   and   this   fraudulent   registration   was   It   is   also   noted   that   the   oppositions   in   the   first   and  
cancelled   in   1979,   stripping   Escobar   of   second   cases   are   based   on   different   laws.   The  
whatsoever  right  she  had  to  the  said  mark;   opposition   in   IPC   No.   686   was   based   on   specific  
  provisions   of   the   Trademark   Law,   i.e.,   Section   4   (d)   on  
(e)   Private   respondent's   trademark   is   entitled   confusing   similarity   of   trademarks   and   Section   8   on   the  
to   protection   as   a   well-­‐known   mark   under   requisite   damage   to   file   an   opposition   to   a   petition   for  
Article   6bis   of   the   Paris   Convention,   Executive   registration.  The  opposition  in  IPC  No.  2049  invoked  the  
Order  No.  913,  and  the  two  Memoranda  dated   Paris   Convention,   particularly   Article   6bis   thereof,   E.O.  
November  20,  1980  and  October  25,  1983  of  the   No.  913  and  the  two  Memoranda  of  the  Minister  of  Trade  
Minister   of   Trade   and   Industry   to   the   Director   and  Industry.  This  opposition  also  invoked  Article  189  of  
of  Patents;   the   Revised   Penal   Code   which   is   a   statute   totally  
  different   from   the   Trademark   Law.   Causes   of   action  
(f)   Escobar's   trademark   is   identical   to   private   which   are   distinct   and   independent   from   each   other,  
respondent's   and   its   use   on   the   same   class   of   although   arising   out   of   the   same   contract,   transaction,  
goods  as  the  latter's  amounts  to  a  violation  of   or  state  of  facts,  may  be  sued  on  separately,  recovery  on  
the   Trademark   Law   and   Article   189   of   the   one   being   no   bar   to   subsequent   actions   on   others.   The  
Revised  Penal  Code.   mere   fact   that   the   same   relief   is   sought   in   the  
  subsequent   action   will   not   render   the   judgment   in   the  

80 z
Intellectual Property Law

prior  action  operative  as  res  judicata,  such  as  where  the   SECTION   137.   REGISTRATION  OF   MARK  AND   ISSUANCE  OF  A  
two   actions   are   based   on   different   statutes.   Res   judicata  
CERTIFICATE  TO  THE  OWNER  OR  HIS  ASSIGNEE.  –    
therefore   does   not   apply   to   the   instant   case   and  
respondent  Court  of  Appeals  did  not  err  in  so  ruling.   137.1.   The   Office   shall   maintain   a   Register   in   which   shall  
  be   registered   marks,   numbered   in   the   order   of   their  
Intellectual   and   industrial   property   rights   cases   are   not   registration,  and  all  transactions  in  respect  of  each  mark,  
simple   property   cases.   Trademarks   deal   with   the   required  to  be  recorded  by  virtue  of  this  law.  
psychological   function   of   symbols   and   the   effect   of    
these  symbols  on  the  public  at  large.  Trademarks  play  a   137.2.   The   registration   of   a   mark   shall   include   a  
significant   role   in   communication,   commerce   and   trade,   reproduction  of  the  mark  and  shall  mention:  its  number;  
and   serve   valuable   and   interrelated   business   functions,   the  name  and  address  of  the  registered  owner  and,  if  the  
both   nationally   and   internationally.   For   this   reason,   all   registered   owner's   address   is   outside   the   country,   his  
agreements  concerning  industrial  property,  like  those  on   address   for   service   within   the   country;   the   dates   of  
trademarks   and   tradenames,   are   intimately   connected   application   and   registration;   if   priority   is   claimed,   an  
with   economic   development.   Industrial   property   indication   of   this   fact,   and   the   number,   date   and   country  
encourages  investments  in  new  ideas  and  inventions  and   of  the  application,  basis  of  the  priority  claims;  the  list  of  
stimulates  creative  efforts  for  the  satisfaction  of  human   goods   or   services   in   respect   of   which   registration   has  
needs.   They   speed   up   transfer   of   technology   and   been   granted,   with   the   indication   of   the   corresponding  
industrialization,   and   thereby   bring   about   social   and   class   or   classes;   and   such   other   data   as   the   Regulations  
economic   progress.   These   advantages   have   been   may  prescribe  from  time  to  time.  
acknowledged   by   the   Philippine   government   itself.   The    
Intellectual   Property   Code   of   the   Philippines   declares   137.3.   A   certificate   of   registration   of   a   mark   may   be  
that   "an   effective   intellectual   and   industrial   property   issued   to   the   assignee   of   the   applicant:   Provided,   That  
system   is   vital   to   the   development   of   domestic   and   the   assignment   is   recorded   in   the   Office.   In   case   of   a  
creative   activity,   facilitates   transfer   of   technology,   it   change   of   ownership,   the   Office   shall   at   the   written  
attracts  foreign  investments,  and  ensures  market  access   request  signed  by  the  owner,  or  his  representative,  or  by  
for   our   products."   The   Intellectual   Property   Code   took   the  new  owner,  or  his  representative  and  upon  a  proper  
effect   on   January   1,   1998   and   by   its   express   provision,   showing   and   the   payment   of   the   prescribed   fee,   issue   to  
repealed   the   Trademark   Law,   the   Patent   Law,   Articles   such  assignee  a  new  certificate  of  registration  of  the  said  
188   and   189   of   the   Revised   Penal   Code,   the   Decree   on   mark   in   the   name   of   such   assignee,   and   for   the  
Intellectual   Property,   and   the   Decree   on   Compulsory   unexpired  part  of  the  original  period.  
Reprinting  of  Foreign  Textbooks.  The  Code  was  enacted    
to   strengthen   the   intellectual   and   industrial   property   137.4.   The   Office   shall   record   any   change   of   address,   or  
system   in   the   Philippines   as   mandated   by   the   country's   address   for   service,   which   shall   be   notified   to   it   by   the  
accession   to   the   Agreement   Establishing   the   World   registered  owner.  
Trade  Organization  (WTO).    
  137.5.  In  the  absence  of  any  provision  to  the  contrary  in  
The  law  does  not  require  the  oppositor  be  a  TM  owner,   this   Act,   communications   to   be   made   to   the   registered  
so   long   as   he   will   sustain   damage   by   the   owner  by  virtue  of  this  Act  shall  be  sent  to  him  at  his  last  
registration,  he  may  oppose.     recorded   address   and,   at   the   same,   at   his   last   recorded  
address  for  service.  
 
 
6. Issuance  and  Publication  of  
SECTION  138.  CERTIFICATES  OF  REGISTRATION.  –    
Certificate   A  certificate  of  registration  of  a  mark  shall  be  prima  facie  
  evidence   of   the   validity   of   the   registration,   the  
SECTION   136.   ISSUANCE  AND  PUBLICATION  OF   CERTIFICATE.   registrant's   ownership   of   the   mark,   and   of   the  
–     registrant's  exclusive  right  to  use  the  same  in  connection  
with   the   goods   or   services   and   those   that   are   related  
When   the   period   for   filing   the   opposition   has   expired,   or  
thereto  specified  in  the  certificate.    
when  the  Director  of  Legal  Affairs  shall  have  denied  the  
 
opposition,  the  Office  upon  payment  of  the  required  fee,  
shall   issue   the   certificate   of   registration.   Upon   issuance   SECTION   139.   PUBLICATION   OF   REGISTERED   MARKS;  
of   a   certificate   of   registration,   notice   thereof   making   INSPECTION  OF  REGISTER.  –    
reference   to   the   publication   of   the   application   shall   be   139.1.  The  Office  shall  publish,  in  the  form  and  within  the  
published  in  the  IPO  Gazette.     period  fixed  by  the  Regulations,  the  marks  registered,  in  
  the   order   of   their   registration,   reproducing   all   the  
particulars  referred  to  in  Subsection  137.2.  
 

o 81
Katrina Michelle Mancao
 
139.2.   Marks   registered   at   the   Office   may   be   inspected   when   IPO   agrees   to   accept   non-­‐use   are   very   rare   and  
free  of  charge  and  any  person  may  obtain  copies  thereof   exceptional.  It  must  be  factors  beyond  your  control.    
at   his   own   expense.   This   provision   shall   also   be   • Bankrupt?   Financial   circumstance   not   an  
applicable   to   transactions   recorded   in   respect   of   any   acceptable  ground    
registered  mark.   • Independent  of  your  will:    
  o Constitutional   prohibition   for  
SECTION  144.  CLASSIFICATION  OF  GOODS  AND  SERVICES.  –     broadcasting   network.   IPO   accepted  
144.1.   Each   registration,   and   any   publication   of   the   Office   it   so   the   mark   is   still   registered,   not  
struck  down  for  use.    
which  concerns  an  application  or  registration  effected  by  
o Government  restriction/regulation    
the   Office   shall   indicate   the   goods   or   services   by   their  
 
names,   grouped   according   to   the   classes   of   the   Nice  
• Within   1st   year   from   5th   anniversary  
Classification,   and   each   group   shall   be   preceded   by   the  
of   mark:   file   declaration   of   actual  
number   of   the   class   of   that   Classification   to   which   that  
use   to   show   that   you   are   still   using  
group   of   goods   or   services   belongs,   presented   in   the  
the  mark  
order  of  the  classes  of  the  said  Classification.  
• 10  years:  file  petition  for  renewal    
 
 
144.2.  Goods  or  services  may  not  be  considered  as  being  
similar  or  dissimilar  to  each  other  on  the  ground  that,  in  
8. Voluntary   Cancellation   of  
any  registration  or  publication  by  the  Office,  they  appear   Certificate  
in  different  classes  of  the  Nice  Classification.    
  SECTION   140.   CANCELLATION   UPON   APPLICATION   BY  
SECTION  4.2.     REGISTRANT;   AMENDMENT   OR   DISCLAIMER   OF  
The   term   "TECHNOLOGY   TRANSFER   ARRANGEMENTS"   refers   to  
REGISTRATION.  –    
contracts   or   agreements   involving   the   transfer   of  
Upon  application  of  the  registrant,  the  Office  may  permit  
systematic   knowledge   for   the   manufacture   of   a   product,  
any   registration   to   be   surrendered   for   cancellation,   and  
the   application   of   a   process,   or   rendering   of   a   service  
upon   cancellation   the   appropriate   entry   shall   be   made   in  
including   management   contracts;   and   the   transfer,  
the   records   of   the   Office.   Upon   application   of   the  
assignment   or   licensing   of   all   forms   of   intellectual  
registrant  and  payment  of  the  prescribed  fee,  the  Office  
property   rights,   including   licensing   of   computer  
for   good   cause   may   permit   any   registration   to   be  
software   except   computer   software   developed   for   mass  
amended  or  to  be  disclaimed  in  part:  Provided,  That  the  
market.  
amendment   or   disclaimer   does   not   alter   materially   the  
 
character   of   the   mark.   Appropriate   entry   shall   be   made  
7. Duration  of  Certificate   in   the   records   of   the   Office   upon   the   certificate   of  
  registration   or,   if   said   certificate   is   lost   or   destroyed,  
SECTION  145.  DURATION.  –     upon  a  certified  copy  thereto.  
A   certificate   of   registration   shall   remain   in   force   for   ten    
(10)   years:   Provided,   That   the   registrant   shall   file   a   9. Correction  of  Mistakes  
declaration  of  actual  use  and  evidence  to  that  effect,  or    
shall   show   valid   reasons   based   on   the   existence   of   SECTION   142.   CORRECTION   OF   MISTAKES   MADE   BY   THE  
obstacles  to  such  use,  as  prescribed  by  the  Regulations,  
within   one   (1)   year   from   the   fifth   anniversary   of   the   date  
OFFICE.  –  
of  the  registration  of  the  mark.  Otherwise,  the  mark  shall    Whenever   a   material   mistake   in   a   registration   incurred  
be  removed  from  the  Register  by  the  Office.   through  the  fault  of  the  Office  is  clearly  disclosed  by  the  
  records   of   the   Office,   a   certificate   stating   the   fact   and  
Marks   acquired   under   RA   166   have   a   20-­‐year   duration   nature   of   such   mistake   shall   be   issued   without   charge,  
until   expiration.   Once   it   expires,   the   mark   can   only   be   recorded  and  a  printed  copy  thereof  shall  be  attached  to  
renewed  for  a  period  of  10  years.   each   printed   copy   of   the   registration.   Such   corrected  
  registration  shall  thereafter  have  the  same  effect  as  the  
Why   require   use   from   a   conceptual   basis?   Warehousing   of   original  certificate;  or  in  the  discretion  of  the  Director  of  
trademarks   is   not   allowed.   What’s   the   point   of   the   Administrative,   Financial   and   Human   Resource  
protecting   trademarks   if   not   use   for   contributing   the   Development   Service   Bureau   a   new   certificate   of  
GDP,  pay  taxes,  hire  people,  no  economic  activity  at  all.     registration   may   be   issued   without   charge.   All  
• There  is  single  6mon  extension  to  make  use  of   certificates   of   correction   heretofore   issued   in  
the  mark.     accordance  with  the  Regulations  and  the  registration  to  
  which   they   are   attached   shall   have   the   same   force   and  
Can   you   file   declaration   of   non-­‐use?   Yes,   but   the   instances   effect  as  if  such  certificates  and  their  issuance  had  been  
authorized  by  this  Act.  

82 z
Intellectual Property Law

  146.4.   An   applicant   for   renewal   not   domiciled   in   the  


SECTION   143.   CORRECTION   OF   MISTAKES   MADE   BY   Philippines   shall   be   subject   to   and   comply   with   the  
APPLICANT.  –   requirements  of  this  Act.    
 
Whenever   a   mistake   is   made   in   a   registration   and   such  
mistake   occurred   in   good   faith   through   the   fault   of   the   Can  a  trademark  registration  be  in  perpetuity  forever  and  
applicant,   the   Office   may   issue   a   certificate   upon   the   ever?   Copyright   can   be   lifetime.   Yes.   For   as  
payment   of   the   prescribed   fee:   Provided,   That   the   long   as   you   keep   renewing   the   mark.   A  
correction   does   not   involve   any   change   in   the   trademark   can   be   forever   for   as   long  as   6   mos  
registration  that  requires  republication  of  the  mark.     prior   to   10th   year   anniversary   of   registration,  
  you   file   petition   for   renewal   which   is   granted  
as   a   matter   of   course.   The   cycle   goes   on   and  
10. Renewal  
on.      
   
SECTION  146.  RENEWAL.  –     H. Rights  conferred  
146.1.   A   certificate   of   registration   may   be   renewed   for    
periods  of  ten  (10)  years  at  its  expiration  upon  payment   SECTION  147.  RIGHTS  CONFERRED.  –    
of   the   prescribed   fee   and   upon   filing   of   a   request.   The  
request  shall  contain  the  following  indications:   147.1.*   Except   in   cases   of   importation   of   drugs   and  
  medicines   allowed   under   Section   72.1   of   this   Act   and   of  
(a)  An  indication  that  renewal  is  sought;   off-­‐patent   drugs   and   medicines,   the   owner   of   a  
  registered  mark  shall  have  the  exclusive  right  to  prevent  
(b)   The   name   and   address   of   the   registrant   or   all   third   parties   not   having   the   owner's   consent   from  
his   successor-­‐in-­‐interest,   hereafter   referred   to   using   in   the   course   of   trade   identical   or   similar   signs   or  
as  the  "right  holder";   containers   for   goods   or   services   which   are   identical   or  
  similar   to   those   in   respect   of   which   the   trademark   is  
(c)  The  registration  number  of  the  registration   registered   where   such   use   would   result   in   a   likelihood   of  
concerned;   confusion.   In   case   of   the   use   of   an   identical   sign   for  
  identical  goods  or  services,  a  likelihood  of  confusion  shall  
(d)   The   filing   date   of   the   application   which   be  presumed.  
resulted   in   the   registration   concerned   to   be    
renewed;   There   shall   be   no   infringement   of   trademarks   or  
  tradenames   of   imported   or   sold   patented   drugs   and  
(e)   Where   the   right   holder   has   a   representative,   medicines  allowed  under  Section  72.1  of  this  Act,  as  well  
the  name  and  address  of  that  representative;   as   imported   or   sold   off-­‐patent   drugs   and   medicines:  
  Provided,   That,   said   drugs   and   medicines   bear   the  
(f)   The   names   of   the   recorded   goods   or   registered   marks   that   have   not   been   tampered,  
services   for   which   the   renewal   is   requested   or   unlawfully   modified,   or   infringed   upon,   under   Section  
the   names   of   the   recorded   goods   or   services   155  of  this  Code.  
for   which   the   renewal   is   not   requested,    
grouped   according   to   the   classes   of   the   Nice   147.2.   The   exclusive   right   of   the   owner   of   a   well-­‐known  
Classification   to   which   that   group   of   goods   or   mark  defined  in  Subsection  123.1(e)**  which  is  registered  
services  belongs  and  presented  in  the  order  of   in   the   Philippines,   shall   extend   to   goods   and   services  
the  classes  of  the  said  Classification;  and   which   are   not   similar   to   those   in   respect   of   which   the  
  mark   is   registered:   Provided,   That   use   of   that   mark   in  
(g)   A   signature   by   the   right   holder   or   his                                                                                                                                                          
representative.   *
 As  amended  by  RA  9502  –  Universally  Accessible  Cheaper  and  
  Quality  Medicines  Act  of  2008  
146.2.  Such  request  shall  be  in  Filipino  or  English  and  may   **
  Section   123.1.   A   mark   cannot   be   registered   if:   (e)   Is   identical  
be   made   at   any   time   within   six   (6)   months   before   the   with,   or   confusingly   similar   to,   or   constitutes   a   translation   of   a  
expiration   of   the   period   for   which   the   registration   was   mark   which   is   considered   by   the   competent   authority   of   the  
issued   or   renewed,   or   it   may   be   made   within   six   (6)   Philippines   to   be   well-­‐known   internationally   and   in   the  
months   after   such   expiration   on   payment   of   the   Philippines,  whether  or  not  it  is  registered  here,  as  being  already  
the   mark   of   a   person   other   than   the   applicant   for   registration,  
additional  fee  herein  prescribed.  
and   used   for   identical   or   similar   goods   or   services:   Provided,  
  That   in   determining   whether   a   mark   is   well-­‐known,   account  
146.3.   If   the   Office   refuses   to   renew   the   registration,   it   shall   be   taken   of   the   knowledge   of   the   relevant   sector   of   the  
shall   notify   the   registrant   of   his   refusal   and   the   reasons   public,  rather  than  of  the  public  at  large,  including  knowledge  in  
therefor.   the   Philippines   which   has   been   obtained   as   a   result   of   the  
  promotion  of  the  mark  

o 83
Katrina Michelle Mancao
 
relation   to   those   goods   or   services   would   indicate   a   87.2.   Those   pursuant   to   which   the   licensor   reserves   the  
connection   between   those   goods   or   services   and   the   right   to   fix   the   sale   or   resale   prices   of   the   products  
owner  of  the  registered  mark:  Provided  further,  That  the   manufactured  on  the  basis  of  the  license;  
interests   of   the   owner   of   the   registered   mark   are   likely    
to  be  damaged  by  such  use.   87.3.   Those   that   contain   restrictions   regarding   the  
  volume  and  structure  of  production;  
SECTION   148.   USE   OF   INDICATIONS   BY   THIRD   PARTIES   FOR    
PURPOSES   OTHER   THAN   THOSE   FOR   WHICH   THE   MARK   IS   87.4.   Those   that   prohibit   the   use   of   competitive  
USED.  –   technologies   in   a   non-­‐exclusive   technology   transfer  
Registration   of   the   mark   shall   not   confer   on   the   agreement;  
registered  owner  the  right  to  preclude  third  parties  from    
using   bona   fide   their   names,   addresses,   pseudonyms,   a   87.5.   Those   that   establish   a   full   or   partial   purchase  
geographical   name,   or   exact   indications   concerning   the   option  in  favor  of  the  licensor;  
kind,   quality,   quantity,   destination,   value,   place   of   origin,    
or   time   of   production   or   of   supply,   of   their   goods   or   87.6.  Those  that  obligate  the  licensee  to  transfer  for  free  
services:   Provided,   That   such   use   is   confined   to   the   to   the   licensor   the   inventions   or   improvements   that   may  
purposes   of   mere   identification   or   information   and   be  obtained  through  the  use  of  the  licensed  technology;  
cannot  mislead  the  public  as  to  the  source  of  the  goods    
or  services.   87.7.   Those   that   require   payment   of   royalties   to   the  
  owners  of  patents  for  patents  which  are  not  used;  
SECTION  4.2.**      
The   term   "TECHNOLOGY   TRANSFER   ARRANGEMENTS"   refers   to   87.8.   Those   that   prohibit   the   licensee   to   export   the  
contracts   or   agreements   involving   the   transfer   of   licensed  product  unless  justified  for  the  protection  of  the  
systematic   knowledge   for   the   manufacture   of   a   product,   legitimate   interest   of   the   licensor   such   as   exports   to  
the   application   of   a   process,   or   rendering   of   a   service   countries   where   exclusive   licenses   to   manufacture  
including   management   contracts;   and   the   transfer,   and/or   distribute   the   licensed   product(s)   have   already  
assignment   or   licensing   of   all   forms   of   intellectual   been  granted;  
property   rights,   including   licensing   of   computer    
software   except   computer   software   developed   for   mass   87.9.   Those   which   restrict   the   use   of   the   technology  
market.   supplied   after   the   expiration   of   the   technology   transfer  
  arrangement,  except  in  cases  of  early  termination  of  the  
SECTION  87.  PROHIBITED  CLAUSES.  –     technology   transfer   arrangement   due   to   reason(s)  
Except   in   cases   under   Section   91 7 ,   the   following   attributable  to  the  licensee;  
provisions   shall   be   deemed   prima   facie   to   have   an    
adverse  effect  on  competition  and  trade:   87.10.   Those  which  require  payments  for  patents  and  
  other   industrial   property   rights   after   their   expiration,  
87.1.   Those   which   impose   upon   the   licensee   the   termination  arrangement;  
obligation  to  acquire  from  a  specific  source  capital  goods,    
intermediate   products,   raw   materials,   and   other   87.11.   Those   which   require   that   the   technology  
technologies,   or   of   permanently   employing   personnel   recipient   shall   not   contest   the   validity   of   any   of   the  
indicated  by  the  licensor;   patents  of  the  technology  supplier;  
   
87.12.   Those   which   restrict   the   research   and  
development   activities   of   the   licensee   designed   to  
absorb   and   adapt   the   transferred   technology   to   local  
                                                                                                                                                        conditions   or   to   initiate   research   and   development  
**
 Relate  to  Sections  87,  88,  92   programs  in  connection  with  new  products,  processes  or  
7
 Section   91.   Exceptional   Cases.   -­‐   In   exceptional   or   meritorious   equipment;  
cases   where   substantial   benefits   will   accrue   to   the   economy,    
such   as   high   technology   content,   increase   in   foreign   exchange   87.13.   Those   which   prevent   the   licensee   from  
earnings,   employment   generation,   regional   dispersal   of   adapting   the   imported   technology   to   local   conditions,   or  
industries   and/or  substitution  with  or  use   of   local   raw   materials,   introducing   innovation   to   it,   as   long   as   it   does   not   impair  
or   in   the   case   of   Board   of   Investments,   registered   companies  
the  quality  standards  prescribed  by  the  licensor;  
with   pioneer   status,   exemption   from   any   of   the   above  
requirements   may   be   allowed   by   the   Documentation,    
Information   and   Technology   Transfer   Bureau   after   evaluation   87.14.   Those  which  exempt  the  licensor  for  liability  for  
thereof  on  a  case  by  case  basis.   non-­‐fulfillment   of   his   responsibilities   under   the  

84 z
Intellectual Property Law

technology   transfer   arrangement   and/or   liability   arising   149.2.   Such  assignment  or  transfer  shall,  however,  be  
from   third   party   suits   brought   about   by   the   use   of   the   null   and   void   if   it   is   liable   to   mislead   the   public,  
licensed  product  or  the  licensed  technology;  and   particularly  as  regards  the  nature,  source,  manufacturing  
  process,   characteristics,   or   suitability   for   their   purpose,  
87.15.   Other  clauses  with  equivalent  effects.   of  the  goods  or  services  to  which  the  mark  is  applied.  
   
SECTION  88.  MANDATORY  PROVISIONS.  –     149.3.   The   assignment   of   the   application   for  
The   following   provisions   shall   be   included   in   voluntary   registration   of   a   mark,   or   of   its   registration,   shall   be   in  
license  contracts:   writing   and   require   the   signatures   of   the   contracting  
  parties.   Transfers   by   mergers   or   other   forms   of  
88.1.   That   the   laws   of   the   Philippines   shall   govern   the   succession   may   be   made   by   any   document   supporting  
interpretation  of  the  same  and  in  the  event  of  litigation,   such  transfer.  
the   venue   shall   be   the   proper   court   in   the   place   where    
the  licensee  has  its  principal  office;   149.4.   Assignments   and   transfers   of   registrations   of  
  marks  shall  be  recorded  at  the  Office  on  payment  of  the  
88.2.   Continued   access   to   improvements   in   techniques   prescribed  fee;  assignment  and  transfers  of  applications  
and   processes   related   to   the   technology   shall   be   made   for   registration   shall,   on   payment   of   the   same   fee,   be  
available   during   the   period   of   the   technology   transfer   provisionally   recorded,   and   the   mark,   when   registered,  
arrangement;   shall  be  in  the  name  of  the  assignee  or  transferee.  
   
88.3.   In   the   event   the   technology   transfer   arrangement   149.5.   Assignments  and  transfers  shall  have  no  effect  
shall  provide  for  arbitration,  the  Procedure  of  Arbitration   against  third  parties  until  they  are  recorded  at  the  Office.  
of   the   Arbitration   Law   of   the   Philippines   or   the    
Arbitration   Rules   of   the   United   Nations   Commission   on   SECTION  150.  LICENSE  CONTRACTS.  –    
International   Trade   Law   (UNCITRAL)   or   the   Rules   of   150.1.  Any  license  contract  concerning  the  registration  of  
Conciliation   and   Arbitration   of   the   International   a   mark,   or   an   application   therefor,   shall   provide   for  
Chamber  of  Commerce  (ICC)  shall  apply  and  the  venue  of   effective   control   by   the   licensor   of   the   quality   of   the  
arbitration   shall   be   the   Philippines   or   any   neutral   goods   or   services   of   the   licensee   in   connection   with  
country;  and   which  the  mark  is  used.  If  the  license  contract  does  not  
  provide  for  such  quality  control,  or  if  such  quality  control  
88.4.  The  Philippine  taxes  on  all  payments  relating  to  the   is   not   effectively   carried   out,   the   license   contract   shall  
technology   transfer   arrangement   shall   be   borne   by   the   not  be  valid.  
licensor.    
  150.2.   A   license   contract   shall   be   submitted   to   the  
SECTION   92.   NON-­‐REGISTRATION   WITH   THE   Office  which  shall  keep  its  contents  confidential  but  shall  
DOCUMENTATION,   INFORMATION   AND   TECHNOLOGY   record   it   and   publish   a   reference   thereto.   A   license  
contract   shall   have   no   effect   against   third   parties   until  
TRANSFER  BUREAU .  –  
such   recording   is   effected.   The   Regulations   shall   fix   the  
 Technology   transfer   arrangements   that   conform   with   procedure  for  the  recording  of  the  license  contract.  
the   provisions   of   Sections   86   and   87   need   not   be    
registered   with   the   Documentation,   Information   and  
SECTION  231.  REVERSE  RECIPROCITY  OF  FOREIGN  LAWS.  –    
Technology  Transfer  Bureau.  Non-­‐conformance  with  any  
Any   condition,   restriction,   limitation,   diminution,  
of   the   provisions   of   Sections   87   and   88,   however,   shall  
requirement,   penalty   or   any   similar   burden   imposed   by  
automatically   render   the   technology   transfer  
the   law   of   a   foreign   country   on   a   Philippine   national  
arrangement   unenforceable,   unless   said   technology  
seeking  protection  of  intellectual  property  rights  in  that  
transfer   arrangement   is   approved   and   registered   with  
country,  shall  reciprocally  be  enforceable  upon  nationals  
the   Documentation,   Information   and   Technology  
of  said  country,  within  Philippine  jurisdiction.  
Transfer   Bureau   under   the   provisions   of   Section   91   on  
 
exceptional  cases.  
 
SECTION   149.   ASSIGNMENT   AND   TRANSFER   OF  
APPLICATION  AND  REGISTRATION.  –    
149.1.   An   application   for   registration   of   a   mark,   or   its  
registration,   may   be   assigned   or   transferred   with   or  
without  the  transfer  of  the  business  using  the  mark.    
 

o 85
Katrina Michelle Mancao
 
Is   there   infringement   even   if   the   goods   are   non-­‐ and   advertisements   of   the   originator   of   said   mark,   to  
competing?   convey   to   the   public   a   false   impression   of   some  
supposed  connection  between  the  manufacturer  of  the  
 
article  sold  under  the  original  mark  and  the  new  articles  
ANG  V.  TEODORO  (1942),  SUPRA  
being  tendered  to  the  public  under  the  same  or  similar  
Refresher:   “Ang   Tibay”   used   for   slippers;   tried   to   be   mark.   As   trade   has   developed   and   commercial   changes  
registered  for  pants  and  shirts.   have   come   about,   the   law   of   unfair   competition   has  
  expanded  to  keep  pace  with  the  times  and  the  element  
Doctrine:   of   strict   competition   in   itself   has   ceased   to   be   the  
This   fundamental   change   in   attitude   first   manifested   determining  factor.  The  owner  of  a  trade-­‐mark  or  trade-­‐
itself   in   the   year   1915-­‐1917.   Until   about   then,   the   courts   name   has   a   property   right   in   which   he   is   entitled   to  
had  proceeded  on  the  theory  that  the  same  trade-­‐mark,   protection,  since  there  is  damage  to  him  from  confusion  
used   on   un-­‐like   goods,   could   not   cause   confusion   in   of   reputation   or   goodwill   in   the   mind   of   the   public   as  
trade   and   that,   therefore,   there   could   be   no   objection   to   well  as  from  confusion  of  goods.  The  modern  trend  is  to  
the  use  and  registration  of  a  well-­‐known  mark  by  a  third   give   emphasis   to   the   unfairness   of   the   acts   and   to  
party  for  a  different  class  of  goods.  Since  1916  however,   classify  and  treat  the  issue  as  a  fraud.  
a  growing  sentiment  began  to  arise  that  in  the  selection    
of   a   famous   mark   by   a   third   party,   there   was   generally  
CHUA  CHE  V.  PHILIPPINE  PATENT  OFFICE    (1965)  
the   hidden   intention   to   "have   a   free   ride"   on   the   trade-­‐
Refresher:   “X-­‐7”   previously   used   as   a   brand   of   toilet  
mark  owner's  reputation  and  good  will.  
articles  was  being  registered  for  soap.  
 
 
In  the  present  state  of  development  of  the  law  on  Trade-­‐
Doctrine:  
Marks,  Unfair  Competition,  and  Unfair  Trading,  the  TEST  
Registration   of   a   trademark   should   be   refused   in   cases  
employed   by   the   courts   to   determine   whether  
where   there   is   a   likelihood   of   confusion,   mistake,   or  
noncompeting  goods  are  or  are  not  of  the  same  class  is  
deception,   even   though   the   goods   fall   into   different  
confusion  as  to  the  origin  of  the  goods  of  the  second  user.  
categories.   The   products   of   appellee   are   common  
Although   two   noncompeting   articles   may   be   classified  
household   items   nowadays,   in   the   same   manner   as  
under  two  different  classes  by  the  Patent  Office  because  
laundry   soap.   The   likelihood   of   purchasers   to   associate  
they   are   deemed   not   to   possess   the   same   descriptive  
those   products   to   a   common   origin   is   not   far-­‐fetched.  
properties,   they   would,   nevertheless,   be   held   by   the  
Both   from   the   standpoint   of   priority   of   use   and   for   the  
courts   to   belong   to   the   same   class   if   the   simultaneous  
protection  of  the  buying  public  and,  of  course,  appellee's  
use   on   them   of   identical   or   closely   similar   trade-­‐marks  
rights   to   the   trademark   "X-­‐7",   it   becomes   manifest   that  
would   be   likely   to   cause   confusion   as   to   the   origin,   or  
the   registration   of   said   trademark   in   favor   of   applicant-­‐
personal  source,  of  the  second  user's  goods.  They  would  
appellant  should  be  denied.  
be  considered  as  not  falling  under  the  same  class  only  if  
 
they   are   so   dissimilar   or   so   foreign   to   each   other   as   to  
make  it  unlikely  that  the  purchaser  would  think  the  first   STA.  ANA  V.  MALIWAT  (1968)  
user  made  the  second  user's  goods.   Refresher:   Maliwat   wanted   to   register   the   trademark  
  FLORMANN,   which   is   used   on   shirts,   pants,   jackets   and  
Such   construction   of   the   law   is   induced   by   cogent   shoes   for   ladies,   men,   and   children.   Sta.   Ana   filed   an  
reasons   of   equity   and   fair   dealing.   The   courts   have   come   application   for   the   registration   of   the   tradename  
to  realize  that  there  can  be  unfair  competition  or  unfair   FLORMEN   SHOE   MANUFACTURERS,   which   he'll   use   for  
trading   even   if   the   goods   are   non-­‐competing,   and   that   manufacturing  ladies'  and  children's  shoes.  
such   unfair   trading   can   cause   INJURY   OR   DAMAGE   TO    
THE   FIRST   USER   of   a   given   trade-­‐mark,   first,   by   Doctrine:  
prevention   of   the   natural   expansion   of   his   business   and,   Modern  law  recognizes  that  the  protection  to  which  the  
second,  by  having  his  business  reputation  confused  with   owner   of   a   trademark   mark   is   entitled   is   not   limited   to  
and   put   at   the   mercy   of   the   second   user.   Then   guarding   his   goods   or   business   from   actual   market  
noncompetitive  products  are  sold  under  the  same  mark,   competition   with   identical   or   similar   products   of   the  
the   gradual   whittling   away   or   dispersion   of   the   identity   parties,   but   extends   to   all   cases   in   which   the   use   by   a  
and  hold  upon  the  public  mind  of  the  mark  created  by  its   junior   appropriator   of   a  trademark  or  tradename  is  likely  
first  user,  inevitably  results.  The  original  owner  is  entitled   to   lead   to   a   confusion   of   source,   as   where   prospective  
to  the  preservation  of  the  valuable  link  between  him  and   purchasers   would   be   misled   into   thinking   that   the  
the  public  that  has  been  created  by  his  ingenuity  and  the   complaining   party   has   extended   his   business   into   the  
merit   of   his   wares   or   services.   Experience   has   field   or   is   in   any   way   connected   with   the   activities   of   the  
demonstrated   that   when   a   well-­‐known   trade-­‐mark   is   infringer;   or   when   it   forestalls   the   normal   potential  
adopted   by   another   even   for   a   totally   different   class   of   expansion  of  his  business.  
goods,   it   is   done   to   get   the   benefit   of   the   reputation    

86 z
Intellectual Property Law

Mere   dissimilarity   of   goods   should   not   preclude   relief   be  injured  by  any  subsequent  appropriation  or  imitation  
where   the   junior   user's   goods   are   not   too   different   or   by  others,  and  the  public  will  not  be  deceived.  
remote  from  any  that  the  owner  would  be  likely  to  make    
or  sell;  and  in  the  present  case,  wearing  apparel  is  not  so   ESSO  STANDARD  EASTERN,  INC.  V.  CA  (1982)  
far  removed  from  shoes  as  to  preclude  relief.   Refresher:   “Esso”   is   used   by   Esso   Standard   for   its  
  petroleum   products.   Private   respondent   (United  
PHILIPPINE  REFINING  COMPANY  V.  NG  SAM  (1982)   Cigarette  Corp.)  wanted  to  use  it  for  its  cigarettes.  
Refresher:   “Camia”   was   used   by   Philippine   Refining    
Sugar  for  its  lard,  butter,  cooking  oil,  and  soap.  Ng  Sam   Doctrine:  
wanted  to  register  it  for  its  ham.   The  law  defines  infringement  as  the  use  without  consent  
  of   the   trademark   owner   of   any   "reproduction,  
Doctrine:   counterfeit,   copy   or   colorable   limitation   of   any  
A   rudimentary   precept   in   trademark   protection   is   that   registered   mark   or   tradename   in   connection   with   the  
"the   right   to   a   trademark   is   a   limited   one,   in   the   sense   sale,   offering   for   sale,   or   advertising   of   any   goods,  
that   others   may   use   the   same   mark   on   unrelated   goods."   business   or   services   on   or   in   connection   with   which   such  
Thus,   as   pronounced   by   the   United   States   Supreme   use  is  likely  to  cause  confusion  or  mistake  or  to  deceive  
Court   in   the   case   of   American   Foundries   vs.   Robertson,   purchasers   or   others   as   to   the   source   or   origin   of   such  
"the  mere  fact  that  one  person  has  adopted  and  used  a   goods   or   services,   or   Identity   of   such   business;   or  
trademark   on   his   goods   does   not   prevent   the   adoption   reproduce,   counterfeit,   copy   or   colorably   imitate   any  
and   use   of   the   same   trademark   by   others   on   articles   of   a   such   mark   or   tradename   and   apply   such   reproduction,  
different  description."   counterfeit,  copy  or  colorable  limitation  to  labels,  signs,  
  prints,   packages,   wrappers,   receptacles   or  
Such   restricted   right   over   a   trademark   is   likewise   advertisements   intended   to   be   used   upon   or   in  
reflected   in   our   Trademark   law.   Under   Section   4(d)   of   connection   with   such   goods,   business   or   services."  
the  law,  registration  of  a  trademark  which  so  resembles   Implicit   in   this   definition   is   the   concept   that   the   goods  
another   already   registered   or   in   use   should   be   denied,   must   be   so   related   that   there   is   a   likelihood   either   of  
where   to   allow   such   registration   could   likely   result   in   confusion   of   goods   or   business.   But   likelihood   of  
confusion,   mistake   or   deception   to   the   consumers.   confusion   is   a   relative   concept;   to   be   determined   only  
Conversely,   where   no   confusion   is   likely   to   arise,   as   in   according   to   the   particular,   and   sometimes   peculiar,  
this  case,  registration  of  a  similar  or  even  Identical  mark   circumstances   of   each   case.   It   is   unquestionably   true   that,  
may  be  allowed.   as   stated   in   Coburn   vs.   Puritan   Mills,   Inc.   "In   trademark  
  cases,   even   more   than   in   other   litigation,   precedent  
The   term   "CAMIA"   is   descriptive   of   a   whole   genus   of   must  be  studied  in  the  light  of  the  facts  of  the  particular  
garden  plants  with  fragrant  white  flowers.  Some  people   case.  
call  the  "CAMIA"  the  "white  ginger  plant"  because  of  its    
tuberous  roots,  while  children  refer  to  it  as  the  butterfly   It  is  undisputed  that  the  goods  on  which  petitioner  uses  
flower  because  of  its  shape.  Being  a  generic  and  common   the   trademark   ESSO,   petroleum   products,   and   the  
term,  its  appropriation  as  a  trademark,  albeit  in  a  fanciful   product   of   respondent,   cigarettes,   are   non-­‐competing.  
manner   in   that   it   bears   no   relation   to   the   product   it   But   as   to   whether   trademark   infringement   exists  
Identifies,   is   valid.   However,   the   degree   of   exclusiveness   depends   for   the   most   part   upon   whether   or   not   the  
accorded  to  each  user  is  closely  restricted.   goods  are  so  related  that  the  public  may  be,  or  is  actually,  
  deceived  and  misled  that  they  came  from  the  same  maker  
A   trademark   is   designed   to   Identify   the   user.   But   it   or   manufacturer.   For   non-­‐competing   goods   may   be  
should   be   so   distinctive   and   sufficiently   original   as   to   those  which,  though  they  are  not  in  actual  competition,  
enable  those  who  come  into  contact  with  it  to  recognize   are  so  related  to  each  other  that  it  might  reasonably  be  
instantly  the  Identity  of  the  user.  "  It  must  be  affirmative   assumed   that   they   originate   from   one   manufacturer.  
and   definite,   significant   and   distinctive,   capable   to   Non-­‐competing   goods   may   also   be   those   which,   being  
indicate  origin."   entirely   unrelated,   could   not   reasonably   be   assumed   to  
  have   a   common   source.   In   the   former   case   of   related  
It   is   evident   that   "CAMIA"   as   a   trademark   is   far   from   goods,   confusion   of   business   could   arise   out   of   the   use  
being  distinctive.  By  itself,  it  does  not  Identify  petitioner   of  similar  marks;  in  the  latter  case  of  non-­‐related  goods,  
as   the   manufacturer   or   producer   of   the   goods   upon   it   could   not.   The   vast   majority   of   courts   today   follow   the  
which   said   mark   is   used,   as   contra-­‐distinguished   to   modern  theory  or  concept  of  "related  goods"  which  the  
trademarks  derived  from  coined  words  such  as  "Rolex",   Court   has   likewise   adopted   and   uniformly   recognized  
"Kodak"   or   "Kotex".   It   has   been   held   that   if  a  mark  is  so   and  applied.  
commonplace   that   it   cannot   be   readily   distinguished    
from   others,   then   it   is   apparent   that   it   cannot   Identify   a   Goods  are  related  when  they  belong  to  the  same  class  or  
particular  business;  and  he  who  first  adopted  it  cannot   have   the   same   descriptive   properties;   when   they   possess  

o 87
Katrina Michelle Mancao
 
the   same   physical   attributes   or   essential   characteristics   deodorant,   talcum   powder,   and   toilet   soap.   Thereafter,  
with   reference   to   their   form,   composition,   texture   or   respondent   court,   through   Justice   Gopengco   with  
quality.  They  may  also  be  related  because  they  serve  the   Justices   Patajo   and   Racela,   Jr.   concurring,   was   initially  
same  purpose  or  are  sold  in  grocery  stores.   persuaded   by   petitioner's   plea   for   reversal   directed  
  against   the   permission   granted   by   the   Director   of  
In   the   situation   before   us,   the   goods   are   obviously   Patents,   but   the   decision   of   the   Second   Special   Cases  
different   from   each   other   with   "absolutely   no   iota   of   Division   handed   down   on   April   29,   1983   was   later  
similitude"   as   stressed   in   respondent   court's   judgment.   reconsidered  in  favor  of  herein  private  respondent.  
They  are  so  foreign  to  each  other  as  to  make  it  unlikely    
that   purchasers   would   think   that   petitioner   is   the   Doctrine:  
manufacturer   of   respondent's   goods.   The   mere   fact   that   Having   thus   reviewed   the   laws   applicable   to   the   case  
one   person   has   adopted   and   used   a   trademark   on   his   before  Us,  it  is  not  difficult  to  discern  from  the  foregoing  
goods   does   not   prevent   the   adoption   and   use   of   the   statutory   enactments   that   private   respondent   may   be  
same   trademark   by   others   on   unrelated   articles   of   a   permitted   to   register   the   trademark   "BRUTE"   for   briefs  
different  kind.   produced   by   it   notwithstanding   petitioner's   vehement  
  protestations  of  unfair  dealings  in  marketing  its  own  set  
Although   petitioner's   products   are   numerous,   they   are   of  items  which  are  limited  to:  after-­‐shave  lotion,  shaving  
of  the  same  class  or  line  of  merchandise  which  are  non-­‐ cream,  deodorant,  talcum  powder  and  toilet  soap.  In  as  
competing   with   respondent's   product   of   cigarettes,   much  as  petitioner  has  not  ventured  in  the  production  of  
which   as   pointed   out   in   the   appealed   judgment   is   briefs,   an   item   which   is   not   listed   in   its   certificate   of  
beyond   petitioner's   "zone   of   potential   or   natural   and   registration,   petitioner   can   not   and   should   not   be  
logical  expansion."  When  a  trademark  is  used  by  a  party   allowed   to   feign   that   private   respondent   had   invaded  
for   a   product   in   which   the   other   party   does   not   deal,   the   petitioner's  exclusive  domain.  To  be  sure,  it  is  significant  
use   of   the   same   trademark   on   the   latter's   product   that   petitioner   failed   to   annex   in   its   Brief   the   so-­‐called  
cannot  be  validly  objected  to.   "eloquent   proof   that   petitioner   indeed   intended   to  
  expand  its  mark  "BRUT"  to  other  goods."    Even  then,  a  
Another   factor   that   shows   that   the   goods   involved   are   mere   application   by   petitioner   in   this   aspect   does   not  
non-­‐competitive  and  non-­‐related  is  the  appellate  court's   suffice   and   may   not   vest   an   exclusive   right   in   its   favor  
finding   that   they   flow   through   different   channels   of   that   can   ordinarily   be   protected   by   the   Trademark   Law.  
trade,  thus:  "The  products  of  each  party  move  along  and   In  short,  paraphrasing  Section  20  of  the  Trademark  Law  
are  disposed  through  different  channels  of  distribution.   as   applied   to   the   documentary   evidence   adduced   by  
  petitioner,   the   certificate   of   registration   issued   by   the  
HICKOK  MANUFACTURING  CO.  V.  CA  (1982)   Director   of   Patents   can   confer   upon   petitioner   the  
Refresher:   exclusive   right   to   use   its   own   symbol   only   to   those  
Petitioner:  diverse  articles  of  men's  wear  such  as  wallets,   goods   specified   in   the   certificate,   subject   to   any  
belts  and  men's  briefs   conditions  and  limitations  stated  therein.    
   
Respondent:  shoes   How   do   We   now   reconcile   the   apparent   conflict  
  between   Section   4(d)   which   was   relied   upon   by   Justice  
Doctrine:   JBL  Reyes  in  the  Sta.  Ana  case  and  Section  20?  It  would  
It  is  established  doctrine,  as  held  in  the  above-­‐cited  cases,   seem  that  Section  4(d)  does  not  require  that  the  goods  
that   "emphasis   should   be   on   the   similarity   of   the   manufactured   by   the   second   user   be   related   to   the  
products  involved  and  not  on  the  arbitrary  classification   goods   produced   by   the   senior   user   while   Section   20  
or   general   description   of   their   properties   or   limits  the  exclusive  right  of  the  senior  user  only  to  those  
characteristics"  and  that  "the  mere  fact  that  one  person   goods  specified  in  the  certificate  of  registration.  But  the  
has   adopted   and   used   a   trademark   on   his   goods   does   rule   has   been   laid   down   that   the   clause   which   comes  
not  prevent  the  adoption  and  use  of  the  same  trademark   later   shall   be   given   paramount   significance   over   an  
by  others  on  unrelated  articles  of  a  different  kind."   anterior  proviso  upon  the  presumption  that  it  expresses  
  the  latest  and  dominant  purpose.  
 
FABERGE  V.  IAC  (1992)  
It   ineluctably   follows   that   Section   20   is   controlling   and,  
Refresher:   therefore,  private  respondent  can  appropriate  its  symbol  
The   Director   of   Patents   authorized   herein   private   for   the   briefs   it   manufactures   because   as   aptly   remarked  
respondent   Co   Beng   Kay   to   register   the   trademark   by   Justice   Sanchez   in   Sterling   Products   International   Inc.  
"BRUTE"   for   the   briefs   manufactured   and   sold   by   his   vs.  Farbenfabriken  Bayer:  
Corporation  in  the  domestic  market  vis-­‐a-­‐vis  petitioner's    
opposition  grounded  on  similarity  of  said  trademark  with   Really,   if   the   certificate   of   registration   were   to   be  
petitioner's   own   symbol   "BRUT"   which   it   previously   deemed  as  including  goods  not  specified  therein,  then  a  
registered   for   after   shave   lotion,   shaving   cream,  

88 z
Intellectual Property Law

situation   may   arise   whereby   an   applicant   may   be   attributes   or   essential   characteristics   with   reference   to  
tempted   to   register   a   trademark   on   any   and   all   goods   their   form,   composition,   texture   or   quality.   They   may  
which   his   mind   may   conceive   even   if   he   had   never   also  be  related  because  they  serve  the  same  purpose  or  
intended   to   use   the   trademark   for   the   said   goods.   We   are  sold  in  grocery  stores.  
believe   that   such   omnibus   registration   is   not    
contemplated  by  our  Trademark  Law.   PEARL  &  DEAN  (PHIL.),  INC.  V.  SHOEMART,  SUPRA  
  Refresher:  
CANON  KABUSHIKI  V.  CA  (2000)   Light  boxes  
Refresher:   Private   respondent   NSR   Rubber   Corporation    
(private  respondent)  filed  an  application  for  registration   Doctrine:  
of  the  mark  CANON  for  sandals.   On  trademark  infringement:  
  The   certificate   of   registration   issued   by   the   Director   of  
Doctrine:   Patents   can   confer   the   exclusive   right   to   use   its   own  
We   find   the   arguments   of   petitioner   to   be   unmeritorious.   symbol   only   to   those   goods   specified   in   the   certificate,  
Ordinarily,  the  ownership  of  a  trademark  or  tradename  is   subject  to  any  conditions  and  limitations  specified  in  the  
a  property  right  that  the  owner  is  entitled  to  protect  as   certificate.  One  who  has  adopted  and  used  a  trademark  
mandated   by   the   Trademark   Law.   However,   when   a   on   his   goods   does   not   prevent   the   adoption   and   use   of  
trademark  is  used  by  a  party  for  a  product  in  which  the   the  same  trademark  by  others  for  products  which  are  of  
other   party   does   not   deal,   the   use   of   the   same   a  different  description.  
trademark   on   the   latter's   product   cannot   be   validly    
objected  to.   Moreover,   the   failure   of   P   &   D   to   secure   a   trademark  
  registration   for   specific   use   on   the   light   boxes   meant  
Herein  petitioner  has  not  made  known  that  it  intends  to   that   there   could   not   have   been   any   trademark  
venture   into   the   business   of   producing   sandals.   This   is   infringement  since  registration  was  an  essential  element  
clearly   shown   in   its   Trademark   Principal   Register   where   thereof.  
the  products  of  the  said  petitioner  had  been  clearly  and    
specifically   described   as   "Chemical   products,   dyestuffs,   246  CORPORATION  V.  DAWAY  (2003)  
pigments,   toner   developing   preparation,   shoe   polisher,   Refresher:  
polishing   agent".   It   would   be   taxing   one's   credibility   to   246  Corporation:  Rolex  Music  Lounge  
aver  at  this  point  that  the  production  of  sandals  could  be    
considered   as   a   possible   "natural   or   normal   expansion"   Doctrine:  
of  its  business  operation.   Under   the   old   Trademark   Law15   where   the   goods   for  
  which   the   identical   marks   are   used   are   unrelated,   there  
The   likelihood   of   confusion   of   goods   or   business   is   a   can  be  no  likelihood  of  confusion  and  there  is  therefore  
relative   concept,   to   be   determined   only   according   to   the   no   infringement   in   the   use   by   the   junior   user   of   the  
particular,   and   sometimes   peculiar,   circumstances   of   registered   mark   on   the   entirely   different   goods.   This  
each  case.16  Indeed,  in  trademark  law  cases,  even  more   ruling,   however,   has   been   to   some   extent,   modified   by  
than  in  other  litigation,  precedent  must  be  studied  in  the   Section   123.1(f)   of   the   Intellectual   Property   Code  
light   of   the   facts   of   the   particular   case.   Contrary   to   (Republic  Act  No.  8293),  which  took  effect  on  January  1,  
petitioner's  supposition,  the  facts  of  this  case  will  show   1998.  The  said  section  reads:  
that   the   cases   of   Sta.   Ana   vs.   Maliwat,   Ang   vs.   Teodoro    
and   Converse   Rubber   Corporation   vs.   Universal   Rubber   Sec.   123.   Registrability.   –   123.1.   A   mark   cannot  
Products,   Inc.   are   hardly   in   point.   The   just   cited   cases   be  registered  if  it:  
involved   goods   that   were   confusingly   similar,   if   not    
identical,  as  in  the  case  of  Converse  Rubber  Corporation   x  x  x                      x  x  x                      x  x  x  
vs.   Universal   Rubber   Products,   Inc.   Here,   the   products    
involved   are   so   unrelated   that   the   public   will   not   be   (f)   Is   identical   with,   or   confusingly   similar   to,   or  
misled   that   there   is   the   slightest   nexus   between   constitutes   a   translation   of   a   mark   considered  
petitioner  and  the  goods  of  private  respondent.   well-­‐known   in   accordance   with   the   preceding  
  paragraph,   which   is   registered   in   the  
In  cases  of  confusion  of  business  or  origin,  the  question   Philippines   with   respect   to   goods   or   services  
that   usually   arises   is   whether   the   respective   goods   or   which   are   not   similar   to   those   with   respect   to  
services   of   the   senior   user   and   the   junior   user   are   so   which  registration  is  applied  for:  Provided,  That  
related  as  to  likely  cause  confusion  of  business  or  origin,   use   of   the   mark   in   relation   to   those   goods   or  
and   thereby   render   the   trademark   or   tradenames   services   would   indicate   a   connection   between  
confusingly  similar.  Goods  are  related  when  they  belong   those  goods  or  services,  and  the  owner  of  the  
to   the   same   class   or   have   the   same   descriptive   registered   mark:   Provided,   further,   That   the  
properties;   when   they   possess   the   same   physical  

o 89
Katrina Michelle Mancao
 
interest   of   the   owner   of   the   registered   mark   (k)  the  outcome  of  litigations  dealing  
are  likely  to  be  damaged  by  such  use;     with  the  issue  of  whether  the  mark  is  
  a  well-­‐known  mark;  and  
A  junior  user  of  a  well-­‐known  mark  on  goods  or  services    
which   are   not   similar   to   the   goods   or   services,   and   are   (l)   the   presence   of   absence   of  
therefore  unrelated,  to  those  specified  in  the  certificate   identical   or   similar   marks   validly  
of  registration  of  the  well-­‐known  mark  is  precluded  from   registered   for   or   used   on   identical   or  
using   the   same   on   the   entirely   unrelated   goods   or   similar   goods   or   services   and   owned  
services,  subject  to  the  following  REQUISITES,  to  wit:   by   persons   other   than   the   person  
  claiming  that  his  mark  is  a  well-­‐known  
1.   The   mark   is   well-­‐known   internationally   and   in   mark.  
the   Philippines.   Under   Rule   102   of   the   Rules    
and  Regulations  on  Trademarks,  Service  Marks,   2.   The   use   of   the   well-­‐known   mark   on   the  
Trade   Names   and   Marked   or   Stamped   entirely   unrelated   goods   or   services   would  
Containers,   in   determining   whether   a   mark   is   indicate   a   connection   between   such   unrelated  
well   known,   the   following   criteria   or   any   goods   or   services   and   those   goods   or   services  
combination   thereof   may   be   taken   into   specified  in  the  certificate  of  registration  in  the  
account:   well   known   mark.   This   requirement   refers   to  
  the  likelihood  of  confusion  of  origin  or  business  
(a)   the   duration,   extent   and   or   some   business   connection   or   relationship  
geographical   area   of   any   use   of   the   between   the   registrant   and   the   user   of   the  
mark,   in   particular,   the   duration,   mark.  
extent   and   geographical   area   of   any    
promotion   of   the   mark,   including   3.  The  interests  of  the  owner  of  the  well-­‐known  
advertising   or   publicity   and   mark  are  likely  to  be  damaged.  For  instance,  if  
presentation,   at   fairs   or   exhibitions,   the   registrant   will   be   precluded   from  
of   the   goods   and/or   services   to   which   expanding  its  business  to  those  unrelated  good  
the  mark  applies;   or  services,  or  if  the  interests  of  the  registrant  
  of   the   well-­‐known   mark   will   be   damaged  
(b)   the   market   share   in   the   because   of   the   inferior   quality   of   the   good   or  
Philippines   and   in   other   countries,   of   services  of  the  user.  
the   goods   and/or   services   to   which    
the  mark  applies;   Section   123.1(f)   is   clearly   in   point   because   the   Music  
  Lounge  of  petitioner  is  entirely  unrelated  to  respondents’  
(c)   the   degree   of   the   inherent   or   business   involving   watches,   clocks,   bracelets,   etc.  
acquired  distinction  of  the  mark;   However,  the  Court  cannot  yet  resolve  the  merits  of  the  
  present   controversy   considering   that   the   requisites   for  
(d)   the   quality-­‐image   or   reputation   the   application   of   Section   123.1(f),   which   constitute   the  
acquired  by  the  mark;   kernel   issue   at   bar,   clearly   require   determination   facts   of  
  which   need   to   be   resolved   at   the   trial   court.   The  
(e)  the  extent  to  which  the  mark  has   existence   or   absence   of   these   requisites   should   be  
been  registered  in  the  world;   addressed   in   a   full-­‐blown   hearing   and   not   on   a   mere  
  preliminary   hearing.   The   respondent   must   be   given  
(f)   the   exclusivity   of   the   registration   ample  opportunity  to  prove  its  claim,  and  the  petitioner  
attained  by  the  mark  in  the  world;   to  debunk  the  same.  
   
(g)  the  extent  to  which  the  mark  has   SOCIETE  DES  PRODUITS  NESTLE  V.  CA  (2001)  
been  used  in  the  world;   Refresher:  
  CFC   Corporation   (Respondent):   “Flavor   Master”   for  
(h)   the   exclusivity   of   use   attained   by   coffee  
the  mark  in  the  world;    
  Nestle:  “Master  Roast”  and  “Master  Blend”  for  coffee  
(i)  the  commercial  value  attributed  to    
the  mark  in  the  world;   Doctrine:  
  The   protection   of   trade-­‐marks   is   the   law’s   recognition   of  
(j)   the   record   of   successful   protection   the   psychological   function   of   symbols.   If   it   is   true   that  
of  the  rights  in  the  mark;   we   live   by   symbols,   it   is   no   less   true   that   we   purchase  
  goods   by   them.   A   trade-­‐mark   is   a   merchandising   short-­‐

90 z
Intellectual Property Law

cut  which  induces  a  purchaser  to  select  what  he  wants,   what   is   of   paramount   consideration   is   the   ordinary  
or  what  he  has  been  led  to  believe  he  wants.  The  owner   purchaser   who   is,   in   general,   undiscerningly   rash   in  
of   a   mark   exploits   this   human   propensity   by   making   buying  the  more  common  and  less  expensive  household  
every   effort   to   impregnate   the   atmosphere   of   the   products   like   coffee,   and   is   therefore   less   inclined   to  
market   with   the   drawing   power   of   a   congenial   symbol.   closely   examine   specific   details   of   similarities   and  
Whatever  the  means  employed,  the  aim  is  the  same  -­‐-­‐-­‐  to   dissimilarities  between  competing  products.  
convey   through   the   mark,   in   the   minds   of   potential    
customers,  the  desirability  of  the  commodity  upon  which   This   Court   cannot   agree   with   the   above   reasoning.   If   the  
it   appears.   Once   this   is   attained,   the   trade-­‐mark   owner   ordinary   purchaser   is   "undiscerningly   rash"   in   buying  
has   something   of   value.   If   another   poaches   upon   the   such   common   and   inexpensive   household   products   as  
commercial   magnetism   of   the   symbol   he   has   created,   instant   coffee,   and   would   therefore   be   "less   inclined   to  
the  owner  can  obtain  legal  redress.   closely   examine   specific   details   of   similarities   and  
  dissimilarities"   between   the   two   competing   products,  
Colorable   imitation   denotes   such   a   close   or   ingenious   then  it  would  be  less  likely  for  the  ordinary  purchaser  to  
imitation  as  to  be  calculated  to  deceive  ordinary  persons,   notice  that  CFC’s  trademark  FLAVOR  MASTER  carries  the  
or   such   a   resemblance   to   the   original   as   to   deceive   an   colors  orange  and  mocha  while  that  of  Nestle’s  uses  red  
ordinary   purchaser   giving   such   attention   as   a   purchaser   and   brown.   The   application   of   the   totality   or   holistic   test  
usually   gives,   as   to   cause   him   to   purchase   the   one   is   improper   since   the   ordinary   purchaser   would   not   be  
supposing  it  to  be  the  other.  In  determining  if  colorable   inclined   to   notice   the   specific   features,   similarities   or  
imitation   exists,   jurisprudence   has   developed   two   kinds   dissimilarities,   considering   that   the   product   is   an  
of   tests   -­‐   the   Dominancy   Test   and   the   Holistic   Test.   The   inexpensive  and  common  household  item.  
test   of   dominancy   focuses   on   the   similarity   of   the    
prevalent   features   of   the   competing   trademarks   which   It   must   be   emphasized   that   the   products   bearing   the  
might  cause  confusion  or  deception  and  thus  constitute   trademarks   in   question   are   "inexpensive   and   common"  
infringement.   On   the   other   side   of   the   spectrum,   the   household  items  bought  off  the  shelf  by  "undiscerningly  
holistic   test   mandates   that   the   entirety   of   the   marks   in   rash"   purchasers.   As   such,   if   the   ordinary   purchaser   is  
question   must   be   considered   in   determining   confusing   "undiscerningly   rash",   then  he  would  not  have  the  time  
similarity.   nor   the   inclination   to   make   a   keen   and   perceptive  
  examination   of   the   physical   discrepancies   in   the  
Nestle   points   out   that   the   dominancy   test   should   have   trademarks   of   the   products   in   order   to   exercise   his  
been  applied  to  determine  whether  there  is  a  confusing   choice.  
similarity   between   CFC’s   FLAVOR   MASTER   and   Nestle’s    
MASTER  ROAST  and  MASTER  BLEND.   While   this   Court   agrees   with   the   Court   of   Appeals’  
  detailed   enumeration   of   differences   between   the  
We  agree.   respective   trademarks   of   the   two   coffee   products,   this  
  Court   cannot   agree   that   totality   test   is   the   one  
As   the   Court   of   Appeals   itself   has   stated,   "[t]he   applicable   in   this   case.   Rather,   this   Court   believes   that  
determination   of   whether   two   trademarks   are   indeed   the   dominancy   test   is   more   suitable   to   this   case   in   light  
confusingly  similar  must  be  taken  from  the  viewpoint  of   of  its  peculiar  factual  milieu.  
the   ordinary   purchasers   who   are,   in   general,    
undiscerningly  rash  in  buying  the  more  common  and  less   Moreover,   the   totality   or   holistic   test   is   contrary   to   the  
expensive   household   products   like   coffee,   and   are   elementary   postulate   of   the   law   on   trademarks   and  
therefore  less  inclined  to  closely  examine  specific  details   unfair   competition   that   confusing   similarity   is   to   be  
of   similarities   and   dissimilarities   between   competing   determined   on   the   basis   of   visual,   aural,   connotative  
products."   comparisons  and  overall  impressions  engendered  by  the  
  marks   in   controversy   as   they   are   encountered   in   the  
The   basis   for   the   Court   of   Appeals’   application   of   the   realities   of   the   marketplace.   The   totality   or   holistic   test  
totality  or  holistic  test  is  the  "ordinary  purchaser"  buying   only   relies   on   visual   comparison   between   two  
the   product   under   "normally   prevalent   conditions   in   trademarks   whereas   the   dominancy   test   relies   not   only  
trade"   and   the   attention   such   products   normally   elicit   on   the   visual   but   also   on   the   aural   and   connotative  
from   said   ordinary   purchaser.   An   ordinary   purchaser   or   comparisons   and   overall   impressions   between   the   two  
buyer   does   not   usually   make   such   scrutiny   nor   does   he   trademarks.  
usually   have   the   time   to   do   so.   The   average   shopper   is    
usually  in  a  hurry  and  does  not  inspect  every  product  on   For  this  reason,  this  Court  agrees  with  the  BPTTT  when  it  
the  shelf  as  if  he  were  browsing  in  a  library.   applied  the  test  of  dominancy  and  held  that:  
   
The   Court   of   Appeals   held   that   the   test   to   be   applied   From   the   evidence   at   hand,   it   is   sufficiently   established  
should   be   the   totality   or   holistic   test   reasoning,   since   that   the   word   MASTER   is   the   dominant   feature   of  

o 91
Katrina Michelle Mancao
 
opposer’s  mark.  The  word  MASTER  is  printed  across  the   shedding  "some  light"  upon  certain  characteristics  of  the  
middle   portion   of   the   label   in   bold   letters   almost   twice   goods   or   services   in   dispute,   they   nevertheless   involve  
the   size   of   the   printed   word   ROAST.   Further,   the   word   "an   element   of   incongruity,"   "figurativeness,"   or   "  
MASTER   has   always   been   given   emphasis   in   the   TV   and   imaginative  effort  on  the  part  of  the  observer."  
radio   commercials   and   other   advertisements   made   in    
promoting   the   product.   This   can   be   gleaned   from   the   LEVI  STRAUSS  V.  CLINTON  APPARELLE  (2005)  
fact   that   Robert   Jaworski   and   Atty.   Ric   Puno   Jr..,   the   Refresher:  
personalities  engaged  to  promote  the  product,  are  given   The   Complaint   alleged   that   LS   &   Co.,   a   foreign  
the   titles   Master   of   the   Game   and   Master   of   the   Talk   corporation   duly   organized   and   existing   under   the   laws  
Show,   respectively.   In   due   time,   because   of   these   of   the   State   of   Delaware,   U.S.A.,   and   engaged   in   the  
advertising   schemes   the   mind   of   the   buying   public   had   apparel   business,   is   the   owner   by   prior   adoption   and   use  
come   to   learn   to   associate   the   word   MASTER   with   the   since   1986   of   the   internationally   famous   "Dockers   and  
opposer’s  goods.   Design"   trademark.   This   ownership   is   evidenced   by   its  
  valid   and   existing   registrations   in   various   member  
x  x  x.  It  is  the  observation  of  this  Office  that  much  of  the   countries   of   the   Paris   Convention.   In   the   Philippines,   it  
dominance   which   the   word   MASTER   has   acquired   has   a   Certificate   of   Registration   No.   46619   in   the  
through   Opposer’s   advertising   schemes   is   carried   over   Principal   Register   for   use   of   said   trademark   on   pants,  
when   the   same   is   incorporated   into   respondent-­‐ shirts,   blouses,   skirts,   shorts,   sweatshirts   and   jackets  
applicant’s   trademark   FLAVOR   MASTER.   Thus,   when   one   under  Class  25.  
looks   at   the   label   bearing   the   trademark   FLAVOR    
MASTER   (Exh.   4)   one’s   attention   is   easily   attracted   to   The   "Dockers   and   Design"   trademark   was   first   used   in  
the  word  MASTER,  rather  than  to  the  dissimilarities  that   the  Philippines  in  or  about  May  1988,  by  LSPI,  a  domestic  
exist.   Therefore,   the   possibility   of   confusion   as   to   the   corporation   engaged   in   the   manufacture,   sale   and  
goods   which   bear   the   competing   marks   or   as   to   the   distribution   of   various   products   bearing   trademarks  
origins  thereof  is  not  farfetched.  x  x  x.   owned   by   LS   &   Co.   To   date,   LSPI   continues   to  
  manufacture   and   sell   Dockers   Pants   with   the   "Dockers  
In  addition,  the  word  "MASTER"  is  neither  a  generic  nor   and  Design"  trademark.  
a   descriptive   term.   As   such,   said   term   can   not   be    
invalidated  as  a  trademark  and,  therefore,  may  be  legally   LS   &   Co.   and   LSPI   further   alleged   that   they   discovered  
protected.   Generic   terms   are   those   which   constitute   the   presence   in   the   local   market   of   jeans   under   the  
"the   common   descriptive   name   of   an   article   or   brand   name   "Paddocks"   using   a   device   which   is  
substance,"   or   comprise   the   "genus   of   which   the   substantially,   if   not   exactly,   similar   to   the   "Dockers   and  
particular   product   is   a   species,"   or   are   "commonly   used   Design"  trademark  owned  by  and  registered  in  the  name  
as  the  name  or  description  of  a  kind  of  goods,"  or  "imply   of   LS   &   Co.,   without   its   consent.   Based   on   their  
reference   to   every   member   of   a   genus   and   the   exclusion   information   and   belief,   they   added,   Clinton   Apparelle  
of  individuating  characters,"  or  "refer  to  the  basic  nature   manufactured   and   continues   to   manufacture   such  
of   the   wares   or   services   provided   rather   than   to   the   "Paddocks"  jeans  and  other  apparel.  
more   idiosyncratic   characteristics   of   a   particular    
product,"   and   are   not   legally   protectable.   On   the   other   Doctrine:  
hand,   a   term   is   descriptive   and   therefore   invalid   as   a   Petitioners   anchor   their   legal   right   to   "Dockers   and  
trademark   if,   as   understood   in   its   normal   and   natural   Design"   trademark   on   the   Certificate   of   Registration  
sense,   it   "forthwith   conveys   the   characteristics,   issued   in   their   favor   by   the   Bureau   of   Patents,  
functions,   qualities   or   ingredients   of   a   product   to   one   Trademarks   and   Technology   Transfer.*   According   to  
who  has  never  seen  it  and  does  not  know  what  it  is,"  or   Section   138   of   Republic   Act   No.   8293,42   this   Certificate  
"if   it   forthwith   conveys   an   immediate   idea   of   the   of   Registration   is   prima   facie   evidence   of   the   validity   of  
ingredients,  qualities  or  characteristics  of  the  goods,"  or   the   registration,   the   registrant’s   ownership   of   the   mark  
if  it  clearly  denotes  what  goods  or  services  are  provided   and  of  the  exclusive  right  to  use  the  same  in  connection  
in   such   a   way   that   the   consumer   does   not   have   to   with   the   goods   or   services   and   those   that   are   related  
exercise  powers  of  perception  or  imagination.   thereto   specified   in   the   certificate.   Section   147.1   of   said  
  law  likewise  grants  the  owner  of  the  registered  mark  the  
Rather,  the  term  "MASTER"  is  a  suggestive  term  brought   exclusive  right  to  prevent  all  third  parties  not  having  the  
about   by   the   advertising   scheme   of   Nestle.   Suggestive   owner’s   consent   from   using   in   the   course   of   trade  
terms  are  those  which,  in  the  phraseology  of  one  court,   identical  or  similar  signs  for  goods  or  services  which  are  
require  "imagination,  thought  and  perception  to  reach  a   identical   or   similar   to   those   in   respect   of   which   the  
conclusion   as   to   the   nature   of   the   goods."   Such   terms,   trademark  is  registered  if  such  use  results  in  a  likelihood  
"which   subtly   connote   something   about   the   product,"   of  confusion.  
are   eligible   for   protection   in   the   absence   of   secondary    
meaning.   While   suggestive   marks   are   capable   of  

92 z
Intellectual Property Law

However,   attention   should   be   given   to   the   fact   that   quality  of  the  mark."  This  is  intended  to  protect  famous  
petitioners’   registered   trademark   consists   of   two   marks  from  subsequent  uses  that  blur  distinctiveness  of  
elements:   (1)   the   word   mark   "Dockers"   and   (2)   the   wing-­‐ the  mark  or  tarnish  or  disparage  it.  
shaped   design   or   logo.   Notably,   there   is   only   one    
registration   for   both   features   of   the   trademark   giving   Based   on   the   foregoing,   to   be   eligible   for   protection  
the   impression   that   the   two   should   be   considered   as   a   from   dilution,   there   has   to   be   a   finding   that:   (1)   the  
single   unit.   Clinton   Apparelle’s   trademark,   on   the   other   trademark   sought   to   be   protected   is   famous   and  
hand,   uses   the   "Paddocks"   word   mark   on   top   of   a   logo   distinctive;  (2)  the  use  by  respondent  of  "Paddocks  and  
which   according   to   petitioners   is   a   slavish   imitation   of   Design"   began   after   the   petitioners’   mark   became  
the   "Dockers"   design.   The   two   trademarks   apparently   famous;   and   (3)   such   subsequent   use   defames  
differ   in   their   word   marks   ("Dockers"   and   "Paddocks"),   petitioners’   mark.   In   the   case   at   bar,   petitioners   have   yet  
but   again   according   to   petitioners,   they   employ   similar   to  establish  whether  "Dockers  and  Design"  has  acquired  
or  identical  logos.  It  could  thus  be  said  that  respondent   a  strong  degree  of  distinctiveness  and  whether  the  other  
only   "appropriates"   petitioners’   logo   and   not   the   word   two   elements   are   present   for   their   cause   to   fall   within  
mark  "Dockers";  it  uses  only  a  portion  of  the  registered   the   ambit   of   the   invoked   protection.   The   Trends   MBL  
trademark  and  not  the  whole.   Survey   Report   which   petitioners   presented   in   a   bid   to  
  establish   that   there   was   confusing   similarity   between  
Given   the   single   registration   of   the   trademark   "Dockers   two  marks  is  not  sufficient  proof  of  any  dilution  that  the  
and   Design"   and   considering   that   respondent   only   uses   trial  court  must  enjoin.  
the   assailed   device   but   a   different   word   mark,   the   right    
to   prevent   the   latter   from   using   the   challenged   RA   166,  SECTION   20.  CERTIFICATE  OF  REGISTRATION  PRIMA  
"Paddocks"  device  is  far  from  clear.  Stated  otherwise,  it  
FACIE  EVIDENCE  O F  VALIDITY.  –    
is   not   evident   whether   the   single   registration   of   the  
A  certificate  of  registration  of  a  mark  or  trade-­‐name  shall  
trademark   "Dockers   and   Design"   confers   on   the   owner  
the   right   to   prevent   the   use   of   a   fraction   thereof   in   the   be   prima   facie   evidence   of   the   validity   of   the  
course   of   trade.   It   is   also   unclear   whether   the   use   registration,   the   registrant's   ownership   of   the   mark   or  
without  the  owner’s  consent  of  a  portion  of  a  trademark   trade-­‐name,   and   of   the   registrant's   exclusive   right   to  
registered   in   its   entirety   constitutes   material   or   use  the  same  in  connection  with  the  goods,  business  or  
substantial  invasion  of  the  owner’s  right.   services   specified   in   the   certificate,   subject   to   any  
  conditions  and  limitations  stated  therein.  
It  is  likewise  not  settled  whether  the  wing-­‐shaped  logo,    
as   opposed   to   the   word   mark,   is   the   dominant   or   central   SECTION  138.  CERTIFICATES  OF  REGISTRATION.  –    
feature   of   petitioners’   trademark—the   feature   that   A  certificate  of  registration  of  a  mark  shall  be  prima  facie  
prevails   or   is   retained   in   the   minds   of   the   public—an   evidence   of   the   validity   of   the   registration,   the  
imitation  of  which  creates  the  likelihood  of  deceiving  the   registrant's   ownership   of   the   mark,   and   of   the  
public   and   constitutes   trademark   infringement.   In   sum,   registrant's   exclusive   right   to   use   the   same   in  
there   are   vital   matters   which   have   yet   and   may   only   be   connection   with   the   goods   or   services   and   THOSE   THAT  
established  through  a  full-­‐blown  trial.   ARE  RELATED  THERETO  specified  in  the  certificate.  
   
From   the   above   discussion,   we   find   that   petitioners’   Question:   Was   Faberge   v.   IAC   (215   SCRA   316)   the  
right   to   injunctive   relief   has   not   been   clearly   and   controlling   precedent   under   RA   166?   State   reasons   for  
unmistakably   demonstrated.   The   right   has   yet   to   be   your   answer.   Research   the   cases   cited   in   your   syllabus  
determined.   Petitioners   also   failed   to   show   proof   that   on  "Is  there  infringement  if  goods  are  not  competing?"  
there  is  material  and  substantial  invasion  of  their  right  to   section  of  your  syllabus.  
warrant  the  issuance  of  an  injunctive  writ.  Neither  were    
petitioners   able   to   show   any   urgent   and   permanent   HW  Answer  (Di  Bonilla):  
necessity  for  the  writ  to  prevent  serious  damage.   No,   Faberge   v.   IAC   was   not   the   controlling  
  precedent   under   RA   166.   Since   Faberge  
Trademark   dilution   is   the   lessening   of   the   capacity   of   a   pronounced   that   Sec.   20   is   controlling   over   Sec  
famous   mark   to   identify   and   distinguish   goods   or   4(d),     it   effectively   reversed   earlier   decisions  
services,   regardless   of   the   presence   or   absence   of:   (1)   which  merely  used  the  "likelihood  of  confusion,  
competition  between  the  owner  of  the  famous  mark  and   deception   or   mistake"   standard   and   which   did  
other   parties;   or   (2)   likelihood   of   confusion,   mistake   or   not   require   similarity   between   the   goods   or  
deception.   Subject   to   the   principles   of   equity,   the   owner   articles  to  which  the  mark  was  being  applied.    
of   a   famous   mark   is   entitled   to   an   injunction   "against    
another  person’s  commercial  use  in  commerce  of  a  mark   Faberge  held  that  the  statement  of  the  Court  in  
or   trade   name,   if   such   use   begins   after   the   mark   has   the   case   of   Sta.   Ana   vs.   Maliwat   that   Sec   4(d)  
become   famous   and   causes   dilution   of   the   distinctive   of   RA   166   does   not   require   that   the   subject  

o 93
Katrina Michelle Mancao
 
articles   possess   the   same   descriptive   property   use   the   mark   in   connection   with   the   goods,  
or   fall   in   the   same   category   and   that   business   or   services   specified   in   the   certificate  
dissimilarity  will  not  preclude  relief  if  the  junior   is  merely  prima  facie  and  is  thus  rebuttable.  
user's   goods   are   not   remote   from   any   other    
product  which  the  first  user  would  likely  make   Third,   the   present   IP   Code   enumerates   the  
or   sell   cannot   prevail   over   the   positive   rights  of  the  owner  of  a  registered  mark  under  
requirement   of   Sec.   20   of   the   same   law   that   Sec.   147.   Subsection   147.1   grants   the   trademark  
only   those   specified   in   the   certificate   of   owner   the   “exclusive   right   to   prevent   all   third  
registration   are   protected.   Effectively   then,   the   parties   not   having   the   owner's   consent   from  
Supreme  Court  in  the  Faberge  case  reversed  its   using  in  the  course  of  trade  identical  or  similar  
earlier  ruling  in  Sta.  Ana  and  other  cases  which   signs  or  containers  for  goods  or  services  which  
merely   relied   on   the     "likelihood   of   confusion,   are   identical   or   similar   to   those   in   respect   of  
deception   or   mistake"   standard   and   which   did   which   the   trademark   is   registered   where   such  
not   require   similarity   between   the   goods   or   use   would   result   in   a   likelihood   of   confusion.”  
articles.   This   was   done   with   the   Supreme   Court   In   other   words,   the   protection   granted   by   law  
not   sitting   en   banc,   in   violation   of   the   to   a   trademark   owner   extends   to   goods   or  
Constitution   which   requires   the   Supreme   Court   services   which   are   only   SIMILAR   to   those   in  
to   be   sitting   en   banc   in   order   to   reverse   an   respect  of  which  the  mark  is  registered.  
earlier   doctrine   enunciated   by   it   (Art.   VIII,   Sec.    
3(3)).   Furthermore,   Subsection   147.2   also   recognizes  
  the   protection   as   applicable   even   to   goods   and  
Question:  Is  the  Faberge  case  still  good  law  under  the  IP   services   NOT   SIMILAR   to   those   in   respect   of  
Code?  State  the  reasons  for  your  answer.   which   the   mark   is   registered   in   the   case   of   an  
  internationally   well   known   mark   which   is  
HW  Answer  (Di  Bonilla)   registered   in   the   Philippines   provided   that   the  
No,   Faberge   vs.   IAC   is   no   longer   good   law   use   would   indicate   a   connection  
under  the  IP  Code.  First,  the  present  law  (under   between   the   goods   or   services   and  
Sec.  138)  now  includes  related  goods,  to  wit:  A   the   owner   of   the   registered   mark  
certificate   of   registration   of   a   mark   shall   be   and   the   interests   of   the   owner   or  
prima   facie   evidence   of   the   validity   of   the   the   registered   mark   are   likely   to   be  
registration,   the   registrant's   ownership   of   the   damaged  by  such  use.    
mark,  and  of  the  registrant's  exclusive  right  to    
use   the   same   in   connection   with   the   goods   or  
Issue  of  parallel  importation  
services   and   those   that   are   related   thereto  
 
specified  in  the  certificate.  (emphasis  supplied)  
  YU  V.  CA  AND  UNISIA  MERCHANDISING  CO.  (1993)  
This   provision   adds   related   goods   or   services   Doctrine:  To  Our  mind,  the  right  to  perform  an  exclusive  
specified  in  the  certificate,  compared  to  Sec.  20   distributorship   agreement   and   to   reap   the   profits  
of  the  old  IP  law  which  states  that:  A  certificate   resulting   from   such   performance   are   proprietary   rights  
of  registration  of  a  mark  or  trade-­‐name  shall  be   which   a   party   may   protect   which   may   otherwise   not   be  
prima   facie   evidence   of   the   validity   of   the   diminished,   nay,   rendered   illusory   by   the   expedient   act  
registration,   the   registrant's   ownership   of   the   of   utilizing   or   interposing   a   person   or   firm   to   obtain  
mark   or   trade-­‐name,   and   of   the   registrant's   goods   from   the   supplier   to   defeat   the   very   purpose   for  
exclusive   right   to   use   the   same   in   connection   which   the   exclusive   distributorship   was   conceptualized,  
with   the   goods,   business   or   services   specified   at  the  expense  of  the  sole  authorized  distributor.  
in  the  certificate,  subject  to  any  conditions  and    
limitations  stated  therein.   RIGHTS  CONFERRED:  
  -­‐ Under   the   IP   Code,   we   have   shifted   to   the  
The  addition  of  the  phrase  “and  those  that  are   REGISTRATION  as  the  basis  of  ownership  
related   thereto”   should   be   given   its   o registration   is   the   source   of   right   –   major  
significance.   change  in  the  IP  Code  (Jan.  1,  1998)  
  o prior   to   IP   Code   –   use   was   the   basis   of  
Second,  Faberge  relied  heavily  upon  Sec.  20  of   ownership  of  tm  (RA  166)  
RA  166,  which  did  not  even  define  the  rights  of   § Prior  user  can  trump  the  rights  of  
a  trademark  owner  but  merely  established  the   a  registrant  
prima  facie  evidentiary  value  of  a  certificate  of   o Who   would   prevail   between   the   prior   user  
registration.   The   evidentiary   value   of   the   before   Jan.   1,   1998   (since   1980),   but  
certificate    as  the  registrant's  exclusive  right  to   neglected  to  file  registration,  and  another  

94 z
Intellectual Property Law

entity   (registrant)   who   has   registered   its   be   using   the   same  


trademark  before  the  prior  user?   harmful  chemicals.  
§ Prior   user   would   prevail.   Section   o Dilution   introduces  
236  –     confusion   to   the  
§ Ma’am:   this   is   a   grey   area   market.   (term   coined   in  
because  it  was  brought  about  by   Ang  Tibay  v.  Teodoro)  
the  transition   § Junior   registrant’s  
• Right   has   been   vested   prior   use   of   the   mark  
to   the   effectivity   of   the   IP   may   undermine  
Code.   Such   vested   right   the  strength  of  the  
cannot   be   impaired   by   the   senior   registrant’s  
IP  Code.   mark.  It  might  also  
-­‐ Rights  of  the  Registrant:   confuse   the  
o Right   to   prevent   others   from   using   your   consuming  public.  
mark,   or   a   confusingly   similar   mark,   in   § Question:   Would   an   ordinary  
respect  to  goods  and  services   registrant   be   able   to   claim  
§ Actual  confusion  is  not  required.   protection   from   dilution   under  
What   is   required   is   a   likelihood   Section  147?  
of  confusion.   • Answer:   Depends   on  
• Used   for   the   very   same   how  “similar”  would  be  
goods   or   services   =   defined.   Existing  
presumption   that   there   is   a   jurisprudence   provide  
likelihood  of  confusion.   for  factors.  
§ Problem:   § Question:   Why   aren’t   ordinary  
• Senior   registrant:   Cosmetics   registrants   not   accorded  
and  toiletries   protection   against   use   by  
• Junior  registrant:  Pesticides   dissimilar  goods?  Why  are  there  2  
• Question:  Competing?   different  rules?  
• Answer:   • Obligation  to  recognize  
o Note   the   IPO   ruling:   if   well-­‐known   marks   is   a  
the   goods   the   same   to   treaty  obligation.  
the   same   class,   even   if   o Ma’am:   But   the  
not   identical   =   treaty   does   not  
subsequent   require   broader  
registration   is   NOT   protection.  
allowed   • Note:   “Well-­‐known”   –  
§ “International   under   the   IP   Code,  
class”  =  based  on  a   internationally   AND  
Treaty   locally  
§ major   factor   to   § Ma’am:   Our   law   does   not  
determine   if   they   sufficiently   protect   locally-­‐
are   similar   =   established   brands.   à   From   a  
CLASSIFICATION   policy   perspective,   we   should  
o Should   not   be   allowed   protect   local   brands,   because   it  
even  if  not  in  the  same   contributes   to   our   national  
class.   The   business,   economy.  
goodwill,   image   and   § Ma’am:   For   local   brands,   you  
reputation   of   the   should   focus   your   arguments   in  
senior   registrant   may   similarity.   Argue   that   it   is   broad  
be   damaged.   It   will   enough   to   cover   the   junior  
cause  dilution.   goods.   Then   argue   the  
§ Buyers  might  think   “trademark   dilution”   doctrine  
that   the   origin   of   from  jurisprudence.  You  can  also  
the   junior   argue   on   the   reasonable   scope  
registrant’s   of  expansion.  
product   is   the   § Ma’am:   Being   “well-­‐known”  
senior   registrant.   internationally   is   difficult   to  
Thus,   they   might   prove.   There   are   several   factors  

o 95
Katrina Michelle Mancao
 
that   should   be   shown.   Note   registered?  Not  protected  against  dilution.  
enumeration   in   246   Corporation   o Ma’am:   But   easy   way   out   is   invoking   Levi  
v.  Daway.   Strauss  by  Justice  Tinga.  
-­‐ Aside:   Article   8,   Sec.   3,   Constitution   –   on   setting   -­‐ Issue   of   Parallel   importation:   Does   a   registrant   have  
aside  decision.  (Important  in  arguing  that  Faberge  is   the   right   to   prevent   the   importation   of   genuine  
not  the  prevailing  ruling  under  RA  166)   goods  by  third  parties?  NO.  
o Under   RA   166,   the   prevailing   doctrine   is   o Note   the   strategy   of   pharmaceutical  
provided   in   Chua   Che   and   Sta.   Ana   cases   companies   –   entering   into   licensing  
(en   banc   decisions).   The   goods   need   not   agreements.  This  is  a  tax  saving  strategy.  
be  similar.   o The   IP   Code   did   not   give   a   trademark  
o Subsequent   cases   (Philippine   Refining,   registrant   the   exclusive   right   to   import  
Esso,   Hickok)   decided   by   division   –   goods   (they  do  not  have  the  right  given  to  patent  
must  be  competing.   holders).   Importation   of   genuine   goods  
o Faberge   –   an   extraordinary   case,   which   is   manufactured  elsewhere  is  not  prohibited  
also   decided   by   division   –   goods   must   be   from  entering  the  Philippines.  
specifically   identified   in   the   certificate   of   o NOW:  Cheaper  Medicines  Act    
registration   (relying   on   Section   20   of   the   o Yu  v.  CA  –  not  about  trademark  law.  This  is  
old  law).   about   contract   law.   Injunction   was   issued  
§ Ma’am:   But   Section   20   does   not   because   the   other   party   fraudulently  
deal   with   the   scope   of   misled  the  trademark  owner    à  the  Court  
protection.   It   only   stated   that   used   third   party   contractual   tort.   Thus,  
the   certificate   of   registration   this   may   not   be   used   to   argue   that  
shall  be  a  prima  facie  evidence  of   trademark  owner  has  the  right  to  prevent  
tm  ownership.   parallel  importation.  
• This   ruling   requires   o Ma’am:   Recourse   of   the   exclusive  
applicant  to  apply  for  a   distributor   should   be   against   the  
lot   of   goods,   and   trademark   owner.   It’s   hard   to   go   against  
applicant  must  also  use   the  parallel  importer  because  it  is  not  privy  
the   same   mark   for   the   to   the   exclusive   distributorship  
products   enumerated   agreement.   Thus,   make   sure  
therein.     that   the   agreement   allows   the  
§ But   this   case   found   its   way   in   exclusive   distributor   to   go  
Canon   Kabushiki   and   Pearl   &   against   the   trademark   owner,  
Dean   cases,   giving   the   or  at  least  to  ask  compensation  
impression   that   it   is   the   for  any  parallel  importation.    
prevailing  ruling.    
• Canon   and   Pearl   &   I. Remedies  
Dean   were   decided    
after   the   IP   Code,   but   SECTION   3.   INTERNATIONAL   CONVENTIONS   AND  
involved   facts  
occurring   prior   to   the   RECIPROCITY.  –  
effectivity   of   the   IP   Any  person  who  is  a  national  or  who  is  domiciled  or  has  
Code.   a  real  and  effective  industrial  establishment  in  a  country  
§ Faberge   is   no   longer   effective   which  is  a  party  to  any  convention,  treaty  or  agreement  
under   the   IP   Code.   There   is   now   relating   to   intellectual   property   rights   or   the   repression  
a   change   in   the   phraseology   of   of   unfair   competition,   to   which   the   Philippines   is   also   a  
the   similar   provision   –   Section   party,   or   extends   reciprocal   rights   to   nationals   of   the  
138   under   the   IP   Code.   Philippines   by   law,   shall   be   entitled   to   benefits   to   the  
Furthermore,   Section   147   was   extent  necessary  to  give  effect  to  any  provision  of  such  
clear   in   defining   the   scope   of   convention,   treaty   or   reciprocal   law,   IN   ADDITION   to  
protection  under  the  IP  Code.   the   rights   to   which   any   owner   of   an   intellectual   property  
o 246  Corporation  –  recognized  the  changed   right  is  otherwise  entitled  by  this  Act.  
in  phraseology.    
-­‐ Illustration   of   dilution:   Seiko   watch   and   Seiko   wallet   SECTION   160.   RIGHT  OF   FOREIGN   CORPORATION  TO   SUE  IN  
à   The   two   have   different   markets.   Because   of   the   TRADEMARK  OR  SERVICE  MARK  ENFORCEMENT  ACTION.  –    
use  of  Seiko  wallet,  Seiko  watch  lost  its  aspirational  
Any   foreign   national   or   juridical   person   who   meets   the  
value.  
requirements   of   Section   3   of   this   Act   and   does   not  
-­‐ What   about   a   well-­‐known   mark   that   is   not  
engage   in   business   in   the   Philippines  may  bring  a  CIVIL  

96 z
Intellectual Property Law

or   ADMINISTRATIVE   action   hereunder   for   opposition,   "facts  showing  the  capacity  of  a  party  to  sue  or  be  sued  
cancellation,   infringement,   unfair   competition,   or   false   or   the   authority   of   a   party   to   sue   or   be   sued   in   a  
designation   of   origin   and   false   description,   whether   or   representative   capacity   or   the   legal   existence   of   an  
not  it  is  licensed  to  do  business  in  the  Philippines  under   organized   association   of   persons   that   is   made   a   party,  
existing  laws.   must  be  averred  "  
   
SECTION  231.  REVERSE  RECIPROCITY  OF  FOREIGN  LAWS.  –     In   the   case   at   bar,   private   respondent   has   chosen   to  
Any   condition,   restriction,   limitation,   diminution,   anchor   its   action   under   the   Trademark   Law   of   the  
requirement,   penalty   or   any   similar   burden   imposed   by   Philippines,   a   law   which,   as   pointed   out,   explicitly   sets  
the   law   of   a   foreign   country   on   a   Philippine   national   down   the   conditions   precedent   for   the   successful  
seeking  protection  of  intellectual  property  rights  in  that   prosecution   thereof.   It   is   therefore   incumbent   upon  
country,  shall  reciprocally  be  enforceable  upon  nationals   private   respondent   to   comply   with   these   requirements  
of  said  country,  within  Philippine  jurisdiction.   or   aver   its   exemption   therefrom,   if   such   be   the   case.   It  
  may   be   that   private   respondent   has   the   right   to   sue  
before   Philippine   courts,   but   our   rules   on   pleadings  
LEVITON  INDUSTRIES  V.  SALVADOR  (1982)  
require   that   the   necessary   qualifying   circumstances  
Doctrine:   which  clothe  it  with  such  right  be  affirmatively  pleaded.  
We   agree   with   petitioners   that   respondent   Leviton    
Marketing  Co.,  Inc.  had  failed  to  allege  the  essential  facts  
PUMA  V.  IAC  (1988)  
bearing  upon  its  capacity  to  sue  before  Philippine  courts.  
Private   respondent's   action   is   squarely   founded   on   Refresher:  
Section  21-­‐A  of  Republic  Act  No.  166,  as  amended,  which   The  petitioner,  a  foreign  corporation  duly  organized  and  
we  quote:   existing   under   the   laws   of   the   Federal   Republic   of  
  Germany  and  the  manufacturer  and  producer  of  "PUMA  
Sec.   21-­‐A.   Any   foreign   corporation   or   juristic   person   to   PRODUCTS,"  filed  a  complaint  for  infringement  of  patent  
which   a   mark   or   tradename   has   been   registered   or   or  trademark  with  a  prayer  for  the  issuance  of  a  writ  of  
assigned   under   this   Act   may   bring   an   action   hereunder   preliminary   injunction   against   the   private   respondent  
for   infringement,   for   unfair   competition,   or   false   before  the  Regional  Trial  Court  of  Makati.  
designation   of   origin   and   false   description,   whether   or    
not  it  has  been  licensed  to  do  business  in  the  Philippines   Doctrine:  
under   Act   numbered   Fourteen   Hundred   and   Fifty-­‐Nine,   The   Petitioner   may   still   bring   the   action.   Court   quoted  
as   amended,   otherwise   known   as   the   Corporation   Law,   the  decision  in  La  Chemise  Lacoste  v.  Fernandez  (infra).  
at   the   time   it   brings   the   complaint;   Provided,   That   the    
country   of   which   the   said   foreign   corporation   or   juristic   Petitioner   maintains   that   it   has   substantially   complied  
person  is  a  citizen,  or  in  which  it  is  domiciled,  by  treaty,   with   the   requirements   of   Section   21-­‐A   of   Republic   Act  
convention  or  law,  grants  a  similar  privilege  to  corporate   R.A.   No.   166,   as   amended.   According   to   the   petitioner,  
or  juristic  persons  of  the  Philippines.  (As  amended  by  R.A.   its   complaint   specifically   alleged   that   it   is   not   doing  
No.  638)   business   in   the   Philippines   and   is   suing   under   the   said  
  Republic   Act;   that   Section   21-­‐A   thereof   provides   that  
Undoubtedly,   the   foregoing   section   grants   to   a   foreign   "the   country   of   which   the   said   corporation   or   juristic  
corporation,   whether   or   not   licensed   to   do   business   in   person  is  a  citizen,  or  in  which  it  is  domiciled,  by  treaty,  
the   Philippines,   the   right   to   seek   redress   for   unfair   convention  or  law,  grants  a  similar  privilege  to  corporate  
competition  before  Philippine  courts.  But  the  said  law  is   or   juristic   persons   of   the   Philippines"   but   does   not  
not  without  qualifications.  Its  literal  tenor  indicates  as  a   mandatorily   require   that   such   reciprocity   between   the  
condition  sine  qua  non  the  registration  of  the  trade  mark   Federal   Republic   of   Germany   and   the   Philippines   be  
of   the   suing   foreign   corporation   with   the   Philippine   pleaded;  that  such  reciprocity  arrangement  is  embodied  
Patent   Office   or,   in   the   least,   that   it   be   an   asignee   of   in   and   supplied   by   the   Union   Convention   for   the  
such   registered   trademark.   The   said   section   further   Protection   of   Industrial   Property   Paris   Convention)   to  
requires   that   the   country,   of   which   the   plaintiff   foreign   which   both   the   Philippines   and   Federal   Republic   of  
corporation   or   juristic   person   is   a   citizen   or   domicilliary,   Germany   are   signatories   and   that   since   the   Paris  
grants   to   Filipino   corporations   or   juristic   entities   the   Convention   is   a   treaty   which,   pursuant   to   our  
same   reciprocal   treatment,   either   thru   treaty,   Constitution,   forms   part   of   the   law   of   the   land,   our  
convention  or  law,   courts   are   bound   to   take   judicial   notice   of   such   treaty,  
  and,   consequently,   this   fact   need   not   be   averred   in   the  
All   that   is   alleged   in   private   respondent's   complaint   is   complaint.  The  Court  agreed.  
that  it  is  a  foreign  corporation.  Such  bare  averment  not    
only   fails   to   comply   with   the   requirements   imposed   by  
the   aforesaid   Section   21-­‐A   but   violates   as   well   the  
directive   of   Section   4,   Rule   8   of   the   Rules   of   Court   that  

o 97
Katrina Michelle Mancao
 
LA  CHEMISE  LACOSTE  V.  FERNANDEZ  (1984)   Corporation   The   latter   is   an   independent   entity   which  
Doctrine:   buys   and   then   markets   not   only   products   of   the  
Hemandas   argues   in   his   comment   on   the   petition   for   petitioner  but  also  many  other  products  bearing  equally  
certiorari  that  the  petitioner  being  a  foreign  corporation   well-­‐known   and   established   trademarks   and   tradenames.  
failed   to   allege   essential   facts   bearing   upon   its   capacity   in   other   words,   Rustan   is   not   a   mere   agent   or   conduit   of  
to  sue  before  Philippine  courts.  He  states  that  not  only  is   the  petitioner.  
the  petitioner  not  doing  business  in  the  Philippines  but  it    
also  is  not  licensed  to  do  business  in  the  Philippines.  He   The   rules   and   regulations   promulgated   by   the   Board   of  
also   cites   the   case   of   Leviton   Industries   v.   Salvador   to   Investments   pursuant   to   its   rule-­‐making   power   under  
support   his   contention   The   Leviton   case,   however,   Presidential   Decree   No.   1789,   otherwise   known   as   the  
involved   a   complaint   for   unfair   competition   under   Omnibus   Investment   Code,   support   a   finding   that   the  
Section  21-­‐A  of  Republic  Act  No.  166  which  provides:   petitioner   is   not   doing   business   in   the   Philippines.   Rule   I,  
  Sec.   1   (g)   of   said   rules   and   regulations   defines   "doing  
Sec.  21  —  A.  Any  foreign  corporation  or  juristic   business"  as  one"  which  includes,  inter  alia:  
person  to  which  a  mark  or  tradename  has  been    
registered  or  assigned  under  this  Act  may  bring   (1)   A   foreign   firm   which   does   business  
an   action   hereunder   for   infringement,   for   through   middlemen   acting   on   their   own   names,  
unfair   competition,   or   false   designation   of   such   as   indentors,   commercial   brokers   or  
origin   and   false   description,   whether   or   not   it   commission   merchants,   shall   not   be   deemed  
has   been   licensed   to   do   business   in   the   doing   business   in   the   Philippines.   But   such  
Philippines   under   Act   numbered   Fourteen   indentors,   commercial   brokers   or   commission  
Hundred   and   Fifty-­‐Nine,   as   amended,   merchants   shall   be   the   ones   deemed   to   be  
otherwise   known   as   the   Corporation   Law,   at   doing  business  in  the  Philippines.  
the  time  it  brings  the  complaint;  Provided,  That    
the   country   of   which   the   said   foreign   (2)   Appointing   a   representative   or  
corporation   or   juristic   person   is   a   citizen,   or   in   distributor   who   is   domiciled   in   the   Philippines,  
which   it   is   domiciled,   by   treaty,   convention   or   unless  said  representative  or  distributor  has  an  
law,   grants   a   similar   privilege   to   corporate   or   independent   status,   i.e.,   it   transacts   business   in  
juristic  persons  of  the  Philippines.   its   name   and   for   its   account,   and   not   in   the  
  name   or   for   the   account   of   a   principal   Thus,  
We   held   that   it   was   not   enough   for   Leviton,   a   foreign   where  a  foreign  firm  is  represented  by  a  person  
corporation  organized  and  existing  under  the  laws  of  the   or   local   company   which   does   not   act   in   its  
State   of   New   York,   United   States   of   America,   to   merely   name   but   in   the   name   of   the   foreign   firm   the  
allege   that   it   is   a   foreign   corporation.   It   averred   in   latter  is  doing  business  in  the  Philippines.  
Paragraph   2   of   its   complaint   that   its   action   was   being    
filed   under   the   provisions   of   Section   21-­‐A   of   Republic   Act   Applying   the   above   provisions   to   the   facts   of   this   case,  
No.   166,   as   amended.   Compliance   with   the   requirements   we   find   and   conclude   that   the   petitioner   is   not   doing  
imposed   by   the   abovecited   provision   was   necessary   business   in   the   Philippines.   Rustan   is   actually   a  
because   Section   21-­‐A   of   Republic   Act   No.   166   having   middleman   acting   and   transacting   business   in   its   own  
explicitly   laid   down   certain   conditions   in   a   specific   name  and  or  its  own  account  and  not  in  the  name  or  for  
proviso,   the   same   must   be   expressly   averred   before   a   the  account  of  the  petitioner.  
successful   prosecution   may   ensue.   It   is   therefore,    
necessary   for   the   foreign   corporation   to   comply   with   But  even  assuming  the  truth  of  the  private  respondent's  
these   requirements   or   aver   why   it   should   be   exempted   allegation   that   the   petitioner   failed   to   allege   material  
from  them,  if  such  was  the  case.  The  foreign  corporation   facts   in   its   petition   relative   to   capacity   to   sue,   the  
may   have   the   right   to   sue   before   Philippine   courts,   but   petitioner   may   still   maintain   the   present   suit   against  
our   rules   on   pleadings   require   that   the   qualifying   respondent   Hemandas.   As   early   as   1927,   this   Court   was,  
circumstances   necessary   for   the   assertion   of   such   right   and   it   still   is,   of   the   view   that   a   foreign   corporation   not  
should  first  be  affirmatively  pleaded.   doing  business  in  the  Philippines  needs  no  license  to  sue  
  before   Philippine   courts   for   infringement   of   trademark  
In   contradistinction,   the   present   case   involves   a   and   unfair   competition.   Thus,   in   Western   Equipment   and  
complaint   for   violation   of   Article   189   of   the   Revised   Supply   Co.   v.   Reyes   (51   Phil.   115),   this   Court   held   that   a  
Penal  Code.  The  Leviton  case  is  not  applicable.   foreign   corporation   which   has   never   done   any   business  
  in   the   Philippines   and   which   is   unlicensed   and  
In  the  present  case,  however,  the  petitioner  is  a  foreign   unregistered   to   do   business   here,   but   is   widely   and  
corporation   not   doing   business   in   the   Philippines.   The   favorably   known   in   the   Philippines   through   the   use  
marketing   of   its   products   in   the   Philippines   is   done   therein   of   its   products   bearing   its   corporate   and  
through   an   exclusive   distributor,   Rustan   Commercial   tradename,  has  a  legal  right  to  maintain  an  action  in  the  

98 z
Intellectual Property Law

Philippines   to   restrain   the   residents   and   inhabitants   Protection  of  Industrial  Property  to  which  the  Philippines  
thereof   from   organizing   a   corporation   therein   bearing   and   France   are   parties.   We   are   simply   interpreting   and  
the   same   name   as   the   foreign   corporation,   when   it   enforcing   a   solemn   international   commitment   of   the  
appears   that   they   have   personal   knowledge   of   the   Philippines  embodied  in  a  multilateral  treaty  to  which  we  
existence   of   such   a   foreign   corporation,   and   it   is   are  a  party  and  which  we  entered  into  because  it  is  in  our  
apparent   that   the   purpose   of   the   proposed   domestic   national  interest  to  do  so.  
corporation   is   to   deal   and   trade   in   the   same   goods   as    
those  of  the  foreign  corporation.   REMEDIES  
  1) Administrative  
We  further  held:   a. Opposition  
  b. Cancellation  
That   company   is   not   here   seeking   to   enforce   c. Intellectual   Property   right   Violation   (IPV   –  
any   legal   or   control   rights   arising   from,   or   new  to  IP  Code)  
growing   out   of,   any   business   which   it   has   d. Customs:   Border   enforcement   (for  
transacted   in   the   Philippine   Islands.   The   sole   counterfeit  goods)  
purpose   of   the   action   is   to   protect   its   2) Criminal  
reputation,   its   corporate   name,   its   goodwill,   a. Infringement  
whenever   that   reputation,   corporate   name   or   b. Unfair  competition  
goodwill   have,   through   the   natural   c. False  designation  of  origin  
development   of   its   trade,   established   d. Section   162   –   no   specific   criminal   remedy,  
themselves.'   And   it   contends   that   its   rights   to   but  you  can  file  for  perjury  
the   use   of   its   corporate   and   trade   name   is   a   3) Civil  
property   right,   a   right   in   rem,   which   it   may   a. Infringement  
assert  and  protect  against  all  the  world,  in  any   b. Unfair  competition  
of  the  courts  of  the  world-­‐even  in  jurisdictions   c. False  designation  of  origin  
where   it   does   not   transact   business-­‐just   the   d. False   or   fraudulent   declaration   (most  
same   as   it   may   protect   its   tangible   property,   useless)  
real   or   personal,   against   trespass,   or    
conversion.   That   point   is   sustained   by   the   Who   has   the   remedy?   All   persons   who   satisfies   the  
authorities,   and   is   well   stated   in   Hanover   Star   requisite  in  Section  3.  
Mining   Co.   v.   Allen   and   Wheeler   Co.   in   which   -­‐ Plead   capacity   to   sue.   Otherwise,   petition   will  
the  syllabus  says,  “Since  it  is  the  trade  and  not   be  vulnerable  to  a  Motion  to  Dismiss.  
the  mark  that  is  to  be  protected,  a  trade-­‐mark   -­‐ Easiest:  Allege  that  it  is  a  domiciliary  of  a  state  
acknowledges   no   territorial   boundaries   of   that   is   a   party   to   a   Convention   or   Treaty   with  
municipalities  or  states  or  nations,  but  extends   the  Philippines.  This  is  easier  because  then,  the  
to   every   market   where   the   trader's   goods   have   court  can  take  judicial  notice.  
become  known  and  Identified  by  the  use  of  the   -­‐ If  foreign  corporation  not  registered  BUT  doing  
mark.”   business  in  the  Philippines,  no  remedy.  General  
  rule   in   Corporation   Code   applies.   Foreign  
Our   recognizing   the   capacity   of   the   petitioner   to   sue   is   corporation  not  given  a  remedy.  
not   by   any   means   novel   or   precedent   setting.   Our    
jurisprudence   is   replete   with   cases   illustrating   instances   Note:   Causes   of   action   in   criminal   and   civil  
when   foreign   corporations   not   doing   business   in   the   remedies   may   also   be   filed   as   IPV.   IPV   is   broad  
Philippines   may   nonetheless   sue   in   our   courts.   In   East   enough   to   cover   infringement,   unfair  
Board   Navigation   Ltd,   v.   Ysmael   and   Co.,   Inc.,   we   competition,   false   designation   of   origin,   false  
recognized   a   right   of   foreign   corporation   to   sue   on   or  fraudulent  declaration.  
isolated   transactions.   In   General   Garments   Corp.   v.    
Director   of   Patents,   we   sustained   the   right   of   Puritan   1. Administrative  
Sportswear   Corp.,   a   foreign   corporation   not   licensed   to  
 
do   and   not   doing   business   in   the   Philippines,   to   file   a  
petition   for   cancellation   of   a   trademark   before   the  
a. Cancellation  
Patent  Office.   proceedings  
   
In   upholding   the   right   of   the   petitioner   to   maintain   the   SECTION  151.  CANCELLATION.  -­‐    
present  suit  before  our  courts  for  unfair  competition  or   151.1.   A   PETITION   TO   CANCEL   a   registration   of   a   mark  
infringement  of  trademarks  of  a  foreign  corporation,  we  
under   this   Act   may   be   filed   with   the   Bureau   of   Legal  
are   moreover   recognizing   our   duties   and   the   rights   of  
Affairs   by   any   person   who   believes   that   he   is   or   will   be  
foreign   states   under   the   Paris   Convention   for   the  

o 99
Katrina Michelle Mancao
 
damaged  by  the  registration  of  a  mark  under  this  Act  as   152.2.   The   use   of   the   mark   in   a   form   different   from  
follows:   the  form  in  which  it  is  registered,  which  does  not  alter  its  
  distinctive   character,   shall   not   be   ground   for  
(a)   Within   five   (5)   years   from   the   date   of   the   cancellation   or   removal   of   the   mark   and   shall   not  
registration  of  the  mark  under  this  Act.   diminish  the  protection  granted  to  the  mark.  
   
(b)   AT  ANY  TIME,  if  the  registered  mark  becomes   152.3.   The   use   of   a   mark   in   connection   with   one   or  
the   generic   name   for   the   goods   or   services,  or  a  portion   more   of   the   goods   or   services   belonging   to   the   class   in  
thereof,   for   which   it   is   registered,   or   has   been   respect  of  which  the  mark  is  registered  shall  prevent  its  
abandoned,   or   its   registration   was   obtained   cancellation   or   removal   in   respect   of   all   other   goods   or  
fraudulently  or  contrary  to  the  provisions  of  this  Act,  or   services  of  the  same  class.  
if   the   registered   mark   is   being   used   by,   or   with   the    
permission   of,   the   registrant   so   as   to   misrepresent   the   152.4.   The  use   of   a   mark   by   a   company   related   with  
source  of  the  goods  or  services  on  or  in  connection  with  
the   registrant   or   applicant   shall   inure   to   the   latter's  
which  the  mark  is  used.  If  the  registered  mark  becomes  
benefit,   and   such   use   shall   not   affect   the   validity   of   such  
the   generic   name   for   less   than   all   of   the   goods   or  
mark  or  of  its  registration:  PROVIDED,  That  such  mark  is  
services  for  which  it  is  registered,  a  petition  to  cancel  the  
not  used  in  such  manner  as  to  deceive  the  public.  If  use  
registration  for  only  those  goods  or  services  may  be  filed.  
of   a   mark   by   a   person   is   controlled   by   the   registrant   or  
A   registered   mark   shall   not   be   deemed   to   be   the   generic  
applicant   with   respect   to   the   nature   and   quality   of   the  
name   of   goods   or   services   solely   because   such   mark   is  
goods  or  services,  such  use  shall  inure  to  the  benefit  of  
also   used   as   a   name   of   or   to   identify   a   unique   product   or  
the  registrant  or  applicant.  
service.   The   primary  significance  of  the  registered  mark  
 
to   the   relevant   public  rather  than  purchaser  motivation  
shall   be   the   TEST   FOR   DETERMINING   WHETHER   THE  
SECTION   153.   REQUIREMENTS   OF   PETITION;   NOTICE   AND  
REGISTERED   MARK   HAS  BECOME   THE   GENERIC  name  of   HEARING.  –    
goods   or   services   on   or   in   connection   with   which   it   has   Insofar   as   applicable,   the   petition   for   cancellation   shall  
been  used.   be   in   the   same   form   as   that   provided   in   Section   1348  
  hereof,   and   notice   and   hearing   shall   be   as   provided   in  
(c)   At   any   time,   if   the   registered   owner   of   the   Section  1359  hereof.  
mark   without   legitimate   reason   fails   to   use   the   mark    
within   the   Philippines,   or   to   cause   it   to   be   used   in   the   SECTION  154.  CANCELLATION  OF  REGISTRATION.  –    
Philippines   by   virtue   of   a   license   during   an   If   the   Bureau   of   Legal   Affairs   finds   that   a   case   for  
uninterrupted  period  of  three  (3)  years  or  longer.   cancellation   has   been   made   out,   it   shall   order   the  
  cancellation   of   the   registration.   When   the   order   or  
151.2.   Notwithstanding   the   foregoing   provisions,   the   judgment  BECOMES  FINAL,  any  right  conferred  by  such  
court   or   the   administrative   agency   vested   with   registration   upon   the   registrant   or   any   person   in  
jurisdiction  to  hear  and  adjudicate  any  action  to  enforce   interest   of   record   shall   terminate.  Notice  of  cancellation  
the   rights   to   a   registered   mark   shall   likewise   exercise   shall  be  published  in  the  IPO  Gazette.  
jurisdiction  to  determine  whether  the  registration  of  said    
mark  may  be  cancelled  in  accordance  with  this  Act.  The  
filing   of   a   suit   to   enforce   the   registered   mark   with   the  
                                                                                                                                                       
proper  court  or  agency  shall  exclude  any  other  court  or   8
 Section   134.   Opposition.   –   x   x   x   Such   opposition   shall   be   in  
agency  from  assuming  jurisdiction  over  a  subsequently   writing   and   verified   by   the   oppositor   or   by   any   person   on   his  
filed   petition   to   cancel   the   same   mark.   On   the   other   behalf   who   knows   the   facts,   and   shall   specify   the   grounds   on  
hand,   the   earlier   filing   of   petition   to   cancel   the   mark   which   it   is   based   and   include   a   statement   of   the   facts   relied  
with   the   Bureau   of   Legal   Affairs   shall   not   constitute   a   upon.   Copies   of   certificates   of   registration   of   marks   registered  
prejudicial   question   that   must   be   resolved   before   an   in  other  countries  or  other  supporting  documents  mentioned  in  
action   to   enforce   the   rights   to   same   registered   mark   the   opposition   shall   be   filed   therewith,   together   with   the  
translation   in   English,   if   not   in   the   English   language.   For   good  
may  be  decided.  
cause  shown  and  upon  payment  of  the  required  surcharge,  the  
 
time  for  filing  an  opposition  may  be  extended  by  the  Director  of  
SECTION  152.  NON-­‐USE  OF  A  MARK  WHEN  EXCUSED.  –     Legal   Affairs,   who   shall   notify   the   applicant   of   such   extension.  
152.1.   Non-­‐use   of   a   mark   may   be   excused   if   caused   by   The   Regulations   shall   fix   the   maximum   period   of   time   within  
which  to  file  the  opposition.  
circumstances   arising   independently   of   the   will   of   the   9
 Section   135.   Notice   and   Hearing.   -­‐   Upon   the   filing   of   an  
trademark  owner.  LACK  OF  FUNDS  shall  not  excuse  non-­‐ opposition,   the   Office   shall   serve   notice   of   the   filing   on   the  
use  of  a  mark.   applicant,   and   of   the   date   of   the   hearing   thereof   upon   the  
  applicant   and   the   oppositor   and   all   other   persons   having   any  
right,  title  or  interest  in  the  mark  covered  by  the  application,  as  
appear  of  record  in  the  Office.  

100 z
Intellectual Property Law

SECTION   230.   EQUITABLE   PRINCIPLES   TO   GOVERN   brassiere,   does   not   affect   its   registrability   as   a  
trademark.”  
PROCEEDINGS.  –    
 
In  all  inter  partes  proceedings  in  the  Office  under  this  Act,  
There   being   no   evidence   of   use   of   the   mark   by   others  
the   equitable   principles   of   laches,   estoppel,   and  
before  1932,  or  that  appellee  abandoned  use  thereof,  the  
acquiescence   where   applicable,   may   be   considered   and   registration   of   the   mark   was   made   in   accordance   with  
applied.  
the   Trademark   Law.   Granting   that   appellant   used   the  
  mark   when   appellee   stopped   using   it   during   the   period  
SECTION  232.2.   of   time   that   the   Government   imposed   restrictions   on  
Unless   expressly   provided   in   this   Act   or   other   statutes,   importation   of   respondent's   brassiere   bearing   the  
appeals  from  decisions  of  administrative  officials  shall  be   trademark,   such   temporary   non-­‐use   did   not   affect   the  
provided  in  the  Regulations.   rights   of   appellee   because   it   was   occasioned   by  
  government   restrictions   and   was   not   permanent,  
ROMERO  V.  MAIDEN  FORM  BRASSIERE  CO.  (1964)   intentional,  and  voluntary.  
Refresher:    
Maiden   Form   filed   an   application   for   registration   of   the   To   work   an   abandonment,   the   disuse   must   be  
trademark  "Adagio"  for  the  brassieres  manufactured  by   permanent   and   not   ephemeral;   it   must   be  
it   with   the   Director   of   Patents.   In   its   application,   intentional   and   voluntary,   and   not   involuntary  
respondent   company   alleged   that   said   trademark   was   or   even   compulsory.   There   must   be   a  
first  used  by  it  in  the  United  States  on  October  26,  1937,   thorough-­‐going   discontinuance   of   any   trade-­‐
and   in   the   Philippines   on   August   31,   1946;   that   it   had   mark  use  of  the  mark  in  question.  
been   continuously   used   by   it   in   trade   in,   or   with   the    
Philippines   for   over   10   years;   that   said   trademark   "is   on   The   use   of   the   trademark   by   other   manufacturers   did  
the  date  of  this  application,  actually  used  by  respondent   not   indicate   an   intention   on   the   part   of   appellee   to  
company  on  the  following  goods,  classified  according  to   abandon  it.  
the  official  classification  of  goods  (Rule  82)   -­‐   Brassieres,    
Class   40";   and   that   said   trademark   is   applied   or   affixed   Non-­‐use   because   of   legal   restrictions   is   not   evidence   of  
by  respondent  to  the  goods  by  placing  thereon  a  woven   an   intent   to   abandon.   Non-­‐use   of   their   ancient   trade-­‐
label  on  which  the  trademark  is  shown.   mark   and   the   adoption   of   new   marks   by   the   Carthusian  
  Monks   after   they   had   been   compelled   to   leave   France  
Petitioner   filed   with   the   Director   a   petition   for   was   consistent   with   an   intention   to   retain   their   right   to  
cancellation  of  the  trademark  on  the  grounds  that  it  is  a   use   their   old   mark.   Abandonment   will   not   be   inferred  
common  descriptive  name  of  an  article  or  substance  on   from   a   disuse   over   a   period   of   years   occasioned   by  
which   the   patent   has   expired.   Petitioner   alleged   that   statutory  restrictions  on  the  name  of  liquor.  
said   trademark   has   not   become   distinctive   of    
respondent   company's   goods   or   business;   that   it   has   PHILIPPINE   NUT   INDUSTRY   V.   STANDARD   BRANDS,   INC.  
been  used  by  respondent  company  to  classify  the  goods   (1975)  
(the  brassieres)  manufactured  by  it,  in  the  same  manner  
Refresher:  
as   petitioner   uses   the   same;   that   said   trademark   has   Philippine   Nut,   a   domestic   corporation,   obtained   from  
been   used   by   petitioner   for   almost   6   years;   that   it   has   the  Patent  office  a  certificate  of  registration  covering  the  
become   a   common   descriptive   name;   and   that   it   is   not  
trademark   "Philippine   Planters   Cordial   Peanuts,"   the  
registered   in   accordance   with   the   requirements   of  
label  used  on  its  product  of  salted  peanuts.  
Section  37(a),  Chapter  XI  of  Republic  Act  No.  166.  
 
  Standard   Brands,   a   foreign   corporation,   filed   a   case   for  
Doctrine:  
the   cancellation   of   Philippine   Nut's   certificate   of  
The   evidence   shows   that   the   trademark   "Adagio"   is   a  
registration   on   the   ground   that   such   trademark   closely  
musical  term,  which  means  slowly  or  in  an  easy  manner,  
resembles,   and   is   confusingly   similar   to   its   trademark  
and  was  used  as  a  trademark  by  the  owners  thereof  (the   "Planters  Cocktail  Peanuts."  
Rosenthals  of  Maiden  Form  Co.,  New  York)  because  they    
are  musically  inclined.  Being  a  musical  term,  it  is  used  in  
Issue  1:  
an  arbitrary  (fanciful)  sense  as  a  trademark  for  brassieres  
The   first   argument   advanced   by   petitioner   which   We  
manufactured   by   respondent   company.   It   also   appears  
believe   goes   to   the   core   of   the   matter   in   litigation   is   that  
that   respondent   company   has,   likewise,   adopted   other   the   Director   of   Patents   erred   in   holding   that   the  
musical   terms   such   as   "Etude",   "Chansonette",  
dominant   portion   of   the   label   of   Standard   Brands   in   its  
"Prelude",  "Over-­‐ture",  and  "Concerto",  to  identify,  as  a  
cans   of   salted   peanuts   consists   of   the   word   PLANTERS  
trademark,  the  different  styles  or  types  of  its  brassieres.  
which  has  been  used  in  the  label  of  Philippine  Nut  for  its  
As   respondent   Director   pointed   out,   "the   fact   that   said   own   product.   According   to   petitioner,   PLANTERS   cannot  
mark   is   used   also   to   designate   a   particular   style   of   be   considered   as   the   dominant   feature   of   the  

o 101
Katrina Michelle Mancao
 
trademarks   in   question   because   it   is   a   mere   descriptive   word   or   phrase   has   come   to   mean   that   the   article   was  
term,   an   ordinary   word   which   is   defined   in   Webster   his  product.  
International   Dictionary   as   "one   who   or   that   which    
plants  or  sows,  a  farmer  or  an  agriculturist."   The  applicability  of  the  doctrine  of  secondary  meaning  to  
  the   situation   now   before   Us   is   appropriate   because  
Held:   there  is  oral  and  documentary  evidence  showing  that  the  
We   find   the   argument   without   merit.   While   it   is   true   that   word  PLANTERS  has  been  used  by  and  closely  associated  
PLANTERS  is  an  ordinary  word,  nevertheless  it  is  used  in   with  Standard  Brands  for  its  canned  salted  peanuts  since  
the  labels  not  to  describe  the  nature  of  the  product,  but   1938   in   this   country.   Not   only   is   that   fact   admitted   by  
to   project   the   source   or   origin   of   the   salted   peanuts   petitioner   in   the   amended   stipulation   of   facts,   but   the  
contained   in   the   cans.   The   word   PLANTERS   printed   matter   has   been   established   by   testimonial   and  
across  the  upper  portion  of  the  label  in  bold  letters  easily   documentary   evidence   consisting   of   invoices   covering  
attracts   and   catches   the   eye   of   the   ordinary   consumer   the  sale  of  "PLANTERS  cocktail  peanuts".  In  other  words,  
and   it   is   that   word   and   none   other   that   sticks   in   his   mind   there   is   evidence   to   show   that   the   term   PLANTERS   has  
when  he  thinks  of  salted  peanuts.   become   a   distinctive   mark   or   symbol   insofar   as   salted  
  peanuts   are   concerned,   and   by   priority   of   use   dating   as  
In  cases  of  this  nature  there  can  be  no  better  evidence   far   back   as   1938,   respondent   Standard   Brands   has  
as  to  what  is  the  dominant  feature  of  a  label  and  as   to   acquired   a   preferential   right   to   its   adoption   as   its  
whether  there  is  a  confusing  similarity  in  the  contesting   trademark   warranting   protection   against   its   usurpation  
trademarks   than   the   labels   themselves.   A   visual   and   by  another.  Ubi  jus  ibi  remedium.  Where  there  is  a  right  
graphic   presentation   of   the   labels   will   constitute   the   there   is   a   remedy.   Standard   Brands   has   shown   the  
best  argument  for  one  or  the  other.   existence   of   a   property   right   and   respondent   Director,  
  has  afforded  the  remedy.  
It   is   true   that   there   are   other   words   used   such   as    
"Cordial"   in   petitioner's   can   and   "Cocktail"   in   Standard   Still   on   this   point,   petitioner   contends   that   Standard  
Brands',  which  are  also  prominently  displayed,  but  these   Brands'  use  of  the  trademark  PLANTERS  was  interrupted  
words   are   mere   adjectives   describing   the   type   of   during   the   Japanese   occupation   and   in   fact   was  
peanuts   in   the   labeled   containers   and   are   not   sufficient   discontinued   when   the   importation   of   peanuts   was  
to   warn   the   unwary   customer   that   the   two   products   prohibited   by   Central   Bank   regulations   effective   July   1,  
come   form   distinct   sources.   As   a   whole   it   is   the   word   1953,   hence   it   cannot   be   presumed   that   it   has   acquired   a  
PLANTERS   which   draws   the   attention   of   the   buyer   and   secondary   meaning.   We   hold   otherwise.   Respondent  
leads  him  to  conclude  that  the  salted  peanuts  contained   Director   correctly   applied   the   rule   that   non-­‐use   of   a  
in   the   two   cans   originate   from   one   and   the   same   trademark   on   an   article   of   merchandize   due   to   legal  
manufacturer.   In   fact,   when   a   housewife   sends   her   restrictions   or   circumstances   beyond   one's   control   is   not  
housemaid  to  the  market  to  buy  canned  salted  peanuts,   to  be  considered  as  an  abandonment.  
she  will  describe  the  brand  she  wants  by  using  the  word    
PLANTERS  and  not  "Cordial"  nor  "Cocktail".   ANCHOR  TRADING  CO.  V.  DIRECTOR  OF  PATENTS  (1956)  
  Doctrine:  
Issue  2:*   The   principal   legal   issue   here   is   whether   or   not   the  
What  is  next  submitted  by  petitioner  is  that  it  was  error   failure   of   the   respondent-­‐appellee   to   register   his  
for   respondent   Director   to   have   enjoined   it   from   using   opposition   to   the   petition   for   registration   in   due   time  
PLANTERS  in  the  absence  of  evidence  showing  that  the   estops   him   from   asking   for   the   cancellation   of   the  
term   has   acquired   secondary   meaning.   Petitioner,   certificate   of   registration   issued   in   favor   of   the  
invoking   American   jurisprudence,   asserts   that   the   first   petitioner-­‐appellant.  The  issue  should  be  resolved  in  the  
user   of   a   tradename   composed   of   common   words   is   negative.   The   only   consequence   resulting   from   a   late  
given  no  special  preference  unless  it  is  shown  that  such   filing   of   an   opposition   to   any   application   for  
words   have   acquired   secondary   meaning,   and   this,   registration   of   a   trademark   is   the   oppositor’s  
respondent   Standard   Brands   failed   to   do   when   no   relinquishment  of  the  privilege  given  to  him  by  laws  to  
evidence  was  presented  to  establish  that  fact.   object  to  such  registration,  but  such  cannot  prevent  him  
  from   asking   later   for   its   cancellation   when   there   are  
Held:   good  grounds  justifying  it.  
This   Court   held   that   the   doctrine   is   to   the   effect   that   a    
word   or   phrase   originally   incapable   of   exclusive  
THE  C LOROX  CO.  V.  DIRECTOR  OF  PATENTS  (1967)  
appropriation  with  reference  to  an  article  on  the  market,  
Refresher:  
because   geographically   or   otherwise   descriptive,   might  
nevertheless   have   been   used   so   long   and   so   exclusively   On   April   7,   1959,   respondent   Go   Siu   Gian   filed   with   the  
Patent   Office   an   application   for   registration   of   the  
by  one  producer  with  reference  to  his  article  that,  in  that  
trademark  "OLDROX,"  with  an  accompanying  statement  
trade   and   to   that   branch   of   the   purchasing   public,   the  
that  he  is  a  citizen  of  China,  residing  and  doing  business  

102 z
Intellectual Property Law

in   the   Philippines   at   838   Folgueras   St.,   Manila;   that   he   choose  which  remedy  it  deems  best  for  the  protection  of  
has   adopted   the   trademark   "OLDROX"   for   his   goods   its  rights.  
(whitening   agent   for   bleaching)   in   trade   and   commerce    
in   the   country;   and   that   said   trademark,   shown   on   WOLVERINE   WORLDWIDE,   INC.   V.   CA   AND   LOLITO   CRUZ  
printed  labels  affixed  to  the  goods,  or  to  the  containers  
(1989)  
thereof,   has   been   used   by   him   since   February   1,   1959.  
Refresher:  
The   application   was   allowed   and   published   in   the   issue  
of   the   Official   Gazette   dated   April   25,   1960,   which   was   On   February   8,   1984,   the   petitioner,   a   foreign  
corporation  organized  and  existing  under  the  laws  of  the  
released  for  circulation  on  August  22,  1960.  
United   States,   brought   a   petition   before   the   Philippine  
 
Patent   Office,   docketed   as   Inter   Partes   Case   No.   1807,  
On   September   21   of   the   same   year,   or   within   30   days  
for   the   cancellation   of   Certificate   of   Registration   No.  
from  the  date  of  its  publication  in  the  Official  Gazette,  an  
24986-­‐B   of   the   trademark   HUSH   PUPPIES   and   DOG  
unverified  opposition  to  the  application  was  filed  by  the  
DEVICE   issued   to   the   private   respondent,   a   Filipino  
law   firm   Lichauco,   Picazo   and   Agcaoili   in   behalf   of   the  
citizen.  
Clorox  Company,  herein  petitioner.  
 
 
Doctrine:   In  support  of  its  petition  for  cancellation,  the  petitioner  
alleged,   inter   alia,   that   it   is   the   registrant   of   the  
The  rule  is  well  settled  that  courts  may  vacate  judgments  
internationally   known   trademark   HUSH   PUPPIES   and   the  
and   grant   new   trials   or   enter   new   judgments   on   the  
DEVICE   of   a   Dog   in   the   United   States   and   in   other  
grounds  of  error  in  fact  or  in  law.  They  have  no  power,  of  
countries  which  are  members  of  the  Paris  Convention  for  
course,   to   vacate   judgments   after   they   have   become  
the  Protection  of  Industrial  Property;  that  the  goods  sold  
final,  in  the  sense  that  the  party  in  whose  favor  they  are  
by  the  private  respondent,  on  the  one  hand,  and  by  the  
rendered   is   entitled   as   of   right,   to   have   execution  
petitioner,   on   the   other   hand,   belong   to   the   same   class  
thereon,   but   prior   thereto,   the   courts   have   plenary  
such   that   the   private   respondent's   use   of   the   same  
control   over   the   proceedings   including   the   judgment,  
and   in   the   exercise   of   a   sound   judicial   discretion,   may   trademark  in  the  Philippines  (which  is  a  member  of  said  
Paris   Convention)   in   connection   with   the   goods   he   sells  
take  such  proper  action  in  this  regard  as  truth  and  justice  
constitutes   an   act   of   unfair   competition,   as   denied   in   the  
may   require.   The   order   of   herein   respondent   dismissing  
Paris  Convention.  
the   opposition   of   petitioner   to   the   registration   of   the  
 
trademark   in   question   may   amount   to   considerable  
Subsequently,  the  private  respondent  moved  to  dismiss  
injustice   to   the   opposer   Clorox   Company,   the   order  
the  petition  on  the  ground  of  res  judicata,  averring  that  
having   been   entered   not   upon   the   merits   of   the  
in  1973,  or  more  than  ten  years  before  this  petition  (Inter  
controversy;   and   the   possibility   of   such   serious  
Partes  Case  No.  1807)  was  filed,  the  same  petitioner  filed  
consequences  necessitates  a  careful  examination  of  the  
grounds   upon   which   it   requests   that   the   order   be   set   two   petitions   for   cancellation   (Inter   Partes   Cases   Nos.  
700   and   701)   and   was   a   party   to   an   interference  
aside.   It   must   be   remembered   that   the   only   discretion  
proceeding   (Inter   Partes   Case   No.   709),   all   of   which  
conferred   upon   officers   is   a   legal   discretion,   and   when  
involved   the   trademark   HUSH   PUPPIES   and   DEVICE,  
anything  is  left  to   any  officer  to  be  done  according  to  his  
before   the   Philippine   Patent   Office.   The   Director   of  
discretion,   the   law   intends   it   to   be   done   with   a   sound  
Patents  had  ruled  in  all  three  inter  parties  cases  in  favor  
discretion   and   according   to   law.   And   when,   as   in   this  
of   Ramon   Angeles,   the   private   respondent's  
case,   the   allegation   of   the   pleading   clearly   show  
predecessor-­‐in-­‐interest,  to  wit:  
circumstances   constituting   mistake   and   excusable  
 
negligence   which   are   grounds   for   a   motion   for  
reconsideration   of   the   order   in   question,   a   dismissal   of   WHEREFORE,   for   all   the   foregoing  
considerations,  
the   motion   and   a   denial   of   the   relief   sought   upon   the  
 
flimsy   excuse   that   the   same   was   filed   as   a   petition   for  
1.   The   petitions   seeking   cancellation   of  
relief,  will  amount  to  an  abuse  of  that  discretion.  Neither  
may   we   consider   the   argument   of   herein   respondent   Registration   Nos.   SR-­‐1099   and   SR-­‐1526,  
respectively,   are   both   denied   and   accordingly  
that   the   petitioner   is   not   totally   deprived   of   its   right   to  
DISMISSED;  
question   the   registration   of   the   trademark   in   question  
 
because   it   may   still   pursue   a   cancellation   proceeding  
2.   Respondent-­‐Registrant/Junior   Party-­‐
under   Sections   17   to   19   of   Republic   Act   No.   166,   and  
Rules   191   to   197   of   the   Rules   of   Practice   in   Trademark   Applicant,   Roman   Angeles,   is   hereby   adjudged  
as  the  prior  user  and  adopter  of  the  trademark  
Cases.   The   opposition   to   a   registration   and   the   petition  
HUSH  PUPPIES  &  DEVICE,  under  Appl.  Serial  No.  
for   cancellation   are   alternative   proceedings   which   a  
17174,   and   therefore,   the   same   given   due  
party  may  avail  of  according  to  his  purposes,  needs,  and  
predicaments,   and   herein   petitioner   has   the   right   to   course;  and  
 

o 103
Katrina Michelle Mancao
 
3.   Registration   No.   14969   of   Dexter   Sales   Between  the  earlier  petitions  and  the  present  one  there  
Company,   assignor   to   Wolverine   Worldwide,   is   substantial   identity   of   parties,   subject   matter,   and  
Inc.,   covering   the   trademark   HUSH   PUPPIES   &   cause  of  action.  
Representation   of   a   Dogie   Head,   is   hereby    
CANCELLED.   The   petitioner   in   all   of   these   cases   is   Wolverine  
  Worldwide,  Inc.  The  respondent-­‐registrant  in  this  case  is  
Doctrine:   the  assignee  of  Randelson  Agro-­‐Industrial  Development,  
The  Court  has  repeatedly  held  that  for  a  judgment  to  be   Inc.  (formerly  known  as  Randelson  Shoes,  Inc.)  which  in  
a  bar  to  a  subsequent  case,  the  following  requisites  must   turn,   acquired   its   right   from   Ramon   Angeles,   the   original  
concur:   (1)   it   must   be   a   final   judgment;   (2)   the   court   respondents-­‐registrant.  
which   rendered   it   had   jurisdiction   over   the   subject    
matter  and  the  parties;  (3)  it  must  be  a  judgment  on  the   As  regards  the  subject  matter,  all  of  these  cases  refer  to  
merits;  and  (4)  there  must  be  Identity  between  the  two   the   cancellation   of   registration   of   the   trademark   HUSH  
cases,   as   to   parties,   —   subject   matter,   and   cause   of   PUPPIES  and  DEVICE  of  a  Dog.  
action.    
  Finally,   there   is   identity   of   cause   of   action,   which   is   the  
Contrary   to   the   petitioner's   assertion,   the   judgment   in   alleged   wrongful   or   erroneous   registration   of   the  
Inter  Partes  Cases  Nos.  700,  701,  and  709  had  long  since   trademark.  
become   final   and   executory.   That   Sec.   17   of   Republic   Act    
166,   also   known   as   the   Trademark   Law,   allows   the   SHANGRILA  V.  CA  (1999)  
cancellation   of   a   registered   trademark   is   not   a   valid   Court  decision:  
premise   for   the   petitioner's   proposition   that   a   decision   The   earlier   institution   of   an   inter   partes   case   for  
granting   registration   of   a   trademark   cannot   be   imbued   cancellation  of  a  registered  service  mark  and  device/logo  
with   the   character   of   absolute   finality   as   is   required   in   with   BPTTT   cannot   effectively   bar   the   subsequent   filing  
res   judicata.   A   judgment   or   order   is   final,   as   to   give   it   the   of  an  infringement  case  by  the  registrant.    
authority  of  res  judicata,  if  it  can  no  longer  be  modified    
by  the  court  issuing  it  or  by  any  other  court.  In  the  case   REASON:   The   certificate   of   registration   upon   which   the  
at   bar,   the   decision   of   the   Court   of   Appeals   affirming   infringement   case   is   based   remains   valid   and   subsisting  
that  of  the  Director  of  Patents,  in  the  cancellation  cases   as  long  as  it  has  not  been  cancelled  by  the  Bureau  or  by  
filed   in   1973,   was   never   appealed   to   us.   Consequently,   an   infringement   court.   As   such,   the   registration  
when  the  period  to  appeal  from  the  Court  of  Appeals  to   continues   as   prima   facie   evidence   of   the   validity   of  
this   Court   lapsed,   with   no   appeal   having   been   perfected,   registration,   registrant’s   ownership   of   the   mark   and  
the   foregoing   judgment   denying   cancellation   of   tradename  and  of  the  registrant’s  exclusive  right  to  use  
registration   in   the   name   of   private   respondent's   the   same   in   connection   with   the   goods,   business   or  
predecessor-­‐in-­‐interest   but   ordering   cancellation   of   services  specified  in  the  certificate.    
registration  in  the  name  of  the  petitioner's  predecessor-­‐  
in-­‐interest,  became  the  settled  law  in  the  case.   In   the   same   light   that   the   infringement   case   can   and  
  should   proceed   independently   from   the   cancellation  
It   must   be   stressed   anew   that,   generally,   the   case   with   the   Bureau   so   as   to   afford   the   owner   of  
fundamental  principle  of  res  judicata  applies  to  all  cases   certificates   of   registration   redress   and   injunctive   reliefs,  
and   proceedings   in   whatever   form   they   may   be.   We   so   must   the   cancellation   with   the   BPTTT   continue  
now   expressly   affirm   that   this   principle   applies,   in   the   independently   from   the   infringement   case   so   as   to  
appropriate   cases,   to   proceedings   for   cancellation   of   determine  whether  a  registered  mark  may  ultimately  be  
trademarks   before   the   Philippine   Patent   Office   (now   cancelled.    
Bureau   of   Patents,   Trademarks   and   Technology    
Transfer).   To  provide  a  judicious  resolution  of  the  issues,  the  Court  
  finds   it   apropos   to   order   the   suspension   of   the  
Undoubtedly,  final  decisions,  orders,  and  resolutions,  of   proceedings   before   the   Bureau   pending   final  
the   Director   of   Patents   are   clothed   with   a   judicial   determination  of  the  infringement  case,  where  the  issue  
character  as  they  are,  in  fact,  reviewable  by  the  Court  of   of  validity  of  the  registration  of  the  subject  TM  and  logo  
Appeals  and  by  us.   in  the  name  of  Developers  group  was  passed  upon.    
   
The  subject  judgment  is  undeniably  on  the  merits  of  the   With   the   decision   of   the   RTC   upholding   the   validity   of  
case,  rendered  after  both  parties  and  actually  submitted   registration  of  the  service  mark  “Shangri-­‐La”  and  S  logo  
their  evidence.   in   the   name   of   Developers   Group,   the   cancellation   case  
  filed   with   the   Bureau   becomes   moot.   To   allow   the  
Bureau  to  proceed  with  the  cancellation  case  would  lead  

104 z
Intellectual Property Law

to   a   possible   result   contradictory   to   what   the   RTC   has   voluntary   assurance   may   include   one   or   more   of   the  
rendered,  albeit  the  same  is  on  appeal.     following:  
   
SUPERIOR   COMMERCIAL   ENTERPRISES   V.   KUNNAN   (1)   An   assurance   to   comply   with   the  
provisions   of   the   intellectual   property   law  
ENTERPRISES  (2010)  
violated;  
Refresher:  
 
Sec.   22   RA   166   provides   that   only   a   registrant   can   file   a   (2)   An   assurance   to   refrain   from  
case   for   infringement.   Corollary   to   this,   Sec.   19   RA   166  
engaging   in   unlawful   and   unfair   acts   and  
provides   that   nay   right   conferred   upon   the   registration  
practices  subject  of  the  formal  investigation;  
under   the   provision   of   RA   166   terminates   when   the  
 
judgment  or  order  of  cancellation  has  become  final.    
(3)   An  assurance  to  recall,  replace,  repair,  
 
or   refund   the   money   value   of   defective   goods  
Hence,   the   cancellation   of   a   registration   of   TM   has   the  
distributed  in  commerce;  and  
effect   of   depriving   the   registrant   of   protection   from  
 
infringement   the   moment   judgment   or   order   of  
(4)   An   assurance   to   reimburse   the  
cancellation  has  become  final.     complainant  the  expenses  and  costs  incurred  in  
 
prosecuting   the   case   in   the   Bureau   of   Legal  
In  trademark  infringement,  title  to  TM  is  indispensable  to  
Affairs.  
a   valid   cause   of   action   and   such   title   is   shown   by   its  
 
certificate  of  registration.    
The   Director   of   Legal   Affairs   may   also   require  
 
the   respondent   to   submit   periodic   compliance  
In   the   absence   of   any   inequitable   conduct   on   the   part   of  
reports   and   file   a   bond   to   guarantee  
the  manufacturer,  an  exclusive  distributor  who  employs  
compliance  of  his  undertaking;  
the  TM  of  the  manufacturer  does  not  acquire  proprietary  
 
rights   of   the   manufacturer   and   a   registration   of   the   TM  
(iii)   The   condemnation   or   seizure   of   products  
by   the   distributor   as   such   belongs   to   the   manufacturer,  
which   are   subject   of   the   offense.   The   goods   seized  
provided   the   fiduciary   relationship   does   not   terminate  
hereunder  shall  be  disposed  of  in  such  manner  as  may  be  
before  application  for  registration  is  filed.  
deemed   appropriate   by   the   Director   of   Legal   Affairs,  
 
such   as   by   sale,   donation   to   distressed   local  
b. Intellectual   property   governments   or   to   charitable   or   relief   institutions,  
rights  violations   exportation,   recycling   into   other   goods,   or   any  
  combination   thereof,   under   such   guidelines   as   he   may  
SECTION  10.2.  THE  BUREAU  OF  LEGAL  AFFAIRS   provide;  
 
(a)   Exercise   original   jurisdiction   in   administrative  
(iv)   The   forfeiture   of   paraphernalia   and   all   real  
complaints   for   violations   of   laws   involving   intellectual  
and   personal   properties   which   have   been   used   in   the  
property   rights:   Provided,   That   its   jurisdiction   is   limited  
commission  of  the  offense;  
to  complaints  where  the  total  damages  claimed  are  not  
 
less   than   Two   hundred   thousand   pesos   (P200,000):  
(v)   The   imposition   of   administrative   fines   in   such  
Provided   further,   That   availment   of   the   provisional  
amount   as   deemed   reasonable   by   the   Director   of   Legal  
remedies   may   be   granted   in   accordance   with   the   Rules  
Affairs,   which   shall   in   no   case   be   less   than   Five   thousand  
of   Court.   The   Director   of   Legal   Affairs   shall   have   the  
pesos   (P5,000)   nor   more   than   One   hundred   fifty  
power   to   hold   and   punish   for   contempt   all   those   who  
thousand   pesos   (P150,000).   In   addition,   an   additional  
disregard   orders   or   writs   issued   in   the   course   of   the  
fine   of   not   more   than   One   thousand   pesos   (P1,000)   shall  
proceedings.    
be  imposed  for  each  day  of  continuing  violation;  
 
 
(b)   After   formal   investigation,   the   Director   for  
(vi)   The   cancellation   of   any   permit,   license,  
Legal   Affairs   may   impose   one   (1)   or   more   of   the  
authority,  or  registration  which  may  have  been  granted  
following  administrative  penalties:  
by   the   Office,   or   the   suspension   of   the   validity   thereof  
 
for   such   period   of   time   as   the   Director   of   Legal   Affairs  
(i)   The   issuance   of   a   cease  and  desist  order   which  
may   deem   reasonable   which   shall   not   exceed   one   (1)  
shall  specify  the  acts  that  the  respondent  shall  cease  and  
year;  
desist   from   and   shall   require   him   to   submit   a   compliance  
 
report   within   a   reasonable   time   which   shall   be   fixed   in  
(vii)   The   withholding   of   any   permit,   license,  
the  order;  
authority,   or   registration  which  is  being  secured  by  the  
 
respondent  from  the  Office;  
(ii)   The   acceptance   of   a   voluntary   assurance   of  
 
compliance  or  discontinuance  as  may  be  imposed.  Such  

o 105
Katrina Michelle Mancao
 
(viii)   The  assessment  of  damages;   Court  of  Appeals  held  that  BLA  did  not  have  jurisdiction  
  over   the   complaint   for   unfair   competition,   as   jurisdiction  
(ix)   Censure;  and   is   vested   in   the   regular   courts.   The   Supreme   Court  
  disagreed  with  the  Court  of  Appeals.  
(x)   Other  analogous  penalties  or  sanctions.    
  Section   10   of   the   Intellectual   Property   Code   specifically  
SECTION  232.  APPEALS.  –     identifies  the  functions  of  the  Bureau  of  Legal  Affairs.  
232.1.   Appeals   from   decisions   of   regular   courts   shall   be    
Unquestionably,  petitioner’s  complaint,  which  seeks  the  
governed   by   the   Rules   of   Court.   Unless   restrained   by   a  
cancellation   of   the   disputed   mark   in   the   name   of  
higher   court,   the   judgment   of   the   trial   court   shall   be  
respondent   Sehwani,   Incorporated,   and   damages   for  
executory   even   pending   appeal   under   such   terms   and  
violation  of  petitioner’s  intellectual  property  rights,  falls  
conditions  as  the  court  may  prescribe.  
within  the  jurisdiction  of  the  IPO  Director  of  Legal  Affairs.  
 
 
232.2.   Unless   expressly   provided   in   this   Act   or   other  
The  Intellectual  Property  Code  also  expressly  recognizes  
statutes,   appeals   from   decisions   of   administrative   the   appellate   jurisdiction   of   the   IPO   Director   General  
officials  shall  be  provided  in  the  Regulations.   over   the   decisions   of   the   IPO   Director   of   Legal   Affairs,   to  
  wit:  
IN-­‐N-­‐OUT  BURGER,  INC.  V.  SEHWANI  (2008)    
Refresher:   Section   7.   The   Director   General   and   Deputies  
Petitioner   IN-­‐N-­‐OUT   BURGER,   INC.,   a   business   entity   Director   General.   7.1   Fuctions.–The   Director  
incorporated  under  the  laws  of  California,  United  States   General   shall   exercise   the   following   powers  
(US)  of  America,  which  is  a  signatory  to  the  Convention   and  functions:  
of   Paris   on   Protection   of   Industrial   Property   and   the    
Agreement   on   Trade   Related   Aspects   of   Intellectual   x  x  x  x  
Property  Rights  (TRIPS).    Petitioner  is  engaged  mainly  in    
the   restaurant   business,   but   it   has   never   engaged   in   b)   Exercise   exclusive   appellate   jurisdiction   over  
business  in  the  Philippines.   all   decisions   rendered   by   the   Director   of   Legal  
  Affairs,  the  Director  of  Patents,  the  Director  of  
Respondents   Sehwani,   Incorporated   and   Benita   Frites,   Trademarks,   and   the   Director   of  
Inc.  are  corporations  organized  in  the  Philippines.   Documentation,   Information   and   Technology  
  Transfer   Bureau.   The   decisions   of   the   Director  
On   2   June   1997,   petitioner   filed   trademark   and   service   General   in   the   exercise   of   his   appellate  
mark  applications  with  the  Bureau  of  Trademarks  (BOT)   jurisdiction   in   respect   of   the   decisions   of   the  
of   the   IPO   for   "IN-­‐N-­‐OUT"   and   "IN-­‐N-­‐OUT   Burger   &   Director   of   Patents,   and   the   Director   of  
Arrow   Design."     Petitioner   later   found   out,   through   the   Trademarks  shall  be  appealable  to  the  Court  of  
Official   Action   Papers   issued   by   the     IPO   on   31   May   2000,   Appeals  in  accordance  with  the  Rules  of  Court;  
that   respondent   Sehwani,   Incorporated   had   already   and   those   in   respect   of   the   decisions   of   the  
obtained   Trademark   Registration   for   the   mark   "IN   N   Director   of   Documentation,   Information   and  
OUT  (the  inside  of  the  letter  "O"  formed  like  a  star)."  By   Technology   Transfer   Bureau   shall   be  
virtue   of   a   licensing   agreement,   Benita   Frites,   Inc.   was   appealable   to   the   Secretary   of   Trade   and  
able  to  use  the  registered  mark  of  respondent  Sehwani,   Industry;  
Incorporated.    
  The  Court  of  Appeals  erroneously  reasoned  that  Section  
Petitioner   eventually   filed   on   4   June   2001   before   the   10(a)  of  the  Intellectual  Property  Code,  conferring  upon  
Bureau   of   Legal   Affairs   (BLA)   of   the   IPO   an   the   BLA-­‐IPO   jurisdiction   over   administrative   complaints  
administrative   complaint   against   respondents   for   unfair   for  violations  of  intellectual  property  rights,  is  a  general  
competition   and   cancellation   of   trademark   registration.     provision,   over   which   the   specific   provision   of   Section  
Petitioner  averred  in  its  complaint  that  it  is  the  owner  of   163   of   the   same   Code,   found   under   Part   III   thereof  
the  trade  name  IN-­‐N-­‐OUT  and  the  following  trademarks:   particularly   governing   trademarks,   service   marks,   and  
(1)   "IN-­‐N-­‐OUT";   (2)   "IN-­‐N-­‐OUT   Burger   &   Arrow   Design";   tradenames,   must   prevail.   Proceeding   therefrom,   the  
and  (3)  "IN-­‐N-­‐OUT  Burger  Logo."    These  trademarks  are   Court   of   Appeals   incorrectly   concluded   that   all   actions  
registered   with   the   Trademark   Office   of   the   US   and   in   involving   trademarks,   including   charges   of   unfair  
various   parts   of   the   world,   are   internationally   well-­‐ competition,   are   under   the   exclusive   jurisdiction   of   civil  
known,  and  have  become  distinctive  of  its  business  and   courts.  
goods  through  its  long  and  exclusive  commercial  use.    
  Such  interpretation  is  not  supported  by  the  provisions  of  
Doctrine:  (jurisdiction  of  the  Bureau  of  Legal  Affairs)   the  Intellectual  Property  Code.  While  Section  163  thereof  

106 z
Intellectual Property Law

vests   in   civil   courts   jurisdiction   over   cases   of   unfair   officers   of   the   customs   service   in   enforcing   this  
competition,   nothing   in   the   said   section   states   that   the   prohibition,   any   person   who   is   entitled   to   the   benefits   of  
regular   courts   have   sole   jurisdiction   over   unfair   this   Act,   may   require   that   his   name   and   residence,   and  
competition   cases,   to   the   exclusion   of   administrative   the   name   of   the   locality   in   which   his   goods   are  
bodies.  On  the  contrary,  Sections  160  and  170,  which  are   manufactured,   a   copy   of   the   certificate   of   registration  
also   found   under   Part   III   of   the   Intellectual   Property   of   his   mark   or   trade   name,   to   be   recorded   in   books  
Code,  recognize  the  concurrent  jurisdiction  of  civil  courts   which   shall   be   kept   for   this   purpose   in   the   Bureau   of  
and   the   IPO   over   unfair   competition   cases.   These   two   Customs,   under   such   regulations   as   the   Collector   of  
provisions  read:   Customs   with   the   approval   of   the   Secretary   of   Finance  
  shall   prescribe,   and   may   furnish   to   the   said   Bureau  
Section   160.   Right   of   Foreign   Corporation   to   facsimiles  of  his  name,  the  name  of  the  locality  in  which  
Sue   in   Trademark   or   Service   Mark   Enforcement   his   goods   are   manufactured,   or   his   registered   mark   or  
Action.–Any  foreign  national  or  juridical  person   trade   name,   and   thereupon   the   Collector   of   Customs  
who   meets   the   requirements   of   Section   3   of   shall   cause   one   (1)   or   more   copies   of   the   same   to   be  
this   Act   and   does   not   engage   in   business   in   the   transmitted   to   each   collector   or   to   other   proper   officer  
Philippines   may   bring   a   civil   or   administrative   of  the  Bureau  of  Customs.  
action   hereunder   for   opposition,   cancellation,    
infringement,   unfair   competition,   or   false   ADMINISTRATIVE  REMEDIES  
designation   of   origin   and   false   description,   Opposition    
whether   or   not   it   is   licensed   to   do   business   in   -­‐ Who   can   file:   Any   person   who   falls   under  
the  Philippines  under  existing  laws.   Section  3.  
  -­‐ When:  
x  x  x  x   -­‐ Remedy:   Denial   of   application.   No   other  
  remedies.  
Section  170.   Penalties.–Independent   of   the   civil    
and  administrative  sanctions  imposed  by  law,  a   Cancellation  (Section  151)  
criminal  penalty  of  imprisonment  from  two  (2)   -­‐ Who   can   file:   Any   person   who   falls   under  
years   to   five   (5)   years   and   a   fine   ranging   from   Section  3.  
Fifty   thousand   pesos   (P50,000)   to   Two   -­‐ When:  After  the  mark  has  been  registered.  
hundred   thousand   pesos   (P200,000),   shall   be   o From  the  point  of  view  of  when  to  file  
imposed   on   any   person   who   is   found   guilty   of   a  petition  à  easier  to  file  cancellation  
committing   any   of   the   acts   mentioned   in   than  opposition.  
Section  155,  Section168,  and  Subsection169.1.   o From   the   point   of   view   of   burden   of  
  proof   à   easier   to   file   opposition.   No  
Based   on   the   foregoing   discussion,   the   IPO   Director   of   prima   facie   presumptions   of   (1)  
Legal   Affairs   had   jurisdiction   to   decide   the   petitioner’s   validity   of   registration,   (2)   ownership  
administrative   case   against   respondents   and   the   IPO   of  mark,  and  (3)  exclusive  right  to  use  
Director   General   had   exclusive   jurisdiction   over   the   mark   as   conferred   upon   the  
appeal   of   the   judgment   of   the   IPO   Director   of   Legal   registrant   by   Section   138.   In  
Affairs.   opposition,   all   the   petitioner   has   to  
  prove  is  damage.  
c. Prohibition   of   § Ma’am:   Why   does   Section  
importation   138   only   give   prima   facie  
presumption   if   registration  
 
confers   the   right   upon   the  
SECTION   166.   GOODS   BEARING   INFRINGING   MARKS   OR   registrant   (i.e.   registration   is  
TRADE  NAMES.  –     the   basis   of   ownership)?  
No  article  of  imported  merchandise  which  shall  copy  or   Because   registration   is   not   a  
simulate   the   name   of   any   domestic   product,   or   perfect   process.   Under   the  
manufacturer,  or  dealer,  or  which  shall  copy  or  simulate   law,   what   actually   confers  
a   mark   registered   in   accordance   with   the   provisions   of   ownership   is   a   registration  
this  Act,  or  shall  bear  a  mark  or  trade  name   calculated   to   validly   obtained,   not   just  
induce   the   public   to   believe   that   the   article   is   any   kind   of   registration.  
manufactured   in   the   Philippines,   or   that   it   is   Thus,   a   person   can   attack  
manufactured   in   any   foreign   country   or   locality   other   the   registration   as   invalid.  
than   the   country   or   locality   where   it   is   in   fact   There’s   always   a   possibility  
manufactured,   shall   be   admitted   to   entry   at   any   that   the   IPO   made   a  
customhouse   of   the   Philippines.   In   order   to   aid   the   mistake.  

o 107
Katrina Michelle Mancao
 
-­‐ Remedy:   Cancellation   of   the   registration.   No   advertising   of   any   goods   or   services   including   other  
other  remedy.   preparatory  steps  necessary  to  carry  out  the  sale  of  any  
-­‐ Grounds  (does  not  prescribe):   goods   or   services   on   or   in   connection   with   which   such  
o Generic  name   use  is  likely  to  cause  confusion,  or  to  cause  mistake,  or  to  
§ Test:  primary  significance  to   deceive;  or  
the  relevant  public    
o Abandonment   155.2.   Reproduce,   counterfeit,   copy   or   colorably  
§ Requisites   (as   enumerated   imitate  a  registered  mark  or  a  dominant  feature  thereof  
in  Romero  v.  Maiden  Form):   and   apply   such   reproduction,   counterfeit,   copy   or  
VIP   colorable   imitation   to   labels,   signs,   prints,   packages,  
• Voluntary   wrappers,  receptacles  or  advertisements  intended  to  be  
• Intentional     used   in   commerce   upon   or   in   connection   with   the   sale,  
• Permanent   offering  for  sale,  distribution,  or  advertising  of  goods  or  
o Registration   was   obtained   services   on   or   in   connection   with   which   such   use   is   likely  
fraudulently   to   cause   confusion,   or   to   cause   mistake,   or   to   deceive,  
o Misrepresents   the   source   of   the   shall   be   liable   in   a   civil   action   for   infringement   by   the  
goods   and   services   in   connection   registrant   for   the   remedies   hereinafter   set   forth:  
with  which  the  marks  was  used   Provided,   That   the   infringement   takes   place   at   the  
o Non-­‐use   of   the   mark   for   an   moment  any  of  the  acts  stated  in  Subsection  155.1  or  this  
uninterrupted  period  of  3  years     subsection   are   committed   REGARDLESS   OF   WHETHER  
§ Circumstances  which  excuse   THERE  IS  ACTUAL  SALE  OF  GOODS  OR  SERVICES  USING  
non-­‐use  of  trademark:     THE  INFRINGING  MATERIAL.    
-­‐ Inter  partes  proceeding  –  expedited,  based  on    
documents.   When   you   file   petition,   attach   all   SECTION   156.   ACTIONS,   AND   DAMAGES   AND   INJUNCTION  
exhibits.  
FOR  INFRINGEMENT.  –    
 
Intellectual  property  violation  (Section  10.2)   156.1.   The   owner   of   a   registered   mark   may   recover  
-­‐ Who   can   file:   Any   person   who   falls   within   damages   from   any   person   who   infringes   his   rights,   and  
Section  3.     the   measure   of   the   damages   suffered   shall   be   either  the  
-­‐ Venue:  Bureau  of  Legal  Affairs  (BLA)   reasonable   profit   which   the   complaining   party   would  
o As  long  as  the  amount  claimed  is  not   have  made,  had  the  defendant  not  infringed  his  rights,  
LESS  than  P200,000.00   OR   the   profit   which   the   defendant   actually   made   out   of  
o It   has   the   power   of   the   courts,   PLUS   the   infringement,   or   in   the   event   such   measure   of  
more.   Except   that   this   “more”   has   damages   cannot   be   readily   ascertained   with   reasonable  
not  really  been  exercised.   certainty,   then   the   court   may   award   as   damages   a  
-­‐ Advantage  of  filing  in  the  BLA  à  you  are  more   reasonable  percentage  based  upon  the  amount  of  gross  
likely   to   obtained   a   resolution   at   an   earlier   sales   of   the   defendant   or   the   value   of   the   services   in  
time.  Also,  no  venue  restrictions  here.  You  can   connection   with   which   the   mark   or   trade   name   was  
file   in   the   BLA   regardless   of   where   used   in  the  infringement  of  the  rights  of  the  complaining  
the  parties  are  located.   party.    
o But   BLA   only   works   if   the    
respondent   is   a   legitimate   156.2.   On   application   of   the   complainant,   the   court  
business.     may   impound   during   the   pendency   of   the   action,   sales  
invoices  and  other  documents  evidencing  sales.    
   
2. Civil   156.3.   In   cases   where   actual   intent   to   mislead   the  
  public   or   to   defraud   the   complainant   is   shown,   in   the  
a. Infringement   discretion  of  the  court,  the  damages  may  be  DOUBLED.    
 
 
156.4.   The   complainant,   upon   proper   showing,   may  
SECTION  155.  REMEDIES;  INFRINGEMENT.  –    
also  be  granted  injunction.    
Any   person   who   shall,   without   the   consent   of   the   owner  
 
of  the  registered  mark:  
SECTION   157.   POWER   OF   COURT   TO   ORDER   INFRINGING  
 
155.1.   Use  in  commerce  any  reproduction,  counterfeit,   MATERIAL  DESTROYED.  –    
copy,   or   colorable   imitation   of   a   registered   mark   or   the   157.1   In   any   action   arising   under   this   Act,   in   which   a  
same   container   or   a   dominant   feature   thereof   in   violation   of   any   right   of   the   owner   of   the   registered  
connection   with   the   sale,   offering   for   sale,   distribution,   mark   is   established,   the   court   may   order   that   goods  

108 z
Intellectual Property Law

found  to  be  infringing  be,  without  compensation  of  any   limited  to  an  injunction  against  the  presentation  of  such  
sort,   disposed   of   outside   the   channels   of   commerce   in   advertising   matter  in  future  issues  of  such  newspapers,  
such  a  manner  as  to  avoid  any  harm  caused  to  the  right   magazines,   or   other   similar   periodicals   or   in   future  
holder,   or   destroyed;   and   all   labels,   signs,   prints,   transmissions   of   such   electronic   communications.   The  
packages,   wrappers,   receptacles   and   advertisements   in   limitations   of   this   subparagraph   shall   apply   only   to  
the  possession  of  the  defendant,  bearing  the  registered   innocent   infringers:   Provided,   That   SUCH   INJUNCTIVE  
mark   or   trade   name   or   any   reproduction,   counterfeit,   RELIEF   SHALL   NOT   BE   AVAILABLE   to   the   owner   of   the  
copy   or   colorable   imitation   thereof,   all   plates,   molds,   right  infringed  with  respect  to  an  issue  of  a  newspaper,  
matrices   and   other   means   of   making   the   same,   shall   be   magazine,   or   other   similar   periodical   or   an   electronic  
delivered  up  and  destroyed.   communication   containing   infringing   matter   where  
  restraining  the  dissemination  of  such  infringing  matter  
157.2.   In   regard   to   counterfeit   goods,   the   simple   in   any   particular   issue   of   such   periodical   or   in   an  
removal   of   the   trademark   affixed   shall   not   be   sufficient   electronic   communication   would   delay   the   delivery   of  
other   than   in   exceptional   cases   which   shall   be   such   issue   or   transmission   of   such   electronic  
determined  by  the  Regulations,  to  permit  the  release  of   communication  is  customarily  conducted  in  accordance  
the  goods  into  the  channels  of  commerce.     with   the   sound   business   practice,   and   not   due   to   any  
  method   or   device   adopted   to   evade   this   section   or   to  
SECTION  158.  DAMAGES;  REQUIREMENT  OF  NOTICE.  –     prevent   or   delay   the   issuance   of   an   injunction   or  
restraining  order  with  respect  to  such  infringing  matter;  
In  any  suit  for  infringement,  the  owner  of  the  registered  
and  
mark  shall  not  be  entitled  to  recover  profits  or  damages  
 
unless   the   acts   have   been   committed   with   knowledge  
159.4.   There   shall   be   no   infringement   of   trademarks   or  
that   such   imitation   is   likely   to   cause   confusion,   or   to  
cause   mistake,   or   to   deceive.   Such   knowledge   is   tradenames   of   imported   or   sold   drugs   and   medicines  
PRESUMED  if  the  registrant  gives  notice  that  his  mark  is   allowed   under   Section   72.1   of   this   Act,   as   well   as  
registered   by   displaying   with   the   mark   the   words   imported   or   sold   off-­‐patent   drugs   and   medicines:  
'"Registered   Mark"   or   the   letter   R   within   a   circle   or   if   Provided,   That   said   drugs   and   medicines   bear   the  
the   defendant   had   otherwise   actual   notice   of   the   registered   marks   that   have   not   been   tampered,  
registration.     unlawfully  modified,  or  infringed  upon  as  defined  under  
  Section  155  of  this  Code.  
 
SECTION  159.   LIMITATIONS  TO  ACTIONS  FOR  INFRINGEMENT.  
SECTION   161.   AUTHORITY   TO   DETERMINE   RIGHT   TO  
–    
REGISTRATION.  –    
Notwithstanding   any   other   provision   of   this   Act,   the  
remedies   given   to   the   owner   of   a   right   infringed   under   In   any   action   involving   a   registered   mark,   the   COURT  
this  Act  shall  be  limited  as  follows:   may   determine   the   right   to   registration,   order   the  
  cancellation   of   a   registration,   in   whole   or   in   part,   and  
159.1.   Notwithstanding   the   provisions   of   Section   155   otherwise   rectify   the   register   with   respect   to   the  
registration  of  any  party  to  the  action  in  the  exercise  of  
hereof,   a   registered   mark   shall   have   no   effect   against  
this.   Judgment   and   orders   shall   be   certified  by  the  court  
any  person  who,  in  good  faith,  before  the  filing  date  or  
to   the   Director,   who   shall   make   appropriate   entry   upon  
the   priority   date,   was   using   the   mark   for   the   purposes  
the   records   of   the   Bureau,   and   shall   be   controlled  
of   his   business   or   enterprise:   Provided,   That   his   right  
thereby.  
may   only   be   transferred   or   assigned   together   with   his  
 
enterprise  or  business  or  with  that  part  of  his  enterprise  
or  business  in  which  the  mark  is  used.   SECTION  163.  JURISDICTION  OF  COURT.  –    
  All   actions   under   Sections   150,   155,   164,   and   166   to   169  
159.2.   Where  an  infringer  who  is  engaged  solely  in  the   shall   be   BROUGHT   BEFORE   THE   PROPER   COURTS   with  
business   of   printing   the   mark   or   other   infringing   appropriate  jurisdiction  under  existing  laws.  
materials  for  others  is  an  innocent   infringer,  the  owner    
of   the   right   infringed   shall   be   entitled   as   against   such   SECTION   164.   NOTICE   OF   FILING   SUIT   GIVEN   TO   THE  
infringer  only  to  an  injunction  against  future  printing.   DIRECTOR.  –  
  It   shall   be   the   duty   of   the   clerks   of   such   courts   within  
159.3.   Where   the   infringement   complained   of   is   one   (1)   month   after   the   filing   of   any   action,   suit,   or  
contained   in   or   is   part   of   paid   advertisement   in   a   proceeding   involving   a   mark   registered   under   the  
newspaper,  magazine,  or  other  similar  periodical  or  in  an   provisions   of   this   Act,   to   notify   the   Director   in   writing  
electronic  communication,  the  remedies  of  the  owner  of   setting   forth:   the   names   and   addresses   of   the   litigants  
the   right   infringed   as   against   the   publisher   or   and   designating   the   number   of   the   registration   or  
distributor   of   such   newspaper,   magazine,   or   other   registrations   and   within   one   (1)   month   after   the  
similar   periodical   or   electronic   communication   shall   be   judgment   is   entered   or   an   appeal   is   taken,   the   clerk   of  

o 109
Katrina Michelle Mancao
 
court   shall   give   notice   thereof   to   the   Office,   and   the   same   trademark   by   others   on   unrelated   articles   of   a  
latter   shall   endorse   the   same   upon   the   filewrapper   of   different   kind.   Although   petitioner's   products   are  
the   said   registration   or   registrations   and   incorporate   the   numerous,   they   are   of   the   same   class   or   line   of  
same  as  a  part  of  the  contents  of  said  filewrapper.   merchandise   which   are   non-­‐competing   with  
  respondent's  product  of  cigarettes,  which  as  pointed  out  
SECTION  232.  APPEALS.  -­‐     in  the  appealed  judgment  is  beyond  petitioner's  "zone  of  
232.1.   Appeals   from   decisions   of   regular   courts   shall   be   potential   or   natural   and   logical   expansion"   21   When   a  
trademark  is  used  by  a  party  for  a  product  in  which  the  
governed   by   the   Rules   of   Court.   Unless   restrained   by   a  
other   party   does   not   deal,   the   use   of   the   same  
higher   court,   the   judgment   of   the   trial   court   shall   be  
trademark   on   the   latter's   product   cannot   be   validly  
executory   even   pending   appeal   under   such   terms   and  
objected  to.    
conditions  as  the  court  may  prescribe.  
 
 
Another   factor   that   shows   that   the   goods   involved   are  
232.2.   Unless   expressly   provided   in   this   Act   or   other  
non-­‐competitive  and  non-­‐related  is  the  appellate  court's  
statutes,   appeals   from   decisions   of   administrative   finding   that   they   flow   through   different   channels   of  
officials  shall  be  provided  in  the  Regulations.   trade,  thus:  "The  products  of  each  party  move  along  and  
  are   disposed   through   different   channels   of   distribution.  
ETEPHA  V.  DIRECTOR  OF  PATENTS  (1966),  SUPRA   The   petitioner's   products   are   distributed   principally  
Refresher:   through   gasoline   service   and   lubrication   stations,  
Atussin  and  Pertussin   automotive   shops   and   hardware   stores.   On   the   other  
  hand,   the   respondent's   cigarettes   are   sold   in   sari-­‐sari  
Doctrine:   stores,   grocery   stores,   and   other   small   distributor  
In   the   solution   of   a   trademark   infringement   problem,   outlets.   Respondent's   cigarettes   are   even   peddled   in   the  
regard  too  should  be  given  to  the  class  of  persons  who   streets  while  petitioner's  'gasul'  burners  are  not.  Finally,  
buy   the   particular   product   and   the   circumstances   there  is  a  marked  distinction  between  oil  and  tobacco,  as  
ordinarily   attendant   to   its   acquisition.   The   medicinal   well   as   between   petroleum   and   cigarettes.   Evidently,   in  
preparation  clothed  with  the  trademarks  in  question,  are   kind   and   nature   the   products   of   respondent   and   of  
unlike  articles  of  everyday  use  such  as  candies,  ice  cream,   petitioner  are  poles  apart."  
milk,   soft   drinks   and   the   like   which   may   be   freely    
obtained  by  anyone,  anytime,  anywhere.  Petitioner's  and   FRUIT  OF  THE  LOOM  V.  CA  AND  GENERAL  GARMENTS  CORP.  
respondent's  products  are  to  be  dispensed  upon  medical  
(1984)  
prescription.   The   respective   labels   say   so.   An   intending  
buyer   must   have   to   go   first   to   a   licensed   doctor   of   Refresher:  
medicine;   he   receives   instructions   as   to   what   to   Respondent:   “Fruit   for   Eve”   –   women’s   panties   and  
purchase;   he   reads   the   doctor's   prescription;   he   knows   pajamas  
what   he   is   to   buy.   He   is   not   of   the   incautious,   unwary,    
unobservant   or   unsuspecting   type;   he   examines   the   Petitioner:   “Fruit   of   the   Loom”   –   men’s,   women’s   and  
product   sold   to   him;   he   checks   to   find   out   whether   it   children’s  underwear  
conforms   to   the   medical   prescription.   The   common    
trade  channel  is  the  pharmacy  or  the  drugstore.  Similarly,   The   main   issue   involved   in   this   case   is   whether   or   not  
the   pharmacist   or   druggist   verifies   the   medicine   sold.   private   respondent's   trademark   FRUIT   FOR   EVE   and   its  
The   margin   of   error   in   the   acquisition   of   one   for   the   hang   tag   are   confusingly   similar   to   petitioner's  
other  is  quite  remote.   trademark  FRUIT  OF  THE  LOOM  and  its  hang  tag  so  as  to  
  constitute   an   infringement   of   the   latter's   trademark  
rights  and  justify  the  cancellation  of  the  former.  
ESSO   STANDARD   EASTERN,   INC.   V.   CA   AND   UNITED  
 
CIGARETTES    (1982)   Doctrine:  
Refresher:   In   determining   whether   the   trademarks   are   confusingly  
“Esso”  on  petroleum  and  cigarette  products   similar,   a   comparison   of   the   words   is   not   the   only  
  determinant   factor.   The   trademarks   in   their   entirety   as  
Doctrine:   they  appear  in  their  respective  labels  or  hang  tags  must  
In   the   situation   before   us,   the   goods   are   obviously   also  be  considered  in  relation  to  the  goods  to  which  they  
different   from   each   other   with   "absolutely   no   iota   of   are   attached.   The   discerning   eye   of   the   observer   must  
similitude"   as   stressed   in   respondent   court's   judgment.   focus   not   only   on   the   predominant   words   but   also   on  
They  are  so  foreign  to  each  other  as  to  make  it  unlikely   the  other  features  appearing  in  both  labels  in  order  that  
that   purchasers   would   think   that   petitioner   is   the   he   may   draw   his   conclusion   whether   one   is   confusingly  
manufacturer   of   respondent's   goods.   The   mere   fact   that   similar  to  the  other.  
one   person   has   adopted   and   used   a   trademark   on   his    
goods   does   not   prevent   the   adoption   and   use   of   the  

110 z
Intellectual Property Law

WE   hold   that   the   trademarks   FRUIT   OF   THE   LOOM   and   may   be   dissipated   as   soon   as   the   court   assumes   to  
FRUIT   FOR   EVE   do   not   resemble   each   other   as   to   analyze  carefully  the  respective  features  of  the  mark.  
confuse   or   deceive   an   ordinary   purchaser.   The   ordinary    
purchaser   must   be   thought   of   as   having,   and   credited   It   has   also   been   held   that   it   is   not   the   function   of   the  
with,   at   least   a   modicum   of   intelligence   to   be   able   to   see   court  in  cases  of  infringement  and  unfair  competition  to  
the  obvious  differences  between  the  two  trademarks  in   educate  purchasers  but  rather  to  take  their  carelessness  
question.   Furthermore,   We   believe   that   a   person   who   for   granted,   and   to   be   ever   conscious   of   the   fact   that  
buys  petitioner's  products  and  starts  to  have  a  liking  for   marks   need   not   be   identical.   A   confusing   similarity   will  
it,   will   not   get   confused   and   reach   out   for   private   justify   the   intervention   of   equity.   The   judge   must   also   be  
respondent's   products   when   she   goes   to   a   garment   aware   of   the   fact   that   usually   a   defendant   in   cases   of  
store.   infringement   does   not   normally   copy   but   makes   only  
  colorable   changes.   Well   has   it   been   said   that   the   most  
DEL   MONTE   CORP.   AND   PHIL.   PACKING   CORP.   V.   CA   AND   successful  form  of  copying  is  to  employ  enough  points  of  
similarity   to   confuse   the   public   with   enough   points   of  
SUNSHINE  SAUCE  MANUFACTURING  INDUSTRIES  (1990)  
difference  to  confuse  the  courts.  
Doctrine:  
 
It   has   been   correctly   held   that   side-­‐by-­‐side   comparison   is  
ASIA  BREWERY,  INC.  V.  CA  AND  SAN  MIGUEL  CORP.   (1993),  
not  the  final  test  of  similarity.  Such  comparison  requires  
a  careful  scrutiny  to  determine  in  what  points  the  labels   SUPRA  
of   the   products   differ,   as   was   done   by   the   trial   judge.   Court  held:  
The   ordinary   buyer   does   not   usually   make   such   scrutiny   Only   registered   trade   marks,   trade   names   and   service  
nor  does  he  usually  have  the  time  to  do  so.  The  average   marks   are   protected   against   infringement   or  
shopper  is  usually  in  a  hurry  and  does  not  inspect  every   unauthorized   use   by   another   or   others.   The   use   of  
product  on  the  shelf  as  if  he  were  browsing  in  a  library.   someone   else's   registered   trademark,   trade   name   or  
Where   the   housewife   has   to   return   home   as   soon   as   service   mark   is   unauthorized,   hence,   actionable,   if   it   is  
possible   to   her   baby   or   the   working   woman   has   to   make   done  "without  the  consent  of  the  registrant."  
quick   purchases   during   her   off   hours,   she   is   apt   to   be    
confused   by   similar   labels   even   if   they   do   have   minute   Infringement   is   determined   by   the   "test   of   dominancy"  
differences.   The   male   shopper   is   worse   as   he   usually   rather   than   by   differences   or   variations   in   the   details   of  
does  not  bother  about  such  distinctions.     one   trademark   and   of   another.   It   has   been   consistently  
  held  that  the  question  of  infringement  of  a  trademark  is  
The   question   is   not   whether   the   two   articles   are   to  be  determined  by  the  test  of  dominancy.  Similarity  in  
distinguishable   by   their   label   when   set   side   by   side   but   size,  form  and  color,  while  relevant,  is  not  conclusive.  If  
whether  the  general  confusion  made  by  the  article  upon   the  competing  trademark  contains  the  main  or  essential  
the  eye  of  the  casual  purchaser  who  is  unsuspicious  and   or   dominant   features   of   another,   and   confusion   and  
off  his  guard,  is  such  as  to  likely  result  in  his  confounding   deception   is   likely   to   result,   infringement   takes   place.  
it   with   the   original.   As   observed   in   several   cases,   the   Duplication   or   imitation   is   not   necessary;   nor   it   is  
general   impression   of   the   ordinary   purchaser,   buying   necessary   that   the   infringing   label   should   suggest   an  
under   the   normally   prevalent   conditions   in   trade   and   effort   to   imitate.   The   question   at   issue   in   cases   of  
giving   the   attention   such   purchasers   usually   give   in   infringement   of   trademarks   is   whether   the   use   of   the  
buying  that  class  of  goods  is  the  touchstone.   marks   involved   would   be   likely   to   cause   confusion   or  
  mistakes  in  the  mind  of  the  public  or  deceive  purchasers.  
It   has   been   held   that   in   making   purchases,   the   consumer    
must  depend  upon  his  recollection  of  the  appearance  of   The  universal  test  question  is  whether  the  public  is  likely  
the   product   which   he   intends   to   purchase.   The   buyer   to   be   deceived.   Nothing   less   than   conduct   tending   to  
having   in   mind   the   mark/label   of   the   respondent   must   pass  off  one  man's  goods  or  business  as  that  of  another  
rely   upon   his   memory   of   the   petitioner's   mark.   Unlike   will   constitute   unfair   competition.   Actual   or   probable  
the   judge   who   has   ample   time   to   minutely   examine   the   deception  and  confusion  on  the  part  of  the  customers  by  
labels  in  question  in  the  comfort  of  his  sala,  the  ordinary   reason  of  defendant's  practices  must  always  appear.  
shopper  does  not  enjoy  the  same  opportunity.    
  CONRAD   AND   CO.   V.   CA,   FITRITE   INC.   AND   VICTORIA  
A  number  of  courts  have  held  that  to  determine  whether  
BISCUITS  CO.  (1995)  
a   trademark   has   been   infringed,   we   must   consider   the  
Refresher:  
mark   as   a   whole   and   not   as   dissected.   If   the   buyer   is  
Trademark  in  question  =  “Sunshine”  
deceived,  it  is  attributable  to  the  marks  as  a  totality,  not  
 
usually   to   any   part   of   it.   The   court   therefore   should   be  
Doctrine:  
guided   by   its   first   impression,   for   a   buyer   acts   quickly  
and   is   governed   by   a   casual   glance,   the   value   of   which   Whether   CONRAD's   acts   of   importing,   selling   and  
distributing   biscuits,   cookies   and   other   food   items  

o 111
Katrina Michelle Mancao
 
bearing   said   registered   "SUNSHINE"   trademark   in   the    
Philippines   without   the   consent   of   its   registrant   As  its  title  implies,  the  test  of  dominancy  focuses  on  the  
(FITRITE)   constitute   infringement   thereof   in   similarity   of   the   prevalent   features   of   the   competing  
contemplation   of   Sec.   22   of   Republic   Act   No.   166,   as   trademarks   which   might   cause   confusion   or   deception  
amended.   Under   Sec.   22,   the   elements   that   constitute   and  thus  constitutes  infringement.  
infringement   are   simply   (1)   the   use   by   any   person,    
without   the   consent   of   the   registrant,   (2)   of   any   On   the   other   side   of   the   spectrum,   the   holistic   test  
registered   mark   or   trade-­‐name   in   connection   with   the   mandates   that   the   entirety   of   the   marks   in   question  
sale,   business   or   services,   among   other   things,   bearing   must  be  considered  in  determining  confusing  similarity.  
such  registered  mark  or  trade-­‐name.      
  Applying   the   foregoing   tenets   to   the   present  
Thus,   having   the   exclusive   right   over   said   trademark,   controversy   and   taking   into   account   the   factual  
FITRITE   should   be   protected   in   the   use   thereof;   and   circumstances   of   this   case,   we   considered   the  
considering   that   it   is   apparent   from   the   record   that   the   trademarks  involved  as  a  whole  and  rule  that  petitioner's  
invasion   of   the   right   FITRITE   sought   to   protect   is   "STYLISTIC  MR.  LEE"  is  not  confusingly  similar  to  private  
material   and   substantial;   that   such   right   of   FITRITE   is   respondent's  "LEE"  trademark.  
clear   and   unmistakable;   and   that   there   is   an   urgent    
necessity   to   prevent   serious   damage   to   FITRITE's   Petitioner's  trademark  is  the  whole  "STYLISTIC  MR.  LEE."  
business   interest,   goodwill   and   profit,   thus   under   the   Although   on   its   label   the   word   "LEE"   is   prominent,   the  
authority   of   Sec.   23   of   said   Republic   Act   No.   166,   as   trademark   should   be   considered   as   a   whole   and   not  
amended,  a  preliminary  injunction  may  be  issued  in  favor   piecemeal.   The   dissimilarities   between   the   two   marks  
of  FITRITE  to  maintain  the  status  quo  pending  trial  of  the   become   conspicuous,   noticeable   and   substantial   enough  
action   a   quo   on   the   merits   without   prejudice   to   the   to  matter  especially  in  the  light  of  the  following  variables  
suspension   of   such   action   if   the   aforesaid   cancellation   that  must  be  factored  in.  
proceeding  before  the  BPTTT  has  not  been  concluded.    
  First,  the  products  involved  in  the  case  at  bar  are,  in  the  
EMERALD   GARMENT   MANUFACTURING   V.   CA   (1995),   main,  various  kinds  of  jeans.  These  are  not  your  ordinary  
household  items  like  catsup,  soysauce  or  soap  which  are  
SUPRA  
of   minimal   cost.   Maong   pants   or   jeans   are   not  
Refresher:  
inexpensive.   Accordingly,   the   casual   buyer   is  
“Stylistics  Mr.  Lee”  and  “Lee”  
predisposed   to   be   more   cautious   and   discriminating   in  
 
and   would   prefer   to   mull   over   his   purchase.   Confusion  
Doctrine:  
and  deception,  then,  is  less  likely.    
The   essential   element   of   infringement   is   colorable  
 
imitation.   This   term   has   been   defined   as   "such   a   close   or   Second,   like   his   beer,   the   average   Filipino   consumer  
ingenious   imitation   as   to   be   calculated   to   deceive  
generally   buys   his   jeans   by   brand.   He   does   not   ask   the  
ordinary   purchasers,   or   such   resemblance   of   the   sales   clerk   for   generic   jeans   but   for,   say,   a   Levis,   Guess,  
infringing  mark  to  the  original  as  to  deceive  an  ordinary  
Wrangler   or   even   an   Armani.   He   is,   therefore,   more   or  
purchaser   giving   such   attention   as   a   purchaser   usually  
less  knowledgeable  and  familiar  with  his  preference  and  
gives,  and  to  cause  him  to  purchase  the  one  supposing  it   will  not  easily  be  distracted.  
to  be  the  other."  
 
 
Finally,   in   line   with   the   foregoing   discussions,   more  
Colorable   imitation   does   not   mean   such   similitude   as  
credit  should  be  given  to  the  "ordinary  purchaser."  Cast  
amounts   to   identity.   Nor   does   it   require   that   all   the  
in   this   particular   controversy,   the   ordinary   purchaser   is  
details   be   literally   copied.   Colorable   imitation   refers   to   not   the   "completely   unwary   consumer"   but   is   the  
such  similarity  in  form,  content,  words,  sound,  meaning,  
"ordinarily   intelligent   buyer"   considering   the   type   of  
special   arrangement,   or   general   appearance   of   the  
product  involved.  
trademark  or  tradename  with  that  of  the  other  mark  or  
 
tradename   in   their   over-­‐all   presentation   or   in   their  
The  definition  laid   down   in   Dy   Buncio   v.   Tan   Tiao  Bok  is  
essential,   substantive   and   distinctive   parts   as   would  
better   suited   to   the   present   case.   There,   the   "ordinary  
likely  mislead  or  confuse  persons  in  the  ordinary  course  
purchaser"   was   defined   as   one   "accustomed   to   buy,   and  
of  purchasing  the  genuine  article.  
therefore   to   some   extent   familiar   with,   the   goods   in  
 
question.  The  test  of  fraudulent  simulation  is  to  be  found  
In   determining   whether   colorable   imitation   exists,  
in   the   likelihood   of   the   deception   of   some   persons   in  
jurisprudence   has   developed   TWO   KINDS   OF   TESTS   —   some   measure   acquainted   with   an   established   design  
the  Dominancy  Test  applied  in  Asia  Brewery,  Inc.  v.  Court   and   desirous   of   purchasing   the   commodity   with   which  
of   Appeals   and   other   cases   and   the   Holistic   Test   that  design  has  been  associated.  The  test  is  not  found  in  
developed  in  Del  Monte  Corporation  v.  Court  of  Appeals  
the   deception,   or   the   possibility   of   deception,   of   the  
and  its  proponent  cases.    

112 z
Intellectual Property Law

person  who  knows  nothing  about  the  design  which  has   infringement.     On   the   other   side   of   the   spectrum,   the  
been   counterfeited,   and   who   must   be   indifferent   holistic   test   mandates   that   the   entirety   of   the   marks   in  
between  that  and  the  other.  The  simulation,  in  order  to   question   must   be   considered   in   determining   confusing  
be   objectionable,   must   be   such   as   appears   likely   to   similarity.  
mislead   the   ordinary   intelligent   buyer   who   has   a   need   to    
supply   and   is   familiar   with   the   article   that   he   seeks   to   As   this   Court   has   often   declared,   each   case   must   be  
purchase."   studied  according  to  the  peculiar  circumstances  of  each  
  case.     That   is   the   reason   why   in   trademark   cases,  
AMIGO  V.  CLUETT  PEABODY  (2001)   jurisprudential   precedents   should   be   applied   only   to   a  
Refresher:   case  if  they  are  specifically  in  point.  
“Gold  Top”  and  “Gold  Toe”  mark  on  socks.    
  In  the  case  at  bar,  other  than  the  fact  that  both  Nestle’s  
Court  decision:   and   CFC’s   products   are   inexpensive   and   common  
In   the   present   case,   a   resort   to   either   the   Dominancy   household  items,  the  similarity  ends  there.    What  is  being  
Test   or   the   Holistic   Test   shows   that   colorable   imitation   questioned   here   is   the   use   by   CFC   of   the   trademark  
exists   between   respondent’s   “Gold   Toe”   and   MASTER.     In   view   of   the   difficulty   of   applying  
petitioner’s   “Gold   Top.”   A   glance   at   petitioner’s   mark   jurisprudential  precedents  to  trademark  cases  due  to  the  
shows  that  it  definitely  has  a  lot  of  similarities  and  in  fact   peculiarity   of   each   case,   judicial   fora   should   not   readily  
looks   like   a   combination   of   the   trademark   and   devices   apply  a  certain  test  or  standard  just  because  of  seeming  
that   respondent   has   already   registered;   namely,   “Gold   similarities.    As  this  Court  has  pointed  above,  there  could  
Toe,”   the   representation   of   a   sock   with   a   magnifying   be   more   telling   differences   than   similarities   as   to   make   a  
glass,  the  “Gold  Toe”  representation  and  “linenized.”   jurisprudential  precedent  inapplicable.  
   
Admittedly,   there   are   some   minor   differences   between   Nestle   points   out   that   the   dominancy   test   should   have  
the  two  sets  of  marks.    The  similarities,  however,  are  of   been  applied  to  determine  whether  there  is  a  confusing  
such   degree,   number   and   quality   that   the   overall   similarity   between   CFC’s   FLAVOR   MASTER   and   Nestle’s  
impression   given   is   that   the   two   brands   of   socks   are   MASTER  ROAST  and  MASTER  BLEND.  
deceptively   the   same,   or   at   least   very   similar   to   each    
another.     An   examination   of   the   products   in   question   We  agree.  
shows   that   their   dominant   features   are   gold   checkered    
lines   against   a   predominantly   black   background   and   a   It   must   be   emphasized   that   the   products   bearing   the  
representation   of   a   sock   with   a   magnifying   glass.     In   trademarks   in   question   are   “inexpensive   and   common”  
addition,   both   products   use   the   same   type   of   lettering.     household  items  bought  off  the  shelf  by  “undiscerningly  
Both   also   include   a   representation   of   a   man’s   foot   rash”   purchasers.     As   such,   if   the   ordinary   purchaser   is  
wearing   a   sock   and   the   word   “linenized”   with   arrows   “undiscerningly  rash”,  then  he  would  not  have  the  time  
printed   on   the   label.     Lastly,   the   names   of   the   brands   are   nor   the   inclination   to   make   a   keen   and   perceptive  
similar  -­‐-­‐   “Gold  Top”  and  “Gold  Toe.”    Moreover,  it  must   examination   of   the   physical   discrepancies   in   the  
also   be   considered   that   petitioner   and   respondent   are   trademarks   of   the   products   in   order   to   exercise   his  
engaged  in  the  same  line  of  business.   choice.  
   
While   this   Court   agrees   with   the   Court   of   Appeals’  
SOCIETES  DES  PRODUITS  NESTLE  V.  CA  (2001),  SUPRA  
detailed   enumeration   of   differences   between   the  
Court  decision:  
respective   trademarks   of   the   two   coffee   products,   this  
Under   Section   36   of   Republic   Act   165   (Patent   Law),   the   Court   cannot   agree   that   totality   test   is   the   one  
Director   of   Patents   is   authorized,   in   case   the   parties  
applicable   in   this   case.     Rather,   this   Court   believes   that  
failed   to   submit   a   licensing   agreement,   to   fix   the   terms  
the   dominancy   test   is   more   suitable   to   this   case   in   light  
and  conditions  of  the  license.  
of  its  peculiar  factual  milieu.  
   
Colorable   imitation   denotes   such   a   close   or   ingenious  
Moreover,   the   totality   or   holistic   test   is   contrary   to   the  
imitation  as  to  be  calculated  to  deceive  ordinary  persons,  
elementary   postulate   of   the   law   on   trademarks   and  
or   such   a   resemblance   to   the   original   as   to   deceive   an  
unfair   competition   that   confusing   similarity   is   to   be  
ordinary   purchaser   giving   such   attention   as   a   purchaser  
determined   on   the   basis   of   visual,   aural,   connotative  
usually   gives,   as   to   cause   him   to   purchase   the   one   comparisons  and  overall  impressions  engendered  by  the  
supposing  it  to  be  the  other.  In  determining  if  colorable  
marks   in   controversy   as   they   are   encountered   in   the  
imitation   exists,   jurisprudence   has   developed   two   kinds  
realities   of   the   marketplace.   The   totality   or   holistic   test  
of   tests   -­‐   the   Dominancy   Test   and   the   Holistic   Test.   The  
only   relies   on   visual   comparison   between   two  
test   of   dominancy   focuses   on   the   similarity   of   the   trademarks   whereas   the   dominancy   test   relies   not   only  
prevalent   features   of   the   competing   trademarks   which  
on   the   visual   but   also   on   the   aural   and   connotative  
might  cause  confusion  or  deception  and  thus  constitute  

o 113
Katrina Michelle Mancao
 
comparisons   and   overall   impressions   between   the   two   in  which  case  defendant’s  goods  are  then  bought  as  the  
trademarks.   plaintiff’s   and   its   poor   quality   reflects   badly   on   the  
  plaintiff’s  reputation.    The  other  is  “confusion  of  business”  
MIGHTY  CORP.  V.  E.  &  J.  GALLO  WINERY  (2004)   wherein   the   goods   of   the   parties   are   different   but   the  
Court  decision:   defendant’s  product  can  reasonably  (though  mistakenly)  
Although   the   laws   on   trademark   infringement   and   unfair   be   assumed   to   originate   from   the   plaintiff,   thus  
competition   have   a   common   conception   at   their   root,   deceiving   the   public   into   believing   that   there   is   some  
that  is,  a  person  shall  not  be  permitted  to  misrepresent   connection   between   the   plaintiff   and   defendant   which,  
his   goods   or   his   business   as   the   goods   or   business   of   in  fact,  does  not  exist.  
another,   the   law   on   unfair   competition   is   broader   and    
more  inclusive  than  the  law  on  trademark  infringement.     In   determining   the   likelihood   of   confusion,   the   Court  
The   latter   is   more   limited   but   it   recognizes   a   more   must  consider:    
exclusive  right  derived  from  the  trademark  adoption  and   [a]  the  resemblance  between  the  trademarks;    
registration   by   the   person   whose   goods   or   business   is   [b]   the   similarity   of   the   goods   to   which   the  
first  associated  with  it.    The  law  on  trademarks  is  thus  a   trademarks  are  attached;    
specialized   subject   distinct   from   the   law   on   unfair   [c]  the  likely  effect  on  the  purchaser  and    
competition,   although   the   two   subjects   are   entwined   [d]  the  registrant’s  express  or  implied  consent  
with   each   other   and   are   dealt   with   together   in   the   and  other  fair  and  equitable  considerations.  
Trademark  Law  (now,  both  are  covered  by  the  IP  Code).      
Hence,   even   if   one   fails   to   establish   his   exclusive   Whether   a   trademark   causes   confusion   and   is   likely   to  
property   right   to   a   trademark,   he   may   still   obtain   relief   deceive   the   public   hinges   on   “colorable   imitation”  
on   the   ground   of   his   competitor’s   unfairness   or   fraud.     which   has   been   defined   as   “such   similarity   in   form,  
Conduct  constitutes  unfair  competition  if  the  effect  is  to   content,  words,  sound,  meaning,  special  arrangement  or  
pass   off   on   the   public   the   goods   of   one   man   as   the   general   appearance   of   the   trademark   or   tradename   in  
goods  of  another.    It  is  not  necessary  that  any  particular   their   overall   presentation   or   in   their   essential   and  
means  should  be  used  to  this  end.   substantive   and   distinctive   parts   as   would   likely   mislead  
  or   confuse   persons   in   the   ordinary   course   of   purchasing  
Trademark  infringement  v.  Unfair  competition:   the  genuine  article.”  
   
(1)   Infringement   of   trademark   is   the   Jurisprudence   has   developed  TWO   TESTS   in   determining  
unauthorized   use   of   a   trademark,   whereas   similarity   and   likelihood   of   confusion   in   trademark  
unfair   competition   is   the   passing   off   of   one's   resemblance   -­‐   the   Dominancy   Test   and   the   Holistic   or  
goods  as  those  of  another.   Totality  Test.  
   
(2)   In   infringement   of   trademark   fraudulent   The   Dominancy   Test   focuses   on   the   similarity   of   the  
intent   is   unnecessary,   whereas   in   unfair   prevalent   features   of   the   competing   trademarks   which  
competition  fraudulent  intent  is  essential.   might   cause   confusion   or   deception,   and   thus  
  infringement.     If   the   competing   trademark   contains   the  
(3)   In   infringement   of   trademark   the   prior   main,   essential   or   dominant   features   of   another,   and  
registration   of   the   trademark   is   a   prerequisite   confusion   or   deception   is   likely   to   result,   infringement  
to   the   action,   whereas   in   unfair   competition   takes  place.  Duplication  or  imitation  is  not  necessary;  nor  
registration  is  not  necessary.   is   it   necessary   that   the   infringing   label   should   suggest   an  
  effort  to  imitate.  The  question  is  whether  the  use  of  the  
A  crucial  issue  in  any  trademark  infringement  case  is  the   marks  involved  is  likely  to  cause  confusion  or  mistake  in  
likelihood   of   confusion,   mistake   or   deceit   as   to   the   the  mind  of  the  public  or  deceive  purchasers.  
identity,  source  or  origin  of  the  goods  or  identity  of  the    
business   as   a   consequence   of   using   a   certain   mark.     On   the   other   hand,   the   Holistic   Test   requires   that   the  
Likelihood   of   confusion   is   admittedly   a   relative   term,   to   entirety   of   the   marks   in   question   be   considered   in  
be   determined   rigidly   according   to   the   particular   (and   resolving   confusing   similarity.     Comparison   of   words   is  
sometimes   peculiar)   circumstances   of   each   case.     Thus,   not   the   only   determining   factor.     The   trademarks   in   their  
in  trademark  cases,  more  than  in  other  kinds  of  litigation,   entirety  as  they  appear  in  their  respective  labels  or  hang  
precedents   must   be   studied   in   the   light   of   each   tags  must  also  be  considered  in  relation  to  the  goods  to  
particular  case.   which   they   are   attached.     The   discerning   eye   of   the  
  observer  must  focus  not  only  on  the  predominant  words  
There   are   TWO   TYPES   OF   CONFUSION   IN   TRADEMARK   but  also  on  the  other  features  appearing  in  both  labels  in  
INFRINGEMENT.    The  first  is  “confusion  of  goods”  when   order   that   he   may   draw   his   conclusion   whether   one   is  
an   otherwise   prudent   purchaser   is   induced   to   purchase   confusingly  similar  to  the  other.  
one   product   in   the   belief   that   he   is   purchasing   another,    

114 z
Intellectual Property Law

In   comparing   the   resemblance   or   colorable   imitation   of   confusion   of   business   remains,   since   the   low-­‐income  
marks,  various  factors  have  been  considered,  such  as  the   group   might   be   led   to   believe   that   the   “Big   Mak”  
dominant   color,   style,   size,   form,   meaning   of   letters,   hamburgers   are   the   low-­‐end   hamburgers   marketed   by  
words,   designs   and   emblems   used,   the   likelihood   of   petitioners.    After  all,  petitioners  have  the  exclusive  right  
deception   of   the   mark   or   name's   tendency   to   confuse   to   use   the   “Big   Mac”   mark.         On   the   other   hand,  
and  the  commercial  impression  likely  to  be  conveyed  by   respondents   would   benefit   by   associating   their   low-­‐end  
the  trademarks  if  used  in  conjunction  with  the  respective   hamburgers,   through   the   use   of   the   “Big   Mak”   mark,  
goods  of  the  parties.   with   petitioners’   high-­‐end   “Big   Mac”   hamburgers,  
  leading   to   likelihood   of   confusion   in   the   identity   of  
MCDONALD’S  CORP  V.  L.C.  BIG  MAK  (2004)   business.  
Court  decision:    
To   establish   trademark   infringement,   the   following   The   registered   trademark   owner   may   use   his   mark   on  
elements  must  be  shown:     the   same   or   similar   products,   in   different   segments   of  
(1)  the  validity  of  plaintiff’s  mark;     the   market,   and   at   different   price   levels   depending   on  
(2)  the  plaintiff’s  ownership  of  the  mark;  and     variations   of   the   products   for   specific   segments   of   the  
(3)   the   use   of   the   mark   or   its   colorable   market.     The   Court   has   recognized   that   the   registered  
imitation   by   the   alleged   infringer   results   in   trademark   owner   enjoys   protection   in   product   and  
“likelihood  of  confusion.”     market  areas  that  are  the  normal  potential  expansion  of  
  his  business.    Thus,  the  Court  has  declared:  
Of   these,   it   is   the   element   of   likelihood   of   confusion   that      
is  the  gravamen  of  trademark  infringement.   Modern   law   recognizes   that   the   protection   to  
  which   the   owner   of   a   trademark   is   entitled   is  
Section   22   covers   two   types   of   confusion   arising   from   not   limited   to   guarding   his   goods   or   business  
the   use   of   similar   or   colorable   imitation   marks,   namely,   from   actual   market   competition   with   identical  
confusion   of   goods   (product   confusion)   and   confusion   or   similar   products   of   the   parties,   but   extends  
of  business  (source  or  origin  confusion).   to   all   cases   in   which   the   use   by   a   junior  
  appropriator   of   a   trade-­‐mark   or   trade-­‐name   is  
In   confusion   of   goods,   the   ordinarily   prudent   purchaser   likely  to  lead  to  a  confusion  of  source,  as  where  
would  be  induced  to  purchase  one  product  in  the  belief   prospective   purchasers   would   be   misled   into  
that   he   was   purchasing   the   other.   In   confusion   of   thinking   that   the   complaining   party   has  
business,  though  the  goods  of  the  parties  are  different,   extended  his  business  into  the  field  or  is  in  any  
the   defendant’s   product   is   such   as   might   reasonably   be   way   connected   with   the   activities   of   the  
assumed   to   originate   with   the   plaintiff,   and   the   public   infringer;   or   when   it   forestalls   the   normal  
would  then  be  deceived  either  into  that  belief  or  into  the   potential  expansion  of  his  business.  
belief   that   there   is   some   connection   between   the    
plaintiff   and   defendant   which,   in   fact,   does   not   exist.   Test  to  determine  likelihood  of  confusion:  
Thus,   while   there   is   confusion   of   goods   when   the   This   Court,   however,   has   relied   on   the   dominancy   test  
products   are   competing,   confusion   of   business   exists   rather   than   the   holistic   test.   The   dominancy   test  
when   the   products   are   non-­‐competing   but   related   considers  the  dominant  features  in  the  competing  marks  
enough  to  produce  confusion  of  affiliation.   in   determining   whether   they   are   confusingly   similar.      
  Under   the   dominancy   test,   courts   give   greater   weight   to  
Petitioners  claim  that  respondents’  use  of  the  “Big  Mak”   the   similarity   of   the   appearance   of   the   product   arising  
mark   on   respondents’   hamburgers   results   in   confusion   from   the   adoption   of   the   dominant   features   of   the  
of   goods,   particularly   with   respect   to   petitioners’   registered   mark,   disregarding   minor   differences.   Courts  
hamburgers  labeled  “Big  Mac.”  Since  respondents  used   will   consider   more   the   aural   and   visual   impressions  
the  “Big  Mak”  mark  on  the  same  goods,  i.e.  hamburger   created   by   the   marks   in   the   public   mind,   giving   little  
sandwiches,   that   petitioners’   “Big   Mac”   mark   is   used,   weight   to   factors   like   prices,   quality,   sales   outlets   and  
trademark  infringement  through  confusion  of  goods  is  a   market  segments.    
proper   issue   in   this   case.   Respondents   also   admit   that    
their   business   includes   selling   hamburger   sandwiches,   The   test   of   dominancy   is   now   explicitly   incorporated  
the   same   food   product   that   petitioners   sell   using   the   into   law   in   Section   155.1   of   the   Intellectual   Property  
“Big   Mac”   mark.   Thus,   trademark   infringement   through   Code   which   defines   infringement   as   the   “colorable  
confusion  of  business  is  also  a  proper  issue  in  this  case.       imitation   of   a   registered   mark   xxx   or   a   dominant   feature  
  thereof.”  
Respondents   assert   that   their   “Big   Mak”   hamburgers    
cater   mainly   to   the   low-­‐income   group   while   petitioners’  
“Big   Mac”   hamburgers   cater   to   the   middle   and   upper  
income   groups.       Even   if   this   is   true,   the   likelihood   of  

o 115
Katrina Michelle Mancao
 
MCDONALD’S  CORP.  V.  MACJOY  FASTFOOD  CORP.  (2007)   In   contrast,   the   Holistic   or   Totality   Test   necessitates   a  
Court  decision:   consideration  of  the  entirety  of  the  marks  as  applied  to  
Essentially,  the  issue  here  is  whether  there  is  a  confusing   the   products,   including   the   labels   and   packaging,   in  
similarity   between   the   MCDONALD’S   marks   of   the   determining   confusing   similarity.   The   discerning   eye   of  
petitioner   and   the   respondent’s   “MACJOY   &   DEVICE”   the   observer   must   focus   not   only   on   the   predominant  
trademark   when   applied   to   Classes   29   and   30   of   the   words,   but   also   on   the   other   features   appearing   on   both  
International   Classification   of   Goods,   i.e.,   food   and   labels   so   that   the   observer   may   draw   conclusion   on  
ingredients  of  food.   whether  one  is  confusingly  similar  to  the  other.  
     
In   determining   similarity   and   likelihood   of   confusion,   Relative  to  the  question  on  confusion  of  marks  and  trade  
jurisprudence   has   developed   two   tests,   the   dominancy   names,   jurisprudence   has   noted   2   types   of   confusion,  
test   and   the   holistic   test.   The   dominancy   test   focuses   on   viz.:   (1)   confusion   of   goods   (product   confusion),   where  
the  similarity  of  the  prevalent  features  of  the  competing   the   ordinarily   prudent   purchaser   would   be   induced   to  
trademarks  that  might  cause  confusion  or  deception.  In   purchase   one   product   in   the   belief   that   he   was  
contrast,   the   holistic   test   requires   the   court   to   consider   purchasing   the   other;   and   (2)   confusion   of   business  
the   entirety   of   the   marks   as   applied   to   the   products,   (source  or  origin  confusion),  where,  although  the  goods  
including   the   labels   and   packaging,   in   determining   of   the   parties   are   different,   the   product,   the   mark   of  
confusing  similarity.  Under  the  latter  test,  a  comparison   which  registration  is  applied  for  by  one  party,  is  such  as  
of  the  words  is  not  the  only  determinant  factor.   might   reasonably   be   assumed   to   originate   with   the  
    registrant   of   an   earlier   product,   and   the   public   would  
In   trademark   cases,   particularly   in   ascertaining   whether   then   be   deceived   either   into   that   belief   or   into   the   belief  
one   trademark   is   confusingly   similar   to   another,   no   set   that  there  is  some  connection  between  the  two  parties,  
rules   can   be   deduced   because   each   case   must   be   though  inexistent.  
decided   on   its   merits.   In   such   cases,   even   more   than   in    
any   other   litigation,   precedent   must   be   studied   in   the   Applying   the   Dominancy   Test   to   the   case   at   bar,   this  
light  of  the  facts  of  the  particular  case.  That  is  the  reason   Court   finds   that   the   use   of   the   stylized   "S"   by  
why   in   trademark   cases,   jurisprudential   precedents   respondent   in   its   Strong   rubber   shoes   infringes   on   the  
should   be   applied   only   to   a   case   if   they   are   specifically   in   mark  already  registered  by  petitioner  with  the  IPO.  While  
point.       it  is  undisputed  that  petitioner's  stylized  "S"  is  within  an  
  oval   design,   to   this   Court's   mind,   the   dominant   feature  
While   we   agree   with   the   CA’s   detailed   enumeration   of   of  the  trademark  is  the  stylized  "S,"  as  it  is  precisely  the  
differences  between  the  2  competing  trademarks  herein   stylized   "S"   which   catches   the   eye   of   the   purchaser.    
involved,  we  believe  that  the  holistic  test  is  not  the  one   Thus,  even  if  respondent  did  not  use  an  oval  design,  the  
applicable   in   this   case,   the   dominancy   test   being   the   one   mere   fact   that   it   used   the   same   stylized   "S",   the   same  
more   suitable.   In   recent   cases   with   a   similar   factual   being   the   dominant   feature   of   petitioner's   trademark,  
milieu   as   here,   the   Court   has   consistently   used   and   already   constitutes   infringement   under   the   Dominancy  
applied   the   dominancy   test   in   determining   confusing   Test.  
similarity  or  likelihood  of  confusion  between  competing    
trademarks.   Withal,   the   protection   of   trademarks   as   intellectual  
  property   is   intended   not   only   to   preserve   the   goodwill  
and  reputation  of  the  business  established  on  the  goods  
SKETCHERS  V.  INTERPACIFIC  INDUSTRIAL  TRADING  (2011)  
bearing   the   mark   through   actual   use   over   a   period   of  
Court  decision:  
time,   but   also   to   safeguard   the   public   as   consumers  
The   essential   element   of   infringement   under   R.A.   No.  
against  confusion  on  these  goods.  
8293   is   that   the   infringing   mark   is   likely   to   cause  
 
confusion.   In   determining   similarity   and   likelihood   of  
Cases  on  bottles  and  containers  
confusion,   jurisprudence   has   developed   tests   –   the  
Dominancy   Test   and   the   Holistic   or   Totality   Test.   The    
Dominancy   Test   focuses   on   the   similarity   of   the   CAGAYAN  VALLEY  ENTERPRISES  V  CA  (1989)  
prevalent   or   dominant   features   of   the   competing   Court  decision:  
trademarks   that   might   cause   confusion,   mistake,   and   RA   623   extends   protection   to   qualified   manufacturer  
deception   in   the   mind   of   the   purchasing   public.   who   successfully   registered   with   the   PPO   its   duly  
Duplication   or   imitation   is   not   necessary;   neither   is   it   stamped   or   marked   bottles,   boxes,   casks,   and   other  
required  that  the  mark  sought  to  be  registered  suggests   similar   containers.   It   merely   requires   that   bottles,   in  
an   effort   to   imitate.   Given   more   consideration   are   the   order  to  be  eligible  for  registration  must  be  stamped  or  
aural   and   visual   impressions   created   by   the   marks   on   the   marked   with   the   names   of   their   manufacturers   or   the  
buyers   of   goods,   giving   little   weight   to   factors   like   prices,   names   of   their   principals   or   products   or   other   marks   of  
quality,  sales  outlets,  and  market  segments.   ownership.   No   drawings   or   labels   are   required   but  
  instead,  two  photos  of  the  container,  duly  signed  by  the  

116 z
Intellectual Property Law

applicant,   showing   clearly   and   legibly   the   names   and   to   the   backyard,   cottage   and   small-­‐scale   manufacturers  
other  marks  of  ownership  sought  to  be  registered  and  a   of  indigenous  native  products  such  as  patis,  sisi  and  toyo  
bottle   showing   the   name   or   other   mark   of   ownership,   who   do   not   have   the   capital   to   buy   brand   new   bottles   as  
irremovably  stamped  or  marked  shall  be  submitted.     containers   nor   afford   to   pass   the   added   cost   to   the  
  majority   of   the   poor   Filipinos   who   use   the   products   as  
Hard   liquor,   although   regulated,   is   not   probity   by   law,   their  daily  condiments  or  viands.    
hence  it  is  within  the  purview  and  coverage  of  RA  623.      
  b. Unfair  competition  
DISTILLERIA  WASHINGTON  V  CA  (1996)    
Court  decision:   SECTION   168.   UNFAIR   COMPETITION,   RIGHTS,   REGULATION  
RA   623   extends   TM   protection   in   the   use   of   containers  
AND  REMEDIES.  –    
duly  registered  with  the  PPO.  The  mere  use  of  registered  
bottles  or  containers  without  the  written  consent  of  the   168.1.   A   person   who   has   identified   in   the   mind   of   the  
manufacturer   is   prohibited.   Only   exceptions:   usage   as   public   the   goods   he   manufactures   or   deals   in,   his  
container   for   sisi,   bagoong,   patis   and   similar   native   business   or   services   from   those   of   others,   whether   or  
products.     not  a  registered  mark  is  employed,  has  a  property  right  
  in   the   goodwill   of   the   said   goods,   business   or   services  
RA   623   does   not   disallow   the   sale   or   transfer   of   so   identified,   which   will   be   protected   in   the   same  
ownership  of  the  marked  bottles  or  containers.     manner  as  other  property  rights.  
   
While   it   may   be   unwarranted   for   the   registrant   to   simply   168.2.   Any   person   who   shall   employ  deception   or   any  
seize   the   empty   containers   from   one   engaged   in   the   other   means   contrary   to   good   faith   by   which   he   shall  
unauthorized   use   of   said   containers,   it   would   be   legally   pass  off  the  goods  manufactured  by  him  or  in  which  he  
absurd   to   still   allow   the   latter   to   recover   possession   deals,   or   his   business,   or   services   for   those   of   the   one  
thereof—the   practical   and   feasible   alternative   is   to   having   established   such   goodwill,   or   who   shall   commit  
merely  require  the  payment  of  just  compensation  to  the   any  acts  calculated  to  produce  said  result,  shall  be  guilty  
latter.     of   unfair   competition,   and   shall   be   subject   to   an   action  
  therefor.  
DISTILLERIA  WASHINGTON  V.  LA  TONDEÑA  (1997)    
Court  decision:   168.3.   In   particular,   and   without   in   any   way   limiting  
It   is   implied   that   Section   2   and   3   of   RA   623   apply   only   the   scope   of   protection   against   unfair   competition,   the  
when   the   registered   manufacturer,   bottler   or   seller   following  shall  be  deemed  guilty  of  unfair  competition:  
retain   ownership   of   the   bottles   and   when   the   bottles    
have  been  transferred  by  way  of  sale,  Section  5  applies,   (a)   Any   person,   who   is   selling   his   goods  
thereby   precluding   the   institution   of   any   action   under   and   gives   them   the   general   appearance   of  
Sections  2  and  3.     goods   of   another   manufacturer   or   dealer,  
  either   as   to   the   goods   themselves   or   in   the  
The   transferee   of   the   marked   bottles   transferred   by   way   wrapping   of   the   packages   in   which   they   are  
of   sale   is   allowed   to   enjoy   all   the   rights   of   an   owner   in   contained,  or  the  devices  or  words  thereon,  or  
regard   to   such   bottles.   What   is   proscribed   is   the   use   of   in  any  other  feature  of  their  appearance,  which  
bottles   in   infringement   of   another’s   trademark   or   would   be   likely   to   influence   purchasers   to  
incorporeal  rights.     believe   that   the   goods   offered   are   those   of   a  
  manufacturer   or   dealer,   other   than   the   actual  
Where  the  marked  bottles  are  transferred  by  way  of  sale,   manufacturer   or   dealer,   or   who   otherwise  
the   registered   owner   relinquishes   all   its   proprietary   clothes   the   goods   with   such   appearance   as  
rights   over   the   bottles   in   favor   of   the   person   who   shall   deceive   the   public   and   defraud   another  
obtains  them  in  due  course.     of   his   legitimate   trade,   or   any   subsequent  
  vendor   of   such   goods   or   any   agent   of   any  
TWIN  ACE  HOLDINGS  V.  CA  (1997)   vendor   engaged   in   selling   such   goods   with   a  
like  purpose;  
Court  decision:  
 
Registered  containers  of  hard  liquor  (rhum,  gin,  brandy,  
(b)   Any   person   who   by   any   artifice,   or  
and  the  like)  are  protected  by  RA  623.    
device,   or   who   employs   any   other   means  
 
calculated   to   induce   the   false   belief   that   such  
RA   623   was   meant   to   protect   the   intellectual   property  
rights   fo   the   registrants   of   the   containers   and   prevent   person  is  offering  the  services  of  another   who  
has   identified   such   services   in   the   mind   of   the  
unfair  trade  practices  and  fraud  on  the  public.  HOWEVER,  
public;  or  
the  exemption  granted  under  the  same  law  was  deemed  
 
extremely  necessary  to  provide  assistance  and  incentive  

o 117
Katrina Michelle Mancao
 
(c)   Any  person  who  shall  make  any  false   circumstances   of   the   case.   The   affidavit   of   a   former  
statement   in   the   course   of   trade   or   who   shall   employee   of   Universal   attesting   to   the   illegal   sale   and  
commit  any  other  act  contrary  to  good  faith  of   manufacture   of   “Spalding”   balls   and   seized   “Spalding”  
a   nature   calculated   to   discredit   the   goods,   products   and   instruments   from   Universal’s   factory   was  
business  or  services  of  another.   sufficient  prima  facie  evidence  to  warrant  prosecution  of  
  private  respondents.    
168.4.   The   remedies   provided   by   Sections   15610,   15711    
and  16112  shall  apply  mutatis  mutandis.     That  a  corporation  other  than  the  certified  owner  of  the  
  TM   is   engaged   in   the   unauthorized   manufacture   of  
SECTION  232.  APPEALS.  –     products   bearing   the   same   TM   engenders   a   reasonable  
belief   that   a   criminal   offense   for   unfair   competition   is  
232.1.   Appeals   from   decisions   of   regular   courts   shall   be  
being  committed.    
governed   by   the   Rules   of   Court.   Unless   restrained   by   a    
higher   court,   the   judgment   of   the   trial   court   shall   be   To   hold   that   the   act   of   selling   is   an   indispensable  
executory   even   pending   appeal   under   such   terms   and   element   of   the   crime   of   unfair   competition   is   illogical  
conditions  as  the  court  may  prescribe.   because  if  the  law  punishes  the  seller  of  imitation  goods,  
  then   with   more   reason   should   the   law   penalize   the  
232.2.   Unless   expressly   provided   in   this   Act   or   other   manufacturer.      
statutes,   appeals   from   decisions   of   administrative    
officials  shall  be  provided  in  the  Regulations.   UNIVERSAL   RUBBER   PRODUCTS   V.   CA,   CONVERSE   RUBBER  
 
CORPORATION,  EDWARSON  MANUFACTURING  CO.  (1984)  
Difference   between   infringement   and   unfair  
Court  decision:  
competition  
General  rule:  On  obtaining  an  injunction  for  infringement  
  of   a   TM,   complainant   is   entitled   to   an   accounting   and  
DEL   MONTE   CORPORATION   V.   CA   AND   SUNSHINE   SAUCE   recovery  of  defendant’s  profits  on  the  goods  sold  under  
(1990)   the  TM  as  part  of  his  property  right.  This  rule  applies  also  
Court  decision:   in  cases  of  unfair  competition.    
Distinctions   between   infringement   of   TM   and   unfair    
competition.     In  such  case,  the  infringer  or  unfair  trader  is  required  in  
INFRINGEMENT   UNFAIR  COMPETITION   equity  to  account  for  and  yield  up  his  gains  on  a  principle  
Unauthorized  use  of  a  TM   Passing  off  of  one’s  goods   analogous   to   that   which   charges   as   trustee   with   the  
as  those  of  another   profits   acquired   by   the   wrongful   use   of   the   property   of  
Intent  is  unnecessary   Fraudulent   intent   is   the   cestui   que   trust   and   defendant’s   profits   are   regarded  
necessary   as   an   equitable   measure   of   the   compensation   plaintiff  
should  receive  for  the  past  harm  suffered  by  him.    
Prior   registration   of   the   Registration   is   not  
 
TM  is  a  prerequisite  to  the   necessary  
In   recovering   the   loss   suffered   by   the   aggrieved   party  
action  
due  to  “unfair  competition,”  Sec.  23  of  RA  166  grants  the  
 
complainant  three  options  within  which  to  ascertain  the  
Lower   court   and   CA   dismissed   Del   Monte’s   complaint  
amount  of  damages  recoverable  either    
against   Sunshine   Suace   for   infringement   of   TM   and  
[1]   reasonable   profit   with   the   complaining   party   would  
unfair   competition   because   of   Del   Monte’s   failure   to  
have   made,   had   the   defendant   not   infringed   his   said  
establish   defendant’s   malice   or   bad   faith,   which   was   an  
rights  
essential   element   of   infringement   of   TM   or   unfair  
[2]   profit   which   the   defendant   actually   made   out   the  
competition.   SC   reversed   the   lower   courts’   ruling   and  
infringement    
found   Sunshine   guilty   of   both   infringement   (logo   was  
[3]   the   court   may   award   as   damages   a   reasonable  
confusingly   similar)   and   unfair   competition   (refilling   of  
percentage   based   upon   the   amount   of   gross   sales   of   the  
Del  Monte’s  bottles  with  their  own  product).    
defendant  of  the  value  of  the  services  in  connection  with  
 
which   the   mark   or   tradename   was   issued   in   the  
PRO   LINE   SPORTS   CENTER   V.   CA,   UNIVERSAL   ATHLETICS   infringement  of  the  rights  of  the  complaining  party.    
INDUSTRIAL  PRODUCTS  INC,  AND  MONICO  SEHWANI  (1997)    
Court  decision:   Nature  of  complaint  of  unfair  competition:  it’s  basically  
The   existence   of   probable   cause   for   unfair   competition   a  suit  for  “injunction  and  damages”  because…  
by   Universal   is   derivable   from   the   facts   and   -­‐   Injunction:   for   the   purpose   of   enjoining   the   unlawful  
                                                                                                                                                        competitor   from   proceeding   further   with   the   unlawful  
10
 For  infringement   competition    
11
 For  destroying  the  material  
12
 For  cancelation  of  registration  

118 z
Intellectual Property Law

-­‐   Damages:   to   allow   the   aggrieved   party   to   recover   the   SOLID   TRIANGLE   SALES   CORPORATION   V.   SHERIFF   OF   RTC  
damage   he   has   suffered   by   virtue   of   the   said   unlawful  
QC    (2001)  
competition.    
Court  decision:  
 
Sanly  Corporation  did  not  pass  off  the  subject  goods  as  
To   determine   the   amount   of   damages   allowable   only  
that   of   another.   Indeed,   it   admits   that   the   goods   are  
after   final   determination   of   the   unfair   competition   case  
genuine   Mitsubishi   photographic   paper,   which   it  
would   render   nugatory   the   right   of   the   complainant  
purchased   from   a   supplier   in   HK.   Petitioner   alleges   that  
under  Sec.  23  RA  166  but  would  be  a  repetitious  process  
private  respondents  “made  it  appear  that  they  were  duly  
causing  only  unnecessary  delay.    
authorized   to   sell   or   distribute   Mitsubishi   Photo   Paper   in  
 
the   PH.”   Assuming   this   act   constitutes   a   crime,   there   is  
CONVERSE  RUBBER   CORPORATION  V.  JACINTO  RUBBER  AND  
no  proof  to  establish  such  allegation.    
PLASTIC  CO  (1980)    
Court  decision:   SONY   COMPUTER   ENTERTAINMENT   V.   SUPERGREEN,   INC.  
The   shoes   manufactured   by   defendants   (Custombuilt)  
(2007)  
contained   practically   all   the   features   of   those   of   the  
Court  decision:  
plaintiff   (Chuck   Taylors),   except   for   the   respective  
Sony  filed  complaint  with  the  NBI  against  Supergreen  for  
brands,   of   course.   The   respective   designs,   shapes,   colors  
reproduction    and  distribution  of  counterfeit  Playstation  
of   the   ankle   patches,   the   bands,   the   toe   patch   and   the  
console   and   game   software.   Manufacture   is   in   Cavite  
soles  of  the  two  products  are  exactly  the  same  such  that  
and   distribution   is   in   Mandaluyong.   Search   warrant   was  
at  a  distance  of  few  meters,  it  is  impossible  to  distinguish  
issued  by  Manila  RTC.  Supergreen  moved  for  quashal  of  
Custombuilt   from   Chuck   Taylor.   These   elements   are  
the  search  warrants.    
more   than   sufficient   to   serve   as   basis   for   a   charge   of  
 
unfair   competition.   Even   if   not   all   the   details   just  
Court  upheld  the  warrants  for  Mandaluyong  because  it  is  
mentioned  were  identical,  with  the  general  appearances  
within  the  judicial  region  of  Manila.    
alone   of   the   two   products,   any   ordinary   or   even   perhaps  
 
even   a   not   too   perceptive   and   discriminating   customer  
Where   a   person’s   imitation   of   the   general   appearance   of  
could   be   deceived,   and   therefore,   Custombuilt   could  
another   person’s   goods   was   done   allegedly   in   Cavite   but  
easily  be  passed  off  for  Chuck  Taylor.    
is  sold  the  goods  in  Metro  Manila,  the  alleged  acts  would  
 
constitute   a   transitory   or   continuing   offense.   The   instant  
The  statute  on  unfair  competition  extends  protection  to  
cause   involves   a   transitory   or   continuing   offense   of  
the   goodwill   of   the   manufacturer/dealer.   It   is   not  
unfair  competition  under  Sec.  168  of  RA  8293.    
required  that:  
 
Goodwill   sought   to   be   protected   in   an   action   for   unfair  
competition   must   have   been   established   in   an   actual   SEHWANI,   INC   AND   BENITA’S   FRIES,   INC   V.   IN-­‐N-­‐OUT  
competitive  situation.   BURGER,  INC  (2007)  
Deception  or  other  means  contrary  to  good  faith  or  any   Court  decision:  
acts  calculated  to  pass  off  other  goods  for  those  of  one   The   question   of   WON   a   trademark   is   well-­‐known   is  
who   has   established   good   will   must   have   been   factual  in  nature,  involving  as  it  does  the  appreciation  of  
committed  in  an  actual  competitive  situation.     evidence  adduced  before  the  BLA-­‐IPO.    
   
ASIA  BREWERY  V.  CA  (1993)   Section   151   (b)   of   RA   8293   specifically   provides   that   a  
Court  decision:   petition   to   cancel   the   registration   of   a   mark   which   is  
In   resolving   cases   of   infringement   and   unfair   registered   contrary   to   the   provisions   thereof,   or   which   is  
competition,   courts   should   take   into   consideration   used  to  misrepresent  the  source  of  the  goods  or  services  
several   factors   which   would   affect   its   conclusion.   E.g.   may   be   filed   anytime.   Laches   may   not   prevail   against   a  
age,   training   and   education   of   the   usual   purchaser,   the   specific   provision   of   law,   since   equity,   which   has   been  
nature   and   cost   of   the   article,   whether   the   article   is   defined   as   “justice   outside   legality”   is   applied   in   the  
bought   for   immediate   consumption   and   also   the   absence   of   and   not   against   statutory   law   or   rules   of  
conditions  under  which  it  is  usually  purchased.     procedure.    
   
The   ruling   in   Del   Monte   should   not   be   applied   here   COCA-­‐COLA  V.  GOMEZ  (2008)  
because   of   the   consideration   of   the   aforementioned   Court  decision:  
factors.   Beer   is   not   usually   picked   up   from   a   store   shelf   Unfair   competition   previously   defined   in   PH  
but   ordered   by   brand   by   the   beer   drinker   himself   from   jurisprudence   in   relation   with   RA   166   and   Articles   188  
the  storekeeper  or  waiter  in  a  pub  or  restaurant.     and   189   of   RPC   is   now   covered   by   Section   168   IPC   as   this  
  Code  has  expressly  repealed  RA  165,  RA  166  and  Articles  
188   and   189   of   the   RPC.   The   law   does   not   cover   every  

o 119
Katrina Michelle Mancao
 
unfair  act  committed  in  the  course  of  business,  it  covers   to   protect   the   property   rights   of   the  
only   acts   characterized   by   “deception   or   any   other   business.    
means   contrary   to   good   faith”   in   the   passing   off   of   • Bottle   is   very   critical,   that’   the   most  
goods   and   services   as   those   of   another   or   who   has   expensive  component.    
established   goodwill   in   relation   with   these   goods   or  
services   or   any   other   act   calculated   to   produce   the   same  
 
result.  
  SUPERIOR   COMMERCIAL   ENTERPRISES   V.   KUNNAN  
Under  Sec.  168  of  RA  8293,  [1]  deception,  [2]  passing  off   ENTERPRISES  LTD  (2010)  
and   [3]   fraud   upon   the   public   are   still   the   key   elements   Court  decision:  
that  must  be  present  for  unfair  competition  to  exist.     There   can   be   trademark   infringement   without   unfair  
  competition,  such  as  when  the  infringer  discloses  on  the  
“True  test  of  unfair  competition”  –  whether  the  acts  of   labels   containing   the   mark   that   he   manufactures   the  
defendant   are   such   as   calculated   to   deceive   the   ordinary   goods,   thus   preventing   the   public   from   being   deceived  
buyer   making   his   purchases   under   the   ordinary   that  the  goods  originate  from  the  TM  owner.    
conditions  which  prevail  in  the  particular  trade  to  which    
the  controversy  relates.     INFRINGEMENT:    
  Elements  of  trademark  infringement:    
Re  hoarding  (collection  of  the  petitioner’s  empty  bottles   1) Validity  of  the  plaintiff’s  mark  
so  that  they  can  be  withdrawn  from  circulation,  and  thus   2) Plaintiff’s  ownership  of  the  mark  
impede   the   circulation   of   the   petitioner’s   bottled   3) Use   of   the   mark   or   its   colorable   imitation   by  
products)     the   alleged   infringer   results   in   “likelihood   or  
à   The   critical   question   is   not   the   intrinsic   unfairness   of   confusion”  
the   act   of   hoarding—what   is   critical   for   purposes   of   Sec.    
168   (3)   is   to   determine   if   the   hoarding,   as   charged,   is   “of   UNFAIR   COMPETITION:   passing   off   (or   palming   off)   or  
a   nature   calculated   to   discredit   the   goods,   business   or   attempting   to   pass   off   upon   the   public   of   the   goods   or  
services”  of  another.     business   of   one   person   as   the   goods   or   business   of  
  another   with   the   end   and   probable   effect   of   deceive   the  
Given  the  IPC’s  specific  focus,     public.    
FIRST   TEST   [that   should   be   made   when   question   arises   Elements  of  unfair  competition:    
on   whether   a   matter   is   covered   by   the   Code]   -­‐   to   ask   if   it   1) confusing   similarity   in   the   general   appearance  
refers  to  an  IP  right  as  defined  in  the  Code   of  goods  
SECOND  TEST  –  if  a  disputed  matter  does  not  expressly   2) intent   to   deceive   the   public   and   defraud   a  
refer   to   an   IP   right   as   defined   above,   is   whether   it   falls   competitor  
under   the   general   “unfair   competition”   concept   and    
definition  under  Sections  168.1  and  168.2  of  the  IPC     c. Action   for   false   or  
 
Hoarding   of   a   competitor’s   product   does   not   fall   within  
fraudulent  
the   coverage   of   the   IPC   and   of   Sec.   168   in   particular.   It   declaration  
does   not   relate   to   any   patent,   TM,   trade   name   or   service    
that   the   respondents   have   invaded,   intruded   into   or   SECTION   162.   ACTION   FOR   FALSE   OR   FRAUDULENT  
used   without   proper   authority   from   the   petitioner   nor  
DECLARATION.  –    
are  the  respondents  alleged  to  have  fraudulently  passed  
of   their   products   or   services   as   those   of   the   petitioner.     Any   person   who   shall   procure   registration   in   the   Office  
  of   a   mark   by   a   false   or   fraudulent   declaration   or  
Hoarding   for   purposes   of   destruction   is   closer   to   RA   representation,   whether   oral   or   in   writing,   or   by   any  
623’s   provision   (which   has   not   been   expressly   repealed   false  means,   shall   be   liable   in   a   civil   action   by   any   person  
by  RA  8293).     injured   thereby   for   any   damages   sustained   in  
  consequence  thereof    
 
• Hoarding   is   not   within   purview   of   unfair  
competition;   there   could   be   no   probable   cause.   SECTION  163.  JURISDICTION  OF  COURT.  –    
Does   not   cover   unfair   act,   only   the   deception   and   All   actions   under   Sections   150,   155,   164,   and   166   to   169  
contrary  to  good  faith.     shall   be   brought   before   the   proper   courts   with  
• Must  be  calculated  to  discredit     appropriate  jurisdiction  under  existing  laws.    
• Adverse  reputational  impact  on  the  business.        
• Obiter:  in  order  for  unfair  competition  committed,  it  
must   be   related   to   intellectual   property.   It’s   not   just  
passing  off,  it’s  broader  than  that.  Objective  is  really  

120 z
Intellectual Property Law

SECTION   164.   NOTICE   OF   FILING   SUIT   GIVEN   TO   THE   169.2.   Any  goods  marked  or  labelled  in  contravention  
DIRECTOR.  –     of   the   provisions   of   this   Section   shall   not   be   imported  
It   shall   be   the   duty   of   the   clerks   of   such   courts   within   into   the   Philippines   or   admitted   entry   at   any  
one   (1)   month   after   the   filing   of   any   action,   suit,   or   customhouse  of  the  Philippines.  The  owner,  importer,  or  
consignee   of   goods   refused   entry   at   any   customhouse  
proceeding   involving   a   mark   registered   under   the  
under   this   section   may   have   any   recourse   under   the  
provisions   of   this   Act,   to   notify   the   Director   in   writing  
customs  revenue  laws  or  may  have  the  remedy  given  by  
setting   forth:   the   names   and   addresses   of   the   litigants  
and   designating   the   number   of   the   registration   or   this  Act  in  cases  involving  goods  refused  entry  or  seized.  
 
registrations   and   within   one   (1)   month   after   the  
judgment   is   entered   or   an   appeal   is   taken,   the   clerk   of   SECTION  232.  APPEALS.  –    
court   shall   give   notice   thereof   to   the   Office,   and   the   232.1.   Appeals   from   decisions   of   regular   courts   shall   be  
latter   shall   endorse   the   same   upon   the   filewrapper   of   governed   by   the   Rules   of   Court.   Unless   restrained   by   a  
the   said   registration   or   registrations   and   incorporate   the   higher   court,   the   judgment   of   the   trial   court   shall   be  
same  as  a  part  of  the  contents  of  said  filewrapper.     executory   even   pending   appeal   under   such   terms   and  
  conditions  as  the  court  may  prescribe.  
SECTION  232.  APPEALS.  –      
232.1.   Appeals   from   decisions   of   regular   courts   shall   be   232.2.   Unless   expressly   provided   in   this   Act   or   other  
governed   by   the   Rules   of   Court.   Unless   restrained   by   a   statutes,   appeals   from   decisions   of   administrative  
higher   court,   the   judgment   of   the   trial   court   shall   be   officials  shall  be  provided  in  the  Regulations.  
executory   even   pending   appeal   under   such   terms   and    
conditions  as  the  court  may  prescribe.   CIVIL  REMEDIES  
  -­‐ Venue:  Commercial  courts  
232.2.   Unless   expressly   provided   in   this   Act   or   other    
statutes,   appeals   from   decisions   of   administrative   CRIMINAL  REMEDIES  
officials  shall  be  provided  in  the  Regulations.   -­‐ Venue:  Commercial  courts  
   
CAUSES  OF  ACTION:  
d. Action   for   false  
Infringement  v.  Unfair  Competition  
declaration  of  origin     Infringement   Unfair  
  Competition  
SECTION   169.   FALSE   DESIGNATIONS   OF   ORIGIN;   FALSE   Definition   Unauthorized   use   Passing   off   of  
DESCRIPTION  OR  REPRESENTATION.  –     of  the  trademark   one’s   goods   as  
goods  of  another  
169.1.   Any   person   who,   on   or   in   connection   with   any  
Registration   Required   NOT   required   –  
goods   or   services,   or   any   container   for   goods,   uses   in  
that’s   why   it's   the  
commerce   any   word,   term,   name,   symbol,   or   device,   or  
last  resort  
any   combination   thereof,   or   any   false   designation   of  
Fraudulent   Not  required   Required  –  harder  
origin,  false  or  misleading  description  of  fact,  or  false  or  
intent   to  prove  
misleading  representation  of  fact,  which:  
Wrongful   Use   of   mark   Damaged   good  
 
act   without  consent   will   of   the  
(a)   Is   likely   to   cause   confusion,   or   to  
business  
cause   mistake,   or   to   deceive   as   to   the  
 
affiliation,   connection,   or   association   of   such  
Ma’am:   Thus,   better   remedy   is   infringement.   Unfair  
person  with  another  person,  or  as  to  the  origin,  
competition  is  resorted  to  only  after  infringement  failed  
sponsorship,   or   approval   of   his   or   her   goods,  
(if   applicable).   Infringement   may   be   proved   upon  
services,   or   commercial   activities   by   another  
showing  certificate  of  registration.    
person;  or  
 
 
Question:   Why   do   we   still   have   the   remedy   of   unfair  
(b)   In   commercial   advertising   or  
competition   if   registration   is   the   basis   of   the   right?   To  
promotion,   misrepresents   the   nature,  
give  a  remedy  to  persons  with  well-­‐known  mark  but  are  
characteristics,   qualities,   or   geographic   origin  
not  registered  in  the  Philippines.  
of   his   or   her   or   another   person's   goods,  
 
services,   or   commercial   activities,   shall   be  
Basis   of   unfair   competition:   Protection   of   the   goodwill  
liable   to   a   civil   action   for   damages   and  
of  the  business,  which  is  a  property  right.  
injunction   provided   in   Sections   156   and   157   of  
 
this  Act  by  any  person  who  believes  that  he  or  
Section   168.3’s   enumeration   –   merely   illustrative.   168.3  
she  is  or  is  likely  to  be  damaged  by  such  act.  
states   “without   in   any   way   limiting   the   scope   of  
 

o 121
Katrina Michelle Mancao
 
protection  against  unfair  competition.”   FALSE  OR  FRAUDULENT  DECLARATION:  
  Ma’am:   This   is   the   most   superfluous   provision.   Only   a  
Note:   Includes   a   false   statement   in   the   course   of   trade   civil  remedy,  no  criminal  aspect  because  not  included  in  
calculated  to  discredit  the  goods,  business  or  services  of   Section   170.   Thus,   it   should   not   be   used   as   an  
another.  à  Spreading  rumors  about  the  competitor.   independent   cause   of   action.   It   may   be   used   only   as   an  
  alternate  cause  of  action.  
Coca   Cola   case   à   hoarding   does   not   fall   within   the    
purview   of   unfair   competition.   Unfair   competition   does   FALSE   DESIGNATION   OF   ORIGIN   OR   FALSE  
not  cover  every  act  unfair  to  the  business.   DESCRIPTION:  
Ma’am:   Note   the   obiter,   which   states   that   for   Ma’am:   Example   –   In-­‐n-­‐Out   Burger   claiming  
there   to   be   unfair   competition,   the   act   must   be   the  same  as  the  foreign  business.    
related  to  IP.    
  3. Criminal  
Infringement   and   unfair   competition   are   based   on    
CONFUSING   SIMILARITY   between   goods,   business,   or  
SECTION  170.  PENALTIES.  –    
service.  
  Independent   of   the   civil   and   administrative   sanctions  
Tests:   imposed  by  law,  a  criminal  penalty  of  imprisonment  from  
1. Dominancy   test   –   does   not   require   that   every   TWO   (2)   YEARS   TO   FIVE   (5)   YEARS   and   a   fine   ranging  
element   be   similar.   Requires   only   that   the   from   Fifty   thousand   pesos   (P50,000)   to   Two   hundred  
dominant   element   be   similar.   Most   strict   test.   thousand   pesos(P200,000),   shall   be   imposed   on   any  
More  often  than  not,  plaintiff  will  win.   person  who  is  found  guilty  of  committing  any  of  the  acts  
  mentioned   in   Section   155   (Infringement),   Section   168  
2. Holistic  test  –  requires  a  side-­‐by-­‐side  comparison   (Unfair   competition)   and   Subsection   169.1   (False  
of   the   goods.   Also   not   limited   to   the   visual   designation  of  origin).  
similarities   and   differences,   includes   a    
consideration   of   how   the   products   are   SY  V.  CA  (1982)  
presented.   Most   lenient   test.   More   often   than   Court  decision:  
not,  defendant  will  win.   Where  the  accused  was  charged  with  unfair  competition,  
  but   the   TC,   being   of   the   opinion   that   the   crime   is  
Del   Monte   case   à   did   not   specify   how   to   determine   infringement   of   TM,   neither   acquitted   nor   found   the  
which  test  to  use,  but  according  to  Ma’am,  it  provides  a   accused  guilty  but  ordered  the  filing  of  a  new  and  proper  
rational   explanation.   According   to   the   Court,   for   information,  there  can  be  no  double  jeopardy.    
ordinary,   everyday,   inexpensive   goods,   the   test   to   be    
applied   is   the   dominancy   test,   because   buyers   do   not   There   can   be   no   double   jeopardy   where   there   was  
usually  scrutinize  inexpensive  goods.  On  the  other  hand,   neither   acquittal   nor   conviction,   but   merely   an   order   to  
for   expensive   goods,   the   test   to   be   applied   is   the   holistic   file  a  new  information  for  the  correct  offense.    
test,   because   buyers   usually   take   the   time   to   scrutinize    
the  products  before  buying.   A   new   preliminary   investigation   is   not   called   for   where  
  court   orders   filing   of   correct   information   involving   a  
Ma’am:   The   guidelines   as   to   what   test   to   apply   are   not   cognate   offense   (unfair   competition   to   infringement   of  
clear-­‐cut.   The   courts   are   actually   given   wide   latitude   of   TM).    
discretion   to   choose   which   test   to   apply.   It   would    
depend  on  the  court’s  appreciation  of  the  facts  and  the   SAMSON  V.  JUDGE  DAWAY  (2004)  
circumstances  of  each  case.     Court  decision:  
  Which  court  has  jurisdiction  over  criminal  and  civil  cases  
Note:   McDonald’s   case,   SC   held   that   dominancy   test   for  violation  of  intellectual  property  rights?  Regional  Trial  
should  be  applied  because  it  has  legal  basis.     Courts  
   
UNFAIR  COMPETITION:   Sec.   163   of   IPC   states   that   actions   (both   civil   and  
Elements:   criminal)   under   Sec.   150,   155,   164,   166,   167,   168   and   169  
1. Confusing  similarity   shall   be   brought   before   proper   courts   with   appropriate  
2. Fraudulent  intent   jurisdiction   “under   existing   laws.”   The   existing   law  
  referred  to  in  Sec.  163  is  Sec.  27  of  RA  166.    
Solid   Triangle   case*   à   no   unfair   competition   because    
the   goods   were   genuine.   Other   party   was   a   parallel   RA   166   is   not   repealed   by   RA   8293   because   of   the  
importer  of  genuine  products.   wording   of   the   repealing   clause   in   239.1   wherein   it  
  mentioned  “parts  of  Acts”  and  “inconsistent  therewith.”  

122 z
Intellectual Property Law

The   use   of   such   phrases   only   mean   that   the   repeal   CONVERSE   RUBBER   CORPORATION   V.   UNIVERSAL   RUBBER  
pertains   only   to   provisions   which   are   repugnant   or   not  
PRODUCTS  (1987)  
susceptible  of  harmonization  with  RA  8293.    
Court  decision:  
 
WON   the   respondent’s   partial   appropriation   of  
YAO,  SR.  V.  PEOPLE  OF  THE  PHILIPPINES  (2007)  
petitioner’s  corporate  name  is  of  such  character  that  it  is  
Court  decision:   calculated  to  deceive  or  confuse  the  public  to  the  injury  
Corporation   owned   by   Yao   family   were   allegedly   of  the  petitioner  to  which  the  name  belongs?    
engaged   in   unlawful   refilling   and   selling   of   LPG   tanks    
owned   by   Shell   and   Petron.   A   search   warrant   was   issued   A   tradename   is   any   individual   name   or   surname,   firm  
against   their   store   by   virtue   of   NBI   seized   several   name,   device   or   word   used   by   manufacturers,  
cylindrical   LPG   tanks.   Yao   family   demands   return   of   the   industrialists,   merchants   and   others   to   identify   their  
items   seized   by   the   raiding   team   considering   that   no   businesses,  vocations  or  occupations.  As  the  tradename  
action   for   infringement   has   been   filed   against   them   or   refers   to   the   business   and   its   goodwill,   the   trademark  
their   corporation,   invoking   Sec.   20   of   AM   02-­‐01-­‐06-­‐SC     refers  to  the  goods.    
which  provides  that:      
  There   is   a   basic   similarity   between   the   tradenames  
Sec.   20.   Failure   to   file   complaint.   –   The   writ   “Universal  Converse  and  Device”  and  “Converse  Rubber  
shall   also,   upon   motion   of   the   expected   Corporation”   as   in   both   names   CONVERSE   is   the  
adverse   party,   be   set   aside   and   the   seized   dominant   word   and   both   parties   engaged   in   similar  
documents   and   articles   returned   to   the   business.    
expected   adverse   party   if   no   case   is   filed   with    
the   appropriate   court   or   authority   within   31   A  tradename  may  not  be  similar  to  a  corporate  name.  –  A  
calendar   days     from   the   date   of   the   issuance   of   corporation  is  entitled  to  the  cancellation  of  a  mark  that  
the  writ.     is   confusingly   similar   to   its   corporate   name.    
  Appropriation   by   another   of   the   dominant   part   of   a  
AM   02-­‐1-­‐06-­‐SC   is   not   applicable   in   the   present   case   corporate  name  is  an  infringement.    
because   it   governs   only   searches   and   seizures   in   civil    
actions   for   infringement   of   intellectual   property   rights.   The  determinative  factor  in  ascertaining  whether  or  not  
The   offense   complained   of   herein   is   for   a   criminal   marks   are   confusingly   similar   to   each   other   is   “not  
violation  of  Sec.  155  in  relation  Sec.  170  of  RA  8293.     whether   the   challenged   mark   would   actually   cause  
  confusion   or   deception   of   the   purchasers   but   whether  
Samson  v.  Daway  case  à  justified  the  jurisdiction  of  the   use  of  such  mark  would  likely  cause  confusion  or  mistake  
RTC  on  the  basis  of  RA  166,  in  connection  with  IP  Code’s   on  the  party  of  the  buying  public.”    
repealing  clause.    
Ma’am:   RA   166   was   resurrected!   But   actually,   The  risk  of  damage  is  not  limited  to  a  possible  confusion  
since   the   IP   Code   was   a   codification   effort,   of  goods  but  also  includes  confusion  of  reputation  if  the  
there  was  actually  an  intent  to  repeal  the  entire   public   could   reasonably   assume   that   the   goods   of   the  
Code.   parties  originated  from  the  same  source.    
   
Better  resolution  would  have  been  that  the  SC   Converse   Rubber   Corporation   has   already   earned   a  
has  a  delegated  power  under  BP  129  .   business   reputation   in   this   country   which   should   be  
  protected.  
Ownership   and   search   warrant   validity.    
Ownership   is   not   material.   Truck   was   seized   2. What  may  not  be  used  as  a  
and  even  if  it  did  not  belong,  as  long  as  it  is  
found   in   crime   scene.   Ownership   would   not   tradename?  
matter.      
  SECTION  165.1.    
J. Tradenames   A   name   or   designation   may   not   be   used   as   a   trade   name  
  if   by   its   nature   or   the   use   to   which   such   name   or  
1. Definition   designation  may  be  put,  it  is  contrary  to  public  order  or  
morals   and   if,   in   particular,   it   is   liable   to   deceive   trade  
 
circles   or   the   public   as   to   the   nature   of   the   enterprise  
SECTION  121.3.     identified  by  that  name.  
"TRADE  NAME"  means  the  name  or  designation  identifying    
or  distinguishing  an  enterprise  
 

o 123
Katrina Michelle Mancao
 
3. Rights   of   the   tradename   This   principle   proceeds   upon   the   theory   that   it   is   fraud  
on   the   corporation   which   has   acquired   a   right   to   that  
owner  
name   and   perhaps   carried   on   its   business   thereunder  
 
that   another   should   attempt   to   use   the   same   name   or  
SECTION  165.2.   the  same  name  with  a  slight  variation  in  such  a  way  as  to  
(a)   Notwithstanding   any   laws   or   regulations   providing   induce  persons  to  deal  with  it  in  the  belief  that  they  are  
for   any   obligation   to   register   trade   names,   such   names   dealing   with   the   corporation   which   has   given   a  
shall  be  protected,  even  prior  to  or  without  registration,   reputation  to  the  name.    
against  any  unlawful  act  committed  by  third  parties.    
  ARMCO  STEEL  CORPORATION  V.  SEC  (1987)  
(b)  In  particular,  any  subsequent  use  of  the  trade  name  
Court  decision:  
by   a   third   party,   whether   as   a   trade   name   or   a   mark   or  
A   trademark   is   entitled   to   protection   in   the   use   thereof  
collective  mark,  or  any  such  use  of  a  similar  trade  name  
in   the   Philippines.   The   owner   has   the   right   to   the  
or   mark,   likely   to   mislead   the   public,   shall   be   deemed  
exclusive  use  and  enjoyment  of  the  term.    
unlawful.  
 
 
Armco   Steel-­‐Philippines   and   Armco   Steel-­‐Ohio   have   not  
SECTION  165.3.   only   identical   name   but   also   a   similarity   in   line   of  
The   remedies   provided   for   in   Sections   153   to   156   and   business.   People   who   are   buying   and   using   products  
Sections  166  and  167  shall  apply  mutatis  mutandis.   bearing  the  TM  Armco  might  be  led  to  believe  that  such  
  products   are   manufactured   by   the   respondent,   when   in  
SECTION  165.4.   fact,  they  might  actually  be  produced  by  the  petitioners.  
Any   change   in   the   ownership   of   a   trade   name   shall   be   Thus,   the   goodwill   that   should   grow   and   inure   to   the  
made  with  the  transfer  of  the  enterprise  or  part  thereof   benefit   of   the   petitioners   could   be   impaired   and  
identified   by   that   name.   The   provisions   of   Subsections   prejudiced  by  the  continued  use  of  the  same  term  by  the  
149.2  to  149.4  shall  apply  mutatis  mutandis.   respondent.    
   
PHILIPS  EXPORT  V.  CA  (1992)   WESTERN  EQUIPMENT  &  SUPPLY  CO  V.  REYES  (1927)  
Court  decision:   Court  decision:  
A   corporation’s   right   to   use   its   corporate   and   trade   An   unregistered   foreign   corporation   which   has   not  
name   is   a   property   right,   a   right   in   rem   which   it   may   personally   transacted   in   the   PH   but   which   has   acquired  
assert  and  protect  against  the  world  in  the  same  manner   valuable   goodwill   and   high   reputation   therein   through  
as   it   may   protect   its   tangible   property,   real   or   personal   the   sale   by   importers   and   the   extensive   use   within   the  
against  trespass  or  conversion.  It  is  regarded  to  a  certain   territory   of   its   products   bearing   either   its   corporate  
extent,   as   a   property   right   and   one   which   cannot   be   name  or  TM  has  a  legal  right  to  restrain  an  officer  of  the  
impaired   or   defeated   by   subsequent   appropriation   by   government,   who   has   full   knowledge   of   those   facts,  
another  corporation  in  the  same  field.     from   issuing   a   certificate   of   incorporation   to   residents   of  
  the  PH  who  are  attempting  to  organize  a  corporation  for  
GR   as   to   corporation:   Each   corporation   must   have   a   the   purpose   of   pirating   the   corporate   name   of   the  
name   by   which   it   is   to   sue   and   be   sued   and   do   all   legal   foreign   corporation   and   of   engaging   in   the   same  
acts.  A  corporation  can  no  more  use  a  corporate  name  in   business,   for   the   purpose   of   making   the   public   believe  
violation   of   the   right   of   others   than   an   individual   can   use   that  the  goods  which  it  proposes  to  sell  are  the  goods  of  
his   name   legally   acquired   so   as   to   mislead   the   public   and   the   foreign   corporation   and   of   defrauding   it   and   its   local  
injure  another.     dealers  of  their  legitimate  trade.    
   
The  right  to  the  exclusive  use  of  a  corporate  name  with   The   purpose   of   such   a   suit   is   to   protect   its   reputation,  
freedom   from   infringement   by   similarity   is   determined   corporate   name   and   goodwill   which   have   been  
with  priority  of  adoption.     established  through  the  natural  development  of  its  trade  
  over   a   long   period   of   years,   in   the   doing   of   which   it   does  
In   determining   the   existence   of   confusing   similarity   in   not   seek   to   enforce   any   legal   or   contract   rights   arising  
corporate  name,  the  test  is  whether  the  similar  is  such  as   from   or   growing   out   of   any   business   which   it   has  
to   mislead   a   person   using   ordinary   care   and   transacted  in  the  Ph.    
discrimination.      
  Under   such   state   of   facts,   the   right   to   the   use   of   the  
A   corporation   has   an   exclusive   right   to   the   use   of   its   corporate  and  trade  name  of  the  foreign  corporation  is  a  
name   which   may   be   protected   by   injunction   upon   a   property   right,   a   right   in   rem,   which   it   may   assert   and  
principle   similar   to   that   upon   which   persons   are   protect   in   any   of   the   courts   of   the   world   even   in  
protected   in   the   use   of   trademarks   and   tradenames.   à   countries   where   it   does   not   personally   transact   any  
business.  

124 z
Intellectual Property Law

  licensed   by,   or   sourced   from,   Harvard   University   without  


It  is  the  trade  and  not  the  mark  that  is  to  be  protected,   the  latter's  consent.  
and   a   TM   does   not   acknowledge   any   territorial    
boundaries,   but   extends   to   every   market   where   the   TRADE  NAME:  
trader’s  goods  have  become  known  and  identified  by  the   Old  law:  Trademark  same  as  trade  name.  
use  of  the  mark.     IP  Code:  Trade  name  is  different.  
   
FREDCO  MANUFACTURING   CORPORATION  V  PRESIDENT  AND   Trade   name   is   a   name   or   designation   identifying   an  
FELLOWS  OF  HARVARD  COLLEGE  (2011)   enterprise.  
 
Court  decision:  
Differences   in   treatment   between   trademark   and   trade  
Article  8  of  the  Paris  Convention  has  been  incorporated  
name:  
in  Section  37  of  R.A.  No.  166,  as  follows:  
-­‐ REGISTRATION   –   Under   the   IP   Code,  
 
registration   is   not   required   to   acquire  
Article   8,   Paris   Convention   -­‐-­‐   A   trade   name   shall  
protection   of   trade   name.   The   basis   of  
be   protected   in   all   the   countries   of   the   Union  
without   the   obligation   of   filing   or   registration,  
protection  is  prior  use  in  Philippine  commerce.  
whether  or  not  it  forms  part  of  a  trademark.   You   can   set   this   up   against   the   trademark  
  owner.  (Note  168.3)  
Section  37,  RA  166.  Rights   of   foreign   registrants.   -­‐-­‐    
Persons   who   are   nationals   of,   domiciled   in,   or   -­‐ REMEDIES   AVAILABLE   –   Note   the   problem  
have   a   bona   fide   or   effective   business   or   with  non-­‐registration  of  trade  name:  There  is  a  
commercial  establishment  in  any  foreign  country,   limitation   of   remedy   available   to   the   trade  
which  is  a  party  to  any  international  convention  or   name   owner.     Trade   name   owner   does   not  
treaty   relating   to   marks   or   trade-­‐names,   or   the   have   criminal   remedy.   Only   civil   and  
repression   of   unfair   competition   to   which   the   administrative.    
Philippines   may   be   a   party,   shall   be   entitled   to   the    
benefits   and   subject   to   the   provisions   of   this   Act   -­‐ ASSIGNABILITY   –   Trade   name   is   NOT   separate  
to   the   extent   and   under   the   conditions   essential   to   the   business.   Trade   name   may   be  
to  give  effect  to  any  such  convention  and  treaties   transferred   only   with   the   transfer   of   the  
so   long   as   the   Philippines   shall   continue   to   be   a  
business.   Trademark   or   service   mark   may   be  
party   thereto,   except   as   provided   in   the   following  
assigned  independently  of  the  business.  
paragraphs  of  this  section.    
 
 
xxxx   SERVICE  MARK:  Trademark  used  in  services.    
   
Trade-­‐names   of   persons   described   in   the   first   K. Collective  Marks  
paragraph   of   this   section   shall   be   protected    
without   the   obligation   of   filing   or   registration   1. Definition  
whether  or  not  they  form  parts  of  marks.  
 
xxxx  
  SECTION  121.2.  
Thus,   under   Philippine   law,   a   trade   name   of   a   national   of   "COLLECTIVE   MARK"   means   any   visible   sign   designated   as  
a   State   that   is   a   party   to   the   Paris   Convention,   whether   such   in   the   application   for   registration   and   capable   of  
or   not   the   trade   name   forms   part   of   a   trademark,   is   distinguishing   the   origin   or   any   other   common  
protected   “without   the   obligation   of   filing   or   characteristic,   including   the   quality   of   goods   or   services  
registration.”   of   different   enterprises   which   use   the   sign   under   the  
  control  of  the  registered  owner  of  the  collective  mark  
“Harvard”   is   the   trade   name   of   the   world   famous    
Harvard  University,  and  it  is  also  a  trademark  of  Harvard   SECTION  167.  COLLECTIVE  MARKS.  –    
University.   Under   Article   8   of   the   Paris   Convention,   as   167.1.  Subject  to  Subsections  167.2  and  167.3,  Sections  122  
well   as   Section   37   of   R.A.   No.   166,   Harvard   University   is  
to   164   and   166   shall   apply   to   collective   marks,   except  
entitled  to  protection  in  the  Philippines  of  its  trade  name  
that   references   therein   to   "mark"   shall   be   read   as  
"Harvard"  even  without  registration  of  such  trade  name  
"collective  mark".  
in  the  Philippines.  This  means  that  no  educational  entity  
 
in   the   Philippines   can   use   the   trade   name   "Harvard"  
167.2.  (a)  An   application   for   registration   of   a   collective  
without  the  consent  of  Harvard  University.  Likewise,  no  
entity  in  the  Philippines  can  claim,  expressly  or  impliedly   mark   shall   designate   the   mark   as   a   collective   mark   and  
through   the   use   of   the   name   and   mark   "Harvard,"   that   shall  be  accompanied  by  a  copy  of  the  agreement,  if  any,  
its   products   or   services   are   authorized,   approved,   or   governing  the  use  of  the  collective  mark.  
 

o 125
Katrina Michelle Mancao
 
(b)   The   registered   owner   of   a   collective   mark   shall   notify   VII. Law  on  Patents  
the   Director   of   any   changes   made   in   respect   of   the    
agreement  referred  to  in  paragraph  (a).   PATENTS  
  This   field   gives   rise   to   serious   issues   on   access.   Unlike  
167.3.   In   addition   to   the   grounds   provided   in   Section   149,   copyright   that   has   a   very   low   standard,   patent   is   very  
the   Court   shall   cancel   the   registration   of   a   collective   exclusionary.   Under   patents,   a   parallel   invention   is   not  
mark   if   the   person   requesting   the   cancellation   proves   allowed.   Patent   is   a   monopoly.   Patent   owner   is   given  
that   only   the   registered   owner  uses  the  mark,  or  that  he   exclusive  right  to  import.  
uses   or   permits   its   use   in   contravention   of   the    
agreements   referred   to   in   Subsection   166.2   or   that   he   Patents   are   costly   to   the   society.   Issuance   of   patent  
uses   or   permits   its   use   in   a   manner   liable   to   deceive   means   that   the   state   allows   the   patent   holder   to  
trade   circles   or   the   public   as   to   the   origin   or   any   other   maximize   the   economic   value   of   the   invention   to   the  
common   characteristics   of   the   goods   or   services   exclusion   of   everybody   else.   Thus,   for   patents   to   work,  
concerned.   the  state  must  ensure  that  it  is  given  only  to  the  one  who  
  deserves  it.  
167.4.   The   registration   of   a   collective   mark,   or   an    
application  therefor  shall  not  be  the  subject  of  a  license   In  the  Philippines,  the  patent  system  is  identified  as  the  
contract.   major   cause   of   the   high   prices   in   medicines.   Philippines  
  are   among   the   states   that   has   the   highest   patent  
COLLECTIVE  MARK   standards,  even  before  it  was  required  to  do  so.    
Mark   designed   to   be   used   by   parties   other   than   the    
registrant.   This   is   usually   used   by   cooperatives.   The   Philippines   also   did   not   adopt   the   flexibilities   under   the  
owner   of   the   mark   is   the   one   who   registers,   but   the   TRIPS   Agreement.   For   example,   under   the   TRIPS,   the  
members   of   the   cooperative   are   the   ones   who   use   it.   country   is   allowed   to   adopt   any   exhaustion   principle   it  
The  registrant  only  holds  it.  But  anyone  who  qualifies  to   chooses.  A  country  may  use  the  international  exhaustion  
be  a  member  (or  meets  the  standard  of  the  agreement)   principle.  The  Philippines,  however,  decided  to  adopt  the  
may  use  the  collective  mark.   domestic  exhaustion  principle.  
   
In  Canada,  this  is  used  to  protect  the  products   Purpose   of   the   patent   –   to   promote   the   advancement   of  
of  indigenous  communities.   technology.  It  was  established  as  an  incentive  system  to  
  encourage   innovation.   But   the   ultimate   objective   is   to  
In  the  Philippines,  it  has  not  been  used.   reward  the  public.  
   
KINDS  OF  PATENTS:  
1. Invention  
2. Utility  model  
3. Industrial  design  
4. Layout  circuits  –  it  was  designed  to  encourage  semi-­‐
conductor   companies   to   invest   in   the   Philippines  
(but  as  of  date,  no  registration  for  this).  
 
STANDARDS  OF  PATENT  PROTECTION:  
-­‐ Key  to  effective  patent  protection.  
-­‐ Note:   Different   rules   apply   to   different  
countries.   You   might   have   to   consider   this   in  
advising  clients  on  registration.  
 
1. Novelty  
-­‐ Not  part  of  prior  art.  Prior  art  is  anything  made  
available  on  any  part  of  the  world.  
-­‐ Anything   found   in   the   public   domain   is   not  
novel   part.   Anything   also   part   of   prior  
application  is  part  of  prior  art.  
-­‐ Philippine   law   exception   –   1   year   of   non-­‐
prejudicial  disclosure.  
 
2. Inventiveness  
-­‐ Anything   that   is   not   obvious   to   a   person   skilled  

126 z
Intellectual Property Law

in  the  art.   SECTION  236.  PRESERVATION  OF  EXISTING  RIGHTS.  –    


-­‐ “Person   skilled   in   the   art”   has   no   uniform   Nothing   herein   shall   adversely   affect   the   rights   on   the  
definition.   Here   in   the   Philippines,   definition   is   enforcement   of   rights   in   patents,   utility   models,  
textbook.   In   some   countries,   term   limited   to   industrial   designs,   marks   and   works,   acquired   in   good  
those  with  doctorate  degree.   faith  prior  to  the  effective  date  of  this  Act.    
-­‐ “Evergreening   of   a   patent”   –   attempt   to    
prolong   the   patent   monopoly   by   RA   9502   -­‐   UNIVERSALLY   ACCESSIBLE   CHEAPER   AND  
changing  one  element.  
  QUALITY  MEDICINES  ACT  OF  2008    
3. Industrial  applicability   -­‐ amended   certain   provision   of   the   Intellectual  
-­‐ Has  some  use.     Property  Code  
   
Industrial   SECTION  47.  SEPARABILITY  CLAUSE.  –    
  Novelty   Inventiveness  
Applicability   Any  portion  or  provision  of  this  Act  that  may  be  declared  
Invention   unconstitutional   or   invalid   shall   not   have   the   effect   of  
ü   ü   ü  
  nullifying  other  portions  and  provisions  hereof  as  long  as  
Utility  
ü   û   ü   such  remaining  portion  or  provision  can  still  subsist  and  
model  
be  given  effect  in  their  entirety.  
Industrial  
ü   û   û    
design  
Layout   SECTION  48.  REPEALING  CLAUSE.  –    
ü   (originality)    
 
circuit   All   laws,   decrees,   executive   orders,   proclamations   and  
  administrative   regulations   or   parts   thereof   inconsistent  
A. Legislative  history   herewith  are  hereby  repealed  or  modified  accordingly.  
   
SECTION  49.  EFFECTIVITY  CLAUSE.  –    
REPUBLIC   ACT   165   -­‐   AN   ACT   CREATING   A   PATENT   OFFICE,  
This   Act   shall   take   effect   fifteen   (15)   days   after   its  
PRESCRIBING   ITS   POWERS   AND   DUTIES,   REGULATING   THE   publication  in  at  least  two  (2)  national  papers  of  general  
ISSUANCE   OF   PATENTS,   AND   APPROPRIATING   FUNDS   circulation.  
THEREFOR    
-­‐ Took  effect  on  20  June  1947   B. Definition  
   
SECTION  239.  REPEALS.  –     “The   right   of   monopoly   secured   by   statute   to   those   who  
239.1.   All   Acts   and   parts   of   Acts   inconsistent   herewith,   invent  or  discover  new  and  useful  devices  and  processes.  
The  exclusive  right  of  manufacture,  sale,  or  use  secured  
more   particularly   Republic   Act   No.   165,   as   amended;  
by   statute   to   one   who   invents   or   discovers   a   new   and  
Republic   Act   No.   166,   as   amended;   and   Articles   188   and  
useful  device  or  process.”  (US  Dictionary)  
189   of   the   Revised   Penal   Code;   Presidential   Decree   No.  
 
49,   including   Presidential   Decree   No.   285,   as   amended,  
are  hereby  repealed.   C. Purpose  
   
SECTION   235.   APPLICATIONS   PENDING   ON   EFFECTIVE   DATE   MANZANO  V.  CA  (1997)  
OF  ACT.  –     Court  decision:  
The   primary   purpose   of   the   patent   system   is   not   the  
235.1.  All  applications  for  patents  pending  in  the  Bureau  
reward  of  the  individual  but  the  advancement  of  the  arts  
of  Patents,  Trademarks  and  Technology  Transfer  shall  be   and   sciences.   The   function   of   a   patent   is   to   add   to   the  
proceeded   with   and   patents   thereon   granted   in   sum  of  useful  knowledge  and  one  of  the  purposes  of  the  
accordance   with   the   Acts   under   which   said   applications   patent   system   is   to   encourage   dissemination   of  
were   filed,   and   said   Acts   are   hereby   continued   to   be   information   concerning   discoveries   and   inventions.   This  
enforced,   to   this   extent   and   for   this   purpose   only,   is   a   matter   which   is   properly   within   the   competence   of  
notwithstanding   the   foregoing   general   repeal   thereof:   the   Patent   Office   the   official   action   of   which   has   the  
Provided,   That   applications   for   utility   models   or   presumption   of   correctness   and   may   not   be   interfered  
industrial   designs   pending   at   the   effective   date   of   this   with   in   the   absence   of   new   evidence   carrying   thorough  
Act,   shall   be   proceeded   with   in   accordance   with   the   conviction   that   the   Office   has   erred.   Since   the   Patent  
provisions   of   this   Act,   unless   the   applicants   elect   to   Office   is   an   expert   body   preeminently   qualified   to  
prosecute   said   applications   in   accordance   with   the   Acts   determine   questions   of   patentability,   its   findings   must  
under  which  they  were  filed.   be   accepted   if   they   are   consistent   with   the   evidence,  
  with   doubts   as   to   patentability   resolved   in   favor   of   the  
Patent  Office.  

o 127
Katrina Michelle Mancao
 
  advances   in   technology   and   design,   in   return  
PEARL  &  DEAN  (PHIL.),  INC.  V.  SHOEMART,  SUPRA   for   the   exclusive   right   to   practice   the   invention  
Refresher:   for  a  number  of  years.    The  inventor  may  keep  
Light  boxes   his   invention   secret   and   reap   its   fruits  
  indefinitely.     In   consideration   of   its   disclosure  
Doctrine:   and  the  consequent  benefit  to  the  community,  
On  patent  infringement:   the  patent  is  granted.    An  exclusive  enjoyment  
For  some  reason  or  another,  petitioner  never  secured  a   is   guaranteed   him   for   17   years,   but   upon   the  
patent   for   the   light   boxes.     It   therefore   acquired   no   expiration  of  that  period,  the  knowledge  of  the  
patent  rights  which  could  have  protected  its  invention,  if   invention   inures   to   the   people,   who   are   thus  
in   fact   it   really   was.     And   because   it   had   no   patent,   enabled  to  practice  it  and  profit  by  its  use.”  
petitioner   could   not   legally   prevent   anyone   from    
manufacturing   or   commercially   using   the   contraption.     In   The   patent   law   has   a   three-­‐fold   purpose:   “first,   patent  
Creser   Precision   Systems,   Inc.   vs.   Court   of   Appeals,   we   law   seeks   to   foster   and   reward   invention;   second,   it  
held  that  “there  can  be  no  infringement  of  a  patent  until   promotes   disclosures   of   inventions   to   stimulate   further  
a   patent   has   been   issued,   since   whatever   right   one   has   to   innovation   and   to   permit   the   public   to   practice   the  
the  invention  covered  by  the  patent  arises  alone  from  the   invention   once   the   patent   expires;   third,   the   stringent  
grant  of  patent.  An  inventor  has  no  common  law  right  to   requirements   for   patent   protection   seek   to   ensure   that  
a   monopoly   of   his   invention.   He   has   the   right   to   make   ideas  in  the  public  domain  remain  there  for  the  free  use  
use   of   and   vend   his   invention,   but   if   he   voluntarily   of  the  public.”  
discloses   it,   such   as   by   offering   it   for   sale,   the   world   is    
free  to  copy  and  use  it  with  impunity.    A  patent,  however,   It  is  only  after  an  exhaustive  examination  by  the  patent  
gives   the   inventor   the   right   to   exclude   all   others.     As   a   office   that   a   patent   is   issued.     Such   an   in-­‐depth  
patentee,   he   has   the   exclusive   right   of   making,   selling   or   investigation   is   required   because   “in   rewarding   a   useful  
using  the  invention.  On  the  assumption  that  petitioner’s   invention,   the   rights   and   welfare   of   the   community   must  
advertising   units   were   patentable   inventions,   petitioner   be  fairly  dealt  with  and  effectively  guarded.    To  that  end,  
revealed   them   fully   to   the   public   by   submitting   the   the   prerequisites   to   obtaining   a   patent   are   strictly  
engineering  drawings  thereof  to  the  National  Library.   observed  and  when  a  patent  is  issued,  the  limitations  on  
  its  exercise  are  equally  strictly  enforced.    To  begin  with,  a  
To  be  able  to  effectively  and  legally  preclude  others  from   genuine   invention   or   discovery   must   be   demonstrated  
copying   and   profiting   from   the   invention,   a   patent   is   a   lest   in   the   constant   demand   for   new   appliances,   the  
primordial  requirement.    No  patent,  no  protection.    The   heavy  hand  of  tribute  be  laid  on  each  slight  technological  
ultimate  goal  of  a  patent  system  is  to  bring  new  designs   advance  in  art.”  
and   technologies   into   the   public   domain   through    
disclosure.   Ideas,   once   disclosed   to   the   public   without   There   is   no   such   scrutiny   in   the   case   of   copyrights   nor  
the   protection   of   a   valid   patent,   are   subject   to   any   notice   published   before   its   grant   to   the   effect   that   a  
appropriation  without  significant  restraint.   person   is   claiming   the   creation   of   a   work.     The   law  
  confers  the  copyright  from  the  moment  of  creation  and  
On   one   side   of   the   coin   is   the   public   which   will   benefit   the  copyright  certificate  is  issued  upon  registration  with  
from   new   ideas;   on   the   other   are   the   inventors   who   the  National  Library  of  a  sworn  ex-­‐parte  claim  of  creation.  
must  be  protected.    As  held  in  Bauer  &  Cie  vs.  O’Donnel,    
“The   act   secured   to   the   inventor   the   exclusive   right   to   D. What  are  patentable?  
make   use,   and   vend   the   thing   patented,   and    
consequently   to   prevent   others   from   exercising   like   1. Inventions  
privileges   without   the   consent   of   the   patentee.     It   was    
passed   for   the   purpose   of   encouraging   useful   invention  
SECTION  21.  PATENTABLE  INVENTIONS.  –    
and   promoting   new   and   useful   inventions   by   the  
Any  technical  solution  of  a  problem  in  any  field  of  human  
protection  and  stimulation  given  to  inventive  genius,  and  
activity   which   is   new,   involves   an   inventive   step   and   is  
was   intended   to   secure   to   the   public,   after   the   lapse   of  
industrially   applicable   shall   be   Patentable.   It   may   be,   or  
the   exclusive   privileges   granted   the   benefit   of   such  
may  relate  to,  a  product,  or  process,  or  an  improvement  
inventions  and  improvements.”  
of  any  of  the  foregoing.  
 
 
The  law  attempts  to  strike  an  ideal  balance  between  the  
two  interests:  
 
“The   patent   system   thus   embodies   a   carefully  
crafted   bargain   for   encouraging   the   creation  
and   disclosure   of   new   useful   and   non-­‐obvious  

128 z
Intellectual Property Law

Standards:   MAGUAN  V.  CA  (1986)  


a. Novelty   Refresher:  
  Patent  for  powder  puff  (in  cosmetics)  
 
SECTION  23.  NOVELTY.  –    
Doctrine:  
An  invention  shall  not  be  considered  new  if  it  forms  part  
It   has   been   repeatedly   held   that   an   invention   must  
of  a  prior  art.    
possess   the   essential   elements   of   novelty   ,   originality  
 
and   precedence   and   for   the   patentee   to   be   entitled   to  
SECTION  24.  PRIOR  ART.  –     protection,   the   invention   must   be   new   to   the   world.  
Prior  art  shall  consist  of:   Accordingly,   a   single   instance   of   public   use   of   the  
  invention   by   a   patentee   for   more   than   two   years   (now  
24.1.   Everything   which   has   been   made   available   for   more   than   one   year   only   under   Sec.   9   of   the   Patent  
to  the  public  anywhere  in  the  world,  before  the   Law)   before   the   date   of   his   application   for   his   patent,  
filing  date  or  the  priority  date  of  the  application   will  be  fatal  to,  the  validity  of  the  patent  when  issued.  
claiming  the  invention;  and    
  It   will   be   noted   that   the   validity   of   petitioner's   patents   is  
24.2.  The  whole  contents  of  an  application  for  a   in   question   for   want   of   novelty.   Private   respondent  
patent,   utility   model,   or   industrial   design   contends  that  powder  puffs  Identical  in  appearance  with  
registration,   published   in   accordance   with   this   that   covered   by   petitioner's   patents   existed   and   were  
Act,   filed   or   effective   in   the   Philippines,   with   a   publicly   known   and   used   as   early   as   1963   long   before  
filing   or   priority   date   that   is   earlier   than   the   petitioner   was   issued   the   patents   in   question.   As  
filing   or   priority   date   of   the   application:   correctly   observed   by   respondent   Court   of   Appeals,  
Provided,  That  the  application  which  has  validly   "since   sufficient   proofs   have   been   introduced   in  
claimed  the  filing  date  of  an  earlier  application   evidence  showing  a  fair  question  of  the  invalidity  of  the  
under   Section   31   of   this   Act,   shall   be   prior   art   patents   issued   for   such   models,   it   is   but   right   that   the  
with   effect   as   of   the   filing   date   of   such   earlier   evidence   be   looked   into,   evaluated   and   determined   on  
application:   Provided   further,   That   the   the   merits   so   that   the   matter   of   whether   the   patents  
applicant   or   the   inventor   identified   in   both   issued  were  in  fact  valid  or  not  may  be  resolved."  
applications  are  not  one  and  the  same.    
  All   these   notwithstanding,   the   trial   court   nonetheless  
SECTION  25.  NON-­‐PREJUDICIAL  DISCLOSURE.  –     issued   the   writ   of   preliminary   injunction   which   under   the  
circumstances  should  be  denied.  
25.1.   The   disclosure   of   information   contained   in   the  
 
application  during  the  twelve  (12)  months  preceding  the   For   failure   to   determine   first   the   validity   of   the   patents  
filing  date  or  the  priority  date  of  the  application  shall  not   before   aforesaid   issuance   of   the   writ,   the   trial   court  
prejudice  the  applicant  on  the  ground  of  lack  of  novelty   failed   to   satisfy   the   two   requisites   necessary   if   an  
if  such  disclosure  was  made  by:   injunction   is   to   issue,   namely:   the   existence   of   the   right  
  to  be  protected  and  the  violation  of  said  right.  
(a)  The  inventor;    
 
VARGAS  V.  F.M.  YAPTICO  &  CO.  (1919)  
(b)   A   patent   office   and   the   information   was  
contained  (a)  in  another  application  filed  by  the   Refresher:  
inventor  and  should  not  have  been  disclosed  by   Adjustable  plow.  
the  office,  or  (b)  in  an  application  filed  without    
the  knowledge  or  consent  of  the  inventor  by  a   Court  decision:  
third   party   which   obtained   the   information   When   a   patent   is   sought   to   be   enforced,   "the   question  
directly  or  indirectly  from  the  inventor;  or   of   invention,   novelty,   or   prior   use,   and   each   of   them,   are  
  open   to   judicial   examination."   The   burden   of   proof   to  
(c)   A   third   party   which   obtained   the   substantiate  a  charge  of  infringement  is  with  the  plaintiff.  
information   directly   or   indirectly   from   the   Where,   however,   the   plaintiff   introduces   the   patent   in  
inventor.   evidence,   if   it   is   in   due   form,   it   affords   a   prima   facie  
  presumption  of  its  correctness  and  validity.  The  decision  
of  the  Commissioner  of  Patents  in  granting  the  patent  is  
25.2.  For  the  purposes  of  Subsection  25.1,  "inventor"  also  
always   presumed   to   be   correct.   The   burden   then   shifts  
means  any  person  who,  at  the  filing  date  of  application,  
to   the   defendant   to   overcome   by   competent   evidence  
had  the  right  to  the  patent.    
this   legal   presumption.   With   all   due   respect,   therefore,  
 
for   the   critical   and   expert   examination   of   the   invention  
by   the   United   States   Patent   Office,   the   question   of   the  
validity   of   the   patent   is   one   for   judicial   determination,  

o 129
Katrina Michelle Mancao
 
and   since   a   patent   has   been   submitted,   the   exact   came   into   existence.   It   may   also   be   noted   that   Adrian   de  
question   is   whether   the   defendant   has   assumed   the   Icsiar  applied  for  a  patent  on  an  invention  which  resulted  
burden  of  proof  as  to  anyone  of  his  defenses.   in   the   rejection   by   the   United   States   Patent   Office   of   the  
  plaintiffs'   original   application   for   a   patent   on   the   so  
On  the  facts,  we  think  the  testimony  shows  such  a  public   called  "spindle"  or  conical  drum  which  was  then  in  actual  
use   of   the   Vargas   plow   as   to   render   the   patent   invalid.   use  in  the  Dringman  and  Icsiar  hemp  stripping  machines.  
For   more   than   two   years   before   the   application   for   the    
original  letters  patent,  or  before  July  22,  1908,  there  was,   b. Inventiveness  
by  the  consent  and  allowance  of  Vargas,  a  public  use  of    
the  invention  covered  by  them.  
SECTION  26.  INVENTIVE  STEP.  –    
 
26.1.   An   invention   involves   an   inventive   step   if,   having  
VARGAS  V.  CHUA  (1933)  
regard   to   prior   art,   it   is   not   obvious   to   a   person   skilled   in  
Court  decision:  
the   art   at   the   time   of   the   filing   date   or   priority   date   of  
We   have   carefully   examined   all   the   plows   presented   as  
the  application  claiming  the  invention.  
exhibits   as   well   as   the   designs   of   those   covered   by   the  
 
patent,   and   we   are   convinced   that   no   substantial  
26.2.   In   the   case   of   drugs   and   medicines,   there   is   no  
difference   exists   between   the   plow,   Exhibit   F,   and   the  
plow,   Exhibit   3-­‐Chua   which   was   originally   patented   by   inventive   step   if   the   invention   results   from   the   mere  
the   appellee,   Vargas.   The   only   difference   noted   by   us   is   discovery   of   a   new   form   or   new   property   of   a   known  
the   suppression   of   the   bolt   and   the   three   holes   on   the   substance  which  does  not  result  in  the  enhancement  of  
metal  strap  attached  to  the  handle  bar.  These  holes  and   the   known   efficacy   of   that   substance,   or   the   mere  
bolt   with   its   nut   were   suppressed   in   Exhibit   F   in   which   discovery   of   any   new   property   or   new   use   for   a   known  
the   beam   is   movable   as   in   the   original   plow.   The   substance,   or   the   mere   use   of   a   known   process   unless  
members   of   this   court,   with   the   plows   in   view,   arrived   at   such   known   process   results   in   a   new   product   that  
the   conclusion   that   not   only   is   there   no   fundamental   employs  at  least  one  new  reactant.  
difference  between  the  two  plows  but  no  improvement    
whatever   has   been   made   on   the   latest   model,   for   the   AGUAS  V.  DE  LEON  (1982)  
same   working   and   movement   of   the   beam   existed   in   the   Court  decision:  
original   model   with   the   advantage,   perhaps,   that   its   The   records   disclose   that   de   Leon's   process   is   an  
graduation   could   be   carried   through   with   more   certainty   improvement  of  the  old  process  of  tile  making.  The  tiles  
by  the  use  of  the  bolt  which  as  has  already  been  stated,   produced   from   de   Leon's   process   are   suitable   for  
was  adjustable  and  movable.   construction   and   ornamentation,   which   previously   had  
  not   been   achieved   by   tiles   made   out   of   the   old   process  
In   view   of   the   foregoing,   we   are   firmly   convinced   that   of   tile   making.   De   Leon's   invention   has   therefore  
the  appellee  is  not  entitled  to  the  protection  he  seeks  for   brought  about  a  new  and  useful  kind  of  tile.  The  old  type  
the   simple   reason   that   his   plow,   Exhibit   F,   does   not   of  tiles  were  usually  intended  for  floors  although  there  is  
constitute   an   invention   in   the   legal   sense,   and   because,   nothing   to   prevent   one   from   using   them   for   walling  
according   to   the   evidence,   the   same   type   of   plows   had   purposes.   These   tiles   are   neither   artistic   nor   ornamental.  
been   manufactured   in   this   country   and   had   been   in   use   They  are  heavy  and  massive.  
in  many  parts  of  the  Philippine  Archipelago,  especially  in    
the   Province   of   Iloilo,   long   before   he   obtained   his   last   The   respondent's   improvement   is   indeed   inventive   and  
patent.   goes   beyond   the   exercise   of   mechanical   skill.   He   has  
  introduced  a  new  kind  of  tile  for  a  new  purpose.  He  has  
FRANK  V.  KOSUYAMA  (1933)   improved   the   old   method   of   making   tiles   and   pre-­‐cast  
Court  decision:   articles   which   were   not   satisfactory   because   of   an  
We  agree  with  the  trial  court  that,  strictly  speaking,  the   intolerable   number   of   breakages,   especially   if   deep  
hemp   stripping   machine   of   the   plaintiffs   does   not   engravings   are   made   on   the   tile.   He   has   overcome   the  
constitute   an   invention   on   the   ground   that   it   lacks   the   problem   of   producing   decorative   tiles   with   deep  
elements  of  novelty,  originality  and  precedence.  In  fact,   engraving,   but   with   sufficient   durability.   Durability  
before   the   plaintiffs   herein   obtained   their   patent,   they   inspite  of  the  thinness  and  lightness  of  the  tile,  is  assured,  
themselves   had   already   publicly   used   the   same   kind   of   provided   that   a   certain   critical   depth   is   maintained   in  
machine   for   some   months,   at   least,   and,   various   other   relation  to  the  dimensions  of  the  tile.  
machines,   having   in   general,   the   same   characteristics    
and   important   parts   as   that   of   the   said   plaintiffs,   were   MANZANO  V.  CA  (1997),  SUPRA  
known   in   the   Province   of   Davao.   Machines   known   as   Court  decision:  
Molo,   Riesgo,   Crumb,   Icsiar,   Browne   and   McFie   were   The   element   of   novelty   is   an   essential   requisite   of   the  
already   known   in   that   locality   and   used   by   the   owners   of   patentability   of   an   invention   or   discovery.   If   a   device   or  
hemp   plantations   before   the   machine   of   the   plaintiffs   process   has   been   known   or   used   by   others   prior   to   its  

130 z
Intellectual Property Law

invention   or   discovery   by   the   applicant,   an   application    


for   a   patent   therefor   should   be   denied;   and   if   the   113.2.   Industrial   designs   dictated   essentially   by   technical  
application   has   been   granted,   the   court,   in   a   judicial   or   functional   considerations   to   obtain   a   technical   result  
proceeding  in  which  the  validity  of  the  patent  is  drawn  in   or   those   that   are   contrary   to   public   order,   health   or  
question,   will   hold   it   void   and   ineffective.   It   has   been   morals  shall  not  be  protected.  
repeatedly   held   that   an   invention   must   possess   the    
essential  elements  of  novelty,  originality  and  precedence,   4. Lay-­‐out   designs   –  
and   for   the   patentee   to   be   entitled   to   the   protection   the  
invention  must  be  new  to  the  world.   Topographies   of  
  Integrated  Circuits  
In   issuing   Letters   Patent   No.   UM-­‐4609   to   Melecia    
Madolaria   for   an   "LPG   Burner"   on   22   July   1981,   the   REPUBLIC   ACT   9150   -­‐   AN   ACT   PROVIDING   FOR   THE  
Philippine   Patent   Office   found   her   invention   novel   and  
PROTECTION   OF   LAYOUT-­‐DESIGNS   (TOPOGRAPHIES)   OF  
patentable.   The   issuance   of   such   patent   creates   a  
presumption   which   yields   only   to   clear   and   cogent   INTEGRATED   CIRCUITS,   AMENDING   FOR   THE   PURPOSE   OF  
evidence   that   the   patentee   was   the   original   and   first   CERTAIN   SECTIONS   OF   RA   8293,   OTHERWISE   KNOWN   AS  
inventor.   The   burden   of   proving   want   of   novelty   is   on   THE  IPC  
him  who  avers  it  and  the  burden  is  a  heavy  one  which  is  
SECTION   1.   Sections   112,113,114,116,117,118,119   and   120  
met   only   by   clear   and   satisfactory   proof   which  
overcomes  every  reasonable  doubt.   under  Chapter  XIII  of  R.  A.  No.  8293  are  hereby  amended  
  to  read  as  follows:  
 
c. Industrial  
CHAPTER  XIII  
application   INDUSTRIAL  DESIGN  AND  LAYOUT-­‐DESIGNS  
  (TOPOGRAPHIES)  OF  INTEGRATED  CIRCUITS  
SECTION  27.  INDUSTRIAL  APPLICABILITY.  –      
An   invention   that   can   be   produced   and   used   in   any   SEC.  112.    DEFINITION  OF  TERMS:  
industry  shall  be  industrially  applicable.    
  1.   An   Industrial   Design   is   any   composition   of  
2. Utility  model   lines   or   colors   or   any   three-­‐   dimensional   form,  
  whether  or  not  associated  with  lines  or  colors:  
Provided,  That  such  composition  or  form  gives  
SECTION   109.   SPECIAL   PROVISIONS   RELATING   TO   UTILITY  
a   special   appearance   to   and   can   serve   as  
MODELS.  –     pattern  for  an  industrial  product  or  handicraft;  
109.1.      
(a)   An   invention   qualifies   for   registration   as   a   2.   Integrated   Circuit   means   a   product,   in   its  
utility   model   if   it   is   new   and   industrially   final   form,   or   an   intermediate   form,   in   which  
applicable.   the  elements,  at  least  one  of  which  is  an  active  
  element,   and   some   or   all   of   the  
(b)   Section   21,   "Patentable   Inventions",   shall   interconnections   are   integrally   formed   in  
apply   except   the   reference   to   inventive   step   as   and/or   on   a   piece   of   material,   and   which   is  
a  condition  of  protection.   intended   to   perform   an   electronic   function;  
  and  
3. Industrial  designs    
  3.   Layout-­‐Design   is   synonymous   with  
'Topography'   and   means   the   three-­‐dimensional  
SECTION  112.  DEFINITION  OF  INDUSTRIAL  DESIGN.  –    
disposition,   however   expressed,   of   the  
An  industrial  design  is  any  composition  of  lines  or  colors   elements,   at   least   one   of   which   is   an   active  
or   any   three-­‐dimensional   form,   whether   or   not   element,   and   of   some   or   all   of   the  
associated-­‐with   lines   or   colors:   Provided,   That   such   interconnections   of   an   integrated   circuit,   or  
composition   or   form   gives   a   special   appearance   to   and   such   a   three-­‐dimensional   disposition   prepared  
can   serve   as   pattern   for   an   industrial   product   or   for   an   integrated   circuit   intended   for  
handicraft.     manufacture."  
   
SECTION   113.   SUBSTANTIVE   CONDITIONS   FOR   PROTECTION.   SEC.  113.  SUBSTANTIVE  CONDITIONS  FOR  PROTECTION.  –  
–     113.1.  Only  industrial  designs  that  are  new  or  ornamental  
113.1.  Only  industrial  designs  that  are  new  or  original  shall   shall  benefit  from  protection  under  this  Act.  
benefit  from  protection  under  this  Act.    

o 131
Katrina Michelle Mancao
 
113.2.   Industrial   designs   dictated   essentially   by   technical   design  or  the  layout-­‐design  or  a  pictorial  representation  
or   functional   considerations   to   obtain   a   technical   result   thereof.  
or   those   that   are   contrary   to   public   order,   health   or    
morals  shall  not  be  protected.   116.2.   If   the   application   does   not   meet   these  
  requirements,   the   filing   date   should   be   that   date   when  
113.3.   Only   layout-­‐designs   of   integrated   circuits   that   are   all   the   elements   specified   in   Sec.   114   are   filed   or   the  
original   shall   benefit   from   protection   under   this   Act.   A   mistakes   corrected.   Otherwise,   if   the   requirements   are  
layout-­‐design  shall  be  considered  original  if  it  is  the  result   not   complied   within   the   prescribed   period,   the  
of   its   creator's   own   intellectual   effort   and   is   not   application  shall  be  considered  withdrawn.  
commonplace   among   creators   of   layout-­‐designs   and    
manufacturers   of   integrated   circuits   at   the   time   of   its   116.3   After   the   application   has   been   accorded   a   filing  
creation.   date   and   the   required   fees   paid   on   time,   the   applicant  
  shall  comply  with  the  requirements  of  Sec.  114  within  the  
113.4.   A   layout-­‐design   consisting   of   a   combination   of   prescribed   period,   otherwise   the   application   shall   be  
elements   and   interconnections   that   are   commonplace   considered  withdrawn.  
shall   be   protected   only   if   the   combination,   taken   as   a    
whole,  is  original.   116.4.   The   Office   shall   examine   whether   the   industrial  
  design   or   layout-­‐design   complies   with   requirements   of  
SEC.  114.  CONTENTS  OF  THE  APPLICATION.  –     Sec.  112  (Definitions)  and  Sec.  113  (Substantive  Conditions  
114.1.   Every   application   for   registration   of   an   industrial   for  Protection).  
design  or  layout-­‐design  shall  contain:    
  SEC.  117.  REGISTRATION.  –    
 (a)   A   request   for   registration   of   the   industrial   117.1.  Where  the  Office  finds  that  the  conditions  referred  
design  or  layout-­‐design;   to   in   Sec.   113   are   fulfilled,   it   shall   order   that   registration  
  be   effected   in   the   industrial   design   or   layout-­‐design  
 (b)  Information  identifying  the  applicant;   register   and   cause   the   issuance   of   an   industrial   design   or  
  layout-­‐design   certificate   of   registration;   otherwise,   it  
 (c)   An   indication   of   the   kind   of   article   of   shall  refuse  the  application.  
manufacture   or   handicraft   to   which   the    
industrial   design   or   layout-­‐design   shall   be   117.2.   The   form   and   contents   of   an   industrial   design   or  
applied:   layout-­‐design   certificate   shall   be   established   by   the  
  Regulations:  Provided,  That  the  name  and  address  of  the  
 (d)   A   representation   of   the   article   of   creator  shall  be  mentioned  in  every  case.  
manufacture  or  handicraft  by  way  of  drawings,    
photographs   or   adequate   graphic   117.3.   Registration   shall   be   published   in   the   form   and  
representation   of   the   industrial   design   or   of   within  the  period  fixed  by  the  Regulations.  
the   layout-­‐design   as   applied   to   the   article   of    
manufacture   or   handicraft   which   clearly   and   117.4.  The  Office  shall  record  in  the  register  any  change  in  
fully   discloses   those   features   for   which   the  identity  of  the  proprietor  of  the  industrial  design  or  
protection  is  claimed;  and   layout-­‐design   or   his   representative,   if   proof   thereof   is  
  furnished   to   it.   A   fee   shall   be   paid,   with   the   request   to  
 (e)   The   name   and   address   of   the   creator,   or   record  the  change  in  the  identity  of  the  proprietor.  If  the  
where   the   applicant   is   not   the   creator,   a   fee  is  not  paid,  the  request  shall  be  deemed  not  to  have  
statement   indicating   the   origin   of   the   right   to   been   filed.   In   such   case,   the   former   proprietor   and   the  
the   industrial   design   or   layout-­‐design   former   representative   shall   remain   subject   to   the   rights  
registration.     and  obligations  as  provided  in  this  Act.  
   
114.2.   The   application   may   be   accompanied   by   a   117.5.   Anyone   may   inspect   the   Register   and   the   files   of  
specimen   of   the   article   embodying   the   industrial   design   registered   industrial   designs   or   layout-­‐designs   including  
or   layout-­‐design   and   shall   be   subject   to   the   payment   of   files  of  cancellation  proceedings.  
the  prescribed  fee.    
  SEC.   118.   THE   TERM   OF   INDUSTRIAL   DESIGN   OR   LAYOUT-­‐DESIGN  
SEC.  116.  EXAMINATION.  –     REGISTRATION.  –    
116.1.  The  Office  shall  accord  as  the  filing  date  the  date  of   118.1.  The  registration  of  an  industrial  design  shall  be  for  
receipt   of   the   application   containing   indications   allowing   a   period   of   five   (5)   years   from   the   filing   date   of   the  
the   identity   of   the   applicant   to   be   established   and   a   application.  
representation   of   the   article   embodying   the   industrial    

132 z
Intellectual Property Law

118.2.   The   registration   of   an   industrial   design   may   be   filing   date   of   the   corresponding   foreign  
renewed  for  not  more  than  two  (2)  consecutive  periods   application;  
of  five  (5)  years  each,  by  paying  the  renewal  fee.    
  SECTION   33   –   Appointment   of   Agent   or  
118.3.   The   renewal   fee   shall   be   paid   within   twelve   (12)   Representative;  
months   preceding   the   expiration   of   the   period   of    
registration.   However,   a   grace   period   of   six   (6)   months   SECTION  51  –  Refusal  of  the  Application;  
shall   be   granted   for   payment   of   the   fees   after   such    
expiration,  upon  payment  of  a  surcharge.   SECTIONS   56   to   60   –   Surrender,   Correction   of  
  and  Changes  in  Patent;  
118.4.   The   Regulations   shall   fix   the   amount   of   renewal    
fee,  the  surcharge  and  other  requirements  regarding  the   CHAPTER   VII   –   Remedies   of   a   Person   with   a  
recording  of  renewals  of  registration.   Right  to  Patent;  
   
118.5.   Registration   of   a   layout-­‐design   shall   be   valid   for   a   Chapter   VIII   –   Rights   of   Patentees   and  
period   often   (10)   years,   without   renewal,   and   such   Infringement  of  Patents;  and  
validity  to  be  counted  from  the  date  of  commencement    
of   the   protection   accorded   to   the   layout-­‐design.   The   Chapter   XI   –   Assignment   and   Transmission   of  
protection   of   a   layout-­‐design   under   this   Act   shall   Rights.  
commence:    
  119.2.   If   the   essential   elements   of   an   industrial   design  
a)   on   the   date   of   the   first   commercial   which   is   the   subject   of   an   application   have   been  
exploitation,   anywhere   in   the   world,   of   the   obtained   from   the   creation   of   another   person   without  
layout-­‐design   by   or   with   the   consent   of   the   his   consent,   protection   under   this   Chapter   cannot   be  
right   holder:   Provided,   That   an   application   for   invoked  against  the  injured  party.  
registration   is   filed   with   the   Intellectual    
Property  Office  within  two  (2)  years  from  such   119.3.   The   following   provisions   relating   to   patents   shall  
date  of  first  commercial  exploitation;  or   apply   mutatis   mutandis   to   a   layout-­‐design   of   integrated  
  circuits  registration:  
b)   on   the   filing   date   accorded   to   the    
application   for   the   registration   of   the   layout-­‐   SECTION  28  –  Right  to  a  Patent;    
design   if   the   layout-­‐design   has   not   been    
previously  exploited  commercially  anywhere  in   SECTION  29  –  First  to  File  Rule;  
the  world.    
  SECTION   30   –   Inventions   Created   Pursuant   to   a  
SEC.  119.  APPLICATION  OF  OTHER  SECTIONS  AND  CHAPTERS.  –   Commission;  
119.1.   The   following   provisions   relating   to   patents   shall    
apply   mutatis   mutandis   to   an   industrial   design   SECTION   33   –   Appointment   of   Agent   or  
registration:   Representative;  
   
SECTION  21  –  Novelty;   SECTION  56  –  Surrender  of  Patent;  
   
SECTION   24   –   Prior   art:   Provided,   That   the   SECTION   57   –   Correction   of   Mistakes   of   the  
disclosure  is  contained  in  printed  documents  or   Office;  
in  any  tangible  form;    
  SECTION   58   –   Correction   of   Mistakes   in   the  
SECTION  25  –  Non-­‐prejudicial  Disclosure;     Application;  
   
SECTION  28  –  Right  to  a  Patent;     SECTION  59  –  Changes  in  Patents;  
   
SECTION  29  –  First  to  File  Rule;   SECTION   60   –   Form   and   Publication   of  
  Amendment;  
SECTION   30   –   Inventions   Created   Pursuant   to   a    
Commission;   Chapter  VII  –  Remedies  of  a  Person  with  a  Right  
  to  Patent;  
SECTION   31   -­‐   Right   of   Priority:   Provided,   That    
the   application   for   industrial   design   shall   be   Chapter   VIII   –   Rights   of   Patentees   and  
filed   within   six   (6)   months   from   the   earliest   Infringement   of   Patents:   Provided,   That   the  

o 133
Katrina Michelle Mancao
 
layout-­‐design   rights   and   limitation   of   layout-­‐ may   perform   any   of   the   said   acts   only   with  
design  rights  provided  hereunder  shall  govern;   respect  to  the  stock  on  hand  or  ordered  before  
  such  time  and  shall  be  liable  to  pay  to  the  right  
Chapter  X  –  Compulsory  Licensing;   holder   a   sum   equivalent   to   at   least   5%   of   net  
  sales  or  such  other  reasonable  royalty  as  would  
Chapter   XI   –   Assignment   and   Transmission   of   be  payable  under  a  freely  negotiated  license  in  
Rights.   respect  of  such  layout-­‐design;  or  
   
119.4.  Rights  Conferred  to  the  Owner  of  a  Layout-­‐Design   (5)   Where   the   act   is   performed   in   respect   of   an  
Registration.  –  The  owner  of  a  layout-­‐design  registration   identical  layout-­‐design  which  is  original  and  has  
shall  enjoy  the  following  rights:   been  created  independently  by  a  third  party.  
   
 (1)   to   reproduce,   whether   by   incorporation   in   SEC.  120.  CANCELLATION  OF  DESIGN  REGISTRATION.  –    
an   integrated   circuit   or   otherwise,   the   120.1.  At  any  time  during  the  term  of  the  industrial  design  
registered   layout-­‐design   in   its   entirety   or   any   registration,   any   person   upon   payment   of   the   required  
part  thereof,  except  the  act  of  reproducing  any   fee,   may   petition   the   Director   of   Legal   Affairs   to   cancel  
part   that   does   not   comply   with   the   the  industrial  design  on  any  of  the  following  grounds:  
requirement  of  originality;  and    
  (a)  If  the  subject  matter  of  the  industrial  design  
 (2)   to   sell   or   otherwise   distribute   for   is  not  registerable  within  the  terms  of  Sections  
commercial   purposes   the   registered   layout-­‐ 112  and  113;  
design,   an   article   or   an   integrated   circuit   in    
which   the   registered   layout-­‐design   is   (b)  If  the  subject  matter  is  not  new;  or  
incorporated.    
  (c)  If  the  subject  matter  of  the  industrial  design  
119.5.   Limitations   of   Layout   Rights.   -­‐   The   owner   of   a   extends  beyond  the  content  of  the  application  
layout   design   has   no   right   to   prevent   third   parties   from   as  originally  filed.  
reproducing   selling   or   otherwise   distributing   for    
commercial  purposes  the  registered  layout-­‐design  in  the   120.2.  Where  the  grounds  for  cancellation  relate  to  a  part  
following  circumstances:   of  the  industrial  design,  cancellation  may  be  effected  to  
  such  extent  only.  The  restriction  may  be  effected  in  the  
(1)   Reproduction   of   the   registered   layout-­‐ form   of   an   alteration   of   the   effected   features   of   the  
design   for   private   purposes   or   for   the   sole   design.  
purpose   of   evaluation,   analysis,   research   or    
teaching;   120.3.   Grounds   for   Cancellation   of   Layout-­‐Design   of  
  Integrated  Circuits.  -­‐   Any  interested  person  may  petition  
(2)   Where   the   act   is   performed   in   respect   of   a   that   the   registration   of   a   layout-­‐design   be   cancelled   on  
layout-­‐design   created   on   the   basis   of   such   the  ground  that:  
analysis   or   evaluation   and   which   is   itself    
original  in  the  meaning  as  provided  herein;   (i)   the   layout-­‐design   is   not   protectable   under  
  this  Act;  
(3)  Where  the  act  is  performed  in  respect  of  a    
registered   layout-­‐design,   or   in   respect   of   an   (ii)   the   right   holder   is   not   entitled   to   protection  
integrated  circuit  in  which  such  a  layout-­‐design   under  this  Act;  or  
is   incorporated,   that   has   been   put   on   the    
market   by   or   with   the   consent   of   the   right   (iii)   where   the   application   for   registration   of  
holder;   the   layout-­‐design,   was   not   filed   within   two   (2)  
  years   from   its   first   commercial   exploitation  
(4)  In  respect  of  an  integrated  circuit  where  the   anywhere  in  the  world.  
person  performing  or  ordering  such  an  act  did    
not   know   and   had   no   reasonable   ground   to   Where  the  grounds  for  cancellation  are  established  with  
know   when   acquiring   the   integrated   circuit   or   respect   only   to   a   part   of   the   layout-­‐design,   only   the  
the   article   incorporating   such   an   integrated   corresponding  part  of  the  registration  shall  be  cancelled.  
circuit,   that   it   incorporated   an   unlawfully    
reproduced   layout-­‐design:   Provided,   however,   Any  cancelled  layout-­‐design  registration  or  part  thereof,  
That   after   the   time   that   such   person   has   shall   be   regarded   as   null   and   void   from   the   beginning  
received   sufficient   notice   that   the   layout-­‐ and   may   be   expunged   from   the   records   of   the  
design  was  unlawfully  reproduced,  that  person   Intellectual   Property   Office.   Reference   to   all   cancelled  

134 z
Intellectual Property Law

layout-­‐design   registration   shall   be   published   in   the   IPO   22.2.  Schemes,  rules  and  methods  of  performing  mental  
Gazette.   acts,  playing  games  or  doing  business,  and  programs  for  
  computers;  
SEC.  2.  IMPLEMENTING  RULES  AND  REGULATIONS.  –      
The   Intellectual   Property   Office   may   issue   Regulations   22.3.   Methods   for   treatment   of   the   human   or   animal  
prescribing   details   for   the   implementation   of   this   law.   body   by   surgery   or   therapy   and   diagnostic   methods  
The   Regulations   may,   in   particular,   provide   for   the   practiced   on   the   human   or   animal   body.   This   provision  
payment  of  fees  in  connection  with  applications  for  the   shall   not   apply   to   products   and   composition   for   use   in  
registration   of   layout-­‐designs   of   integrated   circuits   and   any  of  these  methods;  
matters   related   thereto,   including   Administrative    
Instructions   relating   to   the   procedures   and   other   22.4.   Plant   varieties   or   animal   breeds   or   essentially  
functions  of  the  responsible  unit  duly  designated  by  the   biological  process  for  the  production  of  plants  or  animals.  
Director  General.   This   provision   shall   not   apply   to   micro-­‐organisms   and  
  non-­‐biological  and  microbiological  processes.  
SEC.  3.  APPLICABILITY.  –      
The  provisions  of  this  Act  shall  apply  to  layout-­‐designs  of   Provisions   under   this   subsection   shall   not   preclude  
integrated   circuits   that   were   commercially   exploited   Congress   to   consider   the   enactment   of   a   law   providing  
anywhere   in   the   world   from   and   after   January   1998   sui   generis   protection   of   plant   varieties   and   animal  
provided  they  meet  the  conditions  for  protection  under   breeds   and   a   system   of   community   intellectual   rights  
this  Act.   protection:  
   
SEC.  4.  REPEALING  CLAUSE.  –     22.5.  Aesthetic  creations;  and  
All   acts   and   parts   of   acts   inconsistent   herewith   are    
hereby  repealed  or  amended  accordingly.   22.6.   Anything   which   is   contrary   to   public   order   or  
  morality.  
SEC.  5.  SEPARABILITY  CLAUSE.  –      
If   any   provision   of   this   Act   or   the   application   of   such   UNDER  RA  165  (OLD  PATENT  LAW):  
provision   to   any   circumstance   is   held   invalid,   the  
SECTION  8.  INVENTIONS  NOT  PATENTABLE.  –    
remainder  of  this  Act  shall  not  be  affected  thereby.  
  An   invention   shall   not   be   patentable   if   it   is   contrary   to  
SEC.  6.  EFFECTIVITY.  –     public  order  or  morals,  or  to  public  health  or  welfare,  or  
if   it   constitutes   a   mere   idea,   scientific   principle   or  
This   Act   shall   take   effect   fifteen   (15)   days   after   its  
abstract   theorem   not   embodied   in   an   invention   as  
publication  in  two  (2)  newspapers  of  general  circulation.  
specified   in   section   seven   hereof,   or   any   process   not  
 
directed   to   the   making   or   improving   of   a   commercial  
E. What  are  not  patentable?   product.  
   
SEC.  22.  NON-­‐PATENTABLE  INVENTIONS.  –     SECTION  9.  INVENTION  NOT  CONSIDERED  NEW  OR  PATENTABLE.  –    
The  following  shall  be  excluded  from  patent  protection:   An   invention   shall   not   be   considered   new   or   capable   of  
  being  patented  if  it  was  known  or  used  by  others  in  the  
22.1.   Discoveries,   scientific   theories   and   mathematical   Philippines  before  the  invention  thereof  by  the  inventor  
methods,   and   in   the   case   of   drugs   and   medicines,   the   named  in  an  application  for  patent  for  the  invention;  or  if  
mere   discovery   of   a   new   form   or   new   property   of   a   it  was  patented  or  described  in  any  printed  publication  in  
known   substance   which   does   not   result   in   the   the   Philippines   or   any   foreign   country   more   than   one  
enhancement   of   the   known   efficacy   of   that   substance,   year  before  the  application  for  a  patent  therefor;  or  if  it  
or   the   mere   discovery   of   any   new   property   or   new   use   had   been   in   public   use   or   on   sale   in   the   Philippines   for  
for   a   known   substance,   or   the   mere   use   of   a   known   more   than   one   year   before   the   application   for   a   patent  
process   unless   such   known   process   results   in   a   new   therefor;   or   if   it   is   the   subject   matter   of   a   validly   issued  
product  that  employs  at  least  one  new  reactant.   patent  in  the  Philippines  granted  on  an  application  filed  
  before  the  filing  of  the  application  for  patent  therefor.  
For   the   purpose   of   this   clause,   salts,   esters,   ethers,    
polymorphs,   metabolites,   pure   form,   particle   size,   SECTION  74.  PENALTY  FOR  FALSE  MARKING.  –    
isomers,  mixtures  of  isomers,  complexes,  combinations,   Any  person  who  falsely  represents  or  indicates  that  any  
and   other   derivatives   of   a   known   substance   shall   be   device,   article   or   product   made   or   sold   by   him   is  
considered  to  be  the  same  substance,  unless  they  differ   patented,   or   is   the   subject   of   a   registered   design,   by  
significantly  in  properties  with  regard  to  efficacy;   making   or   having   on   the   device,   article   or   product,   or   on  
  their   containers   or   packages,   or   using   in   advertising   or  
displays   used   in   connection   with   them,   or   with   any  

o 135
Katrina Michelle Mancao
 
process,   words   expressing   or   implying   that   the   device,   property   which   is   possessed   by   no   naturally   occurring  
article,   product   or   process   is   patented   or   registered,   bacteria.   A   patent   examiner's   rejection   of   the   patent  
shall   be   subject   to   a   fine   of   not   less   than   one   hundred   application's  claims  for  the  new  bacteria  was  affirmed  by  
pesos   nor   more   than   one   thousand   pesos,   or   the   Patent   Office   Board   of   Appeals   on   the   ground   that  
imprisonment   for   not   less   than   one   month   nor   more   living   things   are   not   patentable   subject   matter   under   §  
than   one   year,   or   both,   in   the   discretion   of   the   court.   101.  The  Court  of  Customs  and  Patent  Appeals  reversed,  
Actions  hereunder  shall  prescribe  in  two  years.   concluding  that  the  fact  that  micro-­‐organisms  are  alive  is  
  without   legal   significance   for   purposes   of   the   patent   law.  
UNDER  THE  TRIPS  AGREEMENT:   Court  decision:  
ARTICLE  27,  PATENTABLE  SUBJECT  MATTER.   A   live,   human-­‐made   micro-­‐organism   is   patentable  
subject   matter   under   §   101.   Respondent's   micro-­‐
 
organism   constitutes   a   "manufacture"   or   "composition  
1.   Subject   to   the   provisions   of   paragraphs   2   and   3,  
of  matter"  within  that  statute.    
patents   shall   be   available   for   any   inventions,   whether  
 
products   or   processes,   in   all   fields   of   technology,  
In  choosing  such  expansive  terms  as  "manufacture"  and  
provided   that   they   are   new,   involve   an   inventive   step  
"composition   of   matter,"   modified   by   the  
and   are   capable   of   industrial   application.   (5)   Subject   to  
comprehensive   "any,"   Congress   contemplated   that   the  
paragraph  4  of  Article  65,  paragraph  8  of  Article  70  and  
patent   laws   should   be   given   wide   scope,   and   the  
paragraph   3   of   this   Article,   patents   shall   be   available   and  
relevant   legislative   history   also   supports   a   broad  
patent  rights  enjoyable  without  discrimination  as  to  the  
construction.  While  laws  of  nature,  physical  phenomena,  
place   of   invention,   the   field   of   technology   and   whether  
and  abstract  ideas  are  not  patentable,  respondent's  claim  
products  are  imported  or  locally  produced.  
is  not  to  a  hitherto  unknown  natural  phenomenon,  but  to  
 
a  non-­‐naturally  occurring  manufacture  or  composition  of  
2.   Members   may   exclude   from   patentability   inventions,  
matter   -­‐-­‐   a   product   of   human   ingenuity   "having   a  
the   prevention   within   their   territory   of   the   commercial  
distinctive  name,  character  and  use."    
exploitation  of  which  is  necessary  to  protect  ordre  public  
 
or  morality,  including  to  protect  human,  animal  or  plant  
The   passage   of   the   1930   Plant   Patent   Act,   which  
life   or   health   or   to   avoid   serious   prejudice   to   the  
afforded   patent   protection   to   certain   asexually  
environment,   provided   that   such   exclusion   is   not   made  
reproduced   plants,   and   the   1970   Plant   Variety   Protection  
merely   because   the   exploitation   is   prohibited   by   their  
Act,   which   authorized   protection   for   certain   sexually  
law.  
reproduced   plants   but   excluded   bacteria   from   its  
 
protection,   does   not   evidence   congressional  
3.  Members  may  also  exclude  from  patentability:  
understanding   that   the   terms   "manufacture"   or  
 
"composition   of   matter"   in   §   101   do   not   include   living  
(a)   diagnostic,   therapeutic   and   surgical  
things.  
methods   for   the   treatment   of   humans   or  
 
animals;  
Nor   does   the   fact   that   genetic   technology   was  
 
unforeseen   when   Congress   enacted   §   101   require   the  
(b)   plants   and   animals   other   than   micro-­‐
conclusion   that   micro-­‐organisms   cannot   qualify   as  
organisms,  and  essentially  biological  processes  
patentable   subject   matter   until   Congress   expressly  
for   the   production   of   plants   or   animals   other  
authorizes   such   protection.   The   unambiguous   language  
than   non-­‐biological   and   microbiological  
of   §   101   fairly   embraces   respondent's   invention.  
processes.   However,   Members   shall   provide  
Arguments   against   patentability   under   §   101,   based   on  
for   the   protection   of   plant   varieties   either   by  
potential   hazards   that   may   be   generated   by   genetic  
patents  or  by  an  effective  sui  generis  system  or  
research,   should   be   addressed   to   the   Congress   and   the  
by   any   combination   thereof.   The   provisions   of  
Executive,  not  to  the  Judiciary.  
this  subparagraph  shall  be  reviewed  four  years  
 
after   the   date   of   entry   into   force   of   the   WTO  
Remarks:  US  Case  
Agreement.  
   
DIAMOND  V.  CHAKRABARTY,  447  US  303  (1980)   RA  9168  –  PHILIPPINE  PLANT  VARIETY  PROTECTION  ACT  OF  
Refresher:   2002  
Title  35  U.S.C.  §  101  provides  for  the  issuance  of  a  patent   SECTION  2.  STATEMENT  OF  POLICIES.  –  
to   a   person   who   invents   or   discovers   "any"   new   and    
useful   "manufacture"   or   "composition   of   matter."   a)   The   State   recognizes   that   an   effective  
Respondent   filed   a   patent   application   relating   to   his   intellectual  property  system  in  general  and  the  
invention   of   a   human-­‐made,   genetically   engineered   development  of  new  plant  variety  in  particular  
bacterium   capable   of   breaking   down   crude   oil,   a   is  vital  in  attaining  food  security  for  the  country.  

136 z
Intellectual Property Law

To   this   end,   it   shall   protect   and   secure   the   (4)  years  or,  in  the  case  of  vines  or  tress,  more  
exclusive   rights   of   breeders   with   respect   to   than  six  (6)  years  before  the  date  of  filing  of  an  
their   new   plant   variety   particularly   when   application  for  Plant  Variety  Protection.  
beneficial   to   the   people   for   such   periods   as    
provided  for  in  this  Act.   However,  the  requirement  of  novelty  provided  for  in  this  
  Act   shall   not   apply   to   varieties   sold,   offered   for   sale   or  
b)   The   use   of   intellectual   property   bears   a   disposed  of  to  others  for  a  period  of  five  (5)  years  before  
socioeconomic  function.  To  this  end,  the  State   the   approval   of   this   Act.   Provided,   That   application   for  
shall   promote   the   diffusion   of   technology   and   PVP   shall   be   filed   within   one   (1)   year   from   the   approval  
information   for   the   promotion   of   national   of  this  act.  
development   and   progress   for   the   common    
good.   SECTION   6.   DISTINCTNESS.   -­‐   A   variety   shall   be   deemed  
  distinct   if   it   is   clearly   distinguishable   from   any   commonly  
c)   The   State   recognizes   the   indispensable   role   known   variety.   The   filing   of   an   application   for   the  
of   the   private   sector,   encourages   the   granting  of  a  plant  variety  protection  or  for  the  entering  
participation   of   private   enterprises   and   of   a   new   variety   in   an   official   register   of   variety   in   the  
provides   incentives   to   needed   investments   in   Philippines  or  in  any  country,  shall  render  the  said  variety  
the  development  of  new  plant  varieties.   a  matter  of  public  knowledge  from  the  date  of  the  said  
  application:   Provided,   That   the   application   leads   to   the  
d)   The   State   recognizes   that   science   and   granting   of   a   Certificate   of   Plant   Variety   Protection   or  
technology   are   essential   for   national   the   entering   of   the   said   other   variety   in   the   official  
development   and   promotes   the   adaptation   of   register  of  variety  as  the  case  may  be.  
technology   and   knowledge   from   all   sources   for    
the  national  benefit.  The  State  also  recognizes   SECTION   7.   UNIFORMITY.   -­‐   The   variety   shall   be   deemed  
the   need   to   protect   and   secure   the   exclusive   uniform   if,   subject   to   the   variation   that   may   be   expected  
rights   of   scientists   and   other   gifted   citizens   to   from   the   particular   features   of   its   propagation,   it   is  
their  intellectual  property  and  creations.   sufficiently  uniform  in  its  relevant  characteristics.  
   
e)   The   State,   while   recognizing   intellectual   SECTION  8.  STABILITY.  -­‐   The   variety   shall   be   deemed   stable   if  
property  rights  in  the  field  of  agriculture,  does  
its   relevant   characteristics   remain   unchanged   after  
so   in   a   manner   supportive   of   and   not  
repeated   propagation   or,   in   the   case   of   a   particular   cycle  
inconsistent   with   its   obligation   to   maintain   a  
of  propagation,  at  the  end  of  each  such  cycle.  
healthful   ecology   in   accord   with   the   rhythm  
 
and  harmony  of  nature.  
SECTION  17.  ENTITLEMENT.  -­‐   Any   breeder,   with   respect   to   the  
 
SECTION  4.  The  Certificate  of  Plant  Variety  Protection  shall   variety   developed,   may   apply   for   a   plant   variety  
developed,  may  apply  for  a  plant  variety  protection  and  
be  granted  for  varieties  that  are:  
obtain   a   Certificate   of   Plant   Variety   Protection   upon  
 
compliance  with  the  requirements  of  this  Act.  
a)  New;  
 
 
SECTION  18.   CO-­‐OWNERSHIP  OF  THE  RIGHT.   -­‐   If   two   (2)   or   more  
b)  Distinct;  
  persons   contribute   to   the   development   of   a   new   plant  
c)  Uniform;  and   variety,   all   of   them   shall   be   named   in   the   Certificate   of  
  Plant   Variety   Protection   and   shall   be   entitled   to   such  
d)  Stable.   rights   as   agreed   upon   in   writing   or   in   the   absence  
  thereof,  the  rights  in  proportion  to  their  contribution  in  
SECTION  5.  NEWNESS.  -­‐   A   variety   shall   be   deemed   new   if   the   the  development  of  plant  variety.  
 
propagating  or  harvested  material  of  the  variety  has  not  
SECTION   19.   EMPLOYEE-­‐EMPLOYER   RELATIONSHIP.   -­‐   in   case   an  
been   sold,   offered   for   sale   or   otherwise   disposed   of   to  
others,   by   or   with   the   consent   of   the   breeder,   for   employee   develops   a   plant   variety   in   the   course   of   his  
purposes  of  exploitation  of  the  variety;   employment  as  a  result  of  the  performance  of  his  regular  
  duty,   the   plant   variety   protection   shall   belong   to   the  
a)  In  the  Philippines  for  more  than  one  (1)  year   employer,   unless   there   is   a   written   stipulation   to   the  
before   the   date   of   filing   of   an   application   for   contrary.  
plant  variety  protection;  or    
  SECTION  20.  FIRST  TO   FILE  RULE.  -­‐  If  two  (2)  or  more  persons  
b)  In  other  countries  or  territories  in  which  the   develop   a   new   plant   variety   separately   and  
application   has   been   filed,   for   more   than   four   independently   of   each   other,   the   Certificate   of   Plant  

o 137
Katrina Michelle Mancao
 
Variety   Protection   shall   belong   to   the   person   who   files   SECTION  36.  RIGHTS  OF  HOLDERS  OF  PLANT  VARIETY  PROTECTION.  -­‐  
the  application  first.  In  case  two  (2)  or  more  persons  file   In   respect   of   the   propagating   materials,   holders   of   a  
an  application  for  the  same  plant  variety,  the  right  shall   Certificate  of  Plant  Variety  Protection  shall  have  the  right  
be   granted   to   the   person   who   has   the   earliest   filing   date   to  authorize  any  of  the  following  acts:  
or  priority  date.    
  a)  Production  or  reproduction;  
SECTION  21.  PRIORITY  DATE.  -­‐  Any   application   for   a   Certificate    
of   Plant   Variety   Protection   previously   filed   by   a   breeder   b)   Conditioning   for   the   purpose   of  
in   another   country,   which   by   treaty,   convention   or   law   propagation;  
affords   similar   privileges   to   Filipino   citizens,   shall   be    
considered   as   filed   locally   as   of   the   date   of   filing   of   the   c)  Offering  for  sale;  
foreign  application:  Provided,  That:    
  d)  Selling  or  other  marketing;  
a)   The   local   application   expressly   claims    
priority;   e)  Exporting;  
   
b)   It   is   filed   within   twelve   (12)   months   from   the   f)  Importing;  and  
filing   date   of   the   earliest   foreign   application;    
and   g)  Stocking  for  any  purpose  mentioned  above.  
   
c)  The  applicant  submits,  within  six  (6)  months   SECTION   37.   THE   HOLDER   MAY   MAKE   HIS   AUTHORIZATION   SUBJECT  
from   the   filing   of   the   local   application,   TO  CONDITIONS  AND  L IMITATIONS.  
authenticated   copies   of   documents   which    
constitute   the   foreign   application,   samples   or   SECTION   47.   WHAT   CONSTITUTES   INFRINGEMENT.   -­‐   Except   as  
other  evidence  showing  that  the  variety  which  
otherwise  provided  in  this  Act,  any  person  who  without  
is   being   applied   for   protection   is   the   same  
being  entitled  to  do  so,  performs  the  following  acts:  
variety   which   has   been   applied   for   protection  
 
in  a  foreign  country.  
a)  Sell  the  novel  variety,  or  offer  it  or  expose  it  
 
for  sale,  deliver  it,  ship  it,  consign  it,  exchange  
SECTION  22.  FOREIGN  NATIONALS.  -­‐   For   purposes   of   this   Act,   a  
it,   or   solicit   an   offer   to   buy   it,   or   any   other  
person   shall   be   considered   a   national   of   a   foreign   transfer  of  title  or  possession  of  it;  or  
country  if  he  is  a  citizen  of  such  country  according  to  its    
laws,  a  natural  person  residing  therein,  or  is  a  legal  entity   b)   Import   the   novel   variety   into,   or   export   it  
whose  office  is  registered  in  such  foreign  country.   from,  the  Philippines;  or  
   
SECTION   23.   NATIONAL   TREATMENT.   -­‐   Any   application   filed   c)  Sexually  multiply  the  novel  variety  as  a  step  
locally   for   a   Certificate   of   Plant   Variety   Protection   in   marketing   (for   growing   purposes)   the  
previously   granted   to   a   breeder   in   another   country,   variety;  or  
which   by   treaty,   convention   or   law   affords   similar    
privileges  to  Filipino  citizens,  shall  be  issued  a  Certificate   d)   Use   the   novel   variety   in   producing   (as  
of   Plant   Variety   Protection   upon   payment   of   dues   and   distinguished   from   developing)   a   hybrid   or  
compliance   to   all   the   provisions   of   this   Act.   This   Act   shall   different  variety  therefrom;  or  
also   apply   to   the   nationals   of   foreign   countries   that   are    
members  of  intergovernmental  organizations  or  party  to   e)   Use   seed   which   had   been   marked  
any   multilateral   agreement   or   convention   concerning   "unauthorized   propagation   prohibited"   or  
the   granting   of   intellectual   property   protection   to   plant   "unauthorized   seed   multiplication   prohibited"  
varieties.   or   progeny   thereof   to   propagate   the   novel  
  variety;  or  
SECTION   33.   TERM  OF   PROTECTION.   -­‐   For   trees   and   vines,   the    
period  of  protection  shall  be  twenty-­‐five  (25)  years  from   f)   Dispense   the   novel   variety   to   another,   in   a  
the   date   of   the   grant   of   the   Certificate   of   Plant   Variety   form  which  can  be  propagated,  without  notice  
Protection  and  twenty  (20)  years  from  the  said  date  for   as   to   being   a   protected   variety   under   which   it  
all   other   types   of   plants,   unless   declared   void   ab   initio   or   was  received;  or  
cancelled  otherwise,  as  provided  under  Section  61  and  62,    
respectively  of  this  Act.   g)   Fails   to   use   a   variety   denomination   the   use  
  of  which  is  obligatory  under  Section  15;  or  
 

138 z
Intellectual Property Law

h)   Perform   any   of   the   foregoing   acts   even   in   taken  without  their  free  and  prior  informed  consent  or  in  
instances   in   which   the   novel   variety   is   violation  of  their  laws,  traditions  and  customs.  
multiplied   other   than   sexually,   except   in    
pursuance  of  a  valid  Philippine  plant  patent;  or   SEC.   34.   RIGHT   TO   INDIGENOUS   KNOWLEDGE   SYSTEMS   AND  
  PRACTICES   AND   TO   D EVELOP   OWN   SCIENCES   AND   TECHNOLOGIES.-­‐  
i)   Instigate   or   actively   induce   performance   of   ICCs/IPs   are   entitled   to   the   recognition   of   the   full  
any  foregoing  acts,  may  be  sued  by  the  holder,   ownership   and   control   and   protection   of   their   cultural  
who   may   also   avail   of   all   such   relief   as   are   and   intellectual   rights.   They   shall   have   the   right   to  
available   in   any   proceeding   involving   special   measures   to   control,   develop   and   protect   their  
infringements  of  other  proprietary  rights.   sciences,   technologies   and   cultural   manifestations,  
  including   human   and   other   genetic   resources,   seeds,  
SECTION   48.   WHERE  TO   COMMENCE   ACTION.   -­‐   Any   holder   may   including   derivatives   of   these   resources,   traditional  
petition   the   proper   regional   trial   court   for   infringement   medicines   and   health   practices,   vital   medicinal   plants,  
of  his  plant  variety  protection  as  defined  in  this  Act.   animals   and   minerals,   indigenous   knowledge   systems  
  and  practices,  knowledge  of  the  properties  of  fauna  and  
SECTION   49.   PRESUMPTION   OF   VALIDITY.   -­‐   Certificate   of   Plant   flora,   oral   traditions,   literature,   designs,   and   visual   and  
Variety   Protection   shall   be   presumed   valid   and   the   performing  arts.  
burden  of  proof  of  their  invalidity  shall  rest  on  the  party    
assailing  them.   F. Ownership  of  patent  
   
SECTION   50.   DEFENSES   AGAINST   INFRINGEMENT   CHARGES.   -­‐   The   SECTION  28.  RIGHT  TO  A  PATENT.  –    
following   shall   be   valid   defenses   against   infringement  
The   right   to   a   patent   belongs   to   the   inventor,   his   heirs,  
charges:  
or   assigns.   When   two   (2)   or   more   persons   have   jointly  
 
made  an  invention,  the  right  to  a  patent  shall  belong  to  
a)  Non-­‐infringement;  
them  jointly.  
 
 
b)   The   plant   variety   does   not   possess   at   the  
time   of   its   application   criterion   of   novelty   or  
SECTION  29.  FIRST  TO  FILE  RULE.  –    
distinctness;   If   two   (2)   or   more   persons   have   made   the   invention  
  separately  and  independently  of  each  other,  the  right  to  
c)   The   alleged   infringement   was   performed   the   patent   shall   belong   to   the   person   who   filed   an  
under   a   right   adverse   to   it,   prior   to   the   notice   application   for   such   invention,   or   where   two   or   more  
of  infringement;  and/or   applications   are   filed   for   the   same   invention,   to   the  
  applicant  who  has  the  earliest  filing  date  or,  the  earliest  
d)   Other   defenses   that   are   made   available   priority  date.  
under  this  Act.    
  SECTION   30.   INVENTIONS   CREATED   PURSUANT   TO   A  
SECTION  55.  PRESCRIPTION.  -­‐  No  recovery  of  damages  for  any   COMMISSION.  –    
infringement  case  shall  prosper  when  the  cause  of  action   30.1.   The   person   who   commissions   the   work   shall   own  
has   reached   more   than   six   (6)   years   from   the   time   the   the  patent,  unless  otherwise  provided  in  the  contract.  
alleged  infringement  case  was  committed.    
  30.2.   In   case   the   employee   made   the   invention   in   the  
SECTION   56.   CRIMINAL   PENALTY.   -­‐   Any   person   who   violates  
course   of   his   employment   contract,   the   patent   shall  
any   of   the   rights   of   the   holder   provided   for   in   this   Act   belong  to:  
may  also  suffer  the  penalty  of  imprisonment  of  not  less    
than   three   (3)   years   but   not   more   than   six   (6)   years   (a)  The  employee,  if  the  inventive  activity  is  not  
and/or   a   fine   of   up   to   three   (3)   times   the   profit   derived   a   part   of   his   regular   duties   even   if   the  
by   virtue   of   the   infringement   but   in   no   case   should   be   employee   uses   the   time,   facilities   and   materials  
less  than  One  Hundred  Thousand  pesos  (P100,000.00).   of  the  employer.  
   
RA  8371  –  INDIGENOUS  PEOPLES  RIGHTS  ACT  OF  1997   (b)   The   employer,   if   the   invention   is   the   result  
SEC.   32.   COMMUNITY   INTELLECTUAL   RIGHTS.-­‐   ICCs/IPs   have   the   of   the   performance   of   his   regularly-­‐assigned  
right   to   practice   and   revitalize   their   own   cultural   duties,   unless   there   is   an   agreement,   express  
traditions   and   customs.   The   State   shall   preserve,   protect   or  implied,  to  the  contrary.  
and   develop   the   past,   present   and   future   manifestations    
of  their  cultures  as  well  as  the  right  to  the  restitution  of  
cultural,   intellectual,   religious,   and   spiritual   property  

o 139
Katrina Michelle Mancao
 
SECTION   67.   PATENT   APPLICATION   BY   PERSONS   NOT   G. Regular  application  for  patent  
HAVING  THE  RIGHT  TO  A  PATENT.  .  –      
67.1.  If  a  person  referred  to  in  Section  29  other  than  the   1. Who  may  apply?  
applicant,   is   declared   by   final   court   order   or   decision   as    
having   the   right   to   the   patent,   such   person   may,   within   SECTION  28.  RIGHT  TO  A  PATENT.  –    
three  (3)  months  after  the  decision  has  become  final:   The   right   to   a   patent   belongs   to   the   inventor,   his   heirs,  
  or   assigns.   When   two   (2)   or   more   persons   have   jointly  
(a)   Prosecute   the   application   as   his   own   made  an  invention,  the  right  to  a  patent  shall  belong  to  
application  in  place  of  the  applicant;   them  jointly.  
   
(b)   File   a   new   patent   application   in   respect   of   SECTION  29.  FIRST  TO  FILE  RULE.  –    
the  same  invention;  
If   two   (2)   or   more   persons   have   made   the   invention  
 
separately  and  independently  of  each  other,  the  right  to  
(c)  Request  that  the  application  be  refused;  or  
the   patent   shall   belong   to   the   person   who   filed   an  
 
application   for   such   invention,   or   where   two   or   more  
(d)   Seek   cancellation   of   the   patent,   if   one   has  
applications   are   filed   for   the   same   invention,   to   the  
already  been  issued.  
applicant  who  has  the  earliest  filing  date  or,  the  earliest  
 
priority  date.  
67.2.   The   provisions   of   Subsection   38.2   shall   apply    
mutatis   mutandis   to   a   new   application   filed   under   SECTION   30.   INVENTIONS   CREATED   PURSUANT   TO   A  
Subsection  67.  1(b).  
  COMMISSION.  –    
SECTION   68.   REMEDIES   OF   THE   TRUE   AND   ACTUAL   30.1.   The   person   who   commissions   the   work   shall   own  
the  patent,  unless  otherwise  provided  in  the  contract.  
INVENTOR.  –    
 
If  a  person,  who  was  deprived  of  the  patent  without  his  
30.2.   In   case   the   employee   made   the   invention   in   the  
consent   or   through   fraud   is   declared   by   final   court   order  
course   of   his   employment   contract,   the   patent   shall  
or  decision  to  be  the  true  and  actual  inventor,  the  court  
belong  to:  
shall   order   for   his   substitution   as   patentee,   or   at   the  
 
option  of  the  true  inventor,  cancel  the  patent,  and  award  
actual  and  other  damages  in  his  favor  if  warranted  by  the   (a)  The  employee,  if  the  inventive  activity  is  not  
a   part   of   his   regular   duties   even   if   the  
circumstances.  
employee   uses   the   time,   facilities   and   materials  
 
of  the  employer.  
SECTION  69.  PUBLICATION  OF  THE  COURT  ORDER.  –    
 
The  court  shall  furnish  the  Office  a  copy  of  the  order  or   (b)   The   employer,   if   the   invention   is   the   result  
decision  referred  to  in  Sections  67  and  68,  which  shall  be   of   the   performance   of   his   regularly-­‐assigned  
published   in   the   IPO   Gazette   within   three   (3)   months   duties,   unless   there   is   an   agreement,   express  
from   the   date   such   order   or   decision   became   final   and   or  implied,  to  the  contrary.  
executory,   and   shall   be   recorded   in   the   register   of   the    
Office.    
SECTION   68.   REMEDIES   OF   THE   TRUE   AND   ACTUAL  
 
SECTION  70.  TIME  TO  FILE  ACTION  IN  COURT.  –     INVENTOR.  –    
If  a  person,  who  was  deprived  of  the  patent  without  his  
The  actions  indicated  in  Sections  67  and  68  shall  be  filed  
within  one  (1)  year  from  the  date  of  publication  made  in   consent   or   through   fraud   is   declared   by   final   court   order  
or  decision  to  be  the  true  and  actual  inventor,  the  court  
accordance  with  Sections  44  and  51,  respectively.  
shall   order   for   his   substitution   as   patentee,   or   at   the  
 
option  of  the  true  inventor,  cancel  the  patent,  and  award  
SECTION  236.  PRESERVATION  OF  EXISTING  RIGHTS.  –    
actual  and  other  damages  in  his  favor  if  warranted  by  the  
Nothing   herein   shall   adversely   affect   the   rights   on   the   circumstances.  
enforcement   of   rights   in   patents,   utility   models,    
industrial   designs,   marks   and   works,   acquired   in   good  
SECTION   3.   INTERNATIONAL   CONVENTIONS   AND  
faith  prior  to  the  effective  date  of  this  Act.  
  RECIPROCITY.  –    
Any  person  who  is  a  national  or  who  is  domiciled  or  has  a  
real   and   effective   industrial   establishment   in   a   country  
which  is  a  party  to  any  convention,  treaty  or  agreement  
relating   to   intellectual   property   rights   or   the   repression  
of   unfair   competition,   to   which   the   Philippines   is   also   a  

140 z
Intellectual Property Law

party,   or   extends   reciprocal   rights   to   nationals   of   the   (c)   Drawings   necessary   for   the   understanding  
Philippines   by   law,   shall   be   entitled   to   benefits   to   the   of  the  invention;  
extent  necessary  to  give  effect  to  any  provision  of  such    
convention,   treaty   or   reciprocal   law,   in   addition   to   the   (d)  One  or  more  claims;  and  
rights   to   which   any   owner   of   an   intellectual   property    
right  is  otherwise  entitled  by  this  Act.   (e)  An  abstract.  
   
SECTION  231.  REVERSE  RECIPROCITY  OF  FOREIGN  LAWS.  –     32.2.   No   patent   may   be   granted   unless   the   application  
Any   condition,   restriction,   limitation,   diminution,   identifies   the   inventor.   If   the   applicant   is   not   the  
requirement,   penalty   or   any   similar   burden   imposed   by   inventor,   the   Office   may   require   him   to   submit   said  
the   law   of   a   foreign   country   on   a   Philippine   national   authority.    
seeking  protection  of  intellectual  property  rights  in  that    
country,  shall  reciprocally  be  enforceable  upon  nationals   SECTION  33.  APPOINTMENT  OF  AGENT  OR  REPRESENTATIVE.  
of  said  country,  within  Philippine  jurisdiction.     –    
 
An   applicant   who   is   not   a   resident   of   the   Philippines  
SECTION   235.   APPLICATIONS   PENDING   ON   EFFECTIVE   DATE   must   appoint   and   maintain   a   resident   agent   or  
OF  ACT.  –     representative   in   the   Philippines   upon   whom   notice   or  
235.1.  All  applications  for  patents  pending  in  the  Bureau   process   for   judicial   or   administrative   procedure   relating  
of  Patents,  Trademarks  and  Technology  Transfer  shall  be   to   the   application   for   patent   or   the   patent   may   be  
proceeded   with   and   patents   thereon   granted   in   served.  
accordance   with   the   Acts   under   which   said   applications    
were   filed,   and   said   Acts   are   hereby   continued   to   be   SECTION  34.  THE  REQUEST.  –    
enforced,   to   this   extent   and   for   this   purpose   only,   The  request  shall  contain  a  petition  for  the  grant  of  the  
notwithstanding   the   foregoing   general   repeal   thereof:   patent,   the   name   and   other   data   of   the   applicant,   the  
Provided,   That   applications   for   utility   models   or   inventor  and  the  agent  and  the  title  of  the  invention.    
industrial   designs   pending   at   the   effective   date   of   this    
Act,   shall   be   proceeded   with   in   accordance   with   the   SECTION   35.   DISCLOSURE   AND   DESCRIPTION   OF   THE  
provisions   of   this   Act,   unless   the   applicants   elect   to  
INVENTION.  –    
prosecute   said   applications   in   accordance   with   the   Acts  
under  which  they  were  filed.   35.1.   D ISCLOSURE.   -­‐   The   application   shall   disclose   the  
  invention   in   a   manner   sufficiently   clear   and   complete   for  
235.2.   All   applications   for   registration   of   marks   or   trade   it  to  be  carried  out  by  a  person  skilled  in  the  art.  Where  
the  application  concerns  a  microbiological  process  or  the  
names   pending   in   the   Bureau   of   Patents,   Trademarks  
product   thereof   and   involves   the   use   of   a   micro-­‐
and   Technology   Transfer   at   the   effective   date   of   this   Act  
organism   which   cannot   be   sufficiently   disclosed   in   the  
may  be  amended,  if  practicable  to  bring  them  under  the  
application  in  such  a  way  as  to  enable  the  invention  to  be  
provisions   of   this   Act.   The   prosecution   of   such  
carried   out   by   a   person   skilled   in   the   art,   and   such  
applications   so   amended   and   the   grant   of   registrations  
material   is   not   available   to   the   public,   the   application  
thereon  shall  be  proceeded  with  in  accordance  with  the  
provisions  of  this  Act.  If  such  amendments  are  not  made,   shall  be  supplemented  by  a  deposit  of  such  material  with  
an  international  depository  institution.  
the   prosecution   of   said   applications   shall   be   proceeded  
 
with   and   registrations   thereon   granted   in   accordance  
with   the   Acts   under   which   said   applications   were   filed,   35.2.   DESCRIPTION.   -­‐   The   Regulations   shall   prescribe   the  
and   said   Acts   are   hereby   continued   in   force   to   this   contents   of   the   description   and   the   order   of  
extent   for   this   purpose   only,   notwithstanding   the   presentation.    
foregoing  general  repeal  thereof.    
  SECTION  36.  THE  CLAIMS.  –    
2. Application   36.1.  The  application  shall  contain  one  (1)  or  more  claims  
  which   shall   define   the   matter   for   which   protection   is  
SECTION  32.  THE  APPLICATION.  –     sought.   Each   claim   shall   be   clear   and   concise,   and   shall  
be  supported  by  the  description.  
32.1.  The  patent  application  shall  be  in  Filipino  or  English  
 
and  shall  contain  the  following:  
36.2.   The   Regulations   shall   prescribe   the   manner   of   the  
 
(a)  A  request  for  the  grant  of  a  patent;   presentation  of  claims.    
   
(b)  A  description  of  the  invention;   SECTION  37.  THE  ABSTRACT.  –    
  The   abstract   shall   consist   of   a   concise   summary   of   the  
disclosure   of   the   invention   as   contained   in   the  

o 141
Katrina Michelle Mancao
 
description,  claims  and  drawings  in  preferably  not  more   SECTION   108.   APPLICABILITY   OF   PROVISIONS   RELATING   TO  
than  one  hundred  fifty  (150)  words.  It  must  be  drafted   in  
PATENTS.  –    
a   way   which   allows   the   clear   understanding   of   the  
technical   problem,   the   gist   of   the   solution   of   that   108.1.   Subject   to   Section   109,   the   provisions   governing  
problem  through  the  invention,  and  the  principal  use  or   patents   shall   apply,   mutatis   mutandis,   to   the   registration  
uses   of   the   invention.   The   abstract   shall   merely   serve   for   of  utility  models.  
technical  information.      
  108.2.  Where  the  right  to  a  patent  conflicts  with  the  right  
SECTION  38.  UNITY  OF  INVENTION.  –     to   a   utility   model   registration   in   the   case   referred   to   in  
Section   29,   the   said   provision   shall   apply   as   if   the   word  
38.1.  The  application  shall  relate  to  one  invention  only  or  
"patent"   were   replaced   by   the   words   "patent   or   utility  
to   a   group   of   inventions   forming   a   single   general  
model  registration".  
inventive  concept.  
 
 
SECTION   109.   SPECIAL   PROVISIONS   RELATING   TO   UTILITY  
38.2.   If   several   independent   inventions   which   do   not  
form   a   single   general   inventive   concept   are   claimed   in   MODELS.  -­‐  
one   application,   the   Director   may   require   that   the   109.2.   Sections   43   to   49   shall   not   apply   in   the   case   of  
application   be   restricted   to   a   single   invention.   A   later   applications  for  registration  of  a  utility  model.  
application   filed   for   an   invention   divided   out   shall   be    
considered  as  having  been  filed  on  the  same  day  as  the   Industrial  Design  
first   application:   Provided,   That   the   later   application   is  
SECTION  114.  CONTENTS  OF  THE  APPLICATION.  –    
filed   within   four   (4)   months   after   the   requirement   to  
divide   becomes   final   or   within   such   additional   time,   not   114.1.   Every   application   for   registration   of   an   industrial  
exceeding   four   (4)   months,   as   may   be   granted:   Provided   design  shall  contain:  
further,   That   each   divisional   application   shall   not   go    
beyond  the  disclosure  in  the  initial  application.   (a)   A   request   for   registration   of   the   industrial  
  design;  
38.3.   The   fact   that   a   patent   has   been   granted   on   an    
application  that  did  not  comply  with  the  requirement  of   (b)  Information  identifying  the  applicant;  
unity   of   invention   shall   not   be   a   ground   to   cancel   the    
patent.   (c)   An   indication   of   the   kind   of   article   of  
  manufacture  or  handicraft  to  which  the  design  
shall  be  applied;  
SECTION   39.   INFORMATION   CONCERNING   CORRESPONDING  
 
FOREIGN  APPLICATION  FOR  PATENTS.  –     (d)   A   representation   of   the   article   of  
The   applicant   shall,   at   the   request   of   the   Director,   manufacture  or  handicraft  by  way  of  drawings,  
furnish  him  with  the  date  and  number  of  any  application   photographs   or   other   adequate   graphic  
for   a   patent   filed   by   him   abroad,   hereafter   referred   to   as   representation   of   the   design   as   applied   to   the  
the   "foreign   application,"   relating   to   the   same   or   article   of   manufacture   or   handicraft   which  
essentially   the   same   invention   as   that   claimed   in   the   clearly   and   fully   discloses   those   features   for  
application   filed   with   the   Office   and   other   documents   which  design  protection  is  claimed;  and  
relating  to  the  foreign  application.      
  (e)   The   name   and   address   of   the   creator,   or  
SECTION  8.  THE  BUREAU  OF  PATENTS.  –     where   the   applicant   is   not   the   creator,   a  
The  Bureau  of  Patents  shall  have  the  following  functions:   statement   indicating   the   origin   of   the   right   to  
  the  industrial  design  registration.  
8.1.   Search   and   examination   of   patent   applications   and    
the  grant  of  patents;   114.2.   The   application   may   be   accompanied   by   a  
  specimen   of   the   article   embodying   the   industrial   design  
8.2.   Registration   of   utility   models,   industrial   designs,   and   and  shall  be  subject  to  the  payment  of  the  prescribed  fee.  
integrated  circuits;  and    
  SECTION   115.   SEVERAL   INDUSTRIAL   DESIGNS   IN   ONE  
8.3.   Conduct   studies   and   researches   in   the   field   of   APPLICATION.  –    
patents   in   order   to   assist   the   Director   General   in   Two   (2)   or   more   industrial   designs   may   be   the   subject   of  
formulating   policies   on   the   administration   and   the   same   application:   Provided,   That   they   relate   to   the  
examination  of  patents.     same   sub-­‐class   of   the   International   Classification   or   to  
  the  same  set  or  composition  of  articles.    
 

142 z
Intellectual Property Law

3. Priority  date   are  received.  If  the  deficiencies  are  not  remedied  within  
  the   prescribed   time   limit,   the   application   shall   be  
considered  withdrawn.  
SECTION  31.  RIGHT  OF  PRIORITY.  .  –    
 
An   application   for   patent   filed   by   any   person   who   has  
SECTION  116.  EXAMINATION.  –    
previously   applied   for   the   same   invention   in   another  
country   which   by   treaty,   convention,   or   law   affords   116.1.  The  Office  shall  accord  as  the  filing  date  the  date  of  
similar   privileges   to   Filipino   citizens,   shall   be   considered   receipt   of   the   application   containing   indications   allowing  
as   filed   as   of   the   date   of   filing   the   foreign   application:   the   identity   of   the   applicant   to   be   established   and   a  
Provided,  That:  (a)  the  local  application  expressly  claims   representation   of   the   article   embodying   the   industrial  
priority;  (b)  it  is  filed  within  twelve  (12)  months  from  the   design  or  a  pictorial  representation  thereof.  
date   the   earliest   foreign   application   was   filed;   and   (c)   a    
certified  copy  of  the  foreign  application  together  with  an   116.2.   If   the   application   does   not   meet   these  
English   translation   is   filed   within   six   (6)   months   from   the   requirements  the  filing  date  should  be  that  date  when  all  
date  of  filing  in  the  Philippines.   the   elements   specified   in   Section   105   are   filed   or   the  
  mistakes   corrected.   Otherwise   if   the   requirements   are  
BOOTHE  V.  DIRECTOR  OF  PATENTS  (1980)   not   complied   within   the   prescribed   period,   the  
Court  decision:   application  shall  be  considered  withdrawn.  
Under  the  Revised  Rules  of  Practice  in  Patent  Cases,  it  is    
imperative   that   the   application   be   complete   in   order   that   116.3.   After   the   application   has   been   accorded   a   filing  
it  may  be  accepted.  It  is  essential  to  the  validity  of  Letters   date   and   the   required   fees   paid   on   time,   the   applicant  
Patent   that   the   specifications   be   full,   definite,   and   shall  comply  with  the  requirements  of  Section  114  within  
specific.  The  purpose  of  requiring  a  definite  and  accurate   the  prescribed  period,  otherwise  the  application  shall  be  
description   of   the   process   is   to   apprise   the   public   of   considered  withdrawn.  
what  the  patentee  claims  as  his  invention,  to  inform  the    
Courts  as  to  what  they  are  called  upon  to  construe,  and   116.4.   The   Office   shall   examine   whether   the   industrial  
to   convey   to   competing   manufacturers   and   dealers   design   complies   with   requirements   of   Section   112   and  
information  of  exactly  what  they  are  bound  to  avoid.   Subsections  113.2  and  113.3.    
   
4. Filing  date   5. Formality  examination  
   
SECTION  40.  FILING  DATE  REQUIREMENTS.  –     SECTION  42.  FORMALITY  EXAMINATION.  –    
40.1.   The   filing   date   of   a   patent   application   shall   be   the   42.1.   After   the   patent   application   has   been   accorded   a  
date   of   receipt   by   the   Office   of   at   least   the   following   filing  date  and  the  required  fees  have  been  paid  on  time  
elements:   in   accordance   with   the   Regulations,   the   applicant   shall  
  comply   with   the   formal   requirements   specified   by  
(a)   An   express   or   implicit   indication   that   a   Section   32   and   the   Regulations   within   the   prescribed  
Philippine  patent  is  sought;   period,   otherwise   the   application   shall   be   considered  
  withdrawn.  
(b)  Information  identifying  the  applicant;  and    
  42.2.  The  Regulations  shall  determine  the  procedure  for  
(c)   Description   of   the   invention   and   one   (1)   or   the   re-­‐examination   and   revival   of   an   application   as   well  
more  claims  in  Filipino  or  English.   as   the   appeal   to   the   Director   of   Patents   from   any   final  
  action  by  the  examiner.  
40.2.  If  any  of  these  elements  is  not  submitted  within  the    
period   set   by   the   Regulations,   the   application   shall   be   6. Classification  and  search  
considered  withdrawn.      
 
SECTION  43.  CLASSIFICATION  AND  SEARCH.  –    
SECTION  41.  ACCORDING  A  FILING  DATE.  –    
An   application   that   has   complied   with   the   formal  
The  Office  shall  examine  whether  the  patent  application   requirements  shall  be  classified  and  a  search  conducted  
satisfies  the  requirements  for  the  grant  of  date  of  filing   to  determine  the  prior  art.  
as   provided   in   Section   40   hereof.   If   the   date   of   filing    
cannot   be   accorded,   the   applicant   shall   be   given   an  
opportunity   to   correct   the   deficiencies   in   accordance  
with   the   implementing   Regulations.   If   the   application  
does  not  contain  all  the  elements  indicated  in  Section  40,  
the  filing  date  should  be  that  date  when  all  the  elements  

o 143
Katrina Michelle Mancao
 
7. Confidentiality   before   10. Observation   by   third  
publication   parties  
   
SECTION  45.  CONFIDENTIALITY  BEFORE  PUBLICATION.  –     SECTION  47.  OBSERVATION  BY  THIRD  PARTIES.  –    
A  patent  application,  which  has  not  yet  been  published,   Following   the   publication   of   the   patent   application,   any  
and   all   related   documents,   shall   not   be   made   available   person   may   present   observations   in   writing   concerning  
for  inspection  without  the  consent  of  the  applicant.   the   patentability   of   the   invention.   Such   observations  
  shall   be   communicated   to   the   applicant   who   may  
8. Publication   of   patent   comment   on   them.   The   Office   shall   acknowledge   and  
put   such   observations   and   comment   in   the   file   of   the  
application  
application  to  which  it  relates.  
   
SECTION  44.  PUBLICATION  OF  PATENT  APPLICATION.  –     11. Request   for   substantive  
44.1.   The   patent   application   shall   be   published   in   the   IPO  
examination  
Gazette  together  with  a  search  document  established  by  
 
or   on   behalf   of   the   Office   citing   any   documents   that  
reflect   prior   art,   after   the   expiration   of   eighteen   (18)   SECTION  48.  REQUEST  FOR  SUBSTANTIVE  EXAMINATION.  –    
months  from  the  filing  date  or  priority  date.   48.1.   The   application   shall   be   deemed   withdrawn   unless  
  within   six   (6)   months   from   the   date   of   publication   under  
44.2.   After   publication   of   a   patent   application,   any   Section   41,   a   written   request   to   determine   whether   a  
interested  party  may  inspect  the  application  documents   patent  application  meets  the  requirements  of  Sections  21  
filed  with  the  Office.   to  27  and  Sections  32  to  39  and  the  fees  have  been  paid  
  on  time.  
44.3.   The   Director   General   subject   to   the   approval   of   the    
Secretary  of  Trade  and  Industry,  may  prohibit  or  restrict   48.2.  Withdrawal  of  the  request  for  examination  shall  be  
the  publication  of  an  application,  if  in  his  opinion,  to  do   irrevocable  and  shall  not  authorize  the  refund  of  any  fee.  
so   would   be   prejudicial   to   the   national   security   and    
interests  of  the  Republic  of  the  Philippines.   12. Amendment  
   
9. Rights   conferred   by   a   SECTION  49.  AMENDMENT  OF  APPLICATION.  –    
patent  application   An   applicant   may   amend   the   patent   application   during  
  examination:   Provided,   That   such   amendment   shall   not  
SECTION   46.   RIGHTS   CONFERRED   BY   A   PATENT   include   new   matter   outside   the   scope   of   the   disclosure  
contained  in  the  application  as  filed.    
APPLICATION  AFTER  PUBLICATION.  –      
The   applicant   shall   have   all   the   rights   of   a   patentee   13. Conversion    
under   Section   76   against   any   person   who,   without   his  
 
authorization,   exercised   any   of   the   rights   conferred  
under   Section   71   of   this   Act   in   relation   to   the   invention   SECTION   110.   CONVERSION   OF   PATENT   APPLICATIONS   OR  
claimed  in  the  published  patent  application,  as  if  a  patent   APPLICATIONS  FOR  UTILITY  MODEL  REGISTRATION.  –    
had  been  granted  for  that  invention:  Provided,  That  the   110.1.   At   any   time   before   the   grant   or   refusal   of   a   patent,  
said  person  had:   an   applicant   for   a   patent   may,   upon   payment   of   the  
  prescribed   fee,   convert   his   application   into   an  
46.1.   Actual   knowledge   that   the   invention   that   he   was   application   for   registration   of   a   utility   model,   which   shall  
using  was  the  subject  matter  of  a  published  application;   be   accorded   the   filing   date   of   the   initial   application.   An  
or   application  may  be  converted  only  once.  
   
46.2.  Received  written  notice  that  the  invention  that  he   110.2.  At  any  time  before  the  grant  or  refusal  of  a  utility  
was   using   was   the   subject   matter   of   a   published   model   registration,   an   applicant   for   a   utility   model  
application   being   identified   in   the   said   notice   by   its   serial   registration   may,   upon   payment   of   the   prescribed   fee,  
number:  Provided,  That  the  action  may  not  be  filed  until   convert   his   application   into   a   patent   application,   which  
after  the  grant  of  a  patent  on  the  published  application   shall  be  accorded  the  filing  date  of  the  initial  application.  
and   within   four   (4)   years   from   the   commission   of   the    
acts  complained  of.  
 

144 z
Intellectual Property Law

SECTION   111.   PROHIBITION   AGAINST   FILING   OF   PARALLEL   Property  Code  of  the  Philippines,  the  following  rules  on  
PCT  applications  are  hereby  promulgated:  
APPLICATIONS.  –    
   
An   applicant   may   not   file   two   (2)   applications   for   the  
PART  I.  
same   subject,   one   for   utility   model   registration   and   the  
GENERAL  PROVISIONS  
other   for   the   grant   of   a   patent   whether   simultaneously  
 
or  consecutively.  
RULE  1.  TITLE.  -­‐  These  rules  shall  be  known  as  the  Philippine  
 
Rules  on  PCT  Applications  or  “PRo-­‐PCT”.  
H. Philippine  Rules  on  PCT  Applications  
 
  RULE   2.   D EFINITIONS.   -­‐   Unless   otherwise   specified,   the  
WHEREAS,   the   State   recognizes   that   an   effective  
following   terms   and   acronyms   shall   be   understood   as  
industrial  property  system  is  vital  to  the  development  of  
follows:  
domestic   creativity,   facilitates   transfer   of   technology,  
 
attracts  foreign  investments  and  ensures  market  access  
(a)     “Administrative   Instructions”   means   that  
for  our  products;    
body   of   instructions   for   operating   under   the  
 
Patent   Cooperation   Treaty   referred   to   in   PCT  
WHEREAS,   it   is   the   policy   of   the   State   to   streamline  
Rule  89;  
administrative   procedures   in   granting   patents   and  
 
enhance  the  enforcement  of  intellectual  property  rights  
(b)     “designated   Office”   means   a   national  
in  the  Philippines;  
patent   office   or   intergovernmental  
 
organization   of   or   acting   for   the   State  
WHEREAS,   Senate   Resolution   No.   74   dated   05   February  
designated  by  the  applicant  under  Chapter  I  of  
2001   was   passed   whereby   the   Philippines   ratified   the  
the  Treaty;  
Patent  Cooperation  Treaty  (PCT);    
 
 
(c)     “designated   State”   refers   to   the   State   in  
WHEREAS,   since   1978,   the   PCT   system   has   offered  
which  protection  for  the  invention  is  desired  on  
investors   and   industry   an   advantageous   route   for  
the   basis   of   the   international   application   and  
obtaining   patent   protection   internationally;   aims   to  
which   is   designated   under   Chapter   I   of   the  
facilitate   procedures   for   obtaining   legal   protection   for  
Treaty;  
inventions   and   disseminate   technical   information;   seeks  
 
to   simplify   and   to   render   more   effective   and   more  
(d)   “Director   General”   means   the   Head   of   the  
economical   –   in   the   interest   of   the   applicants   and   the  
Intellectual  Property  Office  of  the  Philippines;  
patent   offices   –   previously   established   means   of  
(e)   “elected   Office”   means   a   national   patent  
applying   in   several   countries   for   patent   protection   for  
office   or   intergovernmental   organization   of   or  
invention;   would   substantially   benefit   Philippine   patent  
acting   for   the   State   elected   by   the   applicant  
agents   and   industry   from   the   filing   in   the   Philippines   of  
under  Chapter  II  of  the  Treaty;  
international  applications  under  the  PCT;  
 
 
(f)     “elected   State”   refers   to   the   State   in   which  
WHEREAS,  in  accordance  with  Article  63  of  the  PCT,  the  
protection   for   the   invention   is   desired   on   the  
Treaty   shall   enter   into   force   with   respect   to   the  
basis  of  the  international  application  and  which  
Philippines,   three   (3)   months   after   the   date   on   which   it  
has   been   elected   under   Chapter   II   of   the  
has   deposited   its   Instrument   of   Accession   with   the  
Treaty;  
Director   General   of   the   World   Intellectual   Property  
 
Organization  (WIPO);  
(g)    “IB”  and  “International  Bureau”  mean  the  
   
International   Bureau   of   the   World   Intellectual  
WHEREAS,   the   Philippines   deposited   its   Instrument   of  
Property   Organization   as   referred   to   in   PCT  
Accession   with   the   Director   General   of   the   WIPO   on   17  
Article  2(xix);  
May   2001;   and,   therefore,   the   PCT   entered   into   effect  
 
with  respect  to  the  Philippines  on  17  August  2001;    
(h)     “International   Application”   and   “PCT  
 
Application”   mean   an   application   filed   under  
WHEREAS,   there   is   a   need   to   guide   applicants,   patent  
the  Treaty;  
attorneys/agents,   and   other   interested   parties   in   the  
 
Philippines  on  the  PCT  and  its  rules  and  regulations;  
(i)    “International  Searching  Authority”  or  “ISA”  
 
and   “International   Preliminary   Examining  
NOW,   THEREFORE,   pursuant   to   Senate   Resolution   No.  
Authority”   or   “IPEA”   mean   a   national   patent  
74  dated  05  February  2001,  the  provisions  of  the  Patent  
office   or   intergovernmental   organization   as  
Cooperation   Treaty   and   the   Regulations   thereunder,  
appointed   under   the   Treaty   which   processes  
Republic   Act   No.   8293   also   known   as   the   Intellectual  

o 145
Katrina Michelle Mancao
 
international   applications   as   prescribed   by   the   Other  terms  and  expressions  not  defined  in  this  rule  are  
Treaty  and  PCT  Regulations;   to   be   taken   in   the   sense   of   the   Treaty   and   PCT  
  Regulations.  
(j)     “IP   Code”   means   Republic   Act   No.   8293    
otherwise   known   as   the   Intellectual   Property   RULE  3.   APPLICATION  OF  THE   PCT  AND  PCT  REGULATIONS.  –  In  all  
Code  of  the  Philippines;   matters   not   specifically   provided   for   under   these   Rules,  
  the   provisions   of   the   Treaty,   PCT   Regulations   and   PCT  
(k)     “IPO”   or   “Office”   means   the   Intellectual   Administrative   Instructionsshall   apply   in   the   processing  
Property  Office  of  the  Philippines;   of   an   international   application   during   the   international  
  phase   of   the   application.   In   the   event   of   conflict  
(l)     “IPO   Fee   Structure”   means   the   rules   and   between   these   Rules   and   those   of   the   PCT,   the  
regulations   establishing   the   fees   and   charges   provisions  of  the  latter  shall  apply.    
of   the   Intellectual   Property   Office   of   the    
Philippines;   The   international   phase   covers   the   period   from   the   filing  
  of   the   application   in   accordance   with   the   PCT   until   the  
(m)     “PCT”   and   “Treaty”   mean   the   Patent   application  enters  the  national  phase.  
Cooperation  Treaty;      
  RULE  4.  EFFECT  OF  AN  INTERNATIONAL  APPLICATION.  –    
(n)     “PCT-­‐EASY”   means   the   software   made   4.1.   An   international   application   designating   the  
available   by   the   International   Bureau   for   Philippines   for   the   purposes   of   obtaining   a   national  
facilitating  the  preparation  of  the  request  part   patent   or   a   utility   model,   which   has   been   accorded   an  
of   an   international   application,   and   printing   international   filing   date   in   accordance   with   the   Treaty  
thereof,   for   filing   together   with   a   computer   and   PCT   regulations,   shall   have   the   effect   of   a   national  
diskette   prepared   using   that   software,   application   for   a   patent   or   utility   model,   respectively,  
containing  a  copy  in  electronic  form  of  the  data   regularly  filed  with  the  Intellectual  Property  Office  as  of  
contained  in  the  request  and  of  the  abstract;   the   international   filing   date,   which   date   shall   be  
  considered  to  be  the  actual  filing  date  in  the  Philippines.    
(o)    “PCT  Regulations”  means  the  set  of  rules,    
referred   to   in   PCT   Article   58   and   annexed   to   4.2.   Subject   to   Review   by   the   IPO   under   Rule   37,   an  
the  Treaty  referred  to  as  the  Regulations  Under   international  application  designating  the  Philippines  that  
the  PCT;   is  withdrawn  or  considered  withdrawn  under  the  Treaty  
  and   PCT   Regulations,   before   the   applicant   has   entered  
(p)     “Priority   Date”   for   the   purposes   of   the  national  phase  in  the  Philippines  as  provided  in  Part  3  
computing  time  limits  under  the  PCT  is  defined   of   these   Rules,   that   international   application   is   likewise  
in  PCT  Article  2(xi)  as  follows:   withdrawn  or  considered  withdrawn  in  the  Philippines.    
   
(i)  where  the  international  application   RULE   5.   SEEKING   TWO   (2)   KINDS   OF   PROTECTION;   PROHIBITION  
contains   a   priority   claim   under   PCT  
AGAINST  FILING  OF   PARALLEL  APPLICATIONS.  -­‐  In  respect  of  the  
Article   8,   the   filing   date   of   the  
application   whose   priority   is   so   Philippines  as  a  designated  or  elected  State,  an  applicant  
claimed;   cannot   seek   two   (2)   kinds   of   protection   for   the   same  
(ii)   where   the   international   subject,   one   for   utility   model   registration   and   the   other  
application   contains   several   priority   for   the   grant   of   a   patent   whether   simultaneously   or  
claims   under   PCT   Article   8,   the   filing   consecutively.    
date   of   the   earliest   application   whose    
priority  is  so  claimed;   RULE   6.   DELIVERY   SERVICE.   –   Where   the   applicant   sends   a  
  document   or   letter   through   mail   and   that   document   or  
(iii)   where   the   international   letter   is   lost   or   reaches   the   IPO   after   the   expiration   of  
application   does   not   contain   any   the   applicable   time   limit   under   the   Treaty   and   PCT  
priority  claim  under  PCT  Article  8,  the   Regulations   and   these   Rules,   PCT   Rules   82.1(a)   to   (c)  
international   filing   date   of   such   shall   apply   provided   that   the   delivery   service   used   is  
application;   registered  mail  by  a  postal  authority.  
   
(q)    “Receiving  Office”  means  a  national  patent   RULE   7.  APPOINTMENT  OF  RESIDENT  AGENT  OR  REPRESENTATIVE.   –  
office   or   intergovernmental   organization   with   (a)  An  applicant  who  is  not  a  resident  of  the  Philippines  
which   the   international   application   has   been   shall   appoint   and   maintain   an   agent   or   representative  
filed;   residing   in   the   Philippines   upon   whom   notices   or  
  processes   for   judicial   or   administrative   procedure   may  

146 z
Intellectual Property Law

be   served   relating   to   the   international   application   filed    


with   the   IPO   as   a   receiving,   designated   or   elected   Office.   (5)   Transmitting   the   international   application  
An   applicant   who   is   a   resident   of   the   Philippines   may   to   the   IB   for   processing,   where   the   IPO   is   not  
likewise  appoint  and  maintain  an  agent  or  representative   the   competent   Receiving   Office   as   defined  
residing  in  the  Philippines  for  the  same  purpose.  The  list   under  PCT  Rule  19.  
of   registered   resident   agents   or   representatives   is    
available  in  the  IPO  website.   RULE   9.   INTERNATIONAL   SEARCH   AND   INTERNATIONAL   SEARCHING  
  AUTHORITY.  –    
(b)   Where   the   international   application   is   filed   with   the   9.1.  An  international  application  shall  be  the  subject  of  an  
IPO   as   a   receiving   Office,   the   appointment   of   an   agent   international  search,  which  shall  be  carried  out  by  an  ISA.  
must   be   effected   in   the   Request   form,   signed   by   all   The   objective   of   the   international   search   is   to   discover  
applicants,   or   in   a   separate   power   of   attorney   submitted   relevant  prior  art.  
to   the   IPO   where   the   Request   form   is   signed   by   the    
appointed   agent   in   accordance   with   PCT   Rule   90.4.   The   9.2.   The   Director   General   shall   specify   one   or   more  
separate  power  of  attorney  does  not  require  legalization   competent   ISAs   for   international   applications   filed   with  
or  notarization.   the   IPO   and,   subject   to   any   required   acceptance   or  
  compliance   with   applicable   requirements,   the   IPO   shall  
(c)   For   purposes   of   the   procedure   before   the   IPO   as   a   publish  in  its  website  a  list  of  those  competent  ISAs.    
designated   or   elected   Office,   the   manner   of   appointing    
an   agent   or   representative   shall   comply   with   the   9.3.  The  applicant  shall,  in  his  Request  Form,  indicate  his  
requirements   prescribed   in   the   implementing   rules   and   choice  of  the  competent  ISA.    
regulations  of  the  IP  Code.    
  RULE   10.   INTERNATIONAL   PRELIMINARY   EXAMINATION   AND  
PART  II.  
INTERNATIONAL  PRELIMINARY  EXAMINING  AUTHORITY.  –    
INTERNATIONAL  PHASE  
  10.1.   An   international   application   may   be   the   subject   of  
RULE  8.  THE  IPO  AS  A  RECEIVING  OFFICE.  –     an   international   preliminary   examination   which   shall   be  
carried  out  by  an  IPEA.  
8.1.   The   IPO   shall   act   as   a   Receiving   Office   only   for  
 
applicants   who   are   residents   or   nationals   of   the  
10.2.   The   objective   of   the   international   preliminary  
Philippines.    
examination   is   to   formulate   a   preliminary   and   non-­‐
 
binding   opinion   on   the   questions   whether   the   claimed  
8.2.   The   IPO,   when   acting   as   a   Receiving   Office,   will   be  
invention   appears   to   be   novel,   to   involve   an   inventive  
identified  by  the  full  title  “Philippine  Receiving  Office”  or  
step  (to  be  non-­‐obvious)  and  to  be  industrially  applicable.    
by  the  abbreviation  “RO/PH.”  
 
 
10.3.   The   Director   General   shall   specify   one   or   more  
8.3.   Without   prejudice   to   the   national   security   and  
competent   IPEAs   for   international   applications   filed   with  
interests  of  the  Republic  of  the  Philippines,  the  IPO  shall  
the   IPO   and,   subject   to   any   required   acceptance   or  
perform   all   acts   connected   with   the   discharge   of   duties  
compliance   with   applicable   requirements,   the   IPO   shall  
of   a   Receiving   Office   under   the   Treaty   and   PCT  
publish  in  its  website  a  list  of  those  competent  IPEAs.    
Regulations.   The   functions   of   the   IPO   as   a   Receiving  
 
Office  include:  
10.4  Where  there  are  two  or  more  competent  IPEAs  for  
 
applications  filed  with  the  IPO,  the  applicant  shall,  in  his  
(1)   According   of   international   filing   dates   to  
Demand  Form,  indicate  his  choice  of  the  competent  IPEA.  
international   applications   fulfilling   the  
 
requirements   of   PCT   Article   11(1)   and   in  
RULE   11.   LANGUAGE   PRESCRIBED   BY   THE   RECEIVING   O FFICE.   –   The  
particular,  PCT  Rule  20;  
  language  accepted  by  the  Philippines  receiving  Office  for  
(2)   Checking   that   international   applications   the   filing   of   international   application   is   Filipino   or   English,  
meet   the   standards   for   format   and   content   of   provided   that   it   is   sufficient   that   the   description   and  
PCT  Article  14(1)  and  in  particular,  PCT  Rules  9,   claims   be   in   Filipino   or   English   for   the   purposes   of  
26,  29.1,  37,  38,  91,  and  portions  of  PCT  Rules  3   according   an   international   filing   date   under   PCT   Article  
through  11;     11(1).  
   
(3)  Assessing,  collecting,  and  transmitting  fees   RULE12.     TRANSLATION   FOR   THE   PURPOSES   OF   INTERNATIONAL  
due  for  processing  international  applications;     SEARCH.  –    
  (a)   Where   the   international   application   is   filed   in   Filipino,  
(4)   Transmitting   the   record   and   search   copies   the   applicant   shall,   within   one   (1)   month   from   the   date  
to  the  IB  and  ISA,  respectively;  and   of   receipt   of   the   international   application   by   the   IPO,  

o 147
Katrina Michelle Mancao
 
furnish   to   the   Office   a   translation   of   the   international   for  the  invention  to  be  carried  out  by  a  person  skilled  in  
application  into  English  in  accordance  with  PCT  Rule  12.3   the  art.  
(a)  and  (b).    
  The   requirements   as   to   the   format   and   content   of   the  
(b)   If   the   Engish   translation   is   not   furnished   within   the   description  are  set  forth,  in  particular,  in  PCT  Rules  5,  9,  
time  limit  under  paragraph  (a),  the  Office  shall  invite  the   10,   and   11,   and   Section   204   of   the   PCT   Administrative  
applicant  to  furnish  it  within  one  (1)  month  from  the  date   Instructions.  
of  invitation  or  two  (2)  months  from  the  date  of  receipt    
of   the   international   application   by   the   IPO,   whichever   RULE   18.   THE   CLAIMS.   -­‐   The   claim   or   claims   shall   define   the  
expires  later.   matter   for   which   protection   is   sought.   Claims   shall   be  
  clear   and   concise.   They   shall   be   fully   supported   by   the  
(c)   If   the   applicant   does   not   furnish   the   English   description.    
translation   within   the   applicable   time   limit   under    
paragraph   (b),   the   international   application   shall   be   The  requirements  as  to  the  format  and  content  of  claims  
considered  withdrawn  and  the  IPO  as  a  receiving  Office   are  set  forth,  in  particular,  in  PCT  Rules  6,  9,  10,  and  11.  
shall  so  declare  in  accordance  with  PCT  Rule  12.3(d).    
  RULE  19.  THE  D RAWINGS.  -­‐   Drawings   which   are   necessary   for  
(d)   The   IPO   shall   promptly   furnish   the   IB   and   the   ISA   a   the   understanding   of   the   invention   must   be   part   of   the  
copy  of  the  English  translation.   international   application   as   originally   filed   in   order   to  
  maintain  the  international  filing  date  during  the  national  
RULE  13.  D OCUMENTS  AND  CORRESPONDENCE.  –     stage.   If   drawings   are   referred   to   in   the   international  
(a)  Any  paper  submitted  by  the  applicant,  other  than  the   application  but  they  are  missing  upon  filing,  the  IPO  shall  
international   application   itself,   must,   if   not   itself   in   the   give   the   applicant   thirty   (30)   days   from   the   date   of  
form   of   a   letter,   be   accompanied   by   a   letter   identifying   receipt   of   the   international   application   to   submit   the  
the  international  application  to  which  it  relates  and  must   drawings.  If  the  missing  drawings  are  received  within  the  
be  signed  by  the  applicant.     thirty-­‐day  period,  the  international  filing  date  shall  be  the  
  date   on   which   such   drawings   are   received.   If   such  
(b)   Correspondence   intended   for   the   applicant   shall   be   drawings   are   not   timely   received,   all   references   to  
sent  to  the  agent  appointed  under  Rule  7.   drawings   in   the   international   application   shall   be  
  considered  non-­‐existent.  
RULE   14.   KEEPING  OF   RECORDS  AND   FILES.   -­‐   The   IPO   shall   keep    
the   records   relating   to   the   international   application   or   The   physical   requirements   for   drawings   are   set   forth   in  
purported   international   application   for   at   least   (ten)   10   PCT  Rule  11.3.  
years   from   the   international   filing   date   or,   where   no    
international   filing   date   is   accorded,   from   the   date   of   RULE   20.   THE   ABSTRACT.   -­‐   The   abstract   shall   merely   serve  
receipt  of  the  purported  application.     for  technical  information.  Lack  of  an  abstract  upon  filing  
  of   an   international   application   will   not   affect   the  
RULE   15.   THE   INTERNATIONAL   APPLICATION.   -­‐   An   international   granting   of   a   filing   date.   If   the   abstract   is   missing,   the  
application  shall  contain  a  request,  a  description,  one  or   IPO   shall   invite   the   applicant   to   submit   the   same   within  
more   claims,   one   or   more   drawings   (where   required),   one   (1)   month   from   the   date   of   the   invitation   and   the  
and  an  abstract.     IPO  shall  notify  the  ISA  accordingly.  
   
RULE   16.   THE   REQUEST.   -­‐   The   request   shall   be   made   in   The   requirements   as   to   the   format   and   content   of   the  
accordance   with   PCT   Rules   3   and   4.     Copies   of   the   abstract  are  set  forth  in  PCT  Rule  8.  
Request  form  are  available  at  the  IPO.    
  RULE   21.   PRIORITY   CLAIM.   –   Any   declaration   referred   to   in  
The   Request   Form   may   be   made   using   PCT-­‐EASY   Article   8(1)   (“priority   claim”)   shall   be   indicated   in   the  
software   made   available   by   the   IB,   in   which   case,   the   Request  Form  as  set  forth  in  PCT  Rule  4.10.  
applicant   is   required   to   submit   the   Request   as   a    
computer   print-­‐out   together   with   a   computer   diskette   Any   correction   or   addition   on   the   priority   claim   shall   be  
containing   a   copy   of   the   electronic   form   of   the   data   made  in  accordance  with  PCT  Rule  26bis.  
contained   in   the   Request   and   of   the   abstract   prepared    
using  that  software.   RULE  22.  PHYSICAL  REQUIREMENTS.  –    
  22.1.   An   international   application   and   each   of   the  
RULE   17.   THE   DESCRIPTION.   –     The   description   shall   disclose   documents   that   may   be   referred   to   in   the   checklist   of  
the  invention  in  a  manner  sufficiently  clear  and  complete   the  Request  Form  shall  be  filed  in  one  (1)  copy  only.    
 

148 z
Intellectual Property Law

22.2.   All   sheets   of   the   international   application   must   be   25.2.  If  the  IPO  finds  that  the  applicant  or  not  one  of  the  
on  A4  size  paper  (29.7  cm  x  21  cm)  which  shall  be  flexible,   applicants   is   a   resident   or   national   of   the   Philippines   or  
strong,  white,  smooth,  non-­‐shiny,  and  durable.   that   the   language   is   neither   Filipino   nor   English,   the  
  international   application   shall   be   treated   and  
22.3.  In  order  to  facilitate  processing  of  the  international   transmitted   to   the   IB   in   accordance   with   PCT   Rule   19.4,  
applications,   attention   of   applicants   is   directed   to   the   subject  to  payment  of  the  transmittal  fee  under  PCT  Rule  
other  physical  requirements  for  international  application   14.  
which  are  set  forth  in  PCT  Rule  11  and  Section  207  of  the    
PCT  Administrative  Instructions.   25.3.   If   the   IPO   finds   that   the   international   application  
  did   not,   at   the   time   of   receipt,   contain   the   elements  
RULE  23.  WHO  MAY  FILE  AN  INTERNATIONAL  APPLICATION.  –     listed  in  Rule  25.1  item  (3),  it  shall  invite  the  applicant  to  
A   resident   or   national   of   the   Philippines   may   file   an   file   the   required   correction   within   thirty   (30)   days   from  
international   application.   If   there   are   two   or   more   the  date  of  the  invitation.  If  the  applicant  complies  with  
applicants,   the   right   to   file   an   international   application   the   invitation,   the   IPO   shall   accord   as   the   international  
shall   exist   if   at   least   one   of   them   is   a   resident   or   national   filing   date   the   date   of   receipt   of   the   required   correction.  
of  the  Philippines.   If  the  applicant  does  not  comply  with  the  invitation,  the  
  IPO   shall   promptly   in   accordance   with   PCT   Rule   20.7(i)  
RULE  24.  WHERE  TO  FILE  AN  INTERNATIONAL  APPLICATION.  –     notify   the   applicant   that   his   application   is   not   and   will  
24.1.  A  resident  or  national  of  the  Philippines  may  file  an   not   be   treated   as   an   international   application   and   shall  
international  application  with  the  IPO  or  the  IB.   indicate  the  reasons  therefor.  
   
24.2.   An   international   application   may   be   filed   directly   RULE   26.   TRANSMITTAL   OF   THE   INTERNATIONAL   APPLICATION.   –  
with   the   IPO   or   through   registered   mail   by   postal   26.1.   The   IPO   shall   transmit   the   “record   copy”   of   the  
authority.   international   application   to   the   IB   in   accordance   with  
  PCT  Rule  22.    
RULE   25.   ACCORDING   OF   INTERNATIONAL   FILING   D ATE   AND   ITS    
EFFECT.  –     26.2.   The   IPO   shall   transmit   the   “search   copy”   of   the  
international   application   to   the   competent   ISA   after  
25.1.   The   IPO,   as   the   competent   receiving   Office,   shall  
payment   of   the   search   fee   by   the   applicant   in  
accord   as   the   international   filing   date   the   date   of   receipt  
accordance  with  PCT  Rule  23.  
of   the   international   application,   provided   that,   at   the  
 
time  of  receipt:  
26.3.   The   Director   General   may   prohibit   or   restrict   the  
 
transmittal   of   an   application,   if   in   his   opinion,   to   do   so  
(1)   at   least   one   applicant   is   a   resident   or  
would   be   prejudicial   to   the   national   security   and  
national   of   the   Philippines;chan   robles   virtual  
interests  of  the  Republic  of  the  Philippines.  The  Director  
law  library  
General   may   apply   other   measures,   as   provided   under  
 
PCT   Article   27(8),   that   will   not   prevent   the   transmittal   of  
(2)  the  international  application  is  in  the  Filipino  
the   record   copy   and   search   copy   but   will   restrict   the  
or  English  language;  andchan  robles  virtual  law  
contents  of  the  transmittals.  
library  
 
 
RULE  27.  THE  INTERNATIONAL  PUBLICATION  AND  ITS  EFFECT.  –    
(3)   the   international   application   contains   at  
least  the  following  elements:chan  robles  virtual   27.1.  In  accordance  with  PCT  Article  21  and  PCT  Rule  48,  
law  library     the   IB   shall   publish   the   international   application  
  promptly   after   the   expiration   of   eighteen   (18)   months  
(i)   an   indication   that   it   is   intended   as   from   the   priority   date   of   the   application.   However,   the  
an  international  application;   applicant   may   ask   the   IB   to   publish   his   international  
  application   any   time   before   the   expiration   of   eighteen  
(ii)   the   designation   of   at   least   one   (18)   months   from   the   priority   date   in   accordance   with  
Contracting  State;   PCT  Rule  48.4.  
   
(iii)  the  name  of  the  applicant;   27.2.   In   accordance   with   PCT   Article   29(1),   as   far   as   the  
  protection   of   any   rights   of   the   applicant   under   Section  
(iv)   a   part   which   on   the   face   of   it   46   of   the   IP   Code   in   the   Philippines   is   concerned,   the  
appears  to  be  a  description;  and   international   publication   in   English   of   an   international  
  application  shall  have  the  same  effect  as  a  publication  in  
(v)   a   part   which   on   the   face   of   it   the  IPO  Gazette  under  Section  44  of  the  IP  Code  and  its  
appears  to  be  a  claim  or  claims.   implementing  rules  and  regulations  provided  that  notice  
  of   the   international   publication   and   copy   of   the  

o 149
Katrina Michelle Mancao
 
international   application   has   been   transmitted   by   the   accompanied   by   a   letter   indicating   the   differences  
applicant   to   the   actual   unauthorized   user   of   the   between  the  replaced  sheet  and  the  replacement  sheet.    
invention   claimed   in   the   international   application    
pursuant  to  Sec.  46.2  of  the  IP  Code.     RULE   30.   DELAYS   IN   MEETING   TIME   LIMITS.   Delays   in   meeting  
    time   limits   during   international   processing   of  
27.3.   If   the   language   in   which   the   international   international   applications   may   only   be   excused   as  
publication   has   been   effected   is   a   language   other   than   provided  in  PCT  Rule  82.  
English,   the   protection   of   any   right   of   the   applicant    
under   Section   46   of   the   IP   Code   in   the   Philippines   shall   RULE   31.   RECORDING  OF   CHANGES.   -­‐   Before   the   expiration   of  
be   applicable   only   from   such   time   as   a   translation   into   the   time   limit   referred   to   in   PCT   Articles   22(1)   and  
English   has   been   published   in   the   IPO   Gazette   under   39(1)(a),   the   IPO   may   receive   from   the   applicant   a  
Section  44  of  the  IP  Code  and  its  implementing  rules  and   request   for   the   recording   of   a   change   in   the   person,  
regulations,   and,   such   translation   into   English   has   been   name,   residence,   nationality   or   address   of   the   applicant  
transmitted  by  the  applicant  to  the  actual  unauthorized   or   a   change   in   the   person,   name   or   address   of   the  
user   of   the   invention   claimed   in   the   international   inventor,   the   agent   or   the   common   representative.   The  
application  pursuant  to  Section  46.2  of  the  IP  Code.     IPO   shall   notify   the   IB   of   the   request   for   recording   a  
  change.    
27.4.   Where   the   international   publication   has   been    
effected,   on   the   request   of   the   applicant,   before   the   RULE   32.     WITHDRAWALS.   -­‐   The   IPO   may   receive   from   the  
expiration  of  eighteen  (18)  months  from  the  priority  date,  
applicant   notice   of   withdrawal   of   the   international  
the  rights  provided  for  under  Sec.  46  of  the  IP  Code  shall  
application,   any   designation,   or   priority   claims   in  
be   applicable   only   from   the   expiration   of   eighteen   (18)  
accordance  with  PCT  Rule  90bis(1),  90bis(2),  90bis(3).  
months   from   the   priority   date   subject   to   the   conditions  
 
mentioned  in  the  preceding  paragraphs.  
Withdrawal   of   a   demand   or   any   election   shall   be   made  
 
directly  to  the  IB  in  accordance  with  PCT  Rule  90bis.4.  
RULE   28.   DEMAND   FOR   INTERNATIONAL   PRELIMINARY  
 
EXAMINATION.  –     RULE   33.   CERTIFIED   COPIES   OF   THE   INTERNATIONAL   APPLICATION  
  AND  PRIORITY  DOCUMENT.  –    
28.1.   On   filing   of   a   demand   under   PCT   Article   31,   the  
(a)   The   applicant   may   request   the   IPO   to   prepare   a  
international   application   shall   be   the   subject   of   an  
certified  copy  of  the  international  application  as  filed  and  
international  preliminary  examination.  
of   any   corrections   thereto   upon   payment   of   the  
 
appropriate  fee.    
28.2.   The   demand   for   international   preliminary  
 
examination   shall   be   filed   directly   by   the   applicant   with  
(b)  Where  the  priority  document  is  issued  by  the  IPO,  the  
the   competent   IPEA   prior   to   the   expiration   of   the  
applicant  may,  pursuant  to  PCT  Rule  17.1(b),  request  the  
nineteenth  (19th)  month  from  the  priority  date.    
IPO  to  prepare  and  transmit  the  priority  document  to  the  
 
IB.  Such  request  shall  be  made  not  later  than  sixteen  (16)  
28.3.   The   demand   shall   indicate   the   States   in   which   the  
months   after   the   priority   date   and   shall   be   subject   to  
applicant   intends   to   use   the   results   of   the   international  
payment  of  a  fee.  
preliminary  examination.  
 
 
PART  III.  
28.4.  In  accordance  with  PCT  Article  31  and  PCT  Rules  57  
NATIONAL  PHASE  
and   58,   the   demand   shall   be   subject   to   the   payment   by  
 
the  applicant  of  the  prescribed  fees  directly  to  the  IPEA  
RULE  34.  THE  IPO  AS  A  DESIGNATED  OR  ELECTED  OFFICE.  –    
within   one   (1)   month   from   the   date   on   which   the  
demand  was  submitted.     34.1.  The  IPO  will  act  as  the  Designated  or  Elected  Office  
  for  international  applications  in  which  the  Philippines  has  
Copies  of  the  Demand  Form  are  available  at  the  IPO.   been  designated  or  elected.    
   
RULE   29.   MANNER  OF   MAKING   CORRECTIONS.   –   Any   correction   34.2.   The   IPO,   when   acting   as   a   Designated   or   Elected  
Office   during   international   processing   will   be   identified  
offered  to  the  IPO  as  a  receiving  Office  may  be  stated  in  
by   the   full   title   “Philippine   Designated   Office”   or   by  
a  letter  addressed  to  the  IPO  if  the  correction  is  of  such  a  
abbreviation   “DO/PH”   and   “Philippine   Elected   Office”   or  
nature   that   it   can   be   transferred   from   the   letter   to   the  
by  abbreviation  “EO/PH.”  
record   copy   without   adversely   affecting   the   clarity   and  
 
the   direct   reproducibility   of   the   sheet   on   to   which   the  
34.3.   The   major   functions   of   the   IPO   in   respect   to  
correction   is   to   be   transferred;   otherwise,   the   applicant  
international   applications   in   which   the   Philippines   has  
shall  be  required  to  submit  the  missing  requirement  or  a  
been  designated  or  elected,  shall  include:  
replacement   sheet   containing   the   correction  

150 z
Intellectual Property Law

  with   the   IP   Code   and   its   implementing   rules   and  


(1)   Receiving   various   notifications   throughout   regulations.  
the  international  phase;  and    
  RULE   36.  ENTRY   INTO  THE   NATIONAL   PHASE  OF  AN  INTERNATIONAL  
(2)   Accepting   for   national   phase   examination   APPLICATION   CLAIMING   THE   PRIORITY   OF   AN   EARLIER   PHILIPPINE  
international   applications   which   comply   with   NATIONAL  APPLICATION.  Within  one  (1)  month  from  the  date  
Part  3  of  these  Rules.  
of   entry   into   the   national   phase   of   an   international  
 
application   claiming   the   priority   of   an   earlier   Philippine  
RULE  35.  ENTRY  INTO  NATIONAL  PHASE.  –    
national  application,  the  IPO  shall  invite  the  applicant  to  
(a)  An  international  application  enters  the  national  phase   elect   only   one   (1)   application   for   further   prosecution  
when   the   applicant   furnishes   IPO   a   copy   of   the   within   two   (2)   months   from   the   date   of   the   invitation,   or  
international   application   in   English   (unless   already   within  a  longer  period  which  the  IPO  may  allow  but  not  
transmitted  by  the  IB),  or,  if  the  application  was  filed  in   to   exceed   six   (6)   months   from   the   date   of   entry   into   the  
another   language,   its   English   translation   not   later   than   national  phase.  
thirty   (30)   months   from   the   priority   date   based   on   PCT    
Article  22  (1)  and  PCT  Article  39  (1)  (a).   RULE  37.  REVIEW  BY  IPO  AS  A  D ESIGNATED  OFFICE.  –    
 
(a)   Where   the   receiving   Office   other   than   the   IPO   has  
Subject  to  the  payment  of  an  extension  fee  for  late  entry  
refused   to   accord   an   international   filing   date   to   an  
equal   to   fifty   percent   (50%)   of   the   filing   fee   prescribed   in  
international   application   designating   the   Philippines   or  
the  IPO  Fee  Structure,  the  entry  into  the  national  phase  
has   declared   that   the   international   application   is  
may  be  extended  by  one  (1)  month.  
considered   withdrawn   either   generally   or   as   to   the  
 
Philippines,   or   where   the   IB   has   made   a   finding   under  
(b)  The  filing  fee  prescribed  in  the  IPO  fee  structure  shall  
PCT   Article   12(3)   that   the   international   application   is  
be   paid   within   one   (1)   month   from   the   date   of   entry   into  
considered   withdrawn   because   it   has   not   received   the  
the   national   phase;   provided   that   an   international  
record   copy   within   the   prescribed   time   limit,   the  
application   which   claims   the   priority   of   an   earlier  
applicant   may   request   review   of   the   matter   by   the   IPO  
Philippine   national   application   shall   be   exempt   from  
provided   the   requirements   of   and   the   time   limit  
payment  of  the  filing  fee.If  the  applicant  fails  to  pay  the  
specified  by  the  Treaty  and  PCT  regulations  as  well  as  the  
filing   fee   as   herein   provided,   the   application   shall   be  
requirements  for  entry  into  national  phase  in  these  Rules  
deemed  withdrawn  in  the  Philippines.    
are  complied  with.  
 
 
(c)   The   translation   of   the   international   application,   as  
(b)  Where  the   refusal,   declaration   or   finding   mentioned  
filed,   referred   to   in   paragraph   (a)   shall   include   a  
in   paragraph   (a)   was   the   result   of   an   error   or   omission  
translation  into  English  of:  
on   the   part   of   the   receiving   Office   or   that   the   finding  
 
was  the  result  of  an  error  or  omission  on  the  part  of  the  
(1)  the  description;  
IB,   the   IPO   as   a   designated   Office   may   treat   the  
 
international  application  as  if  such  error  or  omission  has  
(2)  the  claims  as  filed  and,  where  applicable,  as  
not  occurred.  
amended  under  PCT  Article  19;  
 
 
RULE  38.    TIME  LIMIT  TO  SUBMIT  THE  PRIORITY  DOCUMENT.  –    
(3)  any  text  matter  of  the  drawings;  
  38.1.   The   priority   claim   of   an   international   application  
(4)  the  abstract;  and   designating   or   electing   the   Philippines   shall   not   be  
  disregarded   by   IPO,   where   the   priority   document   is  
(5)   where   applicable,   amendments   to   the   received   by   the   IB   not   later   than   sixteen   (16)   months  
description,   claims   or   drawings   under   PCT   after   the   priority   date   in   accordance   with   PCT   Rule   17.1  
Article   34,   as   contained   in   the   annexes   to   the   (a)  and  (b).  
international  preliminary  examination  report.      
  38.2.  If  the  priority  document  is  not  submitted  to  the  IB  
If   the   translation   of   the   amendments   under   PCT   Article   within   sixteen   (16)   months   after   the   priority   date   in  
19  or  PCT  Article  34,  as  the  case  may  be,  is  not  included,   accordance  with  PCT  Rule  17.1  (a)  and  (b),  the  applicant  
those  amendments  will  be  considered  cancelled.   may   submit   to   the   IPO   a   certification   from   the   national  
  Office   concerned   stating   the   following   data   regarding  
(d)   During   the   national   phase,   and   at   anytime   before   a   the  application  the  priority  of  which  is  claimed:  name  of  
patent   is   granted   or   refused   on   the   international   the  applicant,  filing  date,  application  number  and  title  of  
application,   the   applicant   may   present   amendments   to   the   application.   The   certification,   and   its   English  
the   specification,   claims   and   drawings   in   accordance   translation   if   not   in   English,   together   with   payment   of  
the   fee   for   extension   of   time   to   submit   priority  
document  and  the  surcharge  for  late  payment  provided  

o 151
Katrina Michelle Mancao
 
for  in  the  IPO  Fee  Structure,  shall  be  submitted  within  six   902   Priority  Document   500.00  
(6)   months   from   the   date   of   entry   of   the   international   Fee  PCT  Rule  17.1(b)  
application   into   the   national   phase   without   need   of   903   Fee  for  Transmitting   1,750.00  
notice.  The  certification  and  its  English  translation  if  not   Priority  Document  PCT  
Rule  17.1(b)  
in   English   do   not   require   legalization.   Failure   of   the  
904   Confirmation  Fee   Fifty  (50%)  of  the  sum  
applicant   to   comply   with   this   rule   shall   be   ground   to   PCT  Rules  4.9(c)  and   of  designation  fees  
disregard  the  priority  claim.   15.5(a)   under  PCT  Rule  15.5  in  
  Philippine  currency  at  
38.3  Where  the  validity  of  the  priority  claim  is  relevant  to   the  prevailing  rate  of  
the  determination  of  whether  the  invention  concerned  is   exchange  at  the  time  
patentable,   the   applicant   shall   be   required   to   submit   the   the  payment  is  due  
English  translation  of  the  priority  document.   905   Late  Payment  Fee  PCT   3,500.00  
  Rule  16bis.2(a)(ii)  
906   Certified  Copy  of  the   500.00  
RULE  39.  PROCEDURE  FOR  GRANT  OF  PATENT.  –    
International  Application  
An   international   application   that   has   entered   the   Fee  PCT  Rule  20.9  
national   phase   shall   be   governed   by   the   applicable   907   Transmittal  Fee  PCT  Rule   3,500.00  
provisions   under   Republic   Act   No.   8293   and   its   19.4  
implementing   rules   and   regulations   and   the   applicable    
provisions   of   the   Treaty,   PCT   Regulations,   and   PCT   41.2.  Fees  for  the  benefit  of  the  IB  (International  Fee)  
Administrative  Instructions.    
  The   amount   of   the   basic   fee   referred   to   in   PCT   Rule  
RULE  40.  APPLICATION  OF  THE  IPO  FEE  STRUCTURE.  –     15.1(i),  the  designation  fee  referred  to  in  PCT  Rule  15.1(ii)  
The   IPO   Fee   Structure   shall   apply   after   entry   into   the   for   any   designation   made   under   PCT   Rule   4.9(a),   and   the  
national   phase   of   the   international   application   provided   designation   fee   referred   to   in   PCT   Rule   15.5   for   any  
that:   designation   made   under   PCT   Rule   4.9(b)   and   confirmed  
  under  PCT  Rule  4.9(c),  are  published  in  the  PCT  Gazette  
(a)   an   international   application   which   validly   and  are  available  at  the  IPO.    
claims   the   priority   of   an   earlier   Philippine    
national   application   shall   be   exempt   from   41.3.   Fee   for   the   benefit   of   the   ISA   (Search   Fee)chan  
payment   of   fees   that   were   already   paid   in   robles  virtual  law  library  
connection  with  that  earlier  Philippine  national    
application;   The  amount  of  the  search  fee  referred  to  in  PCT  Rule  16.1  
  of   the   competent   ISA   or   ISAs   for   the   Philippines   is  
(b)  the  due  date  for  payment  of  the  first  annual   published  in  the  PCT  Gazette.  
fee   shall   be   computed   from   the   date   of   the    
international   publication   regardless   of   the   RULE   42.   D UE   DATES   AND   EXTENSION   OF   TIME   LIMITS   FOR  
language  of  publication;  and   PAYMENT  OF   FEES   COLLECTED  BY   IPO.   –   All   fees   must   be   paid  
  on   or   before   the   due   date   prescribed   in   the   PCT  
(c)   the   international   application   shall   be   Regulations  and  these  Rules.    
deemed   withdrawn   if   the   substantive    
examination   fee   is   not   paid   within   six   (6)   42.1.  The  transmittal  fee,  basic  fee  and  search  fee  shall  be  
months  from  the  date  of  entry  into  the  national   paid  by  the  applicant  within  one  (1)  month  from  the  date  
phase.   The   applicant   may   use   a   form   for   of  receipt  by  the  IPO  of  the  international  application;    
payment  prescribed  by  IPO  for  this  purpose.        
  42.2.   The   designation   fee   for   any   designation   made  
PART  III.   under  PCT  Rule  4.9(a)  shall  be  paid  within  a  time  limit  of:  
FEES  DURING  INTERNATIONAL  PHASE    
  (a)  one  (1)  year  from  the  priority  date,  or  
RULE   41.   FEES   TO   BE   COLLECTED   BY   THE   IPO.   –   The   following    
fees  shall  be  collected  by  the  IPO:   (b)   one   (1)   month   from   the   date   of   receipt   of  
  the  international  application  if  that  one-­‐month  
41.1.   Fees   for   the   benefit   of   the   IPO.   The   type   and   period  expires  later  than  one  (1)  year  from  the  
amount  of  fees  for  the  benefit  of  the  IPO  are:   priority  date;    
   
Ref.   Type  of  Fee   Amount(Philippine   42.3.   Where   the   fees   due   under   this   rule   have   not   been  
Code   Peso)   paid,  or  that  the  amount  paid  is  insufficient  to  cover  the  
901   Transmittal  Fee  PCT  Rule   3,500.00   total   amount   due,   the   IPO   shall   invite   the   applicant   to  
14  
pay  the  required  fees  together  with  the  late  payment  fee  

152 z
Intellectual Property Law

within  one  (1)  month  without  extension  of  time  from  the   Other  fees  due  to  the  IPEA  in  accordance  with  the  Treaty  
date  of  invitation;   and  PCT  Regulations  shall  be  paid  directly  to  the  IPEA  in  
  the  currency,  form  and  manner  prescribed  by  it.  
42.4.   The   designation   fee   under   PCT   Rule   15.5   for   any    
designation   made   under   PCT   Rule   4.9(b)   and   confirmed   The  Schedules  of  Fees  of  the  IPEAs  are  published  in  the  
under   PCT   Rule   4.9(c),   together   with   the   confirmation   PCT  Gazette  and  are  available  at  the  IPO.  
fee  under  PCT  Rule  15.5,  shall  be  paid  within  fifteen  (15)    
months  from  the  priority  date.   RULE  46.  REFUNDS.  –    
  46.1   The   IPO   shall   refund   the   international   fee   to   the  
RULE   43.   FEES   FOR   THE   BENEFIT   OF   IPO;   FORM,   MANNER   AND   applicant  in  accordance  with  PCT  Rule  15.6:  
CURRENCY  OF  PAYMENT.  –      
(a)  All  fees  for  the  benefit  of  the  IPO  shall  be  paid  in  cash,   (a)   if   the   IPO   has   found   that   the   international  
money   order,   bank   drafts   and/or   checks   in   Philippine   filing   date   cannot   be   accorded   in   accordance  
currency.  In  case  of  cash  payment,  the  date  of  receipt  is   with  PCT  Article  11(1);  
the  date  on  which  the  full  amount  due  was  paid  in  cash.    
In   case   of   money   order,   check   or   bank   draft   payment,   (b)   if,   before   the   transmittal   of   the   record   copy  
the   date   of   receipt   of   the   money   order,   check   or   bank   to   the   IB,   the   international   application   is  
draft  will  be  the  date  of  presentment  provided  that  the   withdrawn  or  considered  withdrawn;  or  
same   is   honored   upon   first   presentment   and   provided    
that  the  payment  covers  the  full  amount  due.   (c)   if,   due   to   prescriptions   concerning   the  
  national   security,   the   international   application  
(b)   Payment   shall   be   made   to   the   IPO   Cashier   during   is  not  treated  as  such.  
regular   working   days   and   business   hours   from   Monday    
to  Friday,  8:00  a.m.  to  12:00  noon  and  from  1:00  p.m.  to   46.2.  The  IPO  shall  refund  the  search  fee  to  the  applicant  
5:00  p.m.   in  accordance  with  PCT  Rule  16.2:  
   
RULE   44.   FEES  FOR   THE   BENEFIT  OF   IB  AND   ISA;   FORM,   MANNER   (a)   if   the   IPO   has   found   that   the   international  
AND  CURRENCY  OF  PAYMENT.  –    
filing   date   cannot   be   accorded   in   accordance  
with  PCT  Article  11(1);  
44.1   All   fees   for   the   benefit   of   the   IB   and   the   ISA  
 
collected  by  IPO  as  receiving  Office  shall  be  paid  to  IPO  
(b)   if,   before   the   transmittal   of   the   search   copy  
in  the  currency  or  any  of  the  currencies,  if  more  than  one,  
to   the   ISA,   the   international   application   is  
specified  by  the  IB  or  the  ISA  as  the  case  may  be,  in  the  
withdrawn  or  considered  withdrawn;  or  
form  of  cash  or  bank  draft.  In  case  of  cash  payment,  the  
 
date  of  receipt  is  the  date  on  which  the  full  amount  due  
(c)   if,   due   to   prescriptions   concerning   the  
was  paid  in  cash.  In  case  of  bank  draft  payment,  the  date  
national   security,   the   international   application  
of   receipt   of   the   bank   draft   will   be   the   date   of  
is  not  treated  as  such.  
presentment   provided   that   the   same   is   honored   upon  
 
first  presentment  and  provided  further  that  the  payment  
RULE  47.    REDUCTION  IN  FEES.  –  An  applicant  may  be  entitled  
covers   the   full   amount   due.   Payment   shall   be   made   to  
the   IPO   Cashier   during   regular   working   days   and   to   a   reduction   in   fees   provided   that   the   requirements,   as  
business   hours   from   Monday   to   Friday,   8:00   a.m.   to   specified   by   the   IB   and   Authorities   under   the   Treaty  
12:00  noon  and  from  1:00  p.m.  to  5:00  p.m.   granting  such  reduction  in  fees,  are  fully  satisfied  and/or  
  duly  complied  with.  
44.2.   Where   the   international   application   incurs   an    
additional  fee  other  than  the  search  fee  under  PCT  Rule   RULE   48.   SEPARABILITY   CLAUSE.   –   If   any   provision   in   these  
16,  the  applicant  shall  pay  directly  to  the  ISA  such  other   Rules   or   application   of   such   provision   to   any  
fees   required   by   the   ISA   in   the   currency,   form   and   circumstance   is   held   invalid,   the   remainder   of   these  
manner  prescribed  by  it.   Rules  shall  not  be  affected.  
   
44.3.   The   Schedules   of   Fees   of   the   IB   and   ISAs   are   RULE   49.   FILING   OF   CERTIFIED   COPIES.   –   Mr.   Eduardo   Joson,  
published  in  the  PCT  Gazette  and  are  available  at  the  IPO.   Records  Officer  II,  is  hereby  directed  to  immediately  file  
  three   (3)   certified   copies   of   these   Rules   with   the  
Rule   45.   Fees   Payable   to   the   IPEA.   -­‐   The   international   University   of   the   Philippines   Law   Center,   and,   one   (1)  
preliminary  examination  fee  and  the  handling  fee  under   certified  copy  each  with  the  Office  of  the  President,  the  
PCT   Rule   58   shall   be   paid   directly   to   the   IPEA   in   the   Senate  of  the  Philippines,  the  House  of  Representatives,  
currency,  form  and  manner  prescribed  by  it.   the   Supreme   Court   of   the   Philippines,   and   the   National  
  Library.  
 

o 153
Katrina Michelle Mancao
 
RULE   50.   EFFECTIVITY.   –   Without   prejudice   to   the   patentee,   he   has   the   exclusive   right   of   making,   selling   or  
performance   by   the   IPO   of   its   functions   as   a   receiving   using  the  invention.  On  the  assumption  that  petitioner’s  
Office,   designated   Office   or   elected   Office   under   the   advertising   units   were   patentable   inventions,   petitioner  
Treaty,   from   and   as   of   the   entering   into   effect   of   the   revealed   them   fully   to   the   public   by   submitting   the  
Treaty   with   respect   to   the   Philippines   on   17   August   2001,   engineering  drawings  thereof  to  the  National  Library.  
these   Rules   shall   take   effect   on   October   22,   2001   after    
publication   in   a   newspaper   of   general   circulation   in   the   To  be  able  to  effectively  and  legally  preclude  others  from  
Philippines  on  October  4,  2001.   copying   and   profiting   from   the   invention,   a   patent   is   a  
  primordial   requirement.     No   patent,   no   protection.   The  
I. Issuance  or  Refusal  of  Patents   ultimate  goal  of  a  patent  system  is  to  bring  new  designs  
and   technologies   into   the   public   domain   through  
 
disclosure.   Ideas,   once   disclosed   to   the   public   without  
SECTION  50.  GRANT  OF  PATENT.  –     the   protection   of   a   valid   patent,   are   subject   to  
50.1.   If   the   application   meets   the   requirements   of   this   appropriation  without  significant  restraint.  
Act,   the   Office   shall   grant   the   patent:   Provided,   That   all    
the  fees  are  paid  on  time.   On   one   side   of   the   coin   is   the   public   which   will   benefit  
  from   new   ideas;   on   the   other   are   the   inventors   who  
50.2.   If   the   required   fees   for   grant   and   printing   are   not   must   be   protected.   The   act   secured   to   the   inventor   the  
paid   in   due   time,   the   application   shall   be   deemed   to   be   exclusive  right  to  make  use,  and  vend  the  thing  patented,  
withdrawn.   and  consequently  to  prevent  others  from  exercising  like  
  privileges   without   the   consent   of   the   patentee.     It   was  
50.3.   A   patent   shall   take   effect   on   the   date   of   the   passed   for   the   purpose   of   encouraging   useful   invention  
publication  of  the  grant  of  the  patent  in  the  IPO  Gazette.   and   promoting   new   and   useful   inventions   by   the  
  protection  and  stimulation  given  to  inventive  genius,  and  
SECTION  51.  REFUSAL  OF  THE  APPLICATION.  –     was   intended   to   secure   to   the   public,   after   the   lapse   of  
the   exclusive   privileges   granted   the   benefit   of   such  
51.1.   The   final   order   of   refusal   of   the   examiner   to   grant   inventions  and  improvements.  
the   patent   shall   be   appealable   to   the   Director   in    
accordance  with  this  Act.   The  law  attempts  to  strike  an  ideal  balance  between  the  
  two  interests:  
51.2.  The  Regulations  shall  provide  for  the  procedure  by    
which   an   appeal   from   the   order   of   refusal   from   the   “The   patent   system   thus   embodies   a   carefully  
Director  shall  be  undertaken.   crafted   bargain   for   encouraging   the   creation  
  and   disclosure   of   new   useful   and   non-­‐obvious  
SECTION  53.  CONTENTS  OF  PATENT.  –     advances   in   technology   and   design,   in   return  
The  patent  shall  be  issued  in  the  name  of  the  Republic  of   for   the   exclusive   right   to   practice   the   invention  
the  Philippines  under  the  seal  of  the  Office  and  shall  be   for  a  number  of  years.    The  inventor  may  keep  
signed  by  the  Director,  and  registered  together  with  the   his   invention   secret   and   reap   its   fruits  
description,   claims,   and   drawings,   if   any,   in   books   and   indefinitely.     In   consideration   of   its   disclosure  
records  of  the  Office.   and  the  consequent  benefit  to  the  community,  
  the  patent  is  granted.    An  exclusive  enjoyment  
PEARL  &  DEAN  V.  SHOEMART  (2003),  SUPRA   is   guaranteed   him   for   17   years,   but   upon   the  
expiration  of  that  period,  the  knowledge  of  the  
Court  decision:  
invention   inures   to   the   people,   who   are   thus  
For  some  reason  or  another,  petitioner  never  secured  a  
enabled  to  practice  it  and  profit  by  its  use.”  
patent   for   the   light   boxes.     It   therefore   acquired   no  
 
patent  rights  which  could  have  protected  its  invention,  if  
The   patent   law   has   a   three-­‐fold   purpose:   “first,   patent  
in   fact   it   really   was.     And   because   it   had   no   patent,  
law   seeks   to   foster   and   reward   invention;   second,   it  
petitioner   could   not   legally   prevent   anyone   from  
promotes   disclosures   of   inventions   to   stimulate   further  
manufacturing   or   commercially   using   the   contraption.  
innovation   and   to   permit   the   public   to   practice   the  
There  can  be  no  infringement  of  a  patent  until  a  patent  
invention   once   the   patent   expires;   third,   the   stringent  
has   been   issued,   since   whatever   right   one   has   to   the  
requirements   for   patent   protection   seek   to   ensure   that  
invention   covered   by   the   patent   arises   alone   from   the  
ideas   in   the   public   domain   remain   there   for   the   free   use  
grant  of  patent.  An  inventor  has  no  common  law  right  to  
of  the  public.”  
a   monopoly   of   his   invention.   He   has   the   right   to   make  
 
use   of   and   vend   his   invention,   but   if   he   voluntarily  
It  is  only  after  an  exhaustive  examination  by  the  patent  
discloses   it,   such   as   by   offering   it   for   sale,   the   world   is  
office   that   a   patent   is   issued.     Such   an   in-­‐depth  
free  to  copy  and  use  it  with  impunity.    A  patent,  however,  
investigation   is   required   because   in   rewarding   a   useful  
gives   the   inventor   the   right   to   exclude   all   others.     As   a  

154 z
Intellectual Property Law

invention,  the  rights  and  welfare  of  the  community  must   on   the   composition   and   use   of   medicines,   be  
be   fairly   dealt   with   and   effectively   guarded.     To   that   end,   they  old  or  new;  on  the  construction  and  use  of  
the   prerequisites   to   obtaining   a   patent   are   strictly   ploughs   or   watches   or   churns;   or   on   the  
observed  and  when  a  patent  is  issued,  the  limitations  on   mixture   and   application   of   colors   for   painting  
its  exercise  are  equally  strictly  enforced.    To  begin  with,  a   or   dyeing;   or   on   the   mode   of   drawing   lines   to  
genuine   invention   or   discovery   must   be   demonstrated   produce   the   effect   of   perspective,   would   be  
lest   in   the   constant   demand   for   new   appliances,   the   the   subject   of   copyright;   but   no   one   would  
heavy  hand  of  tribute  be  laid  on  each  slight  technological   contend   that   the   copyright   of   the   treatise  
advance  in  art.   would   give   the   exclusive   right   to   the   art   or  
  manufacture  described  therein.      The  copyright  
There   is   no   such   scrutiny   in   the   case   of   copyrights   nor   of   the   book,   if   not   pirated   from   other   works,  
any   notice   published   before   its   grant   to   the   effect   that   a   would  be  valid  without  regard  to  the  novelty  or  
person   is   claiming   the   creation   of   a   work.     The   law   want   of   novelty   of   its   subject   matter.     The  
confers  the  copyright  from  the  moment  of  creation  and   novelty   of   the   art   or   thing   described   or  
the  copyright  certificate  is  issued  upon  registration  with   explained  has  nothing  to  do  with  the  validity  of  
the  National  Library  of  a  sworn  ex-­‐parte  claim  of  creation.   the   copyright.     To   give   to   the   author   of   the  
  book   an   exclusive   property   in   the   art   described  
Therefore,   not   having   gone   through   the   arduous   therein,  when  no  examination  of  its  novelty  has  
examination   for   patents,   the   petitioner   cannot   exclude   ever   been   officially   made,   would   be   a   surprise  
others  from  the  manufacture,  sale  or  commercial  use  of   and   a   fraud   upon   the   public.     That   is   the  
the   light   boxes   on   the   sole   basis   of   its   copyright   province   of   letters   patent,   not   of   copyright.    
certificate  over  the  technical  drawings.   The   claim   to   an   invention   of   discovery   of   an   art  
  or   manufacture   must   be   subjected   to   the  
Stated   otherwise,   what   petitioner   seeks   is   exclusivity   examination   of   the   Patent   Office   before   an  
without   any   opportunity   for   the   patent   office   (IPO)   to   exclusive   right   therein   can   be   obtained;   and   a  
scrutinize   the   light   box’s   eligibility   as   a   patentable   patent  from  the  government  can  only  secure  it.  
invention.    The  irony  here  is  that,  had  petitioner  secured    
a   patent   instead,   its   exclusivity   would   have   been   for   17   The  difference  between  the  two  things,  letters  
years   only.     But   through   the   simplified   procedure   of   patent   and   copyright,   may   be   illustrated   by  
copyright-­‐registration   with   the   National   Library   —   reference   to   the   subjects   just   enumerated.    
without   undergoing   the   rigor   of   defending   the   Take   the   case   of   medicines.     Certain   mixtures  
patentability   of   its   invention   before   the   IPO   and   the   are  found  to  be  of  great  value  in  the  healing  art.    
public  —  the  petitioner  would  be  protected  for  50  years.     If   the   discoverer   writes   and   publishes   a   book  
This   situation   could   not   have   been   the   intention   of   the   on   the   subject   (as   regular   physicians   generally  
law.   do),   he   gains   no   exclusive   right   to   the  
  manufacture   and   sale   of   the   medicine;   he   gives  
In   the   oft-­‐cited   case   of   Baker   vs.   Selden,   the   United   that   to   the   public.     If   he   desires   to   acquire   such  
States   Supreme   Court   held   that   only   the   expression   of   exclusive   right,   he   must   obtain   a   patent   for   the  
an  idea  is  protected  by  copyright,  not  the  idea  itself.    In   mixture   as   a   new   art,   manufacture   or  
that   case,   the   plaintiff   held   the   copyright   of   a   book   composition   of   matter.     He   may   copyright   his  
which   expounded   on   a   new   accounting   system   he   had   book,   if   he   pleases;   but   that   only   secures   to  
developed.     The   publication   illustrated   blank   forms   of   him   the   exclusive   right   of   printing   and  
ledgers   utilized   in   such   a   system.     The   defendant   publishing  his  book.    So  of  all  other  inventions  
reproduced   forms   similar   to   those   illustrated   in   the   or  discoveries.  
plaintiff’s   copyrighted   book.     The   US   Supreme   Court    
ruled  that:   The   copyright   of   a   book   on   perspective,   no  
  matter   how   many   drawings   and   illustrations   it  
“There   is   no   doubt   that   a   work   on   the   subject   may   contain,   gives   no   exclusive   right   to   the  
of   book-­‐keeping,   though   only   explanatory   of   modes  of  drawing  described,  though  they  may  
well   known   systems,   may   be   the   subject   of   a   never   have   been   known   or   used   before.     By  
copyright;  but,  then,  it  is  claimed  only  as  a  book.   publishing   the   book   without   getting   a   patent  
x   x   x.     But   there   is   a   clear   distinction   between   for  the  art,  the  latter  is  given  to  the  public.  
the   books,   as   such,   and   the   art,   which   it   is,    
intended   to   illustrate.     The   mere   statement   of   Now,   whilst   no   one   has   a   right   to   print   or  
the   proposition   is   so   evident   that   it   requires   publish   his   book,   or   any   material   part   thereof,  
hardly   any   argument   to   support   it.     The   same   as   a   book   intended   to   convey   instruction   in   the  
distinction   may   be   predicated   of   every   other   art,   any   person   may   practice   and   use   the   art  
art   as   well   as   that   of   bookkeeping.     A   treatise   itself   which   he   has   described   and   illustrated  

o 155
Katrina Michelle Mancao
 
therein.    The  use  of  the  art  is  a  totally  different   three  (3)  months  before  the  due  date.  The  obligation  to  
thing   from   a   publication   of   the   book   explaining   pay   the   annual   fees   shall   terminate   should   the  
it.     The   copyright   of   a   book   on   bookkeeping   application  be  withdrawn,  refused,  or  cancelled.  
cannot  secure  the  exclusive  right  to  make,  sell    
and   use   account   books   prepared   upon   the   plan   55.2.  If  the  annual  fee  is  not  paid,  the  patent  application  
set  forth  in  such  book.    Whether  the  art  might   shall  be  deemed  withdrawn  or  the  patent  considered  as  
or  might  not  have  been  patented,  is  a  question,   lapsed   from   the   day   following   the   expiration   of   the  
which   is   not   before   us.     It   was   not   patented,   period   within   which   the   annual   fees   were   due.   A   notice  
and   is   open   and   free   to   the   use   of   the   public.     that  the  application  is  deemed  withdrawn  or  the  lapse  of  
And,  of  course,  in  using  the  art,  the  ruled  lines   a   patent   for   non-­‐payment   of   any   annual   fee   shall   be  
and   headings   of   accounts   must   necessarily   be   published   in   the   IPO   Gazette   and   the   lapse   shall   be  
used  as  incident  to  it.   recorded  in  the  Register  of  the  Office.  
   
The  plausibility  of  the  claim  put  forward  by  the   55.3.   A   grace   period   of   six   (6)   months   shall   be   granted  
complainant   in   this   case   arises   from   a   for  the  payment  of  the  annual  fee,  upon  payment  of  the  
confusion   of   ideas   produced   by   the   peculiar   prescribed  surcharge  for  delayed  payment.  
nature  of  the  art  described  in  the  books,  which    
have   been   made   the   subject   of   copyright.     In  
M. Surrender,   Correction   and  
describing   the   art,   the   illustrations   and  
diagrams   employed   happened   to   correspond   Amendment  
more   closely   than   usual   with   the   actual   work    
performed   by   the   operator   who   uses   the   art.     SECTION  56.  SURRENDER  OF  PATENT.  –    
The   description   of   the   art   in   a   book,   though   56.1.   The   owner   of   the   patent,   with   the   consent   of   all  
entitled   to   the   benefit   of   copyright,   lays   no   persons   having   grants   or   licenses   or   other   right,   title   or  
foundation   for   an   exclusive   claim   to   the   art   interest   in   and   to   the   patent   and   the   invention   covered  
itself.     The   object   of   the   one   is   explanation;   the   thereby,   which   have   been   recorded   in   the   Office,   may  
object  of  the  other  is  use.    The  former  may  be   surrender  his  patent  or  any  claim  or  claims  forming  part  
secured   by   copyright.     The   latter   can   only   be   thereof  to  the  Office  for  cancellation.  
secured,   if   it   can   be   secured   at   all,   by   letters    
patent.”    
56.2.   A   person   may   give   notice   to   the   Office   of   his  
 
opposition   to   the   surrender   of   a   patent   under   this  
J. Publication  upon  Grant  of  Patents   section,   and   if   he   does   so,   the   Bureau   shall   notify   the  
  proprietor  of  the  patent  and  determine  the  question.  
SECTION  52.  PUBLICATION  UPON  GRANT  OF  PATENT.  –      
52.1.   The   grant   of   the   patent   together   with   other   related   56.3.   If   the   Office   is   satisfied   that   the   patent   may  
information  shall  be  published  in  the  IPO  Gazette  within   properly  be  surrendered,  he  may  accept  the  offer  and,  as  
the  time  prescribed  by  the  Regulations.   from  the  day  when  notice  of  his  acceptance  is  published  
  in   the   IPO   Gazette,   the   patent   shall   cease   to   have   effect,  
52.2.   Any   interested   party   may   inspect   the   complete   but   no   action   for   infringement   shall   lie   and   no   right  
description,   claims,   and   drawings   of   the   patent   on   file   compensation   shall   accrue   for   any   use   of   the   patented  
with  the  Office.     invention   before   that   day   for   the   services   of   the  
  government.    
K. Term  of  Patent    
  SECTION  57.  CORRECTION  OF  MISTAKES  OF  THE  OFFICE.  –    
SECTION  54.  TERM  OF  PATENT.  –     The  Director  shall  have  the  power  to  correct,  without  fee,  
any   mistake   in   a   patent   incurred   through   the   fault   of   the  
The   term   of   a   patent   shall   be   twenty   (20)   years   from   the  
Office   when   clearly   disclosed   in   the   records   thereof,   to  
filing  date  of  the  application.  
make  the  patent  conform  to  the  records.  
 
 
L. Annual  Fees  
SECTION  58.  CORRECTION  OF  MISTAKE  IN  THE   APPLICATION.  
 
–    
SECTION  55.  ANNUAL  FEES.  –    
On   request   of   any   interested   person   and   payment   of   the  
55.1.   To   maintain   the   patent   application   or   patent,   an  
prescribed  fee,  the  Director  is  authorized  to  correct  any  
annual   fee   shall   be   paid   upon   the   expiration   of   four   (4)   mistake   in   a   patent   of   a   formal   and   clerical   nature,   not  
years   from   the   date   the   application   was   published   incurred  through  the  fault  of  the  Office.  
pursuant  to  Section  44  hereof,  and  on  each  subsequent    
anniversary   of   such   date.   Payment   may   be   made   within  

156 z
Intellectual Property Law

SECTION  59.  CHANGES  IN  PATENTS.  –     complete  for  it  to  be  carried  out  by  any  person  
59.1.   The   owner   of   a   patent   shall   have   the   right   to   skilled  in  the  art;  or  
 
request   the   Bureau   to   make   the   changes   in   the   patent   in  
(c)   That   the   patent   is   contrary   to   public   order  
order  to:  
or  morality.  
 
 
(a)  Limit  the  extent  of  the  protection  conferred  
61.2.  Where  the  grounds  for  cancellation  relate  to  some  
by  it;  
  of   the   claims   or   parts   of   the   claim,   cancellation   may   be  
(b)   Correct   obvious   mistakes   or   to   correct   effected  to  such  extent  only.  
clerical  errors;  and    
  SECTION   82.   PATENT   FOUND   INVALID   MAY   BE   CANCELLED.  
(c)  Correct  mistakes  or  errors,  other  than  those   –    
referred   to   in   letter   (b),   made   in   good   faith:   In   an   action   for   infringement,   if   the   court   shall   find   the  
Provided,   That   where   the   change   would   result   patent  or  any  claim  to  be  invalid,  it  shall  cancel  the  same,  
in   a   broadening   of   the   extent   of   protection   and  the  Director  of  Legal  Affairs  upon  receipt  of  the  final  
conferred   by   the   patent,   no   request   may   be   judgment   of   cancellation   by   the   court,   shall   record   that  
made  after  the  expiration  of  two  (2)  years  from   fact  in  the  register  of  the  Office  and  shall  publish  a  notice  
the  grant  of  a  patent  and  the  change  shall  not   to  that  effect  in  the  IPO  Gazette.  
affect   the   rights   of   any   third   party   which   has    
relied  on  the  patent,  as  published.  
SECTION   109.   SPECIAL   PROVISIONS   RELATING   TO   UTILITY  
 
59.2.   No   change   in   the   patent   shall   be   permitted   under   MODELS.  -­‐    
this   section,   where   the   change   would   result   in   the   109.4.  In  proceedings  under  Sections  61  to  64,  the  utility  
disclosure   contained   in   the   patent   going   beyond   the   model   registration   shall   be   canceled   on   the   following  
disclosure  contained  in  the  application  filed.   grounds:  
   
59.3.   If,   and   to   the   extent   to   which   the   Office   changes   (a)  That  the  claimed  invention  does  not  qualify  
the   patent   according   to   this   section,   it   shall   publish   the   for  registration  as  a  utility  model  and  does  not  
same.     meet   the   requirements   of   registrability,   in  
  particular   having   regard   to   Subsection   109.1  
and  Sections  22,  23,  24  and  27;  
SECTION  60.  FORM  AND  PUBLICATION  OF  AMENDMENT.  –    
 
An   amendment   or   correction   of   a   patent   shall   be  
(b)   That   the   description   and   the   claims   do   not  
accomplished   by   a   certificate   of   such   amendment   or  
comply  with  the  prescribed  requirements;  
correction,   authenticated   by   the   seal   of   the   Office   and  
 
signed   by   the   Director,   which   certificate   shall   be  
(c)   That   any   drawing   which   is   necessary   for   the  
attached   to   the   patent.   Notice   of   such   amendment   or  
understanding   of   the   invention   has   not   been  
correction   shall   be   published   in   the   IPO   Gazette   and  
furnished;  
copies   of   the   patent   kept   or   furnished   by   the   Office   shall  
 
include   a   copy   of   the   certificate   of   amendment   or  
(d)   That   the   owner   of   the   utility   model  
correction.    
registration  is  not  the  inventor  or  his  successor  
 
in  title.  
N. Cancellation    
  SECTION  120.  CANCELLATION  OF  DESIGN  REGISTRATION.  –    
1. Grounds   120.1.  At  any  time  during  the  term  of  the  industrial  design  
  registration,   any   person   upon   payment   of   the   required  
SECTION  61.  CANCELLATION  OF  PATENTS.  –     fee,   may   petition   the   Director   of   Legal   Affairs   to   cancel  
61.1.   Any   interested   person   may,   upon   payment   of   the   the  industrial  design  on  any  of  the  following  grounds:  
required   fee,   petition   to   cancel   the   patent   or   any   claim    
thereof,   or   parts   of   the   claim,   on   any   of   the   following   (a)  If  the  subject  matter  of  the  industrial  design  
grounds:   is   not   registrable   within   the   terms   of   Sections  
  112  and  113;  
(a)  That  what  is  claimed  as  the  invention  is  not    
new  or  Patentable;   (b)  If  the  subject  matter  is  not  new;  or  
   
(b)   That   the   patent   does   not   disclose   the   (c)  If  the  subject  matter  of  the  industrial  design  
invention   in   a   manner   sufficiently   clear   and   extends  beyond  the  content  of  the  application  
as  originally  filed.  

o 157
Katrina Michelle Mancao
 
  65.3.   If   the   fee   for   the   printing   of   a   new   patent   is   not  
120.2.  Where  the  grounds  for  cancellation  relate  to  a  part   paid  in  due  time,  the  patent  should  be  revoked.  
of  the  industrial  design,  cancellation  may  be  effected  to    
such  extent  only.  The  restriction  may  be  effected  in  the   65.4.   If   the   patent   is   amended   under   Subsection   65.2  
form   of   an   alteration   of   the   effected   features   of   the   hereof,   the   Bureau   shall,   at   the   same   time   as   it   publishes  
design.   the   mention   of   the   cancellation   decision,   publish   the  
  abstract,   representative   claims   and   drawings   indicating  
2. Proceedings   clearly  what  the  amendments  consist  of.    
   
SECTION  62.  REQUIREMENT  OF  THE  PETITION.  –     SECTION   66.   EFFECT   OF   CANCELLATION   OF   PATENT   OR  
The   petition   for   cancellation   shall   be   in   writing,   verified   CLAIM.  –    
by   the   petitioner   or   by   any   person   in   his   behalf   who   The   rights   conferred   by   the   patent   or   any   specified   claim  
knows   the   facts,   specify   the   grounds   upon   which   it   is   or   claims   cancelled   shall   terminate.   Notice   of   the  
based,  include  a  statement  of  the  facts  to  be  relied  upon,   cancellation  shall  be  published  in  the  IPO  Gazette.  Unless  
and   filed   with   the   Office.   Copies   of   printed   publications   restrained  by  the  Director  General,  the  decision  or  order  
or   of   patents   of   other   countries,   and   other   supporting   to   cancel   by   Director   of   Legal   Affairs   shall   be  
documents   mentioned   in   the   petition   shall   be   attached   immediately  executory  even  pending  appeal.  
thereto,   together   with   the   translation   thereof   in   English,    
if  not  in  the  English  language.   SECTION   230.   EQUITABLE   PRINCIPLES   TO   GOVERN  
 
PROCEEDINGS.  –    
SECTION  63.  NOTICE  OF  HEARING.  –    
In  all  inter  partes  proceedings  in  the  Office  under  this  Act,  
Upon  filing  of  a  petition  for  cancellation,  the  Director  of  
the   equitable   principles   of   laches,   estoppel,   and  
Legal   Affairs   shall   forthwith   serve   notice   of   the   filing  
acquiescence   where   applicable,   may   be   considered   and  
thereof   upon   the   patentee   and   all   persons   having   grants  
applied.  
or   licenses,   or   any   other   right,   title  or   interest   in   and   to  
 
the   patent   and   the   invention   covered   thereby,   as  
appears  of  record  in  the  Office,  and  of  notice  of  the  date  
O. Rights  of  Patentee  
of   hearing   thereon   on   such   persons   and   the   petitioner.    
Notice   of   the   filing   of   the   petition   shall   be   published   in   1. Section  71  
the  IPO  Gazette.    
  SECTION  71.  RIGHTS  CONFERRED  BY  PATENT.  –    
SECTION  64.  COMMITTEE  OF  THREE.  –     71.1.   A   patent   shall   confer   on   its   owner   the   following  
In   cases   involving   highly   technical   issues,   on   motion   of   exclusive  rights:  
any   party,   the   Director   of   Legal   Affairs   may   order   that    
the   petition   be   heard   and   decided   by   a   committee   (a)   Where   the   subject   matter   of   a   patent   is   a  
composed   of   the   Director   of   Legal   Affairs   as   chairman   product,   to   restrain,   prohibit   and   prevent   any  
and   two   (2)   members   who   have   the   experience   or   unauthorized   person   or   entity   from   making,  
expertise  in  the  field  of  technology  to  which  the  patent   using,   offering   for   sale,   selling   or   importing  
sought   to   be   cancelled   relates.   The   decision   of   the   that  product;  
committee  shall  be  appealable  to  the  Director  General.      
  (b)   Where   the   subject   matter   of   a   patent   is   a  
SECTION  65.  CANCELLATION  OF  THE  PATENT.  –     process,   to   restrain,   prevent   or   prohibit   any  
65.1.   If   the   Committee   finds   that   a   case   for   cancellation   unauthorized   person   or   entity   from   using   the  
has   been   proved,   it   shall   order   the   patent   or   any   process,   and   from   manufacturing,   dealing   in,  
specified  claim  or  claims  thereof  cancelled.   using,   selling   or   offering   for   sale,   or   importing  
  any  product  obtained  directly  or  indirectly  from  
65.2.   If   the   Committee   finds   that,   taking   into   such  process.  
 
consideration   the   amendment   made   by   the   patentee  
during  the  cancellation  proceedings,  the  patent  and  the   71.2.   Patent   owners   shall   also   have   the   right   to   assign,   or  
invention   to   which   it   relates   meet   the   requirement   of   transfer   by   succession   the   patent,   and   to   conclude  
this   Act,   it   may   decide   to   maintain   the   patent   as   licensing  contracts  for  the  same.  
amended:   Provided,   That   the   fee   for   printing   of   a   new    
patent   is   paid   within   the   time   limit   prescribed   in   the  
Regulations.  
 

158 z
Intellectual Property Law

PARKE   DAVIS   &   CO.   V.   DOCTOR ’S   PHARMACEUTICALS   the   original   patent   holder   from   unfair   commercial   use  
provided   in   Article   39.3   of   the   Agreement   on   Trade-­‐
(1983)  
Related   Aspects   of   Intellectual   Property   Rights   (TRIPS  
Court  decision:  
Agreement),   the   Intellectual   Property   Office,   in  
An   8%   royalty   rate   on   net   sales   is   reasonable   in   a  
consultation  with  the  appropriate  government  agencies,  
developing   country   like   the   PH,   vis-­‐à-­‐vis   a   giant   foreign  
shall   issue   the   appropriate   rules   and   regulations  
licensors  of  pharmaceutical  products.  It  is  in  fact  midway  
necessary   therein   not   later   than   one   hundred   twenty  
between   the   rates   in   Canada   and   Norway.   Liberal   (120)  days  after  the  enactment  of  this  law;  
treatment   should   be   afforded   to   local   industry   for   as  
 
reasoned   out   by   the   company,   “it   is   so   difficult   to  
72.5.   Where   the   act   consists   of   the   preparation   for  
compete  with  the  industrial  giants  of  the  drug  industry,  
individual   cases,   in   a   pharmacy   or   by   a   medical  
among  them  being  the  petitioner  herein,  that  it  always  is  
professional,  of  a  medicine  in  accordance  with  a  medical  
necessary   that   the   local   drug   companies   should   sell   at  
presciption  or  acts  concerning  the  medicine  so  prepared;  
much  lower  than  the  prices  of  said  foreign  drug  entities.”  
and  
 
 
Foreign  product  licensor  can  later  on  ask  for  an  increase  
in   percentage   rate   of   royalty   fixed   by   the   Director   of   72.6.   Where   the   invention   is   used   in   any   ship,   vessel,  
Patents  if  local  sales  of  licensee  should  increase.   aircraft,  or  land  vehicle  of  any  other  country  entering  the  
  territory   of   the   Philippines   temporarily   or   accidentally":  
SECTION  72.  LIMITATIONS  OF  PATENT  RIGHTS.  –     Provided,   That   such   invention   is     used   exclusively   for   the  
needs  of  the  ship,  vessel,  aircraft,  or  land  vehicle  and  not  
The   owner   of   a   patent   has   no   right   to   prevent   third  
used  for  the  manufacturing  of  anything  to  be  sold  within  
parties   from   performing,   without   his   authorization,   the  
the  Philippines.  
acts   referred   to   in   Section   71   hereof   in   the   following  
 
circumstances:  
 
SECTION  73.  PRIOR  USER.  –    
72.1.   Using   a   patented   product   which   has   been   put   on   73.1.   Notwithstanding   Section   72   hereof,   any   prior   user,  
the   market   in   the   Philippines   by   the   owner   of   the   who,   in   good   faith   was   using   the   invention   or   has  
product,   or   with   his   express   consent,   insofar   as   such   use   undertaken   serious   preparations   to   use   the   invention   in  
is   performed   after   that   product   has   been   so   put   on   the   his   enterprise   or   business,   before   the   filing   date   or  
said   market:   Provided,   That,   with   regard   to   drugs   and   priority   date   of   the   application   on   which   a   patent   is  
medicines,   the   limitation   on   patent   rights   shall   apply   granted,  shall  have  the  right  to  continue  the  use  thereof  
after   a   drug   or   medicine   has   been   introduced   in   the   as   envisaged   in   such   preparations   within   the   territory  
Philippines   or   anywhere   else   in   the   world   by   the   patent   where  the  patent  produces  its  effect.  
owner,  or  by  any  party  authorized  to  use  the  invention:    
Provided,   further,   That   the   right   to   import   the   drugs   and   73.2.   The  right  of  the  prior  user  may  only  be  transferred  
medicines  contemplated  in  this  section  shall  be  available   or   assigned   together   with   his   enterprise   or   business,   or  
to  any  government  agency  or  any  private  third  party;   with  that  part  of  his  enterprise  or  business  in  which  the  
  use  or  preparations  for  use  have  been  made.  
72.2.   Where   the   act   is   done   privately   and   on   a   non-­‐  
commercial   scale   or   for   a   non-­‐commercial   purpose:   SECTION  74.  USE  OF  INVENTION  BY  GOVERNMENT.  –    
Provided,   That   it   does   not   significantly   prejudice   the   74.1.  A  Government  agency  or  third  person  authorized  by  
economic  interests  of  the  owner  of  the  patent;   the  Government  may  exploit  the  invention  even  without  
  agreement  of  the  patent  owner  where:  
72.3.   Where   the   act   consists   of   making   or   using    
exclusively   for   experimental   use   of   the   invention   for   (a)   The   public   interest,   in   particular,   national  
scientific   purposes   or   educational   purposes   and   such   security,   nutrition,   health   or   the   development  
other   activities   directly   related   to   such   scientific   or   of   other   sectors,   as   determined   by   the  
educational  experimental  use;   appropriate   agency   of   the   government,   so  
  requires;  or  
72.4.   In   the   case   of   drugs   and   medicines,   where   the   act    
includes   testing,   using,   making   or   selling   the   invention   (b)   A   judicial   or   administrative   body   has  
including   any   data   related   thereto,   solely   for   purposes   determined  that  the  manner  of  exploitation,  by  
reasonably   related   to   the   development   and   submission   the  owner  of  the  patent  or  his  licensee,  is  anti-­‐
of  information  and  issuance  of  approvals  by  government   competitive;  or  
regulatory   agencies   required   under   any   law   of   the    
Philippines   or   of   another   country   that   regulates   the   (c)  In  the  case  of  drugs  and  medicines,  there  is  
manufacture,   construction,   use   or   sale   of   any   product:   a  national  emergenvy  or  other  circumstance  of  
Provided,  That,  in  order  to  protect  the  data  submitted  by  

o 159
Katrina Michelle Mancao
 
extreme   urgency   requiring   the   use   of   the   exploitation,   which   shall   be   immediately  
invention;  or   executory.  
   
(d)  In  the  case  of  drugs  and  medicines,  there  is   74.3.   All   cases   arising   from   the   implementation   of   this  
public  non-­‐commercial  use  of  the  patent  by  the   provision  shall  be  cognizable  by  courts  with  appropriate  
patentee,  without  satisfactory  reason;  or   jurisdiction  provided  by  law.  
   
(e)   In   the   case   of   drugs   and   medicines,   the   No   court,   except   the   Supreme   Court   of   the   Philippines,  
demand   for   the   patented   article   in   the   shall   issue   any   temporary   restraining   order   or  
Philippines   is   not   being   met   to   an   adequate   preliminary  injunction  or  such  other  provisional  remedies  
extent  and  on  reasonable  terms,  as  determined   that  will  prevent  its  immediate  execution.  
by  the  Secretary  of  the  Department  of  Health.    
  74.4.   The   Intellectual   Property   Office   (IPO),   in  
74.2.   Unless   otherwise   provided   herein,   the   use   by   the   consultation  with  the  appropriate  government  agencies,  
Government,   or   third   person   authorized   by   the   shall   issue   the   appropriate   implementing   rules   and  
Government   shall   be   subject,   where   applicable,   to   the   regulations   for   the   use   or   exploitation   of   patented  
following  provisions:   inventions   as   contemplated   in   this   section   within   one  
  hundred   twenty   (120)   days   after   the   effectivity   of   this  
(a)  In  situations  of  national  emergency  or  other   law.  
circumstances  of  extreme  urgency  as  provided    
under  Section  74.1  (c),  the  right  holder  shall  be   SECTION  231.  REVERSE  RECIPROCITY  OF  FOREIGN  LAWS.  –    
notified  as  soon  as  reasonably  practicable;  
Any   condition,   restriction,   limitation,   diminution,  
 
requirement,   penalty   or   any   similar   burden   imposed   by  
(b)   In   the   case   of   public   non-­‐commercial   use   of  
the   law   of   a   foreign   country   on   a   Philippine   national  
the   patent   by   the   patentee,   without  
seeking  protection  of  intellectual  property  rights  in  that  
satisfactory   reason,   as   provided   under   Section  
country,  shall  reciprocally  be  enforceable  upon  nationals  
74.1   (d),   the   right   holder   shall   be   informed  
of  said  country,  within  Philippine  jurisdiction.  
promptly:   Provided,   That,   the   Government   or  
 
third   person   authorized   by   the   Government,  
without  making  a  patent  search,  knows  or  has  
TRIPS   AGREEMENT   (AGREEMENT   ON   TRADE-­‐RELATED  
demonstrable   ground   to   know   that   a   valid   ASPECTS  OF   INTELLECTUAL   PROPERTY   RIGHTS),   ARTICLE   6.  
patent   is   or   will   be   used   by   or   for   the   EXHAUSTION  –    
Government;   For   the   purposes   of   dispute   settlement   under   this  
  Agreement,  subject  to  the  provisions  of  Articles  3  and  4  
(c)  If  the  demand  for  the  patented  article  in  the   nothing   in   this   Agreement   shall   be   used   to   address   the  
Philippines   is   not   being   met   to   an   adequate   issue  of  the  exhaustion  of  intellectual  property  rights.  
extent   ad   on   reasonable   terms   as   provided    
under  Section  74.1  (e),  the  right  holder  shall  be  
2. Action   for   infringement  
informed  promptly;  
  (Literal  and  Equivalent)  
(d)   The   scope   and   duration   of   such   use   shall   be    
limited   to   the   purpose   for   which   it   was   VARGAS  V.  F.M.  YAPTICO  (1919),  SUPRA  
authorized;   Court  decision:  
  When   a   patent   is   sought   to   be   enforced,   "the   question  
(e)  Such  use  shall  be  non-­‐exclusive;   of   invention,   novelty,   or   prior   use,   and   each   of   them,   are  
  open   to   judicial   examination."   The   burden   of   proof   to  
(f)   The   right   holder   shall   be   paid   adequate   substantiate  a  charge  of  infringement  is  with  the  plaintiff.  
remuneration   in   the   circumstances   of   each   Where,   however,   the   plaintiff   introduces   the   patent   in  
case,  taking  into  account  the  economic  value  of   evidence,   if   it   is   in   due   form,   it   affords   a   prima   facie  
the  authorization;  and   presumption  of  its  correctness  and  validity.  The  decision  
  of  the  Commissioner  of  Patents  in  granting  the  patent  is  
(g)   The   existence   of   a   national   emergency   or   always   presumed   to   be   correct.   The   burden   then   shifts  
other   circumstances   of   extreme   urgency,   to   the   defendant   to   overcome   by   competent   evidence  
referred   to   under   Section   74.1   (c),   shall   be   this   legal   presumption.   With   all   due   respect,   therefore,  
subject   to   the   determination   of   the   President   for   the   critical   and   expert   examination   of   the   invention  
of   the   Philippines   for   the   purpose   of   by   the   United   States   Patent   Office,   the   question   of   the  
determining   the   need   for   such   use   or   other   validity   of   the   patent   is   one   for   judicial   determination,  
and   since   a   patent   has   been   submitted,   the   exact  

160 z
Intellectual Property Law

question   is   whether   the   defendant   has   assumed   the   FRANK  V.  BENITO  (1928)  
burden  of  proof  as  to  anyone  of  his  defenses.   Court  decision:  
  The   burden   of   proof   to   substantiate   a   charge   of  
As   herein   before   stated,   the   defendant   relies   on   three   infringement   is   with   the   plaintiff.   Where,   however,   the  
special   defenses.   One   such   defense,   on   which   the   plaintiff  introduces  the  patent  in  evidence,  if  it  is  in  due  
judgment   of   the   lower   court   is   principally   grounded,   and   form,   it   affords   a   prima   facie   presumption   of   its  
to   which   appellant   devotes   the   major   portion   of   his   correctness   and   validity.   The   decision   of   the  
vigorous   argument,   concerns   the   element   of   novelty,   Commissioner  of  Patents  in  granting  the  patent  is  always  
invention,   or   discovery,   that   gives   existence   to   the   right   presumed   to   be   correct.   The   burden   the   shifts   to   the  
to   a   patent.   On   this   point   the   trial   court   reached   the   defendant   to   overcome   by   competent   evidence   this  
conclusion   that   "the   patented   plow   of   the   plaintiff,   legal  presumption.  
Exhibit  D,  is  not  different  from  the  native  plow,  Exhibit  2,    
except  in  the  material,  in  the  form,  in  the  weight  and  the  
FRANK  V.  KOSUYAMA  (1933),  SUPRA  
grade  of  the  result,  the  said  differences  giving  it  neither  
Refresher:  
a   new   function   nor   a   new   result   distinct   from   the  
Defendant  was  charged  with  infringing  upon  the  patent  
function   and   the   result   obtained   from   the   native   plow;  
rights   of   the   plaintiff   by   manufacturing   and   selling  
consequently,   its   production   does   not   presuppose   the  
machines   similar   to   the   ones   covered   by   plaintiff’s  
exercise   of   the   inventive   faculty   but   merely   of  
patent.  Plaintiff,  in  his  complaint,  described  the  machine  
mechanical  skill,  which  does  not  give  a  right  to  a  patent  
as  having  a  “spindle.”  When  he  amended  his  complaint,  
of   an   invention   under   the   provisions   of   the   Patent   Law."  
he  omitted  such  feature.  
In  thus  finding,  the  court  may  have  been  right,  since  the  
 
Vargas  plow  does  not  appear  to  be  such  a  "combination"  
Court  decision:  
as   contains   a   novel   assemblage   of   parts   exhibiting  
Court  found  that  the  plaintiff’s  invention  shouldn’t  have  
invention.    
been   protected   by   a   patent   for   its   failure   to   satisfy   the  
 
novelty   condition.   However,   since   the   action   was  
A   second   line   of   defense   relates   to   the   fact   that  
initiated   by   the   plaintiff   (for   infringement,   against   the  
defendant   has   never   made   a   complete   Vargas   plow,   but  
defendant),  and  the  defendant  did  not  file  a  cross-­‐claim  
only   points,   shares,   shoes,   and   heel   pieces,   to   serve   as  
against  the  plaintiff,  the  Court  did  not  deem  it  proper  to  
repairs.  Defendant's  contention  is,  that  in  common  with  
cancel  the  patent  of  the  plaintiff.  However,  it  ruled  that  
other   foundries,   he   has   for   years   cast   large   numbers   of  
the  defendant  is  not  liable  for  infringement.  
plow   points   and   shares   suitable   for   use   either   on   the  
 
native  wooden  plow,  or  on  the  Vargas  plow.  A  difference  
has   long   been   recognized   between   repairing   and   G.  SELL  V.  YAP    JUE  (1909)  
reconstructing  a  machine.  If,  for  instance,  partial  injuries,   Refresher:  
whether  they  occur  from  accident  or  from  wear  and  tear,   Petitioner   Gsell,   had   previously   secured   a   decree   of  
to   a   machine   for   agricultural   purposes,   are   made   this   is   permanent   injunction   against   the   respondent,   Yap-­‐Jue  
only  re-­‐fitting  the  machine  for  use,  and  thus  permissible.   prohibiting   the   latter   from   infringing   upon   Gsell's  
Even   under   the   more   rigorous   doctrine   of   Leeds   &   Catlin   patented   process   for   manufacturing   curved   cane  
Co.   vs.   Victor   Talking   Machine   Co.,   it   may   be   possible   handles  for  walking  sticks,  parasols,  and  umbrellas.  
that   all   the   defendant   has   done   is   to   manufacture   and    
sell  isolated  parts  to  be  used  to  replace  worn-­‐out  parts.   The  injunction  described  the  process  as  involving  the  use  
  of  a  lamp  or  blow-­‐  pipe  fed  with  petroleum  or  mineral  oil.  
The   third   defense   is,   that   under   the   provisions   of   the   The  heat  generated  thereby  was  necessary  to  make  the  
statute,   an   inventor's   creation   must   not   have   been   in   canes   pliable   enough   that   they   could   be   worked   into   the  
public   use   or   on   sale   in   the   United   States   (and   the   desired  curvature.  
Philippine   Islands)   for   more   than   two   years   prior   to   his    
application.  Without,  therefore,  committing  ourselves  as   After  the  injunction  issued,  however,  Yap-­‐Jue  continued  
to   the   first   two   defenses,   we   propose   to   base   our   to   use   substantially   the   same   procedure   patented   by  
decision   on   the   one   just   suggested   as   more   easily   Gsell,  with  the  exception  of  the  use  of  a  lamp  fueled  by  
disposing   of   the   case.   We   do   so   with   full   consciousness   alcohol  instead  of  petroleum  or  mineral  oil.  
of   the   doubt   which   arose   in   the   mind   of   the   trial   court,    
but  with  the  belief  that  since  it  has  been  shown  that  the   Court  held:  
invention   was   used   in   public   at   Iloilo   by   others   than   No   one   infringes   a   patent   for   a   combination   who   does  
Vargas,   the   inventor,   more   than   two   years   before   the   not  employ  all  of  the  ingredients  of  the  combination;  but  
application  for  the  patent,  the  patent  is  invalid.   if   he   employs   all   the   ingredients,   or   adopts   mere   formal  
  alterations,   or   substitutes,   for   one   ingredient   another  
which   was   well   known   at   the   date   of   the   patent   as   a  
proper   substitute   for   the   one   withdrawn,   and   which  

o 161
Katrina Michelle Mancao
 
performs   substantially   the   same   function   as   the   one   known   as   such   at   the   date   of   his   invention,   and   which  
withdrawn,  he  does  infringe.     any   constructor   acquainted   with   the   art   will   know   how  
  to  comply.  The  reason  for  the  qualification  of  the  rule  as  
Bona   fide   inventors   of   a   combination   are   as   much   stated  is,  that  such  change  that  is,  the  mere  substitution  
entitled   to   equivalents   as   the   inventors   other   patentable   of   a   well-­‐   known   element   for   another   where   it   appears  
improvements;   by   which   is   meant   that   a   patentee   in   that  the  substituted  element  was  well  known  as  a  usual  
such   a   case   may   substitute   another   ingredient   for   any   substitute  for  the  element  left  out  is  merely  a  formal  one,  
one  of  the  ingredients  of  his  invention,  if  the  ingredient   and   nothing   better   than   a   colorable   evasion   of   the  
substituted   performs   the   same   function   as   the   one   patent.  
omitted   and   as   well   known   at   the   date   of   his   patent   as   a    
proper   substitute   for   the   one   omitted   in   the   patented   MAGUAN  V.  CA,  SUPRA  
combination.   Apply   that   rule   and   it   is   clear   that   an   Court  decision:  
alteration   in   a   patented   combination   which   merely   It   will   be   noted   that   the   validity   of   petitioner's   patents   is  
substitutes   another   old   ingredient   for   one   of   the   in   question   for   want   of   novelty.   Private   respondent  
ingredients   in   the   patented   combination,   is   an   contends  that  powder  puffs  Identical  in  appearance  with  
infringement  of  the  patent,  if  the  substitute  performs  the   that   covered   by   petitioner's   patents   existed   and   were  
same   function   and   was   well   known   at   the   date   of   the   publicly   known   and   used   as   early   as   1963   long   before  
patent  as  a  proper  substitute  for  the  omitted  ingredient.   petitioner   was   issued   the   patents   in   question.   As  
  correctly   observed   by   respondent   Court   of   Appeals,  
Mere   formal   alterations   in   a   combination   in   letters   "since   sufficient   proofs   have   been   introduced   in  
patent  are  no  defense  to  the  charge  of  infringement  and   evidence  showing  a  fair  question  of  the  invalidity  of  the  
the  withdrawal  of  one  ingredient  from  the  same  and  the   patents   issued   for   such   models,   it   is   but   right   that   the  
substitution   of   another   which   was   well   known   at   the   evidence   be   looked   into,   evaluated   and   determined   on  
date   of   the   patent   as   a   proper   substitute   for   the   one   the   merits   so   that   the   matter   of   whether   the   patents  
withdrawn   is   a   mere   formal   alteration   of   the   issued  were  in  fact  valid  or  not  may  be  resolved."  
combination   if   the   ingredient   substituted   performs    
substantially  the  same  function  as  the  one  withdrawn.  
GODINES  V.  CA  (1993)  
 
Doctrine:  
Bona   fide   inventors   of   a   combination   are   as   much  
Tests  have  been  established  to  determine  infringement.  
entitled   to   suppress   every   other   combination   of   the  
These  are:  
same   ingredients   to   produce   the   same   result,   not  
(a)  literal  infringement;  and    
substantially  different  from  what  they  have  invented  and  
(b)  the  doctrine  of  equivalents.    
caused  to  be  patented  as  to  any  other  class  of  inventors.  
 
All   alike   have   the   right   to   suppress   every   colorable  
invasion   of   that   which   is   secured   to   them   by   letters   In   using   LITERAL   INFRINGEMENT   as   a   test,   resort   must  
patent.   be  had,  in  the  first  instance,  to  the  words  of  the  claim.  If  
accused   matter   clearly   falls   within   the   claim,  
 
A  claim  for  the  particular  means  and  mode  of  operation   infringement   is   made   out   and   that   is   the   end   of   it.   To  
described   in   the   specification   extends,   by   operation   of   determine   whether   the   particular   item   falls   within   the  
law,   to   the   equivalent   of   such   means   not   equivalent   literal   meaning   of   the   patent   claims,   the   court   must  
juxtapose   the   claims   of   the   patent   and   the   accused  
simply  because  the  same  result  is  thereby  produced  but  
product   within   the   overall   context   of   the   claims   and  
equivalent   as   being   substantially   the   same   device   in  
structure,  arrangement  and  mode  of  operation.   specifications,   to   determine   whether   there   is   exact  
  identity  of  all  material  elements.  
An   equivalent   device   is   such   as   a   mechanic   of   ordinary    
skill   in   construction   of   similar   machinery,   having   the   Recognizing   that   the   logical   fallback   position   of   one   in  
the   place   of   defendant   is   to   aver   that   his   product   is  
forms,   specifications   and   machine   before   him,   could  
substitute   in   the   place   of   the   mechanism   described   different   from   the   patented   one,   courts   have   adopted  
without  the  exercise  of  the  inventive  faculty.     the   DOCTRINE   OF   EQUIVALENTS   which   recognizes   that  
  minor   modifications   in   a   patented   invention   are  
All  the  elements  of  the  invention  in  this  case  are  old,  and   sufficient   to   put   the   item   beyond   the   scope   of   literal  
the  rule  in  such  cases,  as  before  explained,  undoubtedly   infringement.   Thus,   according   to   this   doctrine,   "an  
is   that   a   purpose   can   not   invoke   the   doctrine   of   infringement   also   occurs   when   a   device   appropriates   a  
equivalents   to   suppress   all   other   improvements   of   the   prior   invention   by   incorporating   its   innovative   concept  
old   machine,   but   he   is   entitled   to   treat   everyone   as   an   and,   albeit   with   some   modification   and   change,   performs  
infringer   who   makes,   uses,   or   vends   his   patented   substantially  the  same  function  in  substantially  the  same  
improvement   without   any   other   change   than   the   way  to  achieve  substantially  the  same  result."  The  reason  
employment  of  a  substitute  for  one  of  its  elements,  well   for   the   doctrine   of   equivalents   is   that   to   permit   the  
imitation   of   a   patented   invention   which   does   not   copy  

162 z
Intellectual Property Law

any   literal   detail   would   be   to   convert   the   protection   of   Doctrine:  


the   patent   grant   into   a   hollow   and   useless   thing.   Such   The  burden  of  proof  to  substantiate  a  charge  for  patent  
imitation  would  leave  room  for  —  indeed  encourage  —   infringement   rests   on   the   plaintiff.   In   the   case   at   bar,  
the   unscrupulous   copyist   to   make   unimportant   and   petitioner’s   evidence   consists   primarily   of   its   Letters  
insubstantial   changes   and   substitutions   in   the   patent   Patent   No.   14561,   and   the   testimony   of   Dr.   Orinion,   its  
which,   though   adding   nothing,   would   be   enough   to   take   general  manager  in  the  Philippines  for  its  Animal  Health  
the   copied   matter   outside   the   claim,   and   hence   outside   Products   Division,   by   which   it   sought   to   show   that   its  
the  reach  of  the  law.     patent   for   the   compound   methyl   5   propylthio-­‐2-­‐
  benzimidazole   carbamate   also   covers   the   substance  
DEL  ROSARIO  V.  CA  (1996)   Albendazole.  
Doctrine:    
It   is   elementary   that   a   patent   may   be   infringed   where   From   a   reading   of   the   9   claims   of   Letters   Patent   No.  
the   essential   or   substantial   features   of   the   patented   14561   in   relation   to   the   other   portions   thereof,   no  
invention   are   taken   or   appropriated,   or   the   device,   mention  is  made  of  the  compound  Albendazole.     All  that  
machine   or   other   subject   matter   alleged   to   infringe   is   the   claims   disclose   are:     the   covered   invention,   that   is,  
substantially   identical   with   the   patented   invention.     In   the   compound   methyl   5   propylthio-­‐2-­‐benzimidazole  
order   to   infringe   a   patent,   a   machine   or   device   must   carbamate;   the   compound’s   being   anthelmintic   but  
perform   the   same   function,   or   accomplish   the   same   nontoxic   for   animals   or   its   ability   to   destroy   parasites  
result   by   identical   or   substantially   identical   means   and   without   harming   the   host   animals;   and   the   patented  
the   principle   or   mode   of   operation   must   be   substantially   methods,   compositions   or   preparations   involving   the  
the  same.   compound  to  maximize  its  efficacy  against  certain  kinds  
  of  parasites  infecting  specified  animals.  
 
SMITH  KLINE  V.  CA  (2003)  
When  the  language  of  its  claims  is  clear  and  distinct,  the  
Refresher:  
patentee  is  bound  thereby  and  may  not  claim  anything  
Smith   Kline   Beckman   Corp.,   a   Pennsylvanian   corp.,   filed  
beyond   them.   And   so   are   the   courts   bound   which   may  
before   the   Philippine   Patent   Office   an   application   for  
not   add   to   or   detract   from   the   claims   matters   not  
patent   over   an   invention   entitled   “Methods   and  
expressed   or   necessarily   implied,   nor   may   they   enlarge  
Compositions   for   Producing   Biphasic   Parasiticide   Activity  
the  patent  beyond  the  scope  of  that  which  the  inventor  
Using   Methyl   5   Propylthio-­‐2-­‐Benzimidazole   Carbamate.”  
claimed   and   the   patent   office   allowed,   even   if   the  
Letters   Patent   was   issued   for   the   same   for   a   term   of   17  
patentee   may   have   been   entitled   to   something   more  
years.  
than  the  words  it  had  chosen  would  include.  
 
 
The   letters   patent   provides   in   its   claims   that   the  
It   bears   stressing   that   the   mere   absence   of   the   word  
patented   invention   consisted   of   a   new   compound  
Albendazole   in   Letters   Patent   No.   14561   is   not  
named   methyl   5   propylthio-­‐2-­‐benzimidazole   carbamate  
determinative   of   Albendazole’s   non-­‐inclusion   in   the  
and   the   methods   or   compositions   utilizing   the  
claims  of  the  patent.     While  Albendazole  is  admittedly  a  
compound   as   an   active   ingredient   in   fighting   infections  
chemical  compound  that  exists  by  a  name  different  from  
caused   by   gastrointestinal   parasites   and   lungworms   in  
that  covered  in  petitioner’s  letters  patent,  the  language  
animals   such   as   swine,   sheep,   cattle,   goats,   horses,   and  
of   Letter   Patent   No.   14561   fails   to   yield   anything   at   all  
even  pet  animals.  
regarding   Albendazole.     And   no   extrinsic   evidence   had  
 
been   adduced   to   prove   that   Albendazole   inheres   in  
Tryco   Pharma   Corporation   (private   respondent)   is   a  
petitioner’s   patent   in   spite   of   its   omission   therefrom   or  
domestic  corporation  that  manufactures,  distributes  and  
that   the   meaning   of   the   claims   of   the   patent   embraces  
sells  veterinary  products  including  Impregon,  a  drug  that  
the  same.  
has   Albendazole   for   its   active   ingredient   and   is   claimed  
While   petitioner   concedes   that   the   mere   literal   wordings  
to   be   effective   against   gastro-­‐intestinal   roundworms,  
of   its   patent   cannot   establish   private   respondent’s  
lungworms,  tapeworms  and  fluke  infestation  in  carabaos,  
infringement,  it  urges  this  Court  to  apply  the  doctrine  of  
cattle  and  goats.  
equivalents.  
 
 
Smith   Kline   sued   Tryco   Pharma   for   infringement.  
The   DOCTRINE   OF   EQUIVALENTS   provides   that   an  
Petitioner  argues  that  under  the  doctrine  of  equivalents  
infringement  also  takes  place  when  a  device  appropriates  
for   determining   patent   infringement,   Albendazole,   the  
a  prior  invention  by  incorporating  its  innovative  concept  
active   ingredient   it   alleges   was   appropriated   by   private  
and,   although   with   some   modification   and   change,  
respondent   for   its   drug   Impregon,   is   substantially   the  
performs  substantially  the  same  function  in  substantially  
same  as  methyl  5  propylthio-­‐2-­‐benzimidazole  carbamate  
the   same   way   to   achieve   substantially   the   same   result.  
covered   by   its   patent   since   both   of   them   are   meant   to  
Yet   again,   a   scrutiny   of   petitioner’s   evidence   fails   to  
combat  worm  or  parasite  infestation  in  animals.  
convince   this   Court   of   the   substantial   sameness   of  
 

o 163
Katrina Michelle Mancao
 
petitioner’s   patented   compound   and   patented  product  or  of  a  product  produced  because  of  a  
Albendazole.     While  both  compounds  have  the  effect  of   patented  process  knowing  it  to  be  especially  adopted  for  
neutralizing   parasites   in   animals,   identity   of   result   does   infringing   the   patented   invention   and   not   suitable   for  
not  amount  to  infringement  of  patent  unless  Albendazole   substantial   non-­‐infringing   use   shall   be   liable   as   a  
operates   in   substantially   the   same   way   or   by   contributory   infringer   and   shall   be   jointly   and   severally  
substantially  the  same  means  as  the  patented  compound,   liable  with  the  infringer.  
even  though  it  performs  the  same  function  and  achieves    
the  same  result.  In  other  words,  the  principle  or  mode  of   SECTION   75.   EXTENT  OF  PROTECTION  AND   INTERPRETATION  
operation  must  be  the  same  or  substantially  the  same.   OF  CLAIMS.  –    
 
75.1.   The   extent   of   protection   conferred   by   the   patent  
The   doctrine   of   equivalents   thus   requires   satisfaction   of  
the  function-­‐means-­‐and-­‐result  test,  the  patentee  having   shall   be   determined   by   the   claims,   which   are   to   be  
the   burden   to   show   that   all   three   components   of   such   interpreted  in  the  light  of  the  description  and  drawings.  
equivalency  test  are  met.    
  75.2.   For   the   purpose   of   determining   the   extent   of  
a. Civil  action   protection  conferred  by  the  patent,  due  account  shall  be  
  taken  of  elements  which  are  equivalent  to  the  elements  
expressed   in   the   claims,   so   that   a   claim   shall   be  
SECTION  76.  CIVIL  ACTION  FOR  INFRINGEMENT.  –    
considered   to   cover   not   only   all   the   elements   as  
76.1.   The   making,   using,   offering   for   sale,   selling,   or   expressed  therein,  but  also  equivalents.  
importing   a   patented   product   or   a   product   obtained    
directly   or   indirectly   from   a   patented   process,   or   the   use   b. Criminal  action    
of   a   patented   process   without   the   authorization   of   the  
 
patentee   constitutes   patent   infringement:   Provided,  
That,   this   shall   not   apply   to   instances   covered   by  
SECTION   84.   CRIMINAL   ACTION   FOR   REPETITION   OF  
Sections   72.1   and   72.4   (Limitations   of   Patent   Rights);   INFRINGEMENT.  –    
Section   74   (Use   of   Invention   by   Government);   Section   If  infringement  is  repeated  by  the  infringer  or  by  anyone  
93.6   (Compulsory   Licensing);   and   Section   93-­‐A   in   connivance   with   him   after   finality   of   the   judgment   of  
(Procedures  on  Issuance  of  a  Special  Compulsory  License   the   court   against   the   infringer,   the   offenders   shall,  
under  the  TRIPS  Agreement)  of  this  Code.   without   prejudice   to   the   institution   of   a   civil   action   for  
  damages,   be   criminally   liable   therefor   and,   upon  
76.2.  Any  patentee,  or  anyone  possessing  any  right,  title   conviction,   shall   suffer   imprisonment   for   the   period   of  
or   interest   in   and   to   the   patented   invention,   whose   not  less  than  six  (6)  months  but  not  more  than  three  (3)  
rights   have   been   infringed,   may   bring   a   civil   action   years   and/or   a   fine   of   not   less   than   One   hundred  
before  a  court  of  competent  jurisdiction,  to  recover  from   thousand   pesos   (P100,000)   but   not   more   than   Three  
the   infringer   such   damages   sustained   thereby,   plus   hundred  thousand  pesos  (P300,000),  at  the  discretion  of  
attorney's   fees   and   other   expenses   of   litigation,   and   to   the   court.   The   criminal   action   herein   provided   shall  
secure  an  injunction  for  the  protection  of  his  rights.   prescribe   in   three   (3)   years   from   date   of   the   commission  
  of  the  crime.  
76.3.  If  the  damages  are  inadequate  or  cannot  be  readily    
ascertained   with   reasonable   certainty,   the   court   may   c. Administrative  
award   by   way   of   damages   a   sum   equivalent   to    
reasonable  royalty.   SECTION  10.2,  SUPRA  
  -­‐ Jurisdiction  of  the  Director  of  Legal  Affairs  
76.4.   The   court   may,   according   to   the   circumstances   of    
the   case,   award   damages   in   a   sum   above   the   amount   d. Who  can  file?  
found   as   actual   damages   sustained:   Provided,   That   the    
award   does   not   exceed   three   (3)   times   the   amount   of  
SECTION   77.   INFRINGEMENT   ACTION   BY   A   FOREIGN  
such  actual  damages.  
  NATIONAL.  –  
76.5.   The   court   may,   in   its   discretion,   order   that   the   Any   foreign   national   or   juridical   entity   who   meets   the  
infringing   goods,   materials   and   implements   requirements   of   Section   3   and   not   engaged   in   business  
predominantly   used   in   the   infringement   be   disposed   of   in   the   Philippines,   to   which   a   patent   has   been   granted   or  
outside  the  channels  of  commerce  or  destroyed,  without   assigned   under   this   Act,   may   bring   an   action   for  
compensation.   infringement   of   patent,   whether   or   not   it   is   licensed   to  
  do  business  in  the  Philippines  under  existing  law.  
76.6.  Anyone  who  actively  induces  the  infringement  of  a    
patent   or   provides   the   infringer   with   a   component   of   a  

164 z
Intellectual Property Law

CRESER  PRECISION  SYSTEM,  INC.  V.  CA  (1998)   SECTION  80.  DAMAGES,  REQUIREMENT  OF  NOTICE.  –    
Court  decision:   Damages   cannot   be   recovered   for   acts   of   infringement  
Section   42   of   R.A.   165,   otherwise   known   as   the   Patent   committed   before   the   infringer   had   known,   or   had  
Law,  explicitly  provides:   reasonable   grounds   to   know   of   the   patent.   It   is  
  presumed   that   the   infringer   had   known   of   the   patent   if  
SECTION.   42.   Civil   action   for   infringement.   –   on  the  patented  product,  or  on  the  container  or  package  
Any   patentee,   or   anyone   possessing   any   right,   in   which   the   article   is   supplied   to   the   public,   or   on   the  
title   or   interest   in   and   to   the   patented   advertising  material  relating  to  the  patented  product  or  
invention,   whose   rights   have   been   infringed,   process,   are   placed   the   words   "Philippine   Patent"   with  
may  bring  a  civil  action  before  the  proper  Court   the  number  of  the  patent.  
of   First   Instance   (now   Regional   Trial   court),   to    
recover   from   the   infringer   damages   sustained   g. Defenses   in   Action  
by   reason   of   the   infringement   and   to   secure   an  
for  Infringement  
injunction  for  the  protection  of  his  right.    
 
 
Under   the   aforequoted   law,   only   the   patentee   or   his   SECTION  81.  DEFENSES  IN  ACTION  FOR  INFRINGEMENT.  –    
successors-­‐in-­‐interest   may   file   an   action   for   infringement.   In  an  action  for  infringement,  the  defendant,  in  addition  
The  phrase  “anyone  possessing  any  right,  title  or  interest   to   other   defenses   available   to   him,   may   show   the  
in  and  to  the  patented  invention”  upon  which  petitioner   invalidity   of   the   patent,   or   any   claim   thereof,   on   any   of  
maintains   its   present   suit,   refers   only   to   the   patentee’s   the   grounds   on   which   a   petition   of   cancellation   can   be  
successors-­‐in-­‐interest,   assignees   or   grantees   since   brought  under  Section  61  hereof.  
actions  for  infringement  of  patent  may  be  brought  in  the    
name   of   the   person   or   persons   interested,   whether   as   h. Patent   Found   Invalid  
patentee,   assignees   or   grantees,   of   the   exclusive   right.   may  be  Cancelled  
Moreover,  there  can  be  no  infringement  of  a  patent  until  
 
a   patent   has   been   issued,   since   whatever   right   one   has  
to   the   invention   covered   by   the   patent   arises   alone   from  
SECTION   82.   PATENT   FOUND   INVALID   MAY   BE   CANCELLED.  
the  grant  of  patent.  In  short,  a  person  or  entity  who  has   –    
not   been   granted   letters   patent   over   an   invention   and   In   an   action   for   infringement,   if   the   court   shall   find   the  
has   not   acquired   any   right   or   title   thereto   either   as   patent  or  any  claim  to  be  invalid,  it  shall  cancel  the  same,  
assignee   or   as   licensee,   has   no   cause   of   action   for   and  the  Director  of  Legal  Affairs  upon  receipt  of  the  final  
infringement   because   the   right   to   maintain   an   judgment   of   cancellation   by   the   court,   shall   record   that  
infringement  suit  depends  on  the  existence  of  the  patent.   fact  in  the  register  of  the  Office  and  shall  publish  a  notice  
  to  that  effect  in  the  IPO  Gazette.  
e. Presumptions    
  i. Assessors  
SECTION  78.  PROCESS  PATENTS;  BURDEN  OF  PROOF.  –      
If   the   subject   matter   of   a   patent   is   a   process   for   SECTION  83.  ASSESSOR  IN  INFRINGEMENT  ACTION.  –    
obtaining   a   product,   any   identical   product   shall   be   83.1.  Two  (2)  or  more  assessors  may  be  appointed  by  the  
presumed   to   have   been   obtained   through   the   use   of   the   court.  The  assessors  shall  be  possessed  of  the  necessary  
patented   process   if   the   product   is   new   or   there   is   scientific   and   technical   knowledge   required   by   the  
substantial   likelihood   that   the   identical   product   was   subject   matter   in   litigation.   Either   party   may   challenge  
made   by   the   process   and   the   owner   of   the   patent   has   the  fitness  of  any  assessor  proposed  for  appointment.  
been   unable   despite   reasonable   efforts,   to   determine    
the   process   actually   used.   In   ordering   the   defendant   to   83.2.   Each   assessor   shall   receive   a   compensation   in   an  
prove  that  the  process  to  obtain  the  identical  product  is   amount   to   be   fixed   by   the   court   and   advanced   by   the  
different   from   the   patented   process,   the   court   shall   complaining  party,  which  shall  be  awarded  as  part  of  his  
adopt   measures   to   protect,   as   far   as   practicable,   his   costs  should  he  prevail  in  the  action.  
manufacturing  and  business  secrets.    
 
3. Licensing    
f. Damages    
 
 
a. Voluntary  
SECTION  79.  LIMITATION  OF  ACTION  FOR  DAMAGES.  –    
 
No   damages   can   be   recovered   for   acts   of   infringement  
SECTION  85.  VOLUNTARY  LICENSE  CONTRACT.  –    
committed   more   than   four   (4)   years   before   the  
institution  of  the  action  for  infringement.   To   encourage   the   transfer   and   dissemination   of  
  technology,   prevent   or   control   practices   and   conditions  

o 165
Katrina Michelle Mancao
 
that   may   in   particular   cases   constitute   an   abuse   of   arrangement,  except  in  cases  of  early  termination  of  the  
intellectual   property   rights   having   an   adverse   effect   on   technology   transfer   arrangement   due   to   reason(s)  
competition   and   trade,   all   technology   transfer   attributable  to  the  licensee;  
arrangements   shall   comply   with   the   provisions   of   this    
Chapter.   87.10.   Those   which   require   payments   for   patents   and  
  other   industrial   property   rights   after   their   expiration,  
SECTION   86.   JURISDICTION   TO   SETTLE   DISPUTES   ON   termination  arrangement;  
ROYALTIES.  –      
The   Director   of   the   Documentation,   Information   and   87.11.  Those  which  require  that  the  technology  recipient  
Technology   Transfer   Bureau   shall   exercise   quasi-­‐judicial   shall   not   contest   the   validity   of   any   of   the   patents   of   the  
jurisdiction   in   the   settlement   of   disputes   between   technology  supplier;  
parties   to   a   technology   transfer   arrangement   arising    
from  technology  transfer  payments,  including  the  fixing   87.12.   Those   which   restrict   the   research   and  
of  appropriate  amount  or  rate  of  royalty.   development   activities   of   the   licensee   designed   to  
  absorb   and   adapt   the   transferred   technology   to   local  
SECTION  87.  PROHIBITED  CLAUSES.  –     conditions   or   to   initiate   research   and   development  
Except   in   cases   under   Section   91,   the   following   programs  in  connection  with  new  products,  processes  or  
provisions   shall   be   deemed   prima   facie   to   have   an   equipment;  
adverse  effect  on  competition  and  trade:    
  87.13.   Those   which   prevent   the   licensee   from   adapting  
87.1.   Those   which   impose   upon   the   licensee   the   the   imported   technology   to   local   conditions,   or  
obligation  to  acquire  from  a  specific  source  capital  goods,   introducing   innovation   to   it,   as   long   as   it   does   not   impair  
intermediate   products,   raw   materials,   and   other   the  quality  standards  prescribed  by  the  licensor;  
technologies,   or   of   permanently   employing   personnel    
indicated  by  the  licensor;   87.14.   Those   which   exempt   the   licensor   for   liability   for  
  non-­‐fulfilment   of   his   responsibilities   under   the  
87.2.   Those   pursuant   to   which   the   licensor   reserves   the   technology   transfer   arrangement   and/or   liability   arising  
right   to   fix   the   sale   or   resale   prices   of   the   products   from   third   party   suits   brought   about   by   the   use   of   the  
manufactured  on  the  basis  of  the  license;   licensed  product  or  the  licensed  technology;  and  
   
87.3.   Those   that   contain   restrictions   regarding   the   87.15.  Other  clauses  with  equivalent  effects.  
volume  and  structure  of  production;    
  SECTION  88.  MANDATORY  PROVISIONS.  –    
87.4.   Those   that   prohibit   the   use   of   competitive   The   following   provisions   shall   be   included   in   voluntary  
technologies   in   a   non-­‐exclusive   technology   transfer   license  contracts:  
agreement;    
  88.1.   That   the   laws   of   the   Philippines   shall   govern   the  
87.5.   Those   that   establish   a   full   or   partial   purchase   interpretation  of  the  same  and  in  the  event  of  litigation,  
option  in  favor  of  the  licensor;   the   venue   shall   be   the   proper   court   in   the   place   where  
  the  licensee  has  its  principal  office;  
87.6.  Those  that  obligate  the  licensee  to  transfer  for  free    
to   the   licensor   the   inventions   or   improvements   that   may   88.2.   Continued   access   to   improvements   in   techniques  
be  obtained  through  the  use  of  the  licensed  technology;   and   processes   related   to   the   technology   shall   be   made  
  available   during   the   period   of   the   technology   transfer  
87.7.   Those   that   require   payment   of   royalties   to   the   arrangement;  
 
owners  of  patents  for  patents  which  are  not  used;  
  88.3.   In   the   event   the   technology   transfer   arrangement  
87.8.   Those   that   prohibit   the   licensee   to   export   the   shall  provide  for  arbitration,  the  Procedure  of  Arbitration  
of   the   Arbitration   Law   of   the   Philippines   or   the  
licensed  product  unless  justified  for  the  protection  of  the  
Arbitration   Rules   of   the   United   Nations   Commission   on  
legitimate   interest   of   the   licensor   such   as   exports   to  
International   Trade   Law   (UNCITRAL)   or   the   Rules   of  
countries   where   exclusive   licenses   to   manufacture  
Conciliation   and   Arbitration   of   the   International  
and/or   distribute   the   licensed   product(s)   have   already  
Chamber  of  Commerce  (ICC)  shall  apply  and  the  venue  of  
been  granted;  
arbitration   shall   be   the   Philippines   or   any   neutral  
 
country;  and  
87.9.   Those   which   restrict   the   use   of   the   technology  
 
supplied   after   the   expiration   of   the   technology   transfer  

166 z
Intellectual Property Law

88.4.  The  Philippine  taxes  on  all  payments  relating  to  the   93.2.   Where   the   public   interest,   in   particular,   national  
technology   transfer   arrangement   shall   be   borne   by   the   security,   nutrition,   health   or   the   development   of   other  
licensor.     vital   sectors   of   the   national   economy   as   determined   by  
  the  appropriate  agency  of  the  Government,  so  requires;  
SECTION  89.  RIGHTS  OF  LICENSOR.  –     or  
In   the   absence   of   any   provision   to   the   contrary   in   the    
technology  transfer  arrangement,  the  grant  of  a  license   93.3.   Where   a   judicial   or   administrative   body   has  
shall   not   prevent   the   licensor   from   granting   further   determined   that   the   manner   of   exploitation   by   the  
licenses   to   third   person   nor   from   exploiting   the   subject   owner  of  the  patent  or  his  licensee  is  anti-­‐competitive;  or  
matter  of  the  technology  transfer  arrangement  himself.   "93.4.  In  case  of  public  non-­‐commercial  use  of  the  patent  
  by  the  patentee,  without  satisfactory  reason;  
SECTION  90.  RIGHTS  OF  LICENSEE.  –      
The   licensee   shall   be   entitled   to   exploit   the   subject   93.5.  If  the  patented  invention  is  not  being  worked  in  the  
matter   of   the   technology   transfer   arrangement   during   Philippines   on   a   commercial   scale,   although   capable   of  
the  whole  term  of  the  technology  transfer  arrangement.   being   worked,   without   satisfactory   reason:   Provided,  
  That   the   importation   of   the   patented   article   shall  
SECTION  91.  EXCEPTIONAL  CASES.  –     constitute  working  or  using  the  patent;  and  
 
In   exceptional   or   meritorious   cases   where   substantial  
93.6.   Where   the   demand   for   patented   drugs   and  
benefits   will   accrue   to   the   economy,   such   as   high  
technology   content,   increase   in   foreign   exchange   medicines  is  not  being  met  to  an  adequate  extent  and  on  
earnings,   employment   generation,   regional   dispersal   of   reasonable   terms,   as   determined   by   the   Secretary   of   the  
industries   and/or   substitution   with   or   use   of   local   raw   Department  of  Health.  
materials,   or   in   the   case   of   Board   of   Investments,    
registered   companies   with   pioneer   status,   exemption   SECTION   93-­‐A.   PROCEDURES   ON   ISSUANCE   OF   A   SPECIAL  
from  any  of  the  above  requirements  may  be  allowed  by   COMPULSORY  LICENSE  UNDER  THE  TRIPS  AGREEMENT.  –    
the   Documentation,   Information   and   Technology   93-­‐A.1.   The   Director   General   of   the   Intellectual   Property  
Transfer   Bureau   after   evaluation   thereof   on   a   case   by   Office,   upon   the   written   recommendation   of   the  
case  basis.   Secretary  of  the  Department  of  Health,  shall,  upon  filing  
  of   a   petition,   grant   a   special   compulsory   license   for   the  
SECTION   92.   NON-­‐REGISTRATION   WITH   THE   importation   of   patented   drugs   and   medicines.   The  
DOCUMENTATION,   INFORMATION   AND   TECHNOLOGY   special   compulsory   license   for   the   importation  
TRANSFER  BUREAU .  –     contemplated  under  this  provision  shall  be  an  additional  
special  alternative  procedure  to  ensure  access  to  quality  
Technology   transfer   arrangements   that   conform   with  
affordable  medicines  and  shall  be  primarily  for  domestic  
the   provisions   of   Sections   86   and   87   need   not   be  
consumption:   Provided,   That   adequate   remuneration  
registered   with   the   Documentation,   Information   and  
shall  be  paid  to  the  patent  owner  either  by  the  exporting  
Technology  Transfer  Bureau.  Non-­‐conformance  with  any  
or   importing   country.   The   compulsory   license   shall   also  
of   the   provisions   of   Sections   87   and   88,   however,   shall  
contain   a   provision   directing   the   grantee   the   license   to  
automatically   render   the   technology   transfer  
exercise   reasonable   measures   to   prevent   the   re-­‐
arrangement   unenforceable,   unless   said   technology  
exportation   of   the   products   imported   under   this  
transfer   arrangement   is   approved   and   registered   with  
provision.  
the   Documentation,   Information   and   Technology  
 
Transfer   Bureau   under   the   provisions   of   Section   91   on  
The   grant   of   a   special   compulsory   license   under   this  
exceptional  cases.  
provision   shall   be   an   exception   to   Sections   100.4   and  
 
100.6  of  Republic  Act  No.  8293  and  shall  be  immediately  
b. Compulsory   executory.  
   
SECTION  93.  GROUNDS  FOR  COMPULSORY  LICENSING.  –     No   court,   except   the   Supreme   Court   of   the   Philippines,  
The   Director   General   of   the   Intellectual   Property   Office   shall   issue   any   temporary   restraining   order   or  
may  grant  a  license  to  exploit  a  patented  invention,  even   preliminary  injunction  or  such  other  provisional  remedies  
without  the  agreement  of  the  patent  owner,  in  favor  of   that   will   prevent   the   grant   of   the   special   compulsory  
any   person   who   has   shown   his   capability   to   exploit   the   license.  
invention,  under  any  of  the  following  circumstances:    
  93-­‐A.2.   A   compulsory   license   shall   also   be   available   for  
93.1.   National   emergency   or   other   circumstances   of   the   manufacture   and   export   of   drugs   and   medicines   to  
extreme  urgency;   any   country   having   insufficient   or   no   manufacturing  
  capacity   in   the   pharmaceutical   sector   to   address   public  

o 167
Katrina Michelle Mancao
 
health   problems:   Provided,   That,   a   compulsory   license    
has   been   granted   by   such   country   or   such   country   has,   95.3.   In   situations   of   national   emergency   or   other  
by   notification   or   otherwise,   allowed   importation   into   its   circumstances   of   extreme   urgency,   the   right   holder   shall  
jurisdiction   of   the   patented   drugs   and   medicines   from   be  notified  as  soon  as  reasonably  practicable.  
the  Philippines  in  compliance  with  the  TRIPS  Agreement.    
  95.4.   In   the   case   of   public   non-­‐commercial   use,   where  
93-­‐A.3.   The   right   to   grant   a   special   compulsory   license   the  government  or  contractor,  without  making  a  patent  
under  this  section  shall  not  limit  or  prejudice  the  rights,   search,   knows   or   has   demonstrable   grounds   to   know  
obligations   and   flexibilities   provided   under   the   TRIPS   that   a   valid   patent   is   or   will   be   used   by   or   for   the  
Agreement   and   under   Philippine   laws,   particularly   government,  the  right  holder  shall  be  informed  promptly.  
Section   72.1   and   Section   74   of   the   Intellectual   Property    
Code,   as   amended   under   this   Act.   It   is   also   without   95.5.   Where   the   demand   for   the   patented   drugs   and  
prejudice   to   the   extent   to   which   drugs   and   medicines   medicines   in   the   Philippines   is   not   being   met   to   an  
produced  under  a  compulsory  license  can  be  exported  as   adequate   extent   and   on   reasonable   terms,   as  
allowed  in  the  TRIPS  Agreement  and  applicable  laws.   determined   by   the   Secretary   of   the   Department   of  
  Health,  the  right  holder  shall  be  informed  promptly.  
SECTION   94.   PERIOD   FOR   FILING   A   PETITION   FOR   A    
COMPULSORY  LICENSE.  –     SECTION   96.   COMPULSORY   LICENSING   OF   PATENTS  
94.1.   A   compulsory   license   may   not   be   applied   for   on   the   INVOLVING  SEMI-­‐CONDUCTOR  TECHNOLOGY.  –    
ground   stated   in   Subsection   93.5   before   the   expiration   In  the  case  of  compulsory  licensing  of  patents  involving  
of  a  period  of  four  (4)  years  from  the  date  of  filing  of  the   semi-­‐conductor   technology,   the   license   may   only   be  
application  or  three  (3)  years  from  the  date  of  the  patent   granted   in   case   of   public   non-­‐commercial   use   or   to  
whichever  period  expires  last.   remedy   a   practice   determined   after   judicial   or  
  administrative  process  to  be  anti-­‐competitive.    
94.2.   A   compulsory   license   which   is   applied   for   on   any   of    
the  grounds  stated  in  Subsections  93.2,  93.3,  93.4,  and     SECTION   97.   COMPULSORY   LICENSE   BASED   ON  
 
INTERDEPENDENCE  OF  PATENTS.  –    
93.6  and  Section  97  may  be  applied  for  at  any  time  after  
If  the  invention  protected  by  a  patent,  hereafter  referred  
the  grant  of  the  patent.  
to  as  the  "second  patent,"  within  the  country  cannot  be  
 
worked   without   infringing   another   patent,   hereafter  
SECTION   95.   REQUIREMENT   TO   OBTAIN   A   LICENSE   ON   referred   to   as   the   "first   patent,"   granted   on   a   prior  
REASONABLE  COMMERCIAL  TERMS.  –     application   or   benefiting   from   an   earlier   priority,   a  
95.1.  The  license  will  only  be  granted  after  the  petitioner   compulsory  license  may  be  granted  to  the  owner  of  the  
has   made   efforts   to   obtain   authorization   from   the   second   patent   to   the   extent   necessary   for   the   working  
patent   owner   on   reasonable   commercial   terms   and   of  his  invention,  subject  to  the  following  conditions:  
conditions   but   such   efforts   have   not   been   successful    
within  a  reasonable  period  of  time.   97.1.  The  invention  claimed  in  the  second  patent  involves  
  an   important   technical   advance   of   considerable  
95.2.   The   requirement   under   Subsection   95.1   shall   not   economic  significance  in  relation  to  the  first  patent;  
apply  in  any  of  the  following  cases:    
  97.2.  The  owner  of  the  first  patent  shall  be  entitled  to  a  
(a)   Where   the   petition   for   compulsory   license   cross-­‐license   on   reasonable   terms   to   use   the   invention  
seeks   to   remedy   a   practice   determined   after   claimed  in  the  second  patent;  
judicial   or   administrative   process   to   be   anti-­‐  
competitive;   97.3.   The   use   authorized   in   respect   of   the   first   patent  
  shall   be   non-­‐assignable   except   with   the   assignment   of  
(b)  In  situations  of  national  emergency  or  other   the  second  patent;  and  
circumstances  of  extreme  urgency;    
  97.4.   The   terms   and   conditions   of   Sections   95,   96   and   98  
(c)  In  cases  of  public  non-­‐commercial  use;  and   to  100  of  this  Act.    
   
(d)   In   cases   where   the   demand   for   the   SECTION  98.  FORM  AND  CONTENTS  OF  PETITION.  –    
patented   drugs   and   medicines   in   the  
The  petition  for  compulsory  licensing  must  be  in  writing,  
Philippines   is   not   being   met   to   an   adequate  
verified   by   the   petitioner   and   accompanied   by   payment  
extent  and  on  reasonable  terms,  as  determined  
of   the   required   filing   fee.   It   shall   contain   the   name   and  
by  the  Secretary  of  the  Department  of  Health.  

168 z
Intellectual Property Law

address   of   the   petitioner   as   well   as   those   of   the   100.6.  The  patentee  shall  be  paid  adequate  remuneration  
respondents,  the  number  and  date  of  issue  of  the  patent   taking   into   account   the   economic   value   of   the   grant   or  
in   connection   with   which   compulsory   license   is   sought,   authorization,  except  that  in  cases  where  the  license  was  
the  name  of  the  patentee,  the  title  of  the  invention,  the   granted   to   remedy   a   practice   which   was   determined  
statutory   grounds   upon   which   compulsory   license   is   after   judicial   or   administrative   process,   to   be   anti-­‐
sought,   the   ultimate   facts   constituting   the   petitioner's   competitive,   the   need   to   correct   the   anti-­‐competitive  
cause  of  action,  and  the  relief  prayed  for.   practice  may  be  taken  into  account  in  fixing  the  amount  
  of  remuneration.    
SECTION  99.  NOTICE  OF  HEARING.  –      
99.1.   Upon   filing   of   a   petition,   the   Director   of   Legal   SECTION  101.  AMENDMENT,  CANCELLATION,  SURRENDER  OF  
Affairs   shall   forthwith   serve   notice   of   the   filing   thereof   COMPULSORY  LICENSE.  –    
upon  the  patent  owner  and  all  persons  having  grants  or   101.1.   Upon   the   request   of   the   patentee   or   the   licensee,  
licenses,  or  any  other  right,  title  or  interest  in  and  to  the  
the   Director   of   Legal   Affairs   may   amend   the   decision  
patent   and   invention   covered   thereby   as   appears   of  
granting   the   compulsory   license,   upon   proper   showing  
record   in   the   Office,   and   of   notice   of   the   date   of   hearing  
of   new   facts   or   circumstances   justifying   such  
thereon,   on   such   persons   and   petitioner.   The   resident  
amendment.  
agent   or   representative   appointed   in   accordance   with  
 
Section   33   hereof,   shall   be   bound   to   accept   service   of  
101.2.   Upon   the   request   of   the   patentee,   the   said  
notice  of  the  filing  of  the  petition  within  the  meaning  of  
this  Section.   Director  may  cancel  the  compulsory  license:  
   
(a)   If   the   ground   for   the   grant   of   the  
99.2.   In   every   case,   the   notice   shall   be   published   by   the  
compulsory   license   no   longer   exists   and   is  
said  Office  in  a  newspaper  of  general  circulation,  once  a  
unlikely  to  recur;  
week   for   three   (3)   consecutive   weeks   and   once   in   the  
 
IPO  Gazette  at  applicant's  expense.  
(b)  If  the  licensee  has  neither  begun  to  supply  
 
the   domestic   market   nor   made   serious  
SECTION   100.   TERMS   AND   CONDITIONS   OF   COMPULSORY   preparation  therefor;  
LICENSE.  –      
The   basic   terms   and   conditions   including   the   rate   of   (c)   If   the   licensee   has   not   complied   with   the  
royalties   of   a   compulsory   license   shall   be   fixed   by   the   prescribed  terms  of  the  license;  
Director   of   Legal   Affairs   subject   to   the   following    
conditions:   101.3.  The  licensee  may  surrender  the  license  by  a  written  
  declaration  submitted  to  the  Office.  
100.1.   The   scope   and   duration   of   such   license   shall   be    
limited  to  the  purpose  for  which  it  was  authorized;   101.4.   The   said   Director   shall   cause   the   amendment,  
  surrender,   or   cancellation   in   the   Register,   notify   the  
100.2.  The  license  shall  be  non-­‐exclusive;   patentee,   and/or   the   licensee,   and   cause   notice   thereof  
  to  be  published  in  the  IPO  Gazette.  
100.3.   The   license   shall   be   non-­‐assignable,   except   with    
that   part   of   the   enterprise   or   business   with   which   the   SECTION  102.  LICENSEE'S  EXEMPTION  FROM  LIABILITY.  –    
invention  is  being  exploited;   Any   person   who   works   a   patented   product,   substance  
  and/or   process   under   a   license   granted   under   this  
100.4.   Use   of   the   subject   matter   of   the   license   shall   be   Chapter,  shall  be  free  from  any  liability  for  infringement:  
devoted   predominantly   for   the   supply   of   the   Philippine   Provided  however,  That  in  the  case  of  voluntary  licensing,  
market:   Provided,   That   this   limitation   shall   not   apply   no   collusion   with   the   licensor   is   proven.   This   is   without  
where   the   grant   of   the   license   is   based   on   the   ground   prejudice   to   the   right   of   the   rightful   owner   of   the   patent  
that   the   patentee's   manner   of   exploiting   the   patent   is   to   recover   from   the   licensor   whatever   he   may   have  
determined   by   judicial   or   administrative   process,   to   be   received  as  royalties  under  the  license.  
anti-­‐competitive.    
  PRICE  V.  UNITED  LABORATORIES  (1988)  
100.5.   The   license   may   be   terminated   upon   proper   Court  decision:  
showing   that   circumstances   which   led   to   its   grant   have   Price   is   owner-­‐assignee   of   Phil   Patent   No.   13540   for   a  
ceased  to  exist  and  are  unlikely  to  recur:  Provided,  That   pharmaceutical   compound   used   for   anti-­‐ulcer.  
adequate   protection   shall   be   afforded   to   the   legitimate   Respondent   UniLab   filed   in   the   PPO   a   petition   for   the  
interest  of  the  licensee;  and   issuance   of   compulsory   license   to   use   the   patented  
  compound   in   its   own   brands   of   medicines   and   to   sell,  

o 169
Katrina Michelle Mancao
 
distribute   or   otherwise   dispose   of   such   med   or   SMITH   KLINE   V.   CA   &   DANLEX   RESEARCH   LABORATORIES  
pharmaceutical   preparation   in   the   country.   After   hearing,  
INC.  (2001)  
PPO   granted   UniLab   compulsory   license   subject   to   10  
Refresher:  
terms  and  conditions.    
Also   on   Cimetidine.   This   time   BPTTT   granted   compulsory  
 
WON  Director’s  unilateral  determination  of  the  terms  and   license  to  Danlex.    
 
conditions  of  the  compulsory  license  without  affording  the  
Court  decision:  
parties   an   opportunity   to   negotiate   the   terms   and  
The   grant   of   the   compulsory   license   satisfies   the  
conditions  freely  and  by  themselves  is  valid?    
requirements  of  Sec.  34  of  Patent  Law.  
Director   of   Patents   authorized   to   fix   the   terms   and  
[1]   More   than   ten   years   have   passed   since   the   patent   for  
conditions   of   the   compulsory   license.   -­‐   The   terms   and  
Cimetidine  was  issued  to  petitioner  and  its  predecessors-­‐
conditions   of   the   compulsory   license   were   fixed   by   the  
in-­‐interest  
Director   of   Patents   after   a   hearing   and   careful  
[2]   The   compulsory   license   applied   for   by   private  
consideration   of   the   evidence   of   the   parties   and   in  
default  of  an  agreement  between  them  as  to  the  terms   respondent   is   for   the   use,   manufacture   and   sale   of   a  
medicinal  product  
of  the  license.  This  he  is  authorized  to  do  under  Section  
[3]   Both   the   appellate   court   and   the   BPTTT   found   that  
36  of  Republic  Act  No.  165.    
private  respondent  had  the  capability  to  work  Cimetidine  
 
or   to   make   use   thereof   in   the   manufacture   of   a   useful  
In   any   event,   since   the   petitioner   Price   will   still   be   paid  
product.  
royalties   on   the   sales   of   any   products   the   licensee   may  
 
manufacture  using  any  or  all  of  the  patented  compounds,  
There   is   no   basis   for   the   allegation   that   the   grant   of  
the   petitioner   cannot   complain   of   a   deprivation   of  
compulsory   license   to   private   respondent   results   in   the  
property  rights  without  just  compensation.  
  deprivation   of   petitioner’s   property   without   just  
compensation.   Even   if   other   entities   like   private  
SMITH   KLINE  V.   CA,   BPTTT  &   DOCTORS  PHARMACEUTICAL    
respondent   are   subsequently   allowed   to   manufacture,  
(1997)   use   and   sell   the   patented   invention   by   virtue   of   a  
Refresher:   compulsory   license,   petitioner   as   owner   of   the   patent  
Smith   Kline   owns   patent   for   Cimetidine.   Doctors   Pharm   would   still   receive   remuneration   for   the   use   of   such  
filed   petition   for   compulsory   licensing   with   BPTTT   for   product  in  the  form  of  royalties.  
authorization  to  manufacture  its  own  brand  of  medicine    
from   the   drug   Cimetidine   and   to   market   the   resulting   The   rate   of   2.5%   of   net   wholesale   price   fixed   by   the  
product   in   the   PH.   BPTTT   granted   the   petition.   Smith   Director   of   the   BPTTT   is   not   unreasonable,   inadequate  
Kline  cries  foul.     and  is  in  fact,  in  accord  with  the  Patent  Law.  Reasons:    
  -­‐ The   compulsory   license   awarded   to   private  
Court  decision:   respondent   consists   only   of   the   bare   right   to  
SC   upheld   the   BPTTT   grant   of   compulsory   license.   use  the  patented  invention  in  the  manufacture  
Section   A   (2)   of   Paris   Convention   R.A.   No.   165   of   another   product,   without   any   technical  
unequivocally   and   explicitly   respects   the   right   of   assistance  from  the  licensor.  
member   countries   to   adopt   legislative   measures   to   -­‐ Identical  royalty  rates  have  been  prescribed  by  
provide  for  the  grant  of  compulsory  licenses  to  prevent   the   Director   of   the   BPTTT   in   numerous   patent  
abuses   which   might   result   from   the   exercise   of   the   cases.  
exclusive  rights  conferred  by  the  patent.      
  4. Right   to   Transfer   and  
Section   34   of   R.A.   No.   165,   even   if   the   Act   was   enacted  
Assign  
prior  to  the  Philippines'  adhesion  to  the  Convention,  fits  
well  within  the  aforequoted  provisions  of  Article  5  of  the    
Paris   Convention.   In   the   explanatory   note   of   Bill   No.   1156   SECTION  4.2.    
which   eventually   became   R.A.   No.   165,   the   legislative   The   term   "TECHNOLOGY   TRANSFER   ARRANGEMENTS"   refers   to  
intent  in  the  grant  of  a  compulsory  license  was  not  only   contracts   or   agreements   involving   the   transfer   of  
to   afford   others   an   opportunity   to   provide   the   public   systematic   knowledge   for   the   manufacture   of   a   product,  
with   the   quantity   of   the   patented   product,   but   also   to   the   application   of   a   process,   or   rendering   of   a   service  
prevent  the  growth  of  monopolies.     including   management   contracts;   and   the   transfer,  
  assignment   or   licensing   of   all   forms   of   intellectual  
Certainly,   the   growth   of   monopolies   was   among   the   property   rights,   including   licensing   of   computer  
abuses   which   Section   A,   Article   5   of   the   Convention   software   except   computer   software   developed   for   mass  
foresaw,   and   which   our   Congress   likewise   wished   to   market.  
prevent  in  enacting  R.A.  No.  165.    
 

170 z
Intellectual Property Law

SECTION  103.  TRANSMISSION  OF  RIGHTS.  –     however,   That   neither   of   the   joint   owners   shall   be  
103.1.   Patents   or   applications   for   patents   and   invention   entitled   to   grant   licenses   or   to   assign   his   right,   title   or  
interest  or  part  thereof  without  the  consent  of  the  other  
to  which  they  relate,  shall  be  protected  in  the  same  way  
owner  or  owners,  or  without  proportionally  dividing  the  
as  the  rights  of  other  property  under  the  Civil  Code.  
proceeds  with  such  other  owner  or  owners.  (Sec.  54,  R.A.  
 
No.  165)  
103.2.  Inventions  and  any  right,  title  or  interest  in  and  to  
 
patents   and   inventions   covered   thereby,   may   be  
SEE  SECTIONS  85-­‐92,  SUPRA  (ON  VOLUNTARY  LICENSING).  
assigned   or   transmitted   by   inheritance   or   bequest   or  
may  be  the  subject  of  a  license  contract.    
  ALBANA  V.  DIRECTOR  OF  PATENTS  (1953)  
SECTION  104.  ASSIGNMENT  OF  INVENTIONS.  –   Court  decision:  
An  assignment  may  be  of  the  entire  right,  title  or  interest   Assignment   of   patents   and   inventions   covered   thereby  
in  and  to  the  patent  and  the  invention  covered  thereby,   may  be  recorded  in  the  books  and  records  and  kept  for  
or   of   an   undivided   share   of   the   entire   patent   and   the  purpose  in  the  Patent  Office  if  presented  in  due  form.  
invention,   in   which   event   the   parties   become   joint   BUT   where   a   person   other   than   the   inventor   files   a  
owners   thereof.   An   assignment   may   be   limited   to   a   motion   with   the   Director   of   Patents   praying   that   the  
specified  territory.     applicant-­‐inventor   be   compelled   to   sign   the   contract  
  executed   by   a   co-­‐applicant-­‐inventor  and  both  applicant-­‐
inventors   to   acknowledge   it   before   a   notary   public   as  
SECTION  105.  FORM  OF  ASSIGNMENT.  –    
well  as  another  document  which  refers  to  the  minutes  of  
The   assignment   must   be   in   writing,   acknowledged   a   meeting   of   the   organizers   of   the   manufacturing  
before   a   notary   public   or   other   officer   authorized   to   corporation,  and  then  to  have  both  documents  recorded  
administer   oath   or   perform   notarial   acts,   and   certified   in   the   Patent   Office   and   in   the   office   of   the   registrar   of  
under   the   hand   and   official   seal   of   the   notary   or   such   deeds,   the   Director   of   Patents   has   no   power   and  
other  officer.     authority   to   compel   the   applicant-­‐inventors   to   do   what  
  the  movant  is  asking  them  to  do.    
SECTION  106.  RECORDING.  –      
106.1.   The   Office   shall   record   assignments,   licenses   and   What  the  movant  asks  the  Director  of  Patents  to  do  for  
other   instruments   relating   to   the   transmission   of   any   him  is  essentially  a  judicial  function  which  would  require  
right,  title  or  interest  in  and  to  inventions,  and  patents  or   the   determination   of   finding   by   a   court   of   competent  
application  for  patents  or  inventions  to  which  they  relate,   jurisdiction   as   to   whether   there   was   a   meeting   of   the  
which   are   presented   in   due   form   to   the   Office   for   minds   of   the   contracting   parties   before   it   could   compel  
registration,  in  books  and  records  kept  for  the  purpose.   any  of  them  to  perform  what  the  movant  prays  the  court  
The   original   documents   together   with   a   signed   duplicate   to  order  him  to  do.  
thereof  shall  be  filed,  and  the  contents  thereof  should  be  
kept   confidential.   If   the   original   is   not   available,   an  
authenticated   copy   thereof   in   duplicate   may   be   filed.  
Upon   recording,   the   Office   shall   retain   the   duplicate,  
return  the  original  or  the  authenticated  copy  to  the  party  
who  filed  the  same  and  notice  of  the  recording  shall  be  
published  in  the  IPO  Gazette.  
 
106.2.   Such   instruments   shall   be   void   as   against   any  
subsequent   purchaser   or   mortgagee   for   valuable  
consideration   and   without   notice,   unless,   it   is   so  
recorded  in  the  Office,  within  three  (3)  months  from  the  
date   of   said   instrument,   or   prior   to   the   subsequent  
purchase  or  mortgage.  
 
SECTION  107.  RIGHTS  OF  JOINT  OWNERS.  –    
If  two  (2)  or  more  persons  jointly  own  a  patent  and  the  
invention  covered  thereby,  either  by  the  issuance  of  the  
patent  in  their  joint  favor  or  by  reason  of  the  assignment  
of   an   undivided   share   in  the   patent   and   invention   or   by  
reason   of   the   succession   in   title   to   such   share,   each   of  
the  joint  owners  shall  be  entitled  to  personally  make,  use,  
sell,   or   import   the   invention   for   his   own   profit:   Provided,  

o 171

S-ar putea să vă placă și